NBME FULL SOLUTION PACK(Bundled exams with complete solutions)

Bulbus cordis

Smooth parts (outflow tract) of left and right ventricles

endocardial cushions

Atrial septum, membranous interventricular septum; AV and semilunar valves

neural crest

left horn of the sinus venosus

coronary sinus

posterior, sub cardinal, and supra cardinal veins

IVC

Right common cardinal vein and right anterior cardinal vein

SVC

Right horn of sinus venosus

Smooth part of right atrium (sinus venarum)

Patent foramen ovale

failure of septum primum and septum secundum to fuse after birth

Transposition of the great vessels
Tetralogy of Fallot
Persistent truncus arteriosus

Conotruncal abnormalities associated with failure of neural crest cells to migrate

ductus venosus

connects the umbilical vein to the inferior vena cava, bypassing the liver

becomes ligamentum venosum

phrenic nerve

innervates the diaphragm and pericardium

S3 heart sound

Increased ventricular filling pressure (e.g., mitral regurgitation, HF), common in dilated ventricles

normal in kids and pregnant women

S4 heart sound

atrial kick late diastole, right before S1

best heard at apex in LLD position

High atrial pressure.

Stiff/hypertrophic ventricle (aortic stenosis, restrictive cardiomyopathy)

Always abnormal

atria contract

a wave of JVP

c wave

RV contraction (closed tricuspid valve bulging into atrium) wave of JVP

x descent

JVP wave corresponding to downward displacement of closed tricuspid valve during rapid ventricular ejection phase

reduced or absent in tricuspid regurge

V wave

JVP wave corresponding to inc’d RA pressure due to filling against closed tricuspid valve

y descent

JVP wave corresponding to RA emptying into RV

absent in cardiac tamponade

plusus parvus et tardus

pulses are weak with delayed peak

Aortic stenosis

PR interval

0.12-0.20 seconds

120 milliseconds

QT interval length

9 – 11 squares = .36 to .44 seconds

Hypokalemia

U wave present on ECG

Mg sulfate

for torsades de pointe, hypokalemia (can lengthen QT and cause torsades), and pre-eclampsia (prevent seizures)

Romano-Ward syndrome

-Congenital long QT syndrome
-Autosomal dominant, pure cardiac phenotype (no deafness).

Jervell and Lange-Nielsen syndrome

-Congenital long QT syndrome
-Autosomal recessive, sensorineural deafness

Brugada syndrome

-Autosomal dominant disorder affecting Na channels most common in Asian males.
-ECG pattern of pseudo-right bundle branch block and ST elevations in V1-V3 (anterior ventricular septum)
-inc risk of ventricular tachyarrhythmias and sudden cardiac deatgh

Prevent SCD with implantable cardioverter-defibrillator (ICD).

Wolff-Parkinson-White Syndrome

Most common type of ventriuclar pre-excitation sydnrome. Abnormal fast accessory conduction pathway from atria to venricle bypasses the rate-slowing AV node causing a delta wave and widening QRS with shortened PR interval. Could lead to a reentrant circuit and suprvaventicular tachy.

First degree AV block

– PRI >5 boxes/.20 sec (200 msec)
– Fixed but prolonged PRI
(consistent but long)
– normally get bradycardia here

second degree AV block mobitz type 2

-PR interval is constant
-atrial conduction to ventricle is intermittent: dropped QRS without increasing PR interval length
-disease below AV node in His bundle

may progress to 3rd degree/complete AV block

Second Degree AV Block Mobitz Type 1 (wenckebach)

Progressive lengthening of pr interval leading to dropped QRS

third degree AV block

The atria and Ventricles are totally dissociated.
-So, the QRSs and the P waves have no relation to each other.

PCWP

4-12 mmHg
est of LA pressure

Williams Syndrome

a genetic condition characterized by mental retardation in most regards but surprisingly good use of language relative to their other abilities, elfin facies
Chromosome 7
assoc with supravalvular aortic stenosis

DiGeorge Syndrome

Maldevelopment of 3 and 4 pharyngeal pouches, fascial dysmorphia, cardiac shunt (trunks arteriosus, tetralogy of Fallot), lack of T-cells, undeveloped paracortex

Corneal arcus

Lipid deposits in the cornea. Common in the elderly, but appears earlier in life with hypercholesterolemia

Image: Corneal arcus

Stanford A aortic dissection

Dissection of the ascending aorta
Tx with surgery

Stanford B aortic dissection

Dissection of the descending aorta below the level o the left subclavian artery
Tx: Beta Blockers then vasodilators

Left bundle branch block

QRS> 120 msec
Deep, broad S waves in V1 and V2
Broad R waves in in V5 and V6

Image: Left bundle branch block

hypertrophic obstructive cardiomyopathy

common AD inherited heart defect of a thick septal wall

mutations in sarcomeric proteins (myosin binding protein C; Beta myosin heavy chain)

sudden death in young athletes

Loffler endocarditis

assoc w hypereosinophilic syndrome; histology shows eosinophilic infiltrates in myocardium

cause restrictive/infiltrative cardiomyopathy with low voltage ECG

Endocardial fibroelastosis

Dense layer of fibrosis and elastic tissue in the endocardium; cause of restrictive cardiomyopathy in children

low voltage ECG

Culture negative bacterial endocarditis

Coxiella
Bartonella
HACEK (Haemophilus, Aggregatibacter/Actinobacillus, Cardiobacterium, Eikenella, Kingella)

Erythema marginatum

rheumatic fever

Image: Erythema marginatum

Erythema multiforme

skin disorder resulting from a generalized allergic reaction to an illness, infection, or medication

Image: Erythema multiforme

Erythema migrans

Lyme disease

Image: Erythema migrans

pulsus paradoxus

beats have weaker amplitude with respiratory inspiration, stronger with expiration

seen in cardiac tamponade, asthma, obstructive sleep apnea, pericarditis, croup

Polyarteritis nodosa

affects multiple medium-sized arteries EXCEPT PULMONARY
Transmural inflammation with fibrinoid necrosis

assoc with Hep B

Bechet’s syndrome

recurrent oral ulcers, genital ulcers, uveitis

immune complex vasculitis, assoc with HLA-B51

cutaneous small vessel vasculitis

Occurs 7-10 days after certain medication (pen, cephalosporins, phenytoin, allopurinol) or infection (HCV,HIV).

Palpable purpura, no visceral involvement

Immune complex mediated leukocytoclastic vasculitis

mixed cryoglobulinemia

Often due to viral infection, especially HCV
Triad of palpable purpura, weakness, arthralgia. May also have peripheral neuropathy and renal disease.

Mixed IgG and IgA immune complex deposition vasculitis

Cardiac myxoma

benign tumor left atrium; embolization; syncope
Gelatinous
Hear early diastolic “tumor plop”
Produce IL-6 leading to constitutional symptoms of fever or weight loss

Kussmaul sign

Inc in JVP on inspiration
Seen in constrictive pericarditis, restrictive cardiomyopathies, right atrial or ventricular tumors

Osler-Weber-Rendu syndrome/Hereditary hemorrhagic telangiectasia

AD disorder of blood vessels

Telangiectasia on skin and mucous membranes, recurrent epistaxis, skin discoloration, AV malformations, GI bleeding, hematuria

paroxysmal atrial fibrillation

“holiday heart syndrome”
subsides in ~7 days
See irregular RR intervals, no discernible P waves, and tachycardia with narrow QRS

Friedrich’s ataxia

GAA trinucleotide repeat expansion on chromosome 9 results in defective frataxin production, which is involved in the synthesis of iron-sulfur clusters, in the mitochondrial respiratory chain. A deficiency of the protein causes iron to accumulate within cells, including in the heart, resulting hypertrophic cardiomyopathy, in pancreatic islets, causing diabetes, and in the spinal cord, causing neurologic dysfunction.

Gs protein coupled receptor

inc’s cAMP and activate protein kinase A

B1, B2, B3
D1
H2
V2

Occlusion of the paramedic branches of the anterior spinal artery and/or vertebral a.

Medial medullary syndrome

Infarct of corticospinal tract: Contralateral hemiparesis;
medial lemniscus: contralateral loss of proprioception; hypoglossal nucleus: ipsilateral tongue deviation

Retroperitoneal fibrosis/Ormond’s disease

rare disease of unknown etiology, characterized by inflammation and fibrosisof the retroperitoneum resulting in compression and encasement of the ureter, and/or the retroperitoneal blood vessels

Contrast CTis the diagnostic test of choice and reveals a retroperitonealmass encasing and obstructing the ureters and/or the aorta and IVC

Physiologic dead space (Vd)

=(PaCO2-PeCO2)/PaCO2

apprx equal to anatomic dead space in healthy lungs

Alveolar ventilation

the amount of air that reaches the alveoli each minute

=(Tidal vol – Physiologic dead space) x RR

atmospheric pressure, negative

At FRC, airway and alveolar pressures equal ___________________ (called zero), and intrapleural pressure is _________.

Normal A-a gradient

10-15 mmHg

(age/4)+4

Alveolar Gas Equation (PAO2)

= PIO2 – (PaCO2 / R)

inspired O2
arterial O2
Resp Quotient = CO2 produced/O2 consumed

@sea level
= 150mmHg – (PaCO2/0.8)

Edwards syndrome (trisomy 18)

have small jaws, small eyes, and malfored low set ears. They also have rocker bottom feet
. But the key buzzword is clenched hands with overlapping fingers.

Patau syndrome (trisomy 13)

Infant with cleft lip/palate, microcephaly or holoprosencephaly, polydactyly, cutis aplasia

Thromboangiitis obliterans (Buerger disease)

Vasculitis with segmental thrombosing inflammation with sparing of the internal elastic lamina

Meyer loop

Optic fibers of the geniculocalcarine tract radiating through the anterior and lateral temporal lobe before reaching the occipital cortex. The involvement of these fibers results in upper quadrantanopsia.

Superior nasal meatus

Drains the sphenoid and posterior ethmoid sinus

Middle nasal meatus

Drains the frontal, maxillary, and anterior ethmoid

Inferior nasal meatus

drains nasolacrimal duct

Moraxella catarrhalis

fastidious, nonmotile, Gram-negative, aerobic, oxidase-positive diplococcus that can cause infections of the respiratory system, middle ear, eye, central nervous system, and joints of humans.

Image: Moraxella catarrhalis

Hamman’s sign

Crepitus/Crunching sound on auscultation=> subcutaneous emphysema, pneumomediastinum

a1-hydroxylase activity of macrophages

Is responsible for the hypercalcemia of sarcoidosis

Caplan syndrome

pneumoconiosis with intrapulmonary nodules + rheumatoid arthritis

asbestos
coal
silica

Mesothelioma

Calretinin + staining
Psammoma bodies

Neimann-Pick disease

hepatosplenomegalycherry-red macular spots, and psychomotor retardation, liver biopsy findings show foam cells due to accumulation of sphingomyelin primarily in lysosomes of the reticuloendothelial system and CNS

normal development until approximately one year

Tay-Sachs disease

AR disorder of hexosaminidase A
Accumulate GM2 ganglioside, onion skin appearance of lysosomes with whorled membranes
Cherry red spot, developmental delay and death in early childhood
NO HEPATOMEGALY, no recurrent pneumonia

Fabry disease

Accumulation of ceramide trihexoside 

The only X-linked recessive sphingolipidosis (more likely in males) and typically includes involvement of the eyes (e.g., cataracts, corneal clouding). 

No cherry-red macular spots, hepatosplenomegaly, or recurrent pneumonia

Furthermore, clinical features usually only start to develop during late childhood or early adolescence.

Gaucher cells

lipid laden macrophages resembling crumpled tissue paper, accumulation of glucocerebroside

Image: Gaucher cells

Krabbe disease (globoid cell leukodystrophy)

rapidly progressive, fatal, autosomal recessive neurologic disorder caused by deficiency of beta-galactocerebrosidase; appears in infancy and is characterized by “globoid cells” in the white matter (macrophages containing undigested galactosylceramide)

Pi Z variant

glutamic acid to lysine substitution at position 342 of a1-AT gene on chrom 14

Riboflavin (B2) deficiency

assessed by measuring the activity coefficient of erythrocyte glutathione reductase

increase in activity upon addition of deficient vitamin indicates a large amount of unsaturated enzyme due to deficiency

Sx: seborrheic dermatitis (erythematous rash with fine yellow scales), pharyngeal hyperemia, glossitis, cheilitis, angular stomatitis, and normocytic-normochromic anemia

Friedriech ataxia

presents with progressive bilateral limb ataxia and weakness, loss of deep tendon reflexes, pallhypesthesia, dysarthria, and skeletal deformities, Pes cavus (inverted feet), hammertoes, and kyphoscoliosis are typical skeletal deformities seen

most common cause of death is heart failure due to hypertrophic cardiomyopathy or arrhythmia

Icatibant

antagonizing the bradykinin-B2-receptor
Tx of hereditary angioedema

Ecallantide

kallekrein inhibitor; decreases conversion of kininogen to bradykinin
Tx of hereditary angioedema

Winter formula

expected respiratory compensation in patients with metabolic acidosis.

Expected pCO₂ (in mm Hg) = (1.5 x HCO3-) + 8 (+/- 2).

CD14

surface marker of monocyte-macrophage cell lineage

eRPF

= U(PAH)xV/P(PAH) = C(PAH)

slight underestimation

Renal blood flow

= RPF / (1 – Hct)

Filtration fraction

= GFR/RPF

normal ~20%

RTA type 1

Inability of a-intercalated cells to secrete H+, no new bicarb formed leading to acidosis

Urine pH>5.5

Hypokalemia

Causes: Ampho B, analgesic nephropathy, congenital obstruction of urinary tract, AI disease

RTA type 2

Defect in PCT bicarb reabsorption leading to acidosis

Urine pH <5.5 due to acidification in a-intercalated cells in collecting duct, but not enough to overcome increased bicarb excretion

Hypokalemia

Causes: Fanconi syndrome, multiple myeloma, CA inhibitors

RTA type 4

Hypoaldosteronism or aldosterone resistance, leads to hyperkalemia and decreased NH4+ excretion

Urine pH <5.5 but variable

Causes: diabetic hyporeninism, ACEIs, ARBs, NSAIDs, heparin, cyclosporine; K+ sparing diuretics, nephropathy due to obstruction, TMP-SMX

Hyaline casts

Nonspecific, can be normal. Form via solidification of Tamm-Horsfall mucoprotein secreted by real tubular cells

Image: Hyaline casts

Calcium oxalate crystals

Are seen with ethylene glycol toxicity, vit C toxicity, Crohn disease

Image: Calcium oxalate crystals

calcium phosphate crystals

Found in alkaline urine
Prisms, tapered at one end

Image: calcium phosphate crystals

Struvite crystals

coffin lids. nephrolithasis. urease-producing UTIs.

Image: Struvite crystals

sodium cyanide nitroprusside test

cystine kidney stones diagnosis

Aldesleukin

recombinant IL-2 (stimulates NK cells and cytotoxic T cells=>recognize the absence of MHC I receptors on infected or malignant cells and destroy them)
Tx: RCC & metastatic melanoma

Hyperosmolar Hyperglycemic State (HHS)

ACUTE COMPLICATION SEEN IN PERSONS WITH TYPE 2 DIABETES, THAT IS CHARACTERIZED BY EXTREME HYPERGLYCEMIA, HYPEROSMOLARITY WITH DEHYDRATION, THE ABSENCE OF KETOACIDOSIS, AND CNS DYSFUNCTION

Mixed cryoglobulinemia

strongly associated with hepatitis C infection. HCV antigen-immune complex formation triggers complement activation and inflammation of blood vessels, resulting in vasculitis, which can manifest with the palpable purpura and membranoproliferative glomerulonephritis

membranous nephropathy

Primary: Ab against phospholipase A2
Secondary: Hep B, Hep C, SLE, malignancy, medications
Majority are idiopathic

Thickened capillary loops and BM, spike and dome appearance on EM w granular subepithelial deposits on IF

Most common cause of nephrotic syndrome in white adults

Familial hypocalciuric hypercalcemia (mutated CaSR= calcium-sensing receptor)

Symptoms: Asymptomatic. Green colored urine.

Lab Values: High PTH levels, hypercalcemia, hypermagnesemia, hypophosphatemia, urine calcium <200mg/day.

Pudendal nerve

Supplies sensation to external genitalia and perineum (external hemorrhoids, below dentate line) and innervation to muscles of the perineum

originates from the S2-S4 spinal cord roots, innervates the external urethral sphincter and plays a role in maintaining continence

uremia (secondary to end stage renal disease)

Altered mental status, elevated BUN, low pH, low bicarbonate level, high anion gap, and reduced pCO2 are consistent with an anion gap metabolic acidosis caused by

Genitofemoral nerve (L1-L2)

genital branch:
through DEEP inguinal ring
Cremasteric muscle and skin in Anterior part of scrotum

Female: w/ round ligament to mons pubis
labium major

femoral branch:
lateral to ext iliac artery to posterior to inguinal ligament, pierces the femoral sheath, innervate skin of sup ant thigh

Ilioinguinal nerve (L1)

enters INGUINAL CANAL –
(sup inguinal ring)

to genital area
upper medial thigh

root of the penis
anterior surface of scotum

***
smaller,inferior than ileohypogastric

usually crosses part of iliacus muscle toward iliac crest

mons pubis and labium major (women)

muscular branches

pelvic splanchnic nerves

originate from the sacral nerve roots S2–S4 and provide parasympathetic innervation to the rectum, erectile tissue, and detrusor muscle of the bladder

Sideroblastic anemia

Seen in:
X-linked defect in delta ALA synthase gene
Myelodysplastic syndromes
Alcohol abuse
Lead poisoning
Vit B6 def (ALA synthase cofactor)
Cu deficiency
Isoniazid or Linezolid therapy

Orotic aciduria

a rare autosomal recessive disorder of de novo pyrimidine synthesis that occurs due to a defect in 5’monophosphate (UMP) synthase. Children typically present with physical and mental retardation, megaloblastic anemia, and large amounts of urinary orotic acid; no hyperammoniemia. 

Not improved by folate or B12 supplementation.

Uridine supplementation (a pyrimidine) can improve symptoms as uridine is converted to UMP via nucleoside kinases

Ornithine transcarbamylase deficiency (OTC)

Only X-linked Urea cycle disorder
Vomiting, tachypnea, and confusion (hyperammonemia) with elevated urinary orotic acid and low BUN
Tx: avoid proteins

Diamond-Blackfan anemia

Congenital pure red cell aplasia in the first 3 months of life
WBC and platelet counts are normal
Short stature, craniofacial abnormalities, and upper extremity malformations (triphalangeal thumbs)

Image: Diamond-Blackfan anemia

Hereditary spherocytosis

Eosin-5-maleimide binding test with decreased mean fluorescence of RBCs

Eculizumab

MOA: targets complement protein C5

Clinical use: paroxysmal nocturnal hemoglinuria

Acute Intermittent Porphyria (AIP)

AD deficiency Porphobilinogen deaminase (aka uroporphyrinogen I synthase) 

↑ porphobilinogen

Painful abdomen

Port wine urine

Polyneuropathy

Psychological disturbance

Precipitated by drugs, alcohol, starvation

Tx: hemin and glucose

Porphyria cutanea tarda (PCT)

AD Uroporphyrinogen decarboxylase deficiency

↑ Urophorphyrin (tea colored urine)

Abdominal pain, neuro symptoms, and photosensitive hyper pigmented blistering

Factor V Leiden mutation

The most common inherited cause of hypercoagulability in Caucasians

Guanin to Adenine DNA point mutation
Leads to Arg506Gln mutation near cleavage site
Resistant to degradation by protein C

cryoprecipitate

fibrinogen, factor 8, factor 13, vWF, and fibronectin

Pseudo-Pelger-Huet anomaly

Neutrophils with bilobed nuclei typically seen after chemotherapy

TdT

marker of pre-T and pre-B cells

t(12;21)

translocation in ALL, better prognosis

Ruxolitinib

JAK2 inhibitor

S-100 stain

Stain for Melanoma, Schwannoma, and Langerhans cell histiocytosis

Hemophagocytic Lymphohistiocytosis

Caused by mutation in a number of different genes involved in the perforin-granzyme killing mechanism used by cytotoxic T lymphocytes and NK cells

Also called macrophage activation syndrome because the CTLs and NKs secrete excessive IFN-gamma, over activating macrophages

Presents as an increase in viral infections
Hemophagocytosis (phagocytosis of RBCs in marrow)
Lymphadenopathy
Hepatosplenomegaly
Very high serum ferritin

Idarucizumab

antidote for dabigatran

Bortezomib, Carfilzomib

MECHANISM Proteasome inhibitors, induce arrest at G2-M phase and apoptosis.

CLINICAL USE Multiple myeloma, mantle cell lymphoma.

ADVERSE EFFECTS Peripheral neuropathy, herpes zoster reactivation.

Trametinib

MEK inhibitor

often given with BRAF inhibitor for treatment of melanoma

External iliac lymph nodes

Drain lymphatics from cervix, superior bladder, and body of uterus

Internal iliac lymph nodes

Drain lymphatics from lower rectum to anal canal (above the pectinate line), bladder, middle third of the vagina, cervix and prostate

Superficial inguinal lymph nodes

Drains lymphatics from the anal canal (below pectinate line), Skin below umbilicus (except popliteal area), scrotum, vulva

Good Syndrome

hypogammaglobulinemia associated with thymoma

IPEX

FOXP3 mutation
Characterized by autoimmunity
Enteropathy
Endocrinopathy
Nail dystrophy
Dermatitis
Assoc with diabetes in male infants

TNFa

Secreted by macrophages

Mediate septic shock, activate endothelium, WBC recruitment, vascular leak; causes cachexia in malignancy; lead to granuloma formation

IL-3

-Secreted by all T cells
-Supports growth and differentiation of bone marrow stem cells. Functions like GM-CSF.

Pemphigus vulgaris

Flaccid intraepidermal bullae

Oral mucosa involved

Nikolsky’s sign (separation of epidermis caused by rubbing of the skin)

Ab’s against anti-desmoglien, anti-desmosomes
“Tombstone” appearance on H&E
Reticular/fishnet pattern on IF

Primary membranous nephropathy

Anti-phospholipase A2 receptor antibodies

p-ANCA

antibody seen in ulcerative colitis

Autoimmune hepatitis type 1

Anti-smooth muscle antibodies

Polymyositis, dermatomyositis

Anti-Jo-1, anti-SRP, anti-Mi-2 antibodies

Diffuse scleroderma

Anti-Scl-70 (anti-DNA topoisomerase I)

Rheumatoid arthritis

anti-CCP antibodies

Drug induced lupus

Anti-histone antibodies

Antiphospholipid syndrome

Anti-B2 glycoprotein antibodies
Anticardiolipin antibodies
Lupus anticoagulant antibodies

Job syndrome (hyper-IgE syndrome)

Deficiency of Th17 cells due to STAT3 mutation= impaired neutrophil chemotaxis to sites of infection

Lack IL-12: no Th1 activation, excess Th2 activation= lots of IgE and Eosinophils

Cold abscesses

Retained baby teeth

Coarse facies

Dermatologic problems/eczema

Inc’d IgE

Fractures from minor trauma

AIRE mutation

A transcription factor expressed in the medulla of the thymus and controls the mechanism by which negative selection removes autoreactive T-cells.

Expression of non-local proteins reduces the threat of of autoimmunity later

-Chronic mucocutaneous candidiasis (antibodies to IL-17 and IL-22),
-IPEX (Autoimmune hypoPTH, autoimmune Addison’s)

SCID

IL-2R y-chain mutation; X-linked recessive, most common

Adenosine deaminase (ADA) deficiency; AR

absent T cells and B cells, no thymus, no germinal centers in lymph nodes

Ataxia telangiectasia (ATM mutation)

Fail to detect DNA damage, accumulate mutations
AR
Ataxia (cerebellar atrophy), Angiomas, IgA deficiency
Inc’d AFP
Inc’d risk of lymphoma/leukemia

Wiskott-Aldrich syndrome

X linked recessive mutation: unable to reorganize actin cytoskeleton= defective Ag presentation

Thrombocytopenia

Eczema

Recurrent pyogenic infections

Inc risk of AI disease and malignancy

Low to normal IgG, IgM

Inc IgE, IgA

LAD type 1 (LFA-1 integrin/CD18 mutation)

Impaired migration and chemotaxis of phagocytes
AR
Recurrent skin and mucosal bacterial infection
Absent pus
Delayed umbilical cord separation
Neutrophilia

Chediak-Higashi syndrome (LYST mutation)

Microtubule dysfunction in phagosome/lysosme fusion
AR

Progressive neuropathy
Lymphohistiocytosis
Albinism
Recurrent infection
Giant granules in granulocytes and platelets
Mild coagulation defects
Pancytopenia

Uremia

causes intrinsic dysfunction of glycoprotein IIb/IIIa as well as an increase in NO levels, which leads to an increase in cyclic GMP and reductions in thromboxane A2 and ADP levels leading to impaired platelet adhesion and aggregation

Inc’d bleeding time with normal PLT count
Normal PT and PTT
Often seen in Px with CKD

Crohn’s disease

anti-Saccharomyces cerevisae antibodies

Parvovirus B19

cause aplastic anemia by binding to blood group P antigens

The absence of P antigens or administration of a monoclonal antibody to globosides, an essential component of the P antigen that functions as a receptor, confers immunity

Monochorionic, diamniotic

Approximately 75% of monozygotic twin pregnancies
Can cause Twin-Twin transfusion syndrome

Donor twin has anemia and oligohydramnios
Recipient twin has polycythemia and polyhydramnios

Cyclosporine

immunosuppressant drug that is commonly used for transplant rejection prophylaxis.
MOA: binds cyclophilin to form a complex that inhibits calcineurin. Inhibition of calcineurin blocks activation of the NFAT transcription factors, which prevents interleukin-2 transcription and, as a result, decreases T cell activation.

Odd ADR: gingival hyperplasia

IL-1 and TNFa

promote the upregulation of cellular adhesion molecules on the endothelium

-interact with integrins on leukocytes, leading to the adhesion of leukocytes to the endothelium in area of injury/damage

hyposthenuria

the secretion of urine of low specific gravity due to inability of the kidney to concentrate the urine normally

azygos vein

receives blood from the right intercostal veins, which in turn receive blood from the right breast (route for metastasis to spine)

has numerous anastomoses with the vertebral venous plexus, which is a longitudinal network of valveless veins in the spine that includes the paravertebral plexus of Batson

CD4<100

HIV associated esophageal candidiasis, pulmonary aspergillosis, cryptococcal meningitis, cerebral toxoplasmosis

CD4<200

HIV associated PCP, PML

CD4<400

HIV assoc TB

CD4 < 50

HIV assoc CMV infection

Artesunate

antimalarial drug used for the treatment of severe falciparum malaria

Image: Artesunate

Ribavirin

inhibits synthesis of guanine nucleotides by competitively inhibiting IMP dehydrogenase

ADRs: teratogenic, GI upset, hemolytic anemia

leukoreduction

process of removing leukocytes from blood products; prevents transmission of CMV

SCID screening

Newborn screening by PCR amplification of T-cell receptor excision circles (TRECs)

Tesamorelin

GHRH analog used to treat HIV-associated lipodystrophy

Neuroblastoma

Most common tumor of adrenal glad in kids

Can cross midline

Usually no hypertension

Present with opsoclonus-myoclonus

Elevated HVA and VMA

Homer-Wright rosettes on histology

Originate from neural crest cells

Bombesin+, Neuron specific enolase+

Amplification of N-myc

Amine precursor uptake decarboxylase (APUD) containing cells

Carcinoid tumors

Stain w chromogranin A and syanptophysin
Inc 5-HIAA
See sx of niacin deficiency
Diarrhea, flushing, asthma, right heart valvular disease

Telotristat

inhibits tryptophan hydroxylase step of 5HT synthesis
For sx control of carcinoid syndrome

Gastrinoma

Positive secretin stimulation test: gastrin levels remained elevated after administration of secretin (normally inhibits gastrin release)

Jod-Basedow phenomenon

thyrotoxicosis upon iodine replenishment in a patient with an iodine deficient goiter

external branch of superior laryngeal nerve

cricothyroid innervation

Pseudohypoparathryoidism Type 1A

AD- defect inherited from mother; inactive Gs protein a-subunit of PTH receptor causing end organ resistance to PTH

Low serum Ca despite elevated PTH

Physical findings= Albright Hereditary Ostreodystrophy
-short 4th/5th digits
-short stature
-obesity
-mental retardation

Pseudopseudohypoparathyroidism

AD defect inherited from father, with normal maternal allele; maintains responsiveness to PTH receptor Gs a-subunit

Normal PTH and serum Ca

Physical findings of Albright Hereditary Ostreodystrophy
-short 4th/5th digits
-short stature
-obesity
-mental retardation

Laron syndrome

AR. Defect in GH receptor; inc GH but dec IGF-1

Dwarfism, small head, saddle nose, delayed skeletal maturation, small genitalia, but a prominent forehead

Sonic hedgehog gene

Produced at base of limbs in zone of polarizing activity. Involved in patterning along anterior- posterior axis. Involved in CNS development; mutation can cause holoprosencephaly.

Wnt-7 gene

Produced at apical ectodermal ridge (thickened ectoderm at distal end of each developing limb). Necessary for proper organization along dorsal-ventral axis.

Fibroblast Growth Factor (FGF) gene

Produced at apical ectodermal ridge. Stimulates mitosis of underling mesoderm, providing for lengthening of limbs

Homeobox (Hox) genes

involved in segmental organization of embryo in a craniocaudal direction. Code for transcription factors. Hox mutations -> appendages in wrong locations.

Pierre Robin Sequence

micrognathia, glossoptosis (tongue placed further bakck), cleft palate, airway obstruction

poor development of the first pharyngeal arch

Treacher Collins Syndrome

AD neural crest dysfunction of 1st/2nd pharyngeal arch
-craniofacial abnormalities (zygomatic bone and mandibular hypoplasia)
-hearing loss
-airway compromise

Image: Treacher Collins Syndrome

Mayer-Rokitansky-Kuster-Hauser syndrome

Mullerian agenisis=> absence or hypoplasia of all or part of the cervix, uterus, and fallopian tubes

Present as primary amenorrhea in females with fully developed secondary sexual characteristics because they still have functional ovaries

hypogastric nerve

Sympathetic T11-L2
Contraction of internal urethral sphincter & Relaxation of detrusor=> urinary retention

Ejaculation

human placental lactogen (hPL)

aka human chorionic somatomammotropin
From syncytiotrophoblast of placenta
Stimulates insulin production; overall inc insulin resistant; can cause gestational diabetes

McCune-Albright syndrome

characterized by triad of fibrous dysplasia of the bone, endocrine abnormalities (such as early puberty or hyperthyroidism) and cafe-au lait spots.

Condition results from an activating mutation in the G protein/CAMP/adenylate cyclase signaling pathway.

Coast of Maine border with the cafe-au-lait (irregular border)

Image: McCune-Albright syndrome

Epididymitis

+ Prehn sign: pain relief with scrotal elevation

Aldose Reductase

catalyzes the reduction of glucose to sorbitol, which is subsequently oxidized to fructose by sorbitol dehydrogenase enzyme.

Diabetes/chronic hyperglycemia, sorbitol formation increases significantly, causing osmotic damage in tissues with low sorbitol dehydrogenase activity (e.g., lens, retina, kidney, and myelin sheath)

Galactose-1-phosphate uridyltransferase

Responsible for converting Galactose-1-phosphate to Glucose-1-P

Absence in this enzyme leads to *Classic Glactosemia*

Much more severe than galactokinase deficiency

– Failure to thrive, Jaundice, Hepatomegaly, *Infantile Cataracts*, 

Treatment: exclude galactose and *Lactose* (galactose+glucose) from diet. 

Phosphate deficiency occurs similar to fructose intolerance (aldolase B deficiency)

Galactokinase deficiency

galactitol accumulates in blood & urine

infantile cataracts (no developmental delay)
No other organ involvement

B-glucuronidase

produced by enteric bacteria and catalyzes the production of unconjugated bilirubin (by deconjugating direct bilirubin), which is then reabsorbed and recycled, increasing the risk of formation of brown pigment gallstones (made of Ca salts of unconj bilirubin; usually seen in conditions where there is infected bile)

fructokinase

found in the liver
phosphorylates fructose into fructose 1 phosphate

deficiency leads to detectable fructose in the blood and urine, has no clinical consequences, as hexokinase shunts excess fructose back into the glycolytic pathway by converting fructose directly to fructose-6-phosphate.

IgG4-related disease

– Constellation of disorders characterized by tissue infiltrates dominated by IgG4-producing plasma cells & lymphocytes (particularly T cells)

– Storiform fibrosis, obliterative phlebitis, increased serum IgG4

– Can occur in virtually any organ system

– Mikulicz syndrome (enlargement & fibrosis of the salivary & lacrimal glands), Riedel thyroiditis, idiopathic retroperitoneal fiborsis, autoimmune pancreatitis, & inflammatory pseudotumors of the orbit, lungs, & kidneys are part of this spectrum of diseases

Myotonic muscular dystrophy type 1

AD CTG repeat expansion in the DMPK gene

Sk muscle weakness and myotonia

Arrhythmias

Cataracts

Gonadal atrophy

Frontal balding (need Toupee)

Hypogammaglobulinemia

Insulin resistance

Cognitive impairment

Third degree perineal laceration

Involves the skin of the fourchette, the perineum, the perineal body and the anal sphincter, may result in fecal incontinence

Danazol

Partial agonist at androgen and progesterone receptors–>decreased LH and FSH secretion–>decreased endometrium and breast growth

Use = tx endometriosis, hereditary angioedema

SE = androgenic SE in women (hirsutism, acne, etc)

Ursodiol

reduces cholesterol absorption and can dissolve cholesterol gallstones; also treat primary biliary cholangitis

Lysyl oxidase

catalyzes the oxidative deamination of nonhydroxylated lysine residues to form polylysine crosslink in elastase

Needs Cu and Vit C

Lynch Syndrome (HNPCC)

AD – DNA mismatch mut (MLH1, MSH2, MSH6, PMS2)
young, family hx
right/proximal colon cancer
tumors w/ mucin
– assoc w/ gastric and endometrial cancer, as well as ovarian cancer and urothelial cancer.

Acute Hepatitis

Liver biopsy: Ballooning degeneration (swelling of hepatocytes) and bridging necrosis (confluent necrosis spanning adjacent lobules). Other histopathological signs include Councilman bodies and periportal inflammation with helper T cells, B cells, and plasma cells

McMurray Test

compression of the meniscus of the knee combined with internal and external rotation while the patient is face-up to assess the integrity of the medial and lateral meniscus
(LIME)

Image: McMurray Test

Guyon canal syndrome

Compression of ulnar nerve at wrist. Classically seen in cyclists due to pressure from
handlebars.

DeQuervain’s Tenosynovitis

Noninflammatory thickening of abductor pollicis longs and exteriors pollicis braves tendons characterized by pain or tenderness at radial styloid.
+Finkelstein test (pain at radial styloid with stretch of thumb tendons)
Inc’d risk in new moms, golfers, racquet sports players.

Osgood-Schlatter disease

inflammation or irritation of the tibia at its point of attachment with the patellar tendon

seen in activity young males with progressive anterior knee pain

Slipped Capital Femoral Epiphysis

obese ~12yo child with hip/knee pain and altered gait
Red arrow in image, yellow arrow is normal

Image: Slipped Capital Femoral Epiphysis

Neonatal lupus (anti-SSA+ mom)

Newborn with congenital heart block, periorbital/diffuse rash, transaminitis, and cytopenias

Puetz-Jeghers syndrome

AD; hyperactive serine-threonine kinase due to a gain of function mutation on the STK11 gene on chrom 19. -melanotic macules on the lips, buccal mucosa, genitalia, palms, and soles

-gastrointestinal hamartomatous polyps

-complications of polyps include bleeding and intussusception

-increased risk of malignancies such as colorectal, pancreatic, ovarian, and breast cancer.

Type III collagen

(Reticulin) Skin, blood vessels, uterus, fetal tissue, granulation tissue

Parinaud syndrome

vertical gaze palsy, pupillary light near dissociation, lid retraction, convergence-retraction nystagmus

cause by stroke, hydrocephalus, pinealoma

Medial medullary syndrome (paramedian branches of ASA and/or vertebral aa.)

Contralateral paralysis upper and lower limbs
(Lateral corticospinal tract)

Contralateral loss of proprioception
(Medial lemniscus)

Ipsilateral hypoglossal dysfunction; ipsilateral tongue deviation
(Caudal medulla-hypoglossalq nerve)

Lateral medullar/Wallenberg syndrome (PICA)

Dysphagia, hoarseness, loss of gag reflex, hiccups

(Nucleus ambiguous- CN 9, 10, 11 motor)

Vomiting, vertigo, nystagmus

(Vestibular nuclei)

Contralateral loss of pain and temperature of body, ipsilateral face

(Lateral spinothalamic tract, spinal trigeminal nucleus)

Ipsilateral Horner syndrome

(sympathetic fibers)

Ipsilateral ataxia, dysmetria

(Inferior cerebellar peduncle)

Lateral pontine syndrome (AICA)

Facial paralysis, loss of lacrimation, loss of salivation, decreased taste in anterior 2/3 of tongue

(Facial nucleus)

Vomiting, vertigo, nystagmus

(Vestibular nuclei)

Dec pain and temp sensation from contralateral body, ipsilateral face

(Spinothalamic tract, spinal trigeminal nucleus)

Ipsilateral Horner syndrome

(sympathetic fibers)

Ipsilateral ataxia, dysmetria 

(Middle and inferior cerebellar peduncle)

Ipsilateral sensorineural deafness, vertigo

(Labyrinthine artery)

Pacinian corpuscles

-sense high-frequency vibrations, deep touch, and mechanical pressure.

-located in the deep layers of the skin (e.g., dermis and subcutaneous layer), ligaments, and joints

-very rapidly-adapting

-innervated by large myelinated fibers, which facilitate very rapid signal propagation=>detect rapid vibrations and facilitate the discrimination of fine surface textures

I-cell disease

Sx:Coarse facial features, clouded corneas, restricted joint movement, high plasma levels of lysosomal enzymes. Fatal in childhood.

Lab Values: Lysosomal enzymes secreted outside the cell; ghost lysosomes

Pathophysiology: Lysosomal storage disease; failure of mannose-6-phosphate addition which is required to traffic proteins to lyosomes

Zellweger syndrome

autosomal recessive disorder of peroxisome biogenesis due to mutated PEX genes
Hypotonia, seizures, hepatomegaly, early death.

Refsum disease

autosomal recessive disorder of a-oxidation=>phytanic acid not metabolized to prostatic acid

Scaly skin, ataxia, cataracts/night blindness, shot 4th toe, epiphyseal dysplasia

Tx: diet, plasmapheresis

Menkes disease

X-Linked Recessive

Defect: 

– Menkes Protein (ATP7A

– Leads to *Copper Deficiency*

– *Copper* is Cofactor for *Lysyl Oxidase* in Collagen *Cross-Linking*

– Causes *Collagen Deficiency*

Symptoms:

– *Kinky Hair*

– *Growth* Retardation

– *Mental* Retardation

– *Seizures*

– *Hypotonia*

OTC deficiency

X-linked
urea cycle disorder
vomitting, seizures, hypotonia
high ammonia (CHECK FOR HYPERAMMONIA IN MOM)
respiratory alkalosis
poor feeding
can be activated by extreme stress in adult onset
cerebral edema, papilledema, lethargy, coma

Hunter syndrome

X-linked recessive deficient L iduronosulfate sulfatase, accumulations of heparin sulfate and dermatan sulfate, hepatospenomegaly, micrognathia, retinal degeneration, joint stiffness, mental retardation, cardiac lesions p59

Hurler syndrome

mucopolysaccharidosis, deficient a L iduronidase, accumulations of heparin sulfate and dermatan sulfate in heart, brain, liver, and other organs, progressive, hepatosplenomagaly, dwarfism, gargoyle-like facies, stubby fingers, corneal clouding, mental retardation, death by 10 years of age p57

Rett Syndrome

X-linked de novo mutation of MECP2

Appears in girls 1-4 yo

progressive neurological developmental disorder featuring constant hand-wringing, intellectual disability, and impaired motor skills, ataxia, seizures

orotic aciduria

Orotic acid in urine, megaloblastic anemia that is not corrected with B12 or folic acid and no hyperammonemia

abetalipoproteinemia

AR mutation in gene that encodes microsomal transfer protein (MTP).

Chylomicrons, VLDL, LDL are absent; deficiency in ApoB48 and ApoB100

Infants present with failure to thrive, fat malabsorption, steatorrhea.

Later manifestations include retinitis pigments, spinocerebellar degeneration due to vit E deficiency, progressive ataxia, acanthocytosis.

Intestinal biopsy shows lipid laden enterocytes

Hyperchylomicronemia (Type I familial dyslipidemia)

Deficient Lipoprotein lipase or apoC-II

Very high chylomicrons, TG, cholesterol

Pancreatitis, hepatosplenomegaly, xanthomas, creamy later in supernatant

Familial hypercholesterolemia (type 2 familial dyslipidemia)

AD absence or defective LDL receptors or ApoB-100

High LDL and cholesterol (300-700)

At risk for accelerated atherosclerosis and CVA
xanthomas
corneal arcus

Dysbetalipoproteinemia (type III familial dyslipidemia)

Defective ApoE, autosomal recessive

High chylomicrons, VLDL

Premature atherosclerosis
Xanthomas

Hypertriglyceridemia (type IV familial dyslipidemia)

AD; overproduction of hepatic VLDL

High VLDL and TGs (>1000 mg/dL)

Acute pancreatitis


62 year old woman – osteoporosis – a bisphosphonate is prescribed. The expected beneficial effect of the drug is due to which of the following?

Decreased Osteoclast Activity

Cohot Study of elderly women – relative risk ratio for hip fractures among those who exercise regularly is 1.2 (95% confidence interval of 1.1 to 1.8). Which of the following is the conclusion about the effect of exercise on the risk of hip fracture?

Statistically Significant Overall Increase Risk

Image: Cohot Study of elderly women - relative risk ratio for hip fractures among those who exercise regularly is 1.2 (95% confidence interval of 1.1 to 1.8). Which of the following is the conclusion about the effect of exercise on the risk of hip fracture?

52 year old man goes to ER with chest pain radiating to his jaw while shoveling snow. Pulse is 80/min and blood pressure is 130/70. The most immediate treatment mechanism of action?

Increased nitric oxide concentration

24 year old woman – spilled hot grease on her left leg while working at a fast-food restaurant. Exam of leg shows 7cm pink, soft, granular, edematous wound. The formation of this tissue was most likely caused by increased activity of which?

a. Vascular Endothelial Growth Factor

VEGF – stimulates angiogenesiss.

TYPE III Collagen = Blood Vessels – early wound repair

Image: 24 year old woman - spilled hot grease on her left leg while working at a fast-food restaurant. Exam of leg shows 7cm pink, soft, granular, edematous wound. The formation of this tissue was most likely caused by increased activity of which?

27 year old man- MVC – skull x-ray shows a linear, nondepressed basal skull fracture – increased serum osm and decreased urin osm. Following desmopressin urine osm increases. Desmopressin’s effect is due to the activation of which of the following?

a. Adenylyl Cyclase

Adenylate Cyclase – ATP – CAMP — PROTEKINASE A — Ca

v2

Image: 27 year old man- MVC - skull x-ray shows a linear, nondepressed basal skull fracture - increased serum osm and decreased urin osm. Following desmopressin urine osm increases. Desmopressin's effect is due to the activation of which of the following?

A 10 month old boy – 4 day history of fever and cough. He attends day care center. Chest exam shows intercostal retractions along with bilateral, diffuse wheezes and expiratory rhonchi. The infectious agent most likely has which of the following properties?

Mediation of Cell Entry via a fusion protein

A 17 year old girl in ED – 15 minutes after being stung by a bee. Mild light headedness but no difficulty swallowing- Bilateral wheezing – Which is most appropriate pharmacotherapy for this patient?

B2- Agonist

14 year old boy – 2 day history of sore throat and fever that peaks in late afternoon. 1 week of fatigue. He recentaly had sex with one partner. Physical exam show cervical lymphadenopathy and pharyngeal erythema with a creamy exudate. DX?

Infectious Mononucleosis

57 year old man – radiation therapy for squamous cell carcinoma of the lung. Despite therapy, tumor increases in size and he dies 6 months later. The progressive tumor growth is due to a defect in cell cycle arrest in which of the following phases of cell cycle?

G1

Image: 57 year old man - radiation therapy for squamous cell carcinoma of the lung. Despite therapy, tumor increases in size and he dies 6 months later. The progressive tumor growth is due to a defect in cell cycle arrest in which of the following phases of cell cycle?

28 year old – lived in sub-Saharan Africa – until he came to the US. Temp of 100.4 – imaging shows bilateral hydroureter and hydronephrosis. Biopsy shows marked fibrosis and scattered granulomas. DX?

Schistosomiasis

A couple with a family history of a-thalassemia. Woman has one gene deletion and man has two gene deletion. If the two gene is trans – what percentage of offspring will have a two gene deletion?

50%

Image: A couple with a family history of a-thalassemia. Woman has one gene deletion and man has two gene deletion. If the two gene is trans - what percentage of offspring will have a two gene deletion?

previously healthy 40 year brought to emergency department by her husband – 2 day history of fever, lethargy, confusion. PE shows scattered petechiae and ecchymoses over the lower extremities – 3+ polychromasia and 3+ schistocytes and Low platelets Dx?

Thrombotic Thrombocytopenia Purpura

16 year old boy – is admitted to the ER because of a knife wound to the left side of his chest. An X-ray of the chest shows an air-fluid level in the left side of the chest, partial collapse of the left lung, and the elevation of the stomach bubble. The mediastinum is midline. DX?

Hemopneumpothorax under tension

49 year old woman – coronary artery disease – BP 140/90 – High Cholesterol, High LDL (190), High triglycerides (350) – TX with atorvastatin and losartan. What are the effects on HDL and Triglycerides?

HDL increased

Triglycerides Decreased

Image: 49 year old woman - coronary artery disease - BP 140/90 - High Cholesterol, High LDL (190), High triglycerides (350) - TX with atorvastatin and losartan. What are the effects on HDL and Triglycerides?

73 yeare old – diffuse weakness and tingling of her arms and legs. Sensation and vibration and position is decreased in all extremities. What vitamin deficiency?

Vitamin B12 – (cyanocobalamin)

Tea and Toast – low B12 in diet

Image: 73 yeare old - diffuse weakness and tingling of her arms and legs. Sensation and vibration and position is decreased in all extremities. What vitamin deficiency?

15 year old girl – 3 month history of acne – which is the underlying cause of the patients acne?

Stimulation of Sebaceous Glands by androgens

b. ACNE = Propionibacterium ACNE 

4 year old from Brazil – PE shows single 12x10cm lesion in the right side of jaw with diffuse regular edges. Photomicrographs of an incisional biopsy (looks like Burkitts Lymphoma/ Starry night) – which of the processes most likely to occur in the region indicated by the arrow?

Apoptosis

b. Endemic Burkitt lymphoma can happen in Brazil as well as Africa (jaw lesion, puffy face).

Image: 4 year old from Brazil - PE shows single 12x10cm lesion in the right side of jaw with diffuse regular edges. Photomicrographs of an incisional biopsy (looks like Burkitts Lymphoma/ Starry night) - which of the processes most likely to occur in the region indicated by the arrow?

51 year old – lump on tongue – 1 pack smoking history for 30 years. 1.5 cm mass on apex of tongue. It is most appropriate to evaluate which lymph nodes first for evidence of metastasis?

Submental

Image: 51 year old - lump on tongue - 1 pack smoking history for 30 years. 1.5 cm mass on apex of tongue. It is most appropriate to evaluate which lymph nodes first for evidence of metastasis?

15 year old boy – ER – 2 hour history of confusion and agitation – fever, headache, stiff neck, and vomiting – since returned from summer camp – patient is hallucinating – lumbar puncture – shows cysts and trophozoites- most likely pathogen?

a. Olfactory Nerve

Naegleria fowleri

Image: 15 year old boy - ER - 2 hour history of confusion and agitation - fever, headache, stiff neck, and vomiting - since returned from summer camp - patient is hallucinating - lumbar puncture - shows cysts and trophozoites- most likely pathogen?

17 year old – ED – 30 minutes after being found with a blank stare. Physical exam shows rigidity. During exam he becomes hostile and assaults physician – Pt ingested which drug?

PCP

Image: 17 year old - ED - 30 minutes after being found with a blank stare. Physical exam shows rigidity. During exam he becomes hostile and assaults physician - Pt ingested which drug?

Placebo controlled clinical trial – 5000 pts with essential hypertension. 2500 patients receive new drug and 2500 patients receive placebo. If alpha is set at 0.01 instead of 0.05, which of the following is most likely result?

Significant findings can be reported with greater confidence

17 year old – gymnast – comes to hospital because of lack of menstrual period for 6 months. BMI 15 Which is the cause of the amenorrhea?

Hypogonadotropic Hypogonadism

male stillborn is delivered at 32 weeks – Oligohydramnios – absence of a urethral opening. Which is most likely finding?

Pulmonary Hypoplasia

Image: A male stillborn is delivered at 32 weeks - Oligohydramnios - absence of a urethral opening. Which is most likely finding?

A 6 day old – breast fed boy in ED – poor weight gain and irritability since delivery – Physical exam shows jaundice and hepatomegaly. The concentration of which of the following metabolites is most likely increased?

a. Galactose – 1 – phosphate

b. Congenital intolerance to breast milk

Image: A 6 day old - breast fed boy in ED - poor weight gain and irritability since delivery - Physical exam shows jaundice and hepatomegaly. The concentration of which of the following metabolites is most likely increased?

A 25-year-old man – comes to ED – severe muscle pain, diffuse, painful swelling of his neck, underarms, and groin after camping in New Mexico Generalized scattered black maculae. Examination of the right upper extremity shows erythematous, solid, tender mass. Mass is draining blood and necrotic material. The most effective antibiotic for patient disorder will interfere with which of the following processes?

a. Ribosomal Assembly

b. Yersenia Pestis

Image: A 25-year-old man - comes to ED - severe muscle pain, diffuse, painful swelling of his neck, underarms, and groin after camping in New Mexico Generalized scattered black maculae. Examination of the right upper extremity shows erythematous, solid, tender mass. Mass is draining blood and necrotic material. The most effective antibiotic for patient disorder will interfere with which of the following processes?

45 year old – progressive weakness – muscle fasciculations of the upper extremities and weakness of the lower extremity – What additional findings?

Atrophy

b. ALS = Lou Gherig

A new severe respiratory illness – Why use a killed vaccine vs a live vaccine?

Avoids Concerns of reversion to virulence

b. Killed vaccines – avoids reversion to virulence

c. live vaccines – can (but rarely do) cause the disease they’re designed to prevent

A 33 year old – keratinizing squamous cell carcinoma of cervix. Which of the following describes pathogenesis of this patient’s disease?

Inactivation of Cellular P53

b. p53 protein = tumor suppressor (it activates apoptosis) – most human cancer

A 54 year old – 40 year history of T1DM – receiving hemodialysis for end stage renal disease while awaiting a kidney transplant. Receives a drug that induces reticulocyte release from bone marrow and stimulates a cytokine receptor that signals Jak/Stat pathway?

Erythropoietin

Jak Stat – erythropoietin

Image: A 54 year old - 40 year history of T1DM - receiving hemodialysis for end stage renal disease while awaiting a kidney transplant. Receives a drug that induces reticulocyte release from bone marrow and stimulates a cytokine receptor that signals Jak/Stat pathway?

During a clinical study examining the effects of exercise. The average pulse is 175/min. Compared with measurement before the session, which is most likely decreased?

Total Peripheral Resistance

An 8-year-old boy – 3-day history of fever, sore throat, and itchy eyes. Returned from week long summer camp that includes hiking trips and swimming – PE shows conjunctival injection and oropharyngeal edema – Outbreak among other campers. Which is most likely cause of this patient’s symptoms?

Adenovirus

Fever+ Sore Throat + Itchy Eyes

transmitted via swimming pools

Conjunctivitis Viral = adenovirus

Image: An 8-year-old boy - 3-day history of fever, sore throat, and itchy eyes. Returned from week long summer camp that includes hiking trips and swimming - PE shows conjunctival injection and oropharyngeal edema - Outbreak among other campers. Which is most likely cause of this patient's symptoms?

44-year-old woman – 10 month history of wide red streaks over her lower trunk (striae) and weight gain in face (moonface). Which additional findings?

Hypertension and muscle weakness

b. Cushing – hypertension and muscle weakness

12 year old boy – pain below left knee –unable to play soccer – An x-ray shown – Which structures attached to the abnormal anterior tibial area?

patellar ligament

b. Osgood-Schlatter

Image: 12 year old boy - pain below left knee -unable to play soccer - An x-ray shown - Which structures attached to the abnormal anterior tibial area?

A 65-year-old health maintenance exams – He lives is a single-family home with his cat and dog. He spend much of his time in his basement woodworking shop. This patient is increased risk for lung cancer due to which of the following environmental exposures?

Radon

54-year-old man – intense overwhelming fearWhich portion of brain stimulated?

Amygdala

b. Fear = amygdala

30 year old woman – recurrent URI – Sweat is Salty- Genetic testing for 36 most common mutations shows detectable G551D in one allele of CFTR – What is patients clinical phenotype?

The Second CFTR Gene was not detected by the testing obtained

74 year old – COPD – “I enjoy coming to see you because you remind me of my daughter. She died 35 years ago”. Which is most appropriate response by physician?

You must miss your daughter very much. Tell me about her.

b. PT = Transference

9 month old boy – awakens and cries at least once nightly and settles back to sleep after drinking a bottle of formula – Explanation for patient’s sleep pattern?

Normal Development

Pubertal gynecomastia in males is norma

A 32 year old – abnormal movement in hands and worse when he feels angry – Unable to fix his gaze on one point or protrude his tongue for more than 30 sec – Pt most likely has anatomic abnormalities in which of the following locations?

Basal Ganglia

A 14-year-old boy – recurrent otitis media since infancy – underwent tonsillectomy for obstructive sleep apnea – Cardiac auscultation exam listen – shows S3 – which is most likely finding?

Normal Findings

b. normal physiologic splitting

37 year old woman – right lower extremity edema – sudden SOB+, If present on physical exam which sign would be the most specific indicator or pulmonary arterial hypertension?

P2 louder than A2

b. Pulmonary Hypertension – Split S2

43 year old – with t2DM – ER – nausea and vomiting – after drinking champagne. Treated for Trichomonas Vaginalis Infection – TX with metronidazole. What is cause of nausea and vomiting?

Accumulation of Acetaldehyde

b. Metronidazole – disulfiram reaction

Image: 43 year old - with t2DM - ER - nausea and vomiting - after drinking champagne. Treated for Trichomonas Vaginalis Infection - TX with metronidazole. What is cause of nausea and vomiting?

45 year old man – right shoulder pain – patient unable to externally rotate the shoulder against resistance – Which of the following tendons is inflamed?

Infraspinatus

b. SITS Muscles

c. Teres Minor and Infraspinatus external rotation

d. Innervated by Suprascapular Nerve

A 44 year old man – 6 week feet numbness – chemotherapy with vincristine – MOA of Vincristine?

Depolymerization of Microtubules

Image: A 44 year old man - 6 week feet numbness - chemotherapy with vincristine - MOA of Vincristine?

10 year old – 95th percent height and 25th weight – Nurse thinks she has marfans – Molecular testing for FBN1 Gene shows single nucleotide change does not change amino acid at the locus – Explain the single nucleotide change in the patient and the mother?

It is a polymorphism

26 year old woman – ED – 2 day history of runny nose – Patient has to wait 6 hours before she is seen. In addition to apologizing, which of the following is the most appropriate remark?

Thank you for waiting. How can I help you today?

A 26 year old man – ER -m 30 minutes after being shot in the leg – Pulse is 120/min, respirations are 16 /min – blood pressure is 80/60Findings compared with healthy adult?

Baroreceptor Firing decreased

SVR increased

Pulmonary Vascular Resistance Increased

Absorption Increased

e. BARORECPTOR – stretch receptors – mirror BP – decreased in hemorrhagic shock

A 36 year old man – fluid is presenting adbomen in which of the following areas as indicated by the arrow?

Omental Bursa (lesser sac)

Image: A 36 year old man - fluid is presenting adbomen in which of the following areas as indicated by the arrow?

What is the positive predictive value (refer to chart)?

.67

Probability that a patient with a positive test has a disease

A/(A+B)

Image: What is the positive predictive value (refer to chart)?

A 75 year old woman with T2DM and HTN – Emigrated from Argentina – Which of the following is the most appropriate person to serve as an interpreter for this patient encounter?

Telephone Interpreter

During a study of renal glomeruli – if efferent arteriole is constricted – which starling forces are likely to change in the glomeruli?

Increased Hydrostatic Pressure

Image: During a study of renal glomeruli - if efferent arteriole is constricted - which starling forces are likely to change in the glomeruli?

2 year old with neurocognitive dysfunction – fine during birth – both parents have learning disabilities – maternal uncle with cognitive disabilities – Irritable making eye contact with flapping his hands?

Trinucleotide Repeat Expansion

b. Fragile X – intellectual disability – trinucleotide repeat in FMR1 gene

A 33 year old man – undergoes radical thyroidectomy – postop serum calcium is 7.5 and albumin concentration of 4g/dl – PTH concentration of 200 – damage to which vessel causes finding?

Branch of Thyrocervical Trunk

b. inferior thyroid arteries, which arise from the thyrocervical trunk.

c. External carotid – superior thyroid

Image: A 33 year old man - undergoes radical thyroidectomy - postop serum calcium is 7.5 and albumin concentration of 4g/dl - PTH concentration of 200 - damage to which vessel causes finding?

46 year old woman – 2 month history of fatigue and muscle weakness – 10 year history of hypertension – treated with thiazide diurectics – measured hypokalemia, hypotension and decreased BP- brisk reflexes. Likely cause?

Adverse Drug Effect

b. Treated with a thiazide diuretic for HTN with measured hypokalemia, hypotension and decreased Hg

Image: 46 year old woman - 2 month history of fatigue and muscle weakness - 10 year history of hypertension - treated with thiazide diurectics - measured hypokalemia, hypotension and decreased BP- brisk reflexes. Likely cause?

55-year old man – nice to nurses – mean to youngest child– Which is the most likely explanation of patient’s behavior?

Splitting

b. Cluster B – borderline personality (BANH)

A Study designed to evaluate the feasibility of acupuncture in children with chronic headache – 60 children recruited – In addition to their usual therapy – all children are treated with acupuncture three times per week for 2 months – Describe findings?

Case-Series

6 year old girl – 1 week history – lots of infections -raised neutrophil count- but cells show a delay in bactericidal activity against Staph Aureus– Most likely cause?

Myeloperoxidase

Image: 6 year old girl - 1 week history - lots of infections -raised neutrophil count- but cells show a delay in bactericidal activity against Staph Aureus- Most likely cause?

2 year old boy – severe pain, swelling, and redness of his left thumb (image). PE shows oral vesicle, cervical lymphadenopathy – infectious agent causing findings in thumb?

DNA Virus

Herpetic Whitlow

Image: 2 year old boy - severe pain, swelling, and redness of his left thumb (image). PE shows oral vesicle, cervical lymphadenopathy - infectious agent causing findings in thumb?

Herpetic Whitlow

vesicles on fingers

Image: Herpetic Whitlow

7 year old – lives in Kentucky – 2 week history of cramping abdominal pain and diarrhea – mother looked in underpants and saw earthworm – no travel history- Transmission via?

Ingestion of Soil

b. Ascaris – fecal oral transmission

23 year old man with multiple sex partners – dysuria and yellow urethral exudate – Gram Stain – numerous neutrophils – many contain gram negative diplococci. He had 3 episodes over the past 2 years. Which property of the organism – explains reinfection?

Antigenic Variation

b. Gram negative Diplococci

c. N. gonorrhea can change its pilus, 

23 year old woman – genetic counseling – brother and maternal uncle had Duchene Muscular Dystrophy – Serum CK of 120 – and mother serum CK 300- What is assessment of carrier status?

The patients DMD carrier status is uncertain because of random X Inactivation

b. Barr Body

c. DMD is X-linked.

20 year old woman – heavy bleeding – Platelet Aggregation is normal – PTT elevated – no easy bruising – Which hemorrhagic disorder cause of patients menorrhagia?

Von Willebrand Disease

B. Platelet aggregation time being normal, ok fine I can see that.

Image: 20 year old woman - heavy bleeding - Platelet Aggregation is normal - PTT elevated - no easy bruising - Which hemorrhagic disorder cause of patients menorrhagia?

32 year old man – ER – for MVC – urethral injury – blood noted at the uretheral meatus. Which portion of the urethra would be at greatest risk of injury?

Membranous

a. Pelvic fracture – MVC

b. Pelvic Straddle injury = Bulbar (spongy) urethera

Image: 32 year old man - ER - for MVC - urethral injury - blood noted at the uretheral meatus. Which portion of the urethra would be at greatest risk of injury?

A 63 year old – ED – unresponsive – Crackles heard over left upper and entire right lung. Pt dies. Photograph of section of right lung obtain – Which is likely cause of the position indicated by the arrow?

C5a

Complement system calls in neutrophils

C5a = neutrophil chemotaxis

Image: A 63 year old - ED - unresponsive - Crackles heard over left upper and entire right lung. Pt dies. Photograph of section of right lung obtain - Which is likely cause of the position indicated by the arrow?

A 22 year old woman – 6 months sever lower backpain – endometrial biopsy shows abundant mitotic figures in the endometrial glands and stroma. Which protein or enzymes regulate the progression of this phase of the menstrual cycle?

Cyclin Dependent Kinase.

29 year old woman – prescribed carbamazepine for trigeminal neuralgia- family history of osteoarthritis – the most likely reason for this recommendation is that carbamazepine may affect which of the following processes?

Metabolism

Image: 29 year old woman - prescribed carbamazepine for trigeminal neuralgia- family history of osteoarthritis - the most likely reason for this recommendation is that carbamazepine may affect which of the following processes?

An 18 year old woman – fever, dizziness, weakness, and vomiting (toxic shock) – using synthethic pads and tampons – Physical exam shows injected conjunctiva and rash– Pts use of which increased risk?

Tampons

A 53 year old man – – 6 month history of blood in stool – 1 cm anal mass – If mass is malignant, evaluate which lymph nodes?

Superficial Inguinal

1 cm mass below dentate line

Image: A 53 year old man - - 6 month history of blood in stool - 1 cm anal mass - If mass is malignant, evaluate which lymph nodes?

24 year old – brought to ED – MVC – face struck steering wheel – facial pain and double vision – Exam of left eye shows infraorbital edema – ocular movement restricted vertically – CT scan shown. Which of the following impaired on left side?

Sensation over the upper lip

V2 = maxillary nerve

Image: 24 year old - brought to ED - MVC - face struck steering wheel - facial pain and double vision - Exam of left eye shows infraorbital edema - ocular movement restricted vertically - CT scan shown. Which of the following impaired on left side?

23 year old – collapsed at finish line – no loss of consciousness Associated with prolonged use of vaginal tampons or nasal packing She is placed on a cot – feet elevated – 30 minutes later her blood pressure is 110/70 mm Hg. Which described patients condition?

Orthostatic Hypotension

.decrease in systolic blood pressure of 20 mm Hg (110-85)

e. decrease in diastolic blood pressure of 10 mm Hg. (70-50)

27 year old – ER – pain in navel – nausea and one episode of vomiting 1 hour ago – underwent appendectomy 2 years ago- CT scan of abdomen shows fat stranding in small bowel with areas of inflammation in terminal illeum – 2 cm bulge on antimesenteric border of the inflamed segment. Histological specimen – most likely to show which of the following?

Gastric Mucosa

b. Crohns with the multiple episodes over the last year, fat stranding, terminal ileum involvement

14 year old girl –parents divorced – starts acting sullen, defiant, and truant at school, sexual intercourse with new partners and no condoms, previously honor studenty- which defense mechanisms?

Acting Out

38 year old – pregnant woman – ultrasound shows oligohydramnios and a full term fetus with a large kidney and empty right renal fossa. Need history on maternal use of which during pregnancy?

ACE Inhibitors

A 65 year old – bilateral wheezing – accumulation of which of the following?

Leukotrienes

Image: A 65 year old - bilateral wheezing - accumulation of which of the following?

19 year old – with asthma – comes to ER multiple times for acute asthma exacerbations – “All doctors don’g understand me” – they keep prescribing steroid inhalers. What is the most appropriate statement by the physician?

Tell me more about what you know about steroid inhalers and how are they supposed to work?

A healthy 22 year old man – in study for glucose metabolism – He consumes an 800 calories meal consisting – 12 hours later serum glucose is within reference range – Which is involved in maintaining serum glucose?

Glycogenolysis of Liver

Image: A healthy 22 year old man - in study for glucose metabolism - He consumes an 800 calories meal consisting - 12 hours later serum glucose is within reference range - Which is involved in maintaining serum glucose?

30 year old man – 6 week history of colicky abdominal pain and diarrhea with occasional blood – on autopsy – small bowel is seen in the wall of a thickend loop of small intestine – Photomicrographs (granulomas)- DX?

Crohns Disease

Image: 30 year old man - 6 week history of colicky abdominal pain and diarrhea with occasional blood - on autopsy - small bowel is seen in the wall of a thickend loop of small intestine - Photomicrographs (granulomas)- DX?

A 3 month old boy – 2 hour respiratory distress – Physical exam shows cyanosis – blood sample appears brown – most likely cause of patients condition”

Decreased activity in the enzyme that reduced Fe3+ to Fe 2

a. the hypoxia and the “blood sample appears brown” should point you towards methemoglobinemia.

b. if you want a fun video from “9-1-1” that actually explains a lot of it:

c. methemoglobinemia = cyanosis and chocolate colored blood.

68 year old – 1 day history of fever and swelling in left leg – cellulitis – gram stain – gram positive, catalase negative cocci – clear zones of blood agar – Which species of Streptococcus?

Strep Pyogenes

67 year old woman – bicuspid aortic valve – admitted to the hospital – 2 day history of fever and chills. Blood culture grows viridans streptococci – In addition to penicillin, which other drug is administered to shorten the duration of treatment?

Binding to 30S protein

12-year-old girl – 2 month history of intermittent yellowing of the eyes and skin – PE – no abnormalities except jaundice. Total bilirubin 3mg/dl unconjugated bilirubin > Direct – Findings in patient?

Decreased activity of UDP glucoronylytransferase

During an experiment, drug X is added to a muscle bath containing a strip of guinea pig intestinal smooth muscle. Agonists are added to the bath, and the resultant effects on muscle tension are shown in table. Acetylcholine increased. Which of the following drugs most likely to produce effects most similar to those of drug X?

Cholinesterase Inhibitor

Image: During an experiment, drug X is added to a muscle bath containing a strip of guinea pig intestinal smooth muscle. Agonists are added to the bath, and the resultant effects on muscle tension are shown in table. Acetylcholine increased. Which of the following drugs most likely to produce effects most similar to those of drug X?

30 year old women – 28 week gestation – Fetal ultrasound normal – Father’s blood group in B, Rh Positive – Physician recommends administration of Rh0D immune globulin – This treatment is most likely to prevent?

Formation of antibodies to RhD

55- year -old –man – hypertension and chronic kidney disease – ECG shows low voltage with electrical alternans. Physical exam is most likely to show which of the following?

BP 85/60

Pulse 120

JVP Increased

Pulse Paradoxus Increased

a. Electrical Alternans = Cardiac Tamponade = pleural effusion

b. Beck Triad – Hypotension, JVP, muffled heart sounds

52-year-old – newly diagnosed with Type 2 diabetes mellitus. Four weeks later, her hepatic glucose out is decreased and target tissue glucose uptake are increased. Which agent?

Metformin

An asymptomatic 44-year-old man – with HIV infection during routine screening to donating blood. A complete blood count- platelets are low – which of the following finds are most likely?

Petechiae

Platelet problem

epistaxis, ecchymoses, petechia, bleeding from superficial scratches

Coagulation problem

late re-bleed, Menorrhagia, GI bleeds, hemarthroses

23 year old – bone marrow failure – given a large dose of rabbit antimyocyte globulin – 10 days later – develops fever, lymphadenopathy, arthralgias, and erythema on her hands and feet. Which is the most likely cause of these symptoms?

Immune complex deposition in tissues

b. Serum sickness – TYPE 3 hypersensitivity

Image: 23 year old - bone marrow failure - given a large dose of rabbit antimyocyte globulin - 10 days later - develops fever, lymphadenopathy, arthralgias, and erythema on her hands and feet. Which is the most likely cause of these symptoms?

42 year old – gunshot wound – 36 hours post-operatively she is receiving morphine. she needs morphine – she has expressed concern she is becoming addicted. Which of the following initial actions by the physician is most appropriate?

Reassure the patient has a miniscule chance of becoming addicted to narcotics

A 22 y/o woman –– itchy, red rash on her arm – new hair dye – picture (contact dermatitis) – Which ligand pairs play a role in the proliferation of the T Lymphocytes

CD28 on T Lymphocytes and CD80 on epidermal Langerhans cells

Image: A 22 y/o woman -- itchy, red rash on her arm - new hair dye - picture (contact dermatitis) - Which ligand pairs play a role in the proliferation of the T Lymphocytes

6 healthy subjects in a study of muscle metabolism – Increased malonyl-CoA directly inhibits which of the following processes in these subjects

Fatty Acid Oxidation

b. Malonyl-CoA inhibits the rate limiting step in beta-oxidation of fatty acid.

Image: 6 healthy subjects in a study of muscle metabolism - Increased malonyl-CoA directly inhibits which of the following processes in these subjects

Over 1 year – study is conducted to assess the antileukemic activity of a new tyrosine kinase inhibitorwith CML in blast crisis – All patients in the study had SML and are informed that they would be treated with tyrosinase inhibitor. What type of study design?

Open Label Clinical Trial

A 63 year old man – 4 day history of increasingly severe left leg pain and swelling on his left calf – CT scan shows no abnormalities – A CT scan of the abdomen shows a 3 cm mass in the body of pancreas – Ultrasound shows a femoropopliteal venous clot – Which of the following is the most likely cause?

Hypercoagulability of advanced malignancy

A 40 year – comes to physician because of 6-month history of facial hair growth – temporal balding and coarse hair on the upper lip chin – ultrasound shows a 12 cm ovarian mass – Describe mass?

Sertoli – Leydig Cell Tumor

35 year old man – pain and swelling in right arm when scraped a tree branch – Temp 101F- Examination of the right arm shows edema around a lesion – primary mechanism of edema?

Separation of endothelial Junction

Image: 35 year old man - pain and swelling in right arm when scraped a tree branch - Temp 101F- Examination of the right arm shows edema around a lesion - primary mechanism of edema?

12-year-old – 2 month history of headaches – 6 day history of nausea and vomiting – bilateral papilledema – broad based gait An MRI – shows tumor in pineal region. Pt most likely oculuomotor impairment?

Upward Gaze

Parinaud Syndrome due to compression of tectum-

vertical gaze palsy occurs with pineolmas

52 year old man – 1 day history of nausea, vomiting, and right sided abdominal pain – radiates to his back – CT scan – shows enlarged right kidney and wedge shaped areas of hypodensity – no nephrolithiasis – strongest predisposing factor?

Atrial Fibrillation

Wedge Shaped renal infarct

coagulative necrosis

24 year old – rash from hot tub- the infectious agent causing these findings most likely began to proliferate in which of the following locations?

Hair Follicle

b. Hot Tub Folliculitis = Pseudomonas

Image: 24 year old - rash from hot tub- the infectious agent causing these findings most likely began to proliferate in which of the following locations?

45-year-old – picture of H. Pylori – epigastric heartburn and weight loss – Which process likely involved?

Elaboration of proteases and urease with local support tissue

Image: 45-year-old - picture of H. Pylori - epigastric heartburn and weight loss - Which process likely involved?

A 14-year-old – hit with baseball bat – unable to dorsiflex his foot. Which of the following nerves is most likely injured?

Common Fibular Nerve (peroneal nerve)

26 year old woman – brought to ED – 8 hour history – severe back and abdominal pain – persistent vaginal bleeding – Ultrasonography shows 2 cm ectopic pregnancy in the ampulla – Ampulla ruptured into surrounding tissue. Fluid will most likely be found in the following locations?

Pouch of Douglas

Image: 26 year old woman - brought to ED - 8 hour history - severe back and abdominal pain - persistent vaginal bleeding - Ultrasonography shows 2 cm ectopic pregnancy in the ampulla - Ampulla ruptured into surrounding tissue. Fluid will most likely be found in the following locations?

46 year od with ankylosing spondylitis – treated with NSAIDs drugs – Sulfasalazine treatment has not resulted in improvement – most appropriate next step in treatment is administration of a drug that inhibits which of the following?

Tumor Necrosis Factor

55-year-old with blisters that do not break easily, and no oral lesions – these blisters are most likely the result of adhesions failure of which of the following?

Basement Membrane

47-year-old with organophosphate poisoning – blurred vision, difficult breathing, diarrhea, muscle weakness, fatigue – Most appropriate immediate pharmacotherapy?

Atropine

b. Organophosphate poisoning – DUMMBELLS

c. Crosses blood brain barrier – reverses muscarinic effects in CNS

72 year old – difficulty swallowing, chest pain, and cough- tachypnea and equal pulses bilaterally- percussion shows dullness over right lung – chest X-ray shows area of opacification in the lower region of the right lower region of lung – Which is most likely cause of patients condition?

Aspiration

62 year old man –abdominal pain – interviewed by physician – used single word and sarcastic answers. No eye contact – FROWNS – tells physician third time he has been asked question- Appropriate response?

You Sound Upset. Tell me a little more about that.

54-year-old – ECG shows P wave and R wave no relation between the two. Which is the most likely diagnosis?

Third Degree AV Block

42 year old – constipation, abdominal discomfort, mild fatigue, CA increased, P04 decreased, treated for renal calculus – PE shows now abnormalities – Most likely cause of patients condition is a small well defined nodule in which location?

Parathyroid Gland

72 year old – ER – stroke symptoms – slurred speech and difficulty walking – left sided hemiparesis – Tongue deviates to right when protruded – proprioception and sensation to light touch are absent over the left upper and lower extremity – Which site in brain stem is damaged

=Area Labeled C

Image: 72 year old - ER - stroke symptoms - slurred speech and difficulty walking - left sided hemiparesis - Tongue deviates to right when protruded - proprioception and sensation to light touch are absent over the left upper and lower extremity - Which site in brain stem is damaged

68 year old woman – end stage renal disease – initially she did well – within 3 months she has been admitted for fluid overload – poor adherence to fluid and salt restrictions – Which is most appropriate initial response?

It is tough to change your diet and fluid intake, but what sorts of things were you doing first when you were following recommendations

During an experiment – southern blot analysis – Figure showed resulting pattern with DNA samples isolated from different organs. Southern blot demonstrates?

Gene rearrangement

Image: During an experiment - southern blot analysis - Figure showed resulting pattern with DNA samples isolated from different organs. Southern blot demonstrates?

17 year old girl – physical exam shows normal female body habitus – normal breast development – refuses to have a pelvic or rectal examination.- no axillary or pubic hair – most likely clinical presentation?

Androgen Insensitivity

16 year old boy – 3 day abdominal pain and vomiting – CT scan shows perforated appendix – Exam of peritoneal fluid shows which organism?

Escherichia Coli

45 year old woman 6 month history of hot flashes, night sweats, and insomnia- Physician informed consent on new treatment – most appropriate next step?

Could you tell me the results of the hormone treatment options we have discussed?

A randomized control trial – assess risk of GI adverse risk using azithromycin compared to erythromycin in the pertussis – 100 children with pertussis enrolled – 50 azithromycin and 50 erythromycin – vomiting among 5 pts in azithromycin group and 15 patients in erythromycin group – Which represents absolute risk reduction?

a. 0.2

. 15/50 – 5/50 = .2

34 – year -old woman with a 10 year history of Hep C – progressive fatigue – vitals normal. – liver biopsy shows hepatocellular injury – infiltration and early fibrosis – which mechanism is most likely to cause hepatocyte injury?

Foreign Peptides bound to Class I MHC Molecules are recognized by CD8+ T Lymphocytes

Image: 34 - year -old woman with a 10 year history of Hep C - progressive fatigue - vitals normal. - liver biopsy shows hepatocellular injury - infiltration and early fibrosis - which mechanism is most likely to cause hepatocyte injury?

A 50 year old – high blood pressure – If left untreated, which of the following is most likely to decrease in this patient?

Baroreceptor Firing

Image: A 50 year old - high blood pressure - If left untreated, which of the following is most likely to decrease in this patient?

A 62-year-old with interstitial pneumonitis – compared with a healthy man, analysis of this patients biopsy is most likely to show the following patterns in the cell population (Type 1, Type II, and Fibroblast)?

Type I pneumocytes decreased

Type II pneumocytes increased

Fibroblasts increased

31 year old – with T2DM – comes to physician because of an oozing, foul smelling wound – patient has crepitant bullae – Gram Statin shows gram-positive rods – The causal organism has which of the following virulence factors?

A -toxin

Image: 31 year old - with T2DM - comes to physician because of an oozing, foul smelling wound - patient has crepitant bullae - Gram Statin shows gram-positive rods - The causal organism has which of the following virulence factors?

4 month year old with SCID received a bone marrow transplant. – six days later – he develops a widespread, erythematous, maculopapular rash – Exam of skin shows diffuse degeneration of basal epidermal cells with mononuclear inflammatory cell infiltrate- Which is cause of rash?

Graft vs Host Disease

Image: 4 month year old with SCID received a bone marrow transplant. - six days later - he develops a widespread, erythematous, maculopapular rash - Exam of skin shows diffuse degeneration of basal epidermal cells with mononuclear inflammatory cell infiltrate- Which is cause of rash?

1

1. 36-year-old F with 2 week history of fatigue, bleeding of the gums, and bone pain. Physical examination shows pallor, hepatosplenomegaly, and ecchymotic lesions over extremities. Labs:Hb 8g/dlHt 25%Leukocytes: 36,000segm neutr 4%eosinophils 4%lymph 6%mono 6% promyelo 80%platelets 25,000Polymerase chain reaction test shows an mRNA corresponding to the retinoic acid receptor- alfa/promyelocytic leukemia fusion gene resulting from a reciprocal translocation of chromosomes 15 and 17Treatment w/ all-trans retinoic acid is started. In response to the therapy, the fusion protein will most likely attract which of the following proteins to form a pre-transcriptional complex?

a. Histone acetylase

b. Histone acetylation allows for relaxation of the DNA 

e. Histone Acetylation makes DNA Active

f. Histone Methylation Mostly Makes DNA Mute.

68-year-old man with 3 days of increasingly severe chest pain, shortness of breath, stridor, hoarseness, difficulty swallowing and nonproductive cough. Long-standing hx of hypertension. Smokes 2 packs of cigarettes for 45 years. T 99 F, pulse 80, rr 15, bp 160/94. PE shows visible pulsation above the manubrium of the sternum and displacement of the trachea to the right. Murmur second right intercostal space. Dx?

Aortic aneurysm

male smoker with HTN + deep chest pain and pulsatile mass = AA all day long

1. 35-year old man w several episodes of squeezing chest pain gets angiogram… gets IV NE. Question shows a graph of coronary blood flow with a drop after the NE and then a rise. 

Which substance causes increased total coronary blood flow 1-2 mins after NE?

Adenosine

Adenosine and Nitric oxide (NO) increases coronary blood flow

Image: 1. 35-year old man w several episodes of squeezing chest pain gets angiogram... gets IV NE. Question shows a graph of coronary blood flow with a drop after the NE and then a rise. 

Which substance causes increased total coronary blood flow 1-2 mins after NE?

4. Man comes to doc for cast removal. Fracture of left humerus that required open reduction, internal fixation, cast immobilization. Muscle strength is 2/5 with extension of elbow and 1/5 with extension of wrist and fingers. Patient most likely sustained a fracture at (which location in humerus)?

Radial groove (radial N. is the extensors of arm, wrist and fingers)

Prevalence of high-grade cervical intraepithelial neoplasia in unscreened population is 5%. Prevalence in population with negative Pap smear results is as high 0.2%. Prevalence decreases, which also decreases?

Predictive value of a positive test result

Higher prevalence – higher predictive vales – positive, predictive, prevalance

45-year-old woman with joint pain due to rheumatoid arthritis comes for infective treatment with over-the-counter agents. Initiate disease-modifying antirheumatic drug (DMARD). Delayed onset of action of DMARD, so physician prescribes another until DMARD is effective. Drug?

Prednisone

You can use three classes of medication to treat RA: NSAIDs, corticosteroids, and DMARDs. NSAIDs and corticosteroids have a short onset of action, while DMARDs take a while to take effect.

28-year-old man infertility, weight lifter and takes anabolic steroidsMechanism of infertility?

Suppression of Gonadotropins

Anabolic steroid use suppresses the Hypothalamic-Pituitary-Gonadal axis which will down regulate GnRH.

FA 2020 p636

62-year-old woman with 3-day hx of fever, shaking chills, and left flank pain. Dx acute pyelonephritis and treated with ciprofloxacin. Five days after, sudden onset watery diarrhea and lower abdominal cramps. T 100.9 F, pulse 80, rr 18, bp 124/88. PE moderate tenderness to palpation in lower quadrants especially on right and increased bowel sounds. Stool is brown and occult blood negative. Next step?

Test of the stool for Clostridium difficile toxin

b. Pseudomembranous colitis

c. Couple days after Ciprofloxin

72-year-old woman comes to the physician because of a 6-month history of increased bruising on her forearms. 

She appears alert and well nourished. Physical examination shows extensive wrinkling, scaly erythematous patches on the face, and irregularly shaped brown macules on the face and forearms

There are ecchymoses in various stages of healing on both forearms; the ecchymoses are more numerous on the right side

Laboratory studies, including a complete blood count and coagulation studies, are within the reference ranges. S

he has noticed no bleeding from her gums after brushing her teeth. Which of the following is the most likely cause of the ecchymoses in this patient?

Extensive solar elastosis

Actinic elastosis, also known as solar elastosis, is an accumulation of abnormal elastin (elastic tissue) in the dermis of the skin, or in the conjunctiva of the eye, which occurs as a result of the cumulative effects of prolonged and excessive sun exposure, a process known as photo=aging

]21-year-old woman with 2-days history of urinary frequency and pain with urination. T 39 C (102.2 F), pulse 125, bp 96/60. Urine grows gram-negative bacteria. Virulence factor for adherence to bladder?

Fimbriae

Ecoli

73-year-old man has poor appetite and lost 25 lb over 4 months. Labs show normochromatic normocytic anemia. Xray of chest shows 2-cm perihilar mass. Biopsy shows small cell carcinoma of lung. Which is responsible for weight loss?

Cytokine effect

Cachexia = increased pro-inflammatory cytokines

Mediated by TNF-α, IFN-γ, IL-1, and IL-6

34-year-old man with herniated lumbar intervertebral disc. Laminectomy and removal of hernia scheduled. CT scan of vertebrae shown, which is surgical entrance location into neural canal?

Area labeled D

Image: 34-year-old man with herniated lumbar intervertebral disc. Laminectomy and removal of hernia scheduled. CT scan of vertebrae shown, which is surgical entrance location into neural canal?

24-year-old woman overdose on drug X and has serum concentration of 32. Drug X follows first-order kinetics. 6 hours later, the serum concentration is 16. Cannot be moved from intensive care unit until concentration 1.1 or lower. Which is minimum number of hours from first blood sample (32) that patient must remain in icu?

30 hours (5 half lives)

b. First Order – Constant Fraction

c. Zero Order – Constant Amount

45-year-old woman dx with cholelithiasis. She asks why abdominal pain is intermittent and not constant. Pain is produced when gallbladder contracts against gallstone obstructing cystic ductWhere is hormone released from that causes gallbladder contraction?

Enteroendocrine cells of the small intestine (CCK)

CCK = produced by the I cells in the duodenum and jejunum in response to fatty acids and amino acids.

CCK – cause gallbladder to contract

Image: 45-year-old woman dx with cholelithiasis. She asks why abdominal pain is intermittent and not constant. Pain is produced when gallbladder contracts against gallstone obstructing cystic duct. Where is hormone released from that causes gallbladder contraction?

. 35-year-old with Chagas disease and 2-hour history of moderate chest pain. 4.4 lb weight loss in 2 months. Vitals stable. Barium swallow shows dilated esophagus with beak-like narrowing at lower esophageal sphincter (LES). Biopsy shows?

Loss of neurons in myenteric plexus

failure of LES relaxation, causing distal stenosis of the LES (“bird’s beak” sign) proximal dilation indicative or achalasia

Image: . 35-year-old with Chagas disease and 2-hour history of moderate chest pain. 4.4 lb weight loss in 2 months. Vitals stable. Barium swallow shows dilated esophagus with beak-like narrowing at lower esophageal sphincter (LES). Biopsy shows?

45-year-old man comes to physician for follow up after appendectomy. There is mild scleral icterus and well healing surgical incision. 

Lab values show 

Total bilirubin 3.2 

Direct: 0.2 

Indirect 3 mg/dL

Dx?

a. Gilbert

Stress related – Mild jaundice with increased unconjugated in an older fellow is decreased UDP-glucuronyltransferase activity.

35yo F has congestive cardiomyopathy and pitting edema. Her serum urea nitrogen concentration is 25mg/dl and serum creatinine is 1.8mg/dl. Furosemide therapy is started. 

After 5 days, labs show:

Na 130K 4.5Cl 90HCO 30

BUN 85 Creatinine 2.2

Albumin 3Urine specific gravity 1.023, 

rbc 0 wbc0 sediment none

Urinary fractional excretion of sodium is less than 1%. 

Explanation?

Prerenal azotemia

After the addition of furosemide, her BUN/Cr ratio went up to 38 and Fraction Excretion of Na < 1

Image: 35yo F has congestive cardiomyopathy and pitting edema. Her serum urea nitrogen concentration is 25mg/dl and serum creatinine is 1.8mg/dl. Furosemide therapy is started. 
After 5 days, labs show:
Na 130K 4.5Cl 90HCO 30
BUN 85 Creatinine 2.2
Albumin 3Urine specific gravity 1.023, 
rbc 0 wbc0 sediment none
Urinary fractional excretion of sodium is less than 1%. 
Explanation?

33-year-old woman who three weeks ago, underwent oophorectomy for epithelial ovarian cancer. Recommends adjuvant chemotherapy with paclitaxel. Mechanism?

a. Inhibits microtubule disassembly

“taxes STABILIZE the economy”

4-year-old boy with fatigue and irritability for 2 months. Family visited rural Louisiana 5 months ago and ran around barefoot. Conjunctivae are pale. Labs shows normal wbc with 15% eosinophils. Stool prep shows parasite egg (picture). Cause of fatigue?

Microcytic anemia

Necator or Ancylostoma = running around barefoot (entrance site for laravae

hookworms suck blood from the intestines leading to iron deficiency anemia.

Parasite in US – unable to absorb nutrient like iron

Image: 4-year-old boy with fatigue and irritability for 2 months. Family visited rural Louisiana 5 months ago and ran around barefoot. Conjunctivae are pale. Labs shows normal wbc with 15% eosinophils. Stool prep shows parasite egg (picture). Cause of fatigue?

Heart of 76-year-old woman shows concentric enlargement of left ventricle. Micro exam shows enlarged myocardial cells with large nuclei. Dx that causes this cardiac enlargement?

Hypertension

b. concentric ventricular hypertrophy = increased afterload

c. eccentric ventricular hypertrophy = increased preload

d. chronic HTN, Friedreich ataxia

23-year-old woman g1p1, not felt well since delivery 2 weeks ago; ongoing fatigue, inability to breast-feed, light-headedness in upright position. Pregnancy complicated by preeclampsia and required cesarean delivery, after she required blood transfusions because of hypotension. PE appears lethargic. HR 80, BP 100/50, HR standing 85, BP standing 86/44. Hb 11.6, hct 36%. Serum hormone concentrations?

Prolactin decreased

ACTH decreased

TSH decreased

Aldosterone increased

Sheehan syndrome

c. severe bleeding requiring transfusions + inability to breastfeed (decreased prolactin

d. All hormones from pituitary – destroyed

50-year-old man with pulmonary embolus. Treated with intravenous heparin. 24 hours later, warfarin added. Day 2, partial thromboplastin time is 52 seconds (control 26 sec), and prothrombin time is 12 seconds (control 12.1 sec; INR = 1). Best explanation for normal prothrombin time and INR?

Long half-life of factor II (prothrombin)

Factor II (prothrombin) has the longest half-life

Warfarin acts on Factor II, VII, IX, X

Heparin = factor IIa inhibited by ATIII – thus

low

PTT is prolonged

Image: 50-year-old man with pulmonary embolus. Treated with intravenous heparin. 24 hours later, warfarin added. Day 2, partial thromboplastin time is 52 seconds (control 26 sec), and prothrombin time is 12 seconds (control 12.1 sec; INR = 1). Best explanation for normal prothrombin time and INR?

8-year-old boy with persistent disruptive behavior. Teacher says he’s in “constant motion” and never completes assignments. Treated in ER several times because of skateboarding injuries. Tx?

Methylphenidate

sxs in 2+ settings–> school, and ER for skateboarding injuries Tx= methylphenidate

MOA = stimulates NE and DA

65-year-old man with cancer of cecum found to have metastatic lesion to right lobe liver. Venous route of mets?

Ileocolic –> superior mesenteric –> portal –> right hepatic branch of portal

right lobe of the liver = all drain into the Superior Mesenteric Vein.

left sided structures drain into the Inferior Mesenteric Vein.

Image: 65-year-old man with cancer of cecum found to have metastatic lesion to right lobe liver. Venous route of mets?

48yo man with possible hypertension. based on 20 measurements, his average diastolic pressure is 94mmHg, SD is 8mmHg. If only four measurements were made rather than 20, which of the following statements would best describe the width of the 95% CI with regard the mean blood pressure

Larger

Width of confidence interval larger with fewer cases

Width will become smaller – with more cases

98-year-old woman who lives with daughter and grandson comes for routine exam with type 2 diabetes and hypertension but no complaints. She has had 14-lb weight loss since last visit 3 months ago. BMI 15. PE shows multiple ecchymoses in various stages of healing upper extremities and torso. Physician should ask?

Are you safe at home?

A transcription factor that activates expression of HMG coA reductase is identified, this TF is initially synthetized as a large precursor protein, with 2 membrane spanning domains. The transcriptional domain is released by proteolysis when the membrane has a decreased cholesterol concentration. The precursor protein is most likely initially produced in which of the following labeled area on this labeled diagram of a hepatocyte?

Area labeled F 

Endoplasmic Reticulum

Image: A transcription factor that activates expression of HMG coA reductase is identified, this TF is initially synthetized as a large precursor protein, with 2 membrane spanning domains. The transcriptional domain is released by proteolysis when the membrane has a decreased cholesterol concentration. The precursor protein is most likely initially produced in which of the following labeled area on this labeled diagram of a hepatocyte?

18-year-old woman comes 12 hours after ingesting 100 aspirin tablets in suicide attempt. PE shows tachypnea. Labs?pH/pCO2/HCO3-

pH 7.32

pCO2 15

HCO3 8

aspirin overdose (early) = Resp Alkalosis

aspirin overdose (Later) = Metabolic acidosis

HCO3- should also be less than normal (22-28)

12 hours = Late Stages

Image: 18-year-old woman comes 12 hours after ingesting 100 aspirin tablets in suicide attempt. PE shows tachypnea. Labs?pH/pCO2/HCO3-

10-day-old male newborn 1-day history red eyes with discharge. Pregnancy uncomplicated but no prenatal care in third trimester. PE bilateral conjunctival injection with water discharge. Cause?

Chlamydia trachomatis

b. chlamydia occurs 1-2 weeks after birth

c. conjunctivitis from gonorrhea occurs 2-5 days after birth

54-year-old man with normal renal functions gets a heart transplantation. One year later, bp 170/110 and serum creatinine 2.1. Which immunosuppressive drug caused these findings?

Cyclosporine

ADR: hypertension and nephrotoxicity

2-year-old girl with febrile seizure. PE shows nuchal rigidity and bacterial meningitis suspected. LP and immediate abx therapy planned, but parents unavailable for consent. Next step?

Initiate the procedure and treatment without consent

Implied consent in emergency

14-year-old girl with 4 months of heavy menstrual flow. Menarche at 13 with regular intervals. History of frequent nosebleeds and easy bruisability. Father has problems with nosebleeds and clotting. PE shows mild gum bleeding and ecchymoses. Labs: hb 8.2, hct 24.6, platelet 250,000, bleeding time 10, PT 14 (INR 1.5), PTT 60Dx?

von Willebrand disease

prolonged bleeding time + prolonged PTT =

A study is conducted to compare the incidence of myocardial infarction in patients undergoing two different types of angioplasty or an operative procedure to manage single-vessel coronary artery disease. A total of 1000 patients are enrolled. Through a chance process, 500 are assigned to undergo the operative procedure, 250 are assigned to undergo one type of angioplasty, and 250 are assigned to undergo a second type of angioplasty. All patients are followed for 3 years to determine the incidence of myocardial infarction. Which of the following best describes this study design?

Randomized clinical trial

Study design with randomly assigned experimental and control groups is still a randomized clinical trial

In cohort study, you don’t assign the exposure, you just simply follow them (risk factors).

51-year-old with Graves disease develops ulcerating pharyngitis after 6 months of propylthiouracil. What’s the underlying cause of ulcerative pharyngeal disease?

Agranulocytosis

(rare side effect of PTU) commonly presents with acute pharyngitis.

. 22-year-old nulligravid woman has menses that occur at irregular 26- to 32-day intervals. Height 5 ft 4 in and weighs 187 lb. BMI 32. PE shows mild hirsutism and velvety brown, thickened skin at the base of the neck and around axillae. Patient has which condition?

Hyperinsulinemia

b. PCOS

c. Acanthosis Nigrans

3-year-old boy 7 days after dx with severe chronic diarrhea due to Giardia. Immunological studies show peripheral leukocytes that express both CD3 and CD4 and fail to express CD40 ligand. Based on this, the immunoglobulin isotype that predominates has which biological properties?

Complement activation

Hyper IgM syndrome; without CD40 ligand, T cells are unable to provide the secondary signal to activate B cell class switching.

prevents IgM from switching into IgD, G, E and A, and thus an accumulation of IgM – hence the name “hyper IgM” – and deficiency in the rest

Complement activation= IgM and IgG

67-year-old woman comes to the physician because of a 1-month history of low back pain. She has hypertension well controlled with a thiazide diuretic. Her pulse is 140/min, and blood pressure is 140/85 mm Hg. Physical examination shows tenderness to palpation over the L2-3 vertebrae. Laboratory studies show:Hemoglobin 13.5 g/dLHematocrit 41%Leukocyte count 10,500/mm3Segmented neutrophils 65%Eosinophils 1%Basophils 1%Lymphocytes 30%Monocytes 3%Platelet count 250,000/mm3

Serum electrophoresis shows a monoclonal spike of IgG kappa. A chest x-ray shows cardiomegaly with bilateral pleural and pericardial effusions

An x-ray of the spine shows a lytic lesion. Echocardiography shows an echodense thickened left ventricle and poor diastolic compliance. A photomicrograph of a specimen obtained on myocardial biopsy is shown. Which of the following is the most likely cause of the cardiac findings in this patient?

Amyloid infiltration

Multiple Myleoma = monoclonal IgG kappa (light chain) spike on electrophoresis, lytic bone lesions in the spine, cardiomegaly all point to multiple myeloma.

Amyloid infiltration- Causes a primary amyloidosis that can deposit in multiple tissues,

Image: 67-year-old woman comes to the physician because of a 1-month history of low back pain. She has hypertension well controlled with a thiazide diuretic. Her pulse is 140/min, and blood pressure is 140/85 mm Hg. Physical examination shows tenderness to palpation over the L2-3 vertebrae. Laboratory studies show:Hemoglobin 13.5 g/dLHematocrit 41%Leukocyte count 10,500/mm3Segmented neutrophils 65%Eosinophils 1%Basophils 1%Lymphocytes 30%Monocytes 3%Platelet count 250,000/mm3

Serum electrophoresis shows a monoclonal spike of IgG kappa. A chest x-ray shows cardiomegaly with bilateral pleural and pericardial effusions. 

An x-ray of the spine shows a lytic lesion. Echocardiography shows an echodense thickened left ventricle and poor diastolic compliance. A photomicrograph of a specimen obtained on myocardial biopsy is shown. Which of the following is the most likely cause of the cardiac findings in this patient?

Investigator studying immune response to fungi. Fungi express beta-glucans on cell surface and that triggers innate immune response. Which tx decreases glucan expression?

Caspofungin

Inhibit cell wall synthesis by inhibiting synthesis of β-glucan

Image: Investigator studying immune response to fungi. Fungi express beta-glucans on cell surface and that triggers innate immune response. Which tx decreases glucan expression?

6-year-old boy with 5-day history of intermittent vomiting and 3-month hx of progressive clumsiness. Can no longer ride bicycle and difficulty getting out of car. Funduscopic exam shows bilateral papilledema. Neuro exam shows impaired upward gaze and pupil response to light. Walks shuffling gait. CT shows enlarged lateral and third ventricles and a 2-cm mass. Location of mass?

Pineal gland

Pinealoma

Parinaud syndrome (compression of the tectum –> vertical gaze palsy)

Obstructive Hydrocephalus

Image: 6-year-old boy with 5-day history of intermittent vomiting and 3-month hx of progressive clumsiness. Can no longer ride bicycle and difficulty getting out of car. Funduscopic exam shows bilateral papilledema. Neuro exam shows impaired upward gaze and pupil response to light. Walks shuffling gait. CT shows enlarged lateral and third ventricles and a 2-cm mass. Location of mass?

.34-year-old man burned hands firing pots 3 months ago. He has no pain during or after burn. PE shows mild atrophy of arms and hands, absence of deep tendon reflexes in upper extremities, and decreased pain and temperature sensations in C4 to T1 dermatomes. Touch is preserved. Dx?

Syringomyelia

Cape like distribution of loss of pain and temperature 

Touch preserved – lack of degeneration in the DCML.

57-year-old man with alcoholism dies of klebsiella pneumonia. Abscess cavities filled with purulent exudate on autopsy. Pattern of necrosis in lung tissue?

Liquefaction

Abscess (proteolytic enzymes from neutrophils liquify tissue 

An investigator conducts a meta-analysis of three genome-wide association studies of Crohn Disease. The studies encompassed 3200 cases and 4800 controls, all of European descent. The initial studies identified 11 significant loci with odds ratios above 1.3 and 1.5; the combined meta-analysis identified an additional 21 loci with odd ratio of 1.1 to 1.3. It is estimated that the 32 loci identified explain about 10% of the variance in disease risk with 2 loci accounting for 2% of the variance. Previous studies of twins indicated a 50% heritability rate for Crohn’s disease. Which of the following best explains these results?

The identified loci account for a relatively small part of the variance

5-year-old girls with severe leg pain 1 day after fell off sled. Immigrate to USA from Iceland. Has had three bone fractures since birth. Parents are vegan and don’t give dairy products. PE shows tenderness, swelling, and limited range of motion of left lower extremity. Xray shows fracture of fibula. Pt has vitamin deficiency that affects which of the following?

Intestinal calcium absorption

Vegan

vitamin D deficiency

36-year-old woman diagnosed with HIV. Two months ago, started antiretroviral therapy with efavirenz, emtricitabine, and tenofovir. Labs: CD4 352 and undetectable HIV viral load. Two months ago, CD4 count was 158 and plasma HIV viral load was 5500. Next step?

Continue efavirenz, emtricitabine, and tenofovir with no changes

If it ain’t broke don’t fix it.

Typical antiretroviral HIV therapy regime is:

3 NRTIs OR 2 NRTIs AND 1 NNRTI OR 1 Protease inhibitor OR 1 Integrase inhibitor

18-year-old woman get general anesthesia for wisdom teeth extraction. After 5 minutes, gets hypertonicity of skeletal muscles and increased body temperature. The treatment is a drug that decreases?

Sarcoplasmic Ca release

Malignant hyperthermia– rare, life-threatening condition in which inhaled anesthetics or succinylcholine induce severe muscle contractions and hyperthermia

Dantrolene – TX- Prevents release of Ca2+from sarcoplasmic reticulum of skeletal muscle by inhibiting the ryanodine receptor

48. 66-year-old man with 1-month hx of nonproductive cough and 6 months shortness of breath. Bilateral end-inspiratory crackles heard. CT chest shows diffuse reticular opacities at periphery and bases of lungs. Micro exam of biopsy shows patchy interstitial fibrosis, several fibroblastic foci and no granulomas. Dx?

Idiopathic pulmonary fibrosis

honeycomb lung

Image: 48. 66-year-old man with 1-month hx of nonproductive cough and 6 months shortness of breath. Bilateral end-inspiratory crackles heard. CT chest shows diffuse reticular opacities at periphery and bases of lungs. Micro exam of biopsy shows patchy interstitial fibrosis, several fibroblastic foci and no granulomas. Dx?

35-year-old man with several episodes of dizziness and fainting during the past 2 months. Father and several paternal uncles died suddenly. PE and lipid studies normal. Angiography shows no coronary artery blockage. ECG shows prolonged QTDecreased activity in which of following causes this?

Outward (delayed) rectifying potassium channel

QT is correlated with the repolarization (Phase III),

Image: 35-year-old man with several episodes of dizziness and fainting during the past 2 months. Father and several paternal uncles died suddenly. PE and lipid studies normal. Angiography shows no coronary artery blockage. ECG shows prolonged QT. Decreased activity in which of following causes this?

4-month-old boy with 1-day respiratory distress. Has progressive weakness and difficulty feeding for a month. RR 50/min. PE shows enlarged tongue and generalized hypotonia. Increased lactate, pyruvate, glucose, and uric acid. CXR shows cardiomegaly. Biopsy of skeletal muscle shows increased glycogenEnzyme impaired?

alpha-1,4-Glucosidase

Pompe Disease (Type 2) 

Lack of – Lysosomal Debranching Enzyme (α-1,6 Glucosidase)

58-year-old woman with 6-month shortness of breath and chronic nonproductive cough. 2-year history difficulty swallowing, joint stiffness, diffuse tightening of skin on face, neck, shoulders, arms, fingers. Sensitivity to cold weather, fingers turn white. Hx of esophageal reflux. Biopsy showed atrophy of epidermis and deposition of collagen throughout dermis with loss of dermal appendages. PE cutaneous ulceration, clawlike flexion deformity, decreased joint mobility. At risk for which pulmonary disorder?

Pulmonary hypertension

systemic sclerosis/CREST

complications = interstitial fibrosis, pulmonary HTN

CREST – 

Calcinosis, anti-Centromere antibody, 

Raynaud phenomenon, 

Esophageal dysmotility,

Sclerodactyly

Telangiectasia

2-year-old woman has operation for hyperparathyroidism. Three parathyroid glands found but one does not appear in normal superior location on right side. Embryologic event that led to this?

Abnormal migration of endoderm from fourth pharyngeal pouch

4th pharyngeal pouch gives rise to superior parathyroid

EAR Tonsils – Bottom to Top

Image: 2-year-old woman has operation for hyperparathyroidism. Three parathyroid glands found but one does not appear in normal superior location on right side. Embryologic event that led to this?

46-year-old woman with 1-week low-grade fever and joint pain. Has chronic headaches and takes ibuprofen several times. PE diffuse maculopapular rash. UA 2+ protein, 10-20 WBC and eosinophils. Renal biopsy would show what?

Inflammatory infiltrates in the interstitium

acute interstitial nephritis

urine eosinophils + NSAID use + Rash

47-year-old woman with irregular, raised, multicolored dark lesion on left forearm with frequent sunlight exposure. Biopsy shows malignant pigmented cells. Worst prognosis with involvement of which layer?

Subcutaneous tissue

highly vascularized and has larger blood vessels than the dermis, leading to a higher risk of metastasis and worse prognosis (TNM staging system)

Deeper than basement membrane – no longer carcinoma in situ

33-year-old man dx with epilepsy age 10 yearsMost recent generalized tonic-clonic was 5 years ago. Medication was adjusted. Current meds include carbamazepine. He’s never had any collisions while driving his motor vehicle. Patient’s status with respect to driving?

He is medically qualified to drive

most of the states a 1-2-year free seizure period is required for epileptic patients to get their license

35-year-old from group home comes for worsening behavior for 2 weeks. He believes CIA is spying on him through television set. Reports hearing voices in hall outside and that CIA now plans to kill him. Appears disheveled with unkempt hair and poor hygiene, difficulty answering questions because listening to internal stimuli. Mental status exam will show which?

Flattened affect

Schizophrenia

Positive—hallucinations, delusions, unusual thought processes, disorganized speech, bizarre behavior

Negative—flat or blunted affect, apathy, anhedonia, alogia, socialwithdrawal

45 yr old man with SOB on exertion x 6 monthsnosebleeds since adolescence, 2 pics: clubbing + hemorrhagic lesions in tongue (IMAGE), inhaled albuterol doesn’t improve his symptoms. Cause of symptoms?

Pulmonary AV shunting

osler weber rendua 

hereditary hemorrhagic telangiectasia

Image: 45 yr old man with SOB on exertion x 6 months, nosebleeds since adolescence, 2 pics: clubbing + hemorrhagic lesions in tongue (IMAGE), inhaled albuterol doesn't improve his symptoms. Cause of symptoms?

55-year-old man northern European descent with 2-month weakness, altered skin color, bilateral knee pain. Siblings have type 2 diabetes and cirrhosis. PE bronzed skin, hepatomegaly, arthritis. Increased saturation of transferrin and ferritin. Liver biopsy increased iron content and cirrhosis Greatest risk?

Hepatocellular carcinoma

hemochromatosis (bronzed skin, hepatomegaly, arthritis, increased iron content and cirrhosis)

Cirrhosis -> hepatocellular carcinoma

52-year-old female with a history of breast cancer who received 4week course of radiations 6 months ago comes for follow-up. Exam shows no cancer recurrence. Respiratory Rate 26/min. CT chest shows b/l atelectasis in upper lung fields. Primary pathophysiological cause?

Contraction

radiation therapy caused fibrosis and the myofibroblasts “contract” during fibrosis

19-year-old woman 2-day history of pain in left index finger. Injured it when catching a ball. PE shows erythema of left index finger. Unable to flex the distal phalanx when proximal interphalangeal joint metacarpophalangeal joints restrained. Xray normal. Injured structure?

Flexor digitorum profundus tendon

profound go far – profundus to distal/furthest phalynx

Image: 19-year-old woman 2-day history of pain in left index finger. Injured it when catching a ball. PE shows erythema of left index finger. Unable to flex the distal phalanx when proximal interphalangeal joint metacarpophalangeal joints restrained. Xray normal. Injured structure?

21-year-old woman with 10-days difficulty walkingTwo years ago had loss of vision in left eye which improved. Neuro exam shows decreased visual acuity in left eye with pallor of optic disc. Has past-pointing on a finger-nose test. Broad-based gait. MRI shows brain lesions in white matter of cerebellum. Pathogenesis?

CD4+ T lymphocytes are activated by myelin basic protein

Multiple Sclerosis

increased IgG level and myelin basic protein in CSF

Oligoclonal Bands

69-year-old woman with 3-week history of muscle cramps, weakness, abdominal pain, and constipation. Hypertension treated with metoprolol and hydrochlorothiazide for past 4 months. Labs show hypokalemia. Which drug should be added?

Triamterene

(potassium-sparing) = Keep your SEAT

Spirolactone, Eplenone Amiloride, Triametrene

Work in Distal Tubule Collecting Duct

Image: 69-year-old woman with 3-week history of muscle cramps, weakness, abdominal pain, and constipation. Hypertension treated with metoprolol and hydrochlorothiazide for past 4 months. Labs show hypokalemia. Which drug should be added?

82-year-old woman comes to the physician because of constant severe lower abdominal pain and fever for 24 hours. Laparoscopic examination shows severe diverticulosis and perforated diverticulitis. In spite of appropriate therapy, she dies 2 days later. Liver autopsy shown. Which of the following is the primary component of the material shown on the hepatic surface (IMAGE)?

Fibrin

Fibrin activated – perforated – for coagulation

Fibrin deposition local defense mechanism 

Image: 82-year-old woman comes to the physician because of constant severe lower abdominal pain and fever for 24 hours. Laparoscopic examination shows severe diverticulosis and perforated diverticulitis. In spite of appropriate therapy, she dies 2 days later. Liver autopsy shown. Which of the following is the primary component of the material shown on the hepatic surface (IMAGE)?

58-year-old man comes to physician for benzodiazepine prescription for situation at work. Feels anxious and thinks he can complete last 2 years of work if anxiety decreases. Gives 1-month regimen of benzodiazepine. Wife calls, “My husband got fired! I know it was because he was taking too much of that drug you gave him. Didn’t you know he had history of alcoholism?” Initial action?

Contact the patient to discuss the situation

Reach out to patient

4-year-old boy (pedigree shown) has clumsy gait for past year. Exam calf hypertrophy and proximal muscle weakness. Creatine kinase increased. Muscle biopsy shows loss of muscle tissue, regenerating muscles fibers and fibrosis. Maternal uncle had similar findings and died at 15 years. Patient’s sister is pregnant. Ultrasound identifies male fetus. Probability fetus has disorder?

¼

boy you only look at half, so the chance of him having it is 1/2.so 1/2 * 1/2= 1/4

Image: 4-year-old boy (pedigree shown) has clumsy gait for past year. Exam calf hypertrophy and proximal muscle weakness. Creatine kinase increased. Muscle biopsy shows loss of muscle tissue, regenerating muscles fibers and fibrosis. Maternal uncle had similar findings and died at 15 years. Patient's sister is pregnant. Ultrasound identifies male fetus. Probability fetus has disorder?

2-year-old boy 2-day history of fever, sore throat, rash. T 101.7 F (38.7 C). Widespread, red, sandpaper-like rash on extremities. Purulent exudate over tonsils. Dx streptococcus pyogenes (group A) infection. Cause of skin findings?

Erythrogenic toxin

causes a Toxic shock-like syndrome

Fever, Rash, Shock, Scarlet Fever.

Streptococcus Pyogens (group A strep)

35-year-old woman pain in left leg 2 days. PE shows deep venous thrombosis. Labs: platelet 200,000, PT 12 (INR 1), PTT 37. Heparin started. Five days laterplatelet 120,000

Reason for decreased platelets?

Drug-related antibodies

Heparin induced thrombocytopenia:

IgG against platelet factor 4

FA 2020

1-year-old has numerous infections of skin and oral mucosa since birth. Infections slow to respond to antibiotic therapy. T 100.4F. PE multiple erythematous lesions of skin some with superficial ulceration. WBC 21,000 77% segmented neutrophils, 6% bands, 14% lymphocytes and 3% monocytes. Biopsy shows no neutrophils in dermis or epidermis. Culture of lesion grows Staphylococcus aureus. Defective?

An integrin

Leukocyte adhesion deficiency = integrin defect

Neutrophils are unable to adhere and migrate to infection sites, instead accumulating in blood.

Image: 1-year-old has numerous infections of skin and oral mucosa since birth. Infections slow to respond to antibiotic therapy. T 100.4F. PE multiple erythematous lesions of skin some with superficial ulceration. WBC 21,000 77% segmented neutrophils, 6% bands, 14% lymphocytes and 3% monocytes. Biopsy shows no neutrophils in dermis or epidermis. Culture of lesion grows Staphylococcus aureus. Defective?

A 25-year-old woman comes to the physician because of a 2-year history of intermittent, diffuse, cramping lower abdominal pain. The pain is usually associated with 2 to 6 days of loose, watery stools, and is typically relieved with defecation. Between these episodes, her stools are normal. Her vital signs are within normal limits. Physical examination shows no abnormalities. Laboratory studies, including complete blood count, metabolic panel, and thyroid function tests show no abnormalities. A drug targeting which of the following mechanisms of action is most appropriate for this patient?

Accentuation of μ-opioid myenteric plexus receptor

Irritable Bowel Syndrome.

Loperamide = opioid anti-diarrheal

35-year-old woman with infertile, receive injection of contrast material into cervix. On hysterosalpingogram (shown), contrast material (indicated by arrows) also seen in peritoneal cavity, which explain this finding?

Spillage of contrast which normal

fallopian tubes – leak contrast – a gap between fallopian tubes and ovary.

no spillage, that means there is a blockage = infertility

Image: 35-year-old woman with infertile, receive injection of contrast material into cervix. On hysterosalpingogram (shown), contrast material (indicated by arrows) also seen in peritoneal cavity, which explain this finding?

27-year-old primigravid woman at 18 weeks gestation comes to physician for routine prenatal examination. The uterus consistent in size with 18-week gestation. Ultrasonography shows a male fetus. The collecting system and pelvis of the left kidney is dilated and the renal cortex appears compressed. The left and right ureters are not dilated. The right kidney appears normal. Amniotic fluid volume is normal. Which causing renal finding in this fetus?

Incomplete recanalization of proximal ureter

ureteropelvic junction, – LAST to canalize and can lead to congenital obstruction. 

Image: 27-year-old primigravid woman at 18 weeks gestation comes to physician for routine prenatal examination. The uterus consistent in size with 18-week gestation. Ultrasonography shows a male fetus. The collecting system and pelvis of the left kidney is dilated and the renal cortex appears compressed. The left and right ureters are not dilated. The right kidney appears normal. Amniotic fluid volume is normal. Which causing renal finding in this fetus?

25-year-old-man who has a history of facial flushing and rapid heart rate each time he consumes a small amount of alcohol participates in study of alcohol intolerance. Molecular analysis shows presence of a lysine (K487, oriental variant) for glutamate (E487, native variant) substitution in aldehyde dehydrogenase. Kinetic characteristics of the enzyme variants shown:(NAD+))E487 Km 37 KCat 180K487 Km 5600 kCat 9.5based on these findings, which of the following is the most likely cause of this man’s condition?

Decreased catalytic efficiency of K487

Catalytic efficiency is defined as Kcat/KM

21-year-old man loses 15% total blood volume 2 minute after motor vehicle collision. Finding most likely?

Increased sympathetic nerve traffic to sinoatrial node

Image: 21-year-old man loses 15% total blood volume 2 minute after motor vehicle collision. Finding most likely?

48-year-old woman with gradual onset back pain past 2 weeks. No trauma. Doesn’t smoke cigarettes, drink alcohol or use drugs. Hemogram, serologic studies and urinalysis unremarkable. X-ray of spine shows two lytic lesions, in T-10 and L-1. Dx?

Metastatic carcinoma of the breast

Prostate, Breast >Kidney, Thyroid, Lung

Workbench wiped down with alcohol, successfully inactivates viruses with which characteristic?

Enveloped virion

Alcohol-based products disrupt the lipid membrane envelope

52-year-old with chronic pancreatitis is deficient in which enzymes that causes inability to digest triglycerides?

Colipase

cofactor for pancreatic lipase,

35-year-old primigravid woman 36 weeks’ gestation with 6-hour history of heavy vaginal bleeding. No prenatal care. Ultrasound shows placenta over cervical os. Can’t stop bleeding and has cesarean. Dx?

Placenta previa

Opening of Cervical OS – preview of Placement

Image: 35-year-old primigravid woman 36 weeks' gestation with 6-hour history of heavy vaginal bleeding. No prenatal care. Ultrasound shows placenta over cervical os. Can't stop bleeding and has cesarean. Dx?

45-year-old woman with 4-month joint pain, muscles aches and fatigue. Had a renal calculus 6 months ago and had cholecystectomy 1 year ago. Appears fatigue, depressed, there is mucosal pallor. No mass45-year-old woman with 4-month joint pain, muscles aches and fatigue. Had a renal calculus 6 months ago and had cholecystectomy 1 year ago. Appears fatigue, depressed, there is mucosal pallor. 

No masses palpated in neck. 

Labs: erythrocyte count 3 million, K+ 4, Cl- 106, Ca2+ 13, phosphorous 2.7, alk phos 125. 

Technetium-99m scan shows 1.2 cm nodule in neckMutation?s palpated in neck. Labs: erythrocyte count 3 million, K+ 4, Cl- 106, Ca2+ 13, phosphorous 2.7, alk phos 125. Technetium-99m scan shows 1.2 cm nodule in neckMutation?

MEN1

Pituitary, Pancreatic, Parathyroid – 3ps = MEN 1

Parathyroid tumor releasing PTH, increasing Ca and ALP. 

Image: 45-year-old woman with 4-month joint pain, muscles aches and fatigue. Had a renal calculus 6 months ago and had cholecystectomy 1 year ago. Appears fatigue, depressed, there is mucosal pallor. No mass45-year-old woman with 4-month joint pain, muscles aches and fatigue. Had a renal calculus 6 months ago and had cholecystectomy 1 year ago. Appears fatigue, depressed, there is mucosal pallor. 
No masses palpated in neck. 
Labs: erythrocyte count 3 million, K+ 4, Cl- 106, Ca2+ 13, phosphorous 2.7, alk phos 125. 

Technetium-99m scan shows 1.2 cm nodule in neck. Mutation?s palpated in neck. Labs: erythrocyte count 3 million, K+ 4, Cl- 106, Ca2+ 13, phosphorous 2.7, alk phos 125. Technetium-99m scan shows 1.2 cm nodule in neck. Mutation?

6-year-old girl with 15-minute history of severe shortness of breath. Diagnosed with throat tumor 3 years ago. RR 32. PE nasal flaring. Laryngoscopy shows multiple raised, finger-shaped lesions from vocal cords and epiglottis. Lesions excised and shows finger-shaped fibrovascular cores lined with benign squamous epithelium. Causal virus?

Human papillomavirus, type 6

Laryngopapiloma

A 68-year-old man comes to the physician because of a 1-month history of light-headedness and tightness in his chest with exertion. He adds that the pain is worse after arguing with his wife, and the symptoms resolve with rest. He has a past history of lower gastrointestinal bleeding; evaluation at that time was negative on upper endoscopy and colonoscopy. His temperature is 37°C (98.6°F), pulse is 85/min, respirations are 15/min, and blood pressure is 110/75 mm Hg. Physical examination shows no abnormalities. His hemoglobin concentration is 8.2 g/dL, and hematocrit is 24%. Test of the stool for occult blood is positive. An ECG shows no abnormalities. Repeat colonoscopy shows no abnormalities. Which of the following is the most likely cause of this patient’s gastrointestinal symptoms?

Angiodysplasia

Hematochezia is the passage of fresh blood per anus

Tortuous dilation of vessels hematochezia

24-year-old man with 3-day progressive numbness of both feet ascended to thighs. Last 24 hours, numbness and tingling of hands. PE ataxic gait. Deep tendon reflexes diminished in upper extremities and absent in knees and ankles. Vibration and joint position absent in fingertips and feet bilaterally. Mild weakness distal upper extremities ad moderate weakness of lower extremities. Structure involved?

Myelinated primary afferents

Most common subtype of Guillain-Barré syndrome

27-year-old man for psychiatric eval sent by employer because he is “very odd.” Is a computer repair specialist and lives alone, Refuses to socialize and has no friends. Extremely preoccupied with science fiction, occult, afterlife. Personality disorder?

Schizotypal

Eccentric appearance, odd beliefs or magical thinking

Cluster A 

45-year-old man bmi 26, total cholesterol 200, HDL 50, triglycerides 550. Which drug to prescribe?

Fenofibrate

Triglycerides 550

MOA = Activates PPAR-αto induce HDL synthesis

60-year-old man for routine health examination. Has had normal blood pressure measurements. BP today 170/95mmHg. Physical examination shows no other abnormalities. Serum show hypokalemia and metabolic alkalosis. Plasma renin activity and serum aldosterone concentrations are increased. Following the administration of captopril, there is a marked increase in plasma renin activity. Which of the following is the most likely cause of the findings in this patient?

Renal artery stenosis

Decreased renal perfusion (one or both kidneys)

Increased renin

Increased angiotensin —HTN

After operation, 65-year-old patient has lung region that is under ventilated but well perfused. Which increases?

Physiologic shunt

– Mismatching of ventilation and perfusio

– insufficient ventilation to provide the oxygen needed to oxygenate the blood

Physiologic shunt

-Mismatching of ventilation and perfusion with the lung
-Results in insufficient ventilation to provide the oxygen needed to oxygenate the blood flowing through the alveolar capillaries

Image: Physiologic shunt

Physiologic dead space

portion of the tracheobronchial tree that does not participate in gas exchange

63-year-old man 2-week sensation of fullness in left upper quadrant. Has lethargy and shortness of breath. 20-lb weight loss during 3 months. PE shows pallor. Spleen tip palpated. Labs: hb 9, hct 27%, wbc 4000, serum uric acid 15. Peripheral blood smear numerous erythrocytes with abnormal shapes and sizes, nucleated erythrocytes and myelocytes. Aspiration of bone marrow dry tap. Biopsy shows thickened bony trabeculae with increased reticulum. Dx?

Myelofibrosis

“Bone marrow cries because it’s fibrosed and is a dry tap”

17-year-old boy brought by mother with concern that puberty is delayed. When mother leaves room, patient states, “I’m fine. I don’t know what’s the matter with her. She wants me to be tall like my dad.” Patient is 175 cm (5 ft 9 in) tall and weighs 70 kg (155 lb); BMI 23. Sex development is Tanner stage 4. After reassurance, Most appropriate next statement to mother

Tell me more about your concerns about your son’s height

1-year-old boy with rash for 2 weeks. 10th percentile for height and weight. PE scaly, seborrheic eruption over scalp, palms, back, diaper region and soles of feet. Generalized lymphadenopathy and hepatosplenomegaly. Xray of skull shows osteolytic lesions. EM biopsy of skin shows tennis racket-shaped bilamellar granule in cytoplasm. Immuno studies show CD1a antigen expression. Abnormal cells in patient are derived from which cell?

Dendritic cells

Langerhans cells (LC) are tissue-resident dendritic cells of the skin, and contain organelles 

Image: 1-year-old boy with rash for 2 weeks. 10th percentile for height and weight. PE scaly, seborrheic eruption over scalp, palms, back, diaper region and soles of feet. Generalized lymphadenopathy and hepatosplenomegaly. Xray of skull shows osteolytic lesions. EM biopsy of skin shows tennis racket-shaped bilamellar granule in cytoplasm. Immuno studies show CD1a antigen expression. Abnormal cells in patient are derived from which cell?

2-month-old boy well child exam, mother with no concerns. 25th percentile for length and 30th percentile for weight. Cardiac exam shows blowing holosystolic murmur best heard at lower left sternal border. Cause?

Ventricular septal defect

Blowing Holosystolic Murmur at Lower left sternal border

Study designed to test the effectiveness of a new drug in the treatment of endometriosis, 100 women randomly assigned to one of two groups48 of women receive new drug, 52 receive standard therapy. The primary purpose of this method of assigning patients to different groups is to create which of the following?

Two groups with similar underlying characteristics

A 70-year-old woman comes to the physician for a routine pelvic examination. During speculum examination of the vagina and cervix, the Valsalva maneuver causes a bulge of the anterior vaginal wall. Which of the following is the most likely cause of this finding?

Cystocele

prolapsed bladder, is a medical condition in which a woman’s bladder bulges into her vagina. Some may have no symptoms

Image: A 70-year-old woman comes to the physician for a routine pelvic examination. During speculum examination of the vagina and cervix, the Valsalva maneuver causes a bulge of the anterior vaginal wall. Which of the following is the most likely cause of this finding?

32-year-old man with HIV infection follow up examination, has been treated with HAART for the past 6 years HIV plasma viral load has been undetectable. HIV viral load now increase, antiretroviral resistance suspected. HIV genotype analysis confirms that the virus has resistance mutations, which of the following most likely mutated?

Reverse transcriptase and protease

HAART therapy = reverse transcriptase and protease enzymes.

32-year-old man with X-linked recessive disease has deafness, hematuria and progressive renal failure. Protein abnormality?

Type IV collagen

Alport Syndrome

Can’t See, Can’t pee, Can’t hear a Bee

65-year-old woman with 6 hour Hx fever and shaking chills. 4 hours ago, she took 325 mg aspirine tablets. Temp 39.4 C, pulse 96/m, respiration 18/m and blood pressure 102/60 mmHg. Physical examination shows marked tenderness bilaterally in the costovertebral areas. 

Lab studies show:

Hb 13 g/dlHt 39%

Leukocyte count 32,.000/mm3

Urine ph 6

Nitrites 4+

Leukocyte esterase 4+

A photograph of wright stained peripheral blood smear, which the most likely cause this patient leukocytosis?

Leukemoid reaction

Image: 65-year-old woman with 6 hour Hx fever and shaking chills. 4 hours ago, she took 325 mg aspirine tablets. Temp 39.4 C, pulse 96/m, respiration 18/m and blood pressure 102/60 mmHg. Physical examination shows marked tenderness bilaterally in the costovertebral areas. 
Lab studies show:
Hb 13 g/dlHt 39%
Leukocyte count 32,.000/mm3
Urine ph 6
Nitrites 4+
Leukocyte esterase 4+

A photograph of wright stained peripheral blood smear, which the most likely cause this patient leukocytosis?

25-year-old man just returned from work as worker from Africa begins oral chloroquine therapy for malaria caused by Plasmodium vivax. His initial therapeutic response is good, but he develops recurrent parasitemia 2 months later. Which of the following best explains the recurrence?

Chloroquine is ineffective on the exoerythrocytic malaria tissue stages

P vivax/ovale, add primaquine for hypnozoite

Hypnozoite = inactive dormant – only in P. Vivax and Ovale

28-year-old woman comes to the physician because of a 4-day history of palpitations, severe neck pain, fatigue, and malaise. Her pulse is 120/min and regular. Physical examination shows a diffusely tender, mildly enlarged thyroid gland. There is no exophthalmos. Serum studies show a thyroid-stimulating hormone concentration of 0.01 μU/mL. Which of the following is the most likely diagnosis?

Subacute granulomatous thyroiditis

de quervain’s = pain

68-year-old man with a 10-month history of shortness of breath and swelling of his feet, family history of cardiovascular disease. He smoked 2 packs of cigarettes daily for 50 years. Pulse 80/min, rr 24/min, BP 150/80. PE: 3+ pitting edema of lower extremities. Diffuse, scattered wheezes are heard bilaterally on auscultation of the chest. Grade 2/6 pansystolic mumur heard best at lower left sternal border, which increases on inspiration. Maximal impulse palpated in sub-xiphoid area. S1 and S2 sounds are distant. Liver span 14 cm. Diagnosis?

Cor pulmonale

Backfilling of blood from the lungs into the R ventricle is stretching out the R side (dilation) and also remodeling the heart via hypertrophy

cor pulmonale

right ventricular hypertrophy and heart failure due to pulmonary hypertension

Image: cor pulmonale

17-year-old boy in septic shock unresponsive to ADH (vasopressin). Treat is discontinued, and high-dose dopamine in started. Which receptors are stimulated?

alpha1-adrenoreceptors

low doses d1 = dilates renal vessels

medium b1 agonist = ionotropic and chonotropic

high doses a1 agonist = vasoconstriction

8-year-old girl is brought to the physician by her mother because of a 3-week history of poor feeding chronic diarrhea and pale foul-smelling stools. Mother says the symptoms began with colicky abdominal pain following introduction of solid food to the infant’s diet. History of gluten sensitivity. 60th percentile for length and 25th percentile for weight. Which cell is dysfunctional?

Enterocyte

Celiac disease, an autoimmune intolerance to gliadin leading to enterocyte damage and subsequent malabsorption

25-year-old woman comes for counseling prior to conception. Has seizure disorder on valproic acid. Fetus at greatest risk for drug-related adverse effect during which pregnancy stage?

3 to 8 weeks

Neural tube defects Neuropores fail to fuse (4th week)

16-year-old boy who frequently thinks about sex, daydreams about girls before going to sleep, and masturbates one to two times daily. After counseling about safe sex, best next step?

Schedule next routine examination

Normal behavior for a teen boy

35-year-old woman with 3-month progressive shortness of breath with exertion. RR 26. Physical shows jvd and prominent a wave. Lungs clear. Cardiac exam shows loud pulmonic component of S2 and right-sided S4 gallop. Increased pulmonary expression of what?

Endothelin-1

pulmonary hypertension = imbalance between vasodilation substances and vasoconstrictors.

7-year-old boy is about undergo an appendectomy. An intravenous catheter needs to be inserted, but the patient is fearful of being stuck with needle. The most appropriate anesthesia administered by mask to anesthetize this patient quickly would have which of the following characteristics?

Low blood solubility

–> faster onset –> faster recovery

Lipid solubility determines potency

Image: 7-year-old boy is about undergo an appendectomy. An intravenous catheter needs to be inserted, but the patient is fearful of being stuck with needle. The most appropriate anesthesia administered by mask to anesthetize this patient quickly would have which of the following characteristics?

66-year-old man dies 7 days after myocardial infarction. Gross of heart shown (perforated interventricular wall). Histology?

Erythrocytes, cellular debris, macrophages, and early granulation tissue

0-24 hours Dark Mottling

3-14 days – Macrophages, then granulation tissue at margins

Image: 66-year-old man dies 7 days after myocardial infarction. Gross of heart shown (perforated interventricular wall). Histology?

68-year-old man with creatinine 2.3 due to chronically increased hydrostatic pressure in Bowman space. Cause?

Benign prostatic hyperplasia

sexually active 32-year-old woman has vaginal pain with urination. Pelvic examination show bilateral vesicoulcerative lesions of introitus. Tx?

Acyclovir

inhibit viral DNA polymerase by chain termination

26-year-old man comes in 2 hours after injuring arm skiing. Sensation to pinprick absent over lateral aspect of shoulder. Xray of right shoulder shown (fracture of surgical neck of humerus). Nerve damaged?

Axillary

fracture of the surgical neck of the humerus.

axillary nerve and the posterior circumflex humeral artery 

Radial does lateral elbow

musculocutaneous does lateral forearm

Image: 26-year-old man comes in 2 hours after injuring arm skiing. Sensation to pinprick absent over lateral aspect of shoulder. Xray of right shoulder shown (fracture of surgical neck of humerus). Nerve damaged?

Patient making sexual advances towards physician. Appropriate measure?

Have a chaperone join them for the remainder of the examination

Inappropriate behavior – sexual advances towards physician

A 70-year-old man comes with skin blistering for 1 week. No oral lesions. Physical shows tense bullae in joint folds of upper and lower extremities. Biopsy shows subepidermal blister formation. Immunofluorescence microscopy shows antibodies against proteins at the dermal-epidermal junction. Target by antibodies?

Hemidesmosome

Subepidermal blister formation + Tense Bullae

An experimental animal is created that has a defect in an innate gastrointestinal defense mechanism. Organism is found to have decreased HCl prod. After 2 months on biopsy gastric fundus and body show decreased mucosal thickness and hyperplasia of enterochromaffin like cells. This resembles?

Chronic Gastritis

mucosal inflammation leading to atrophy

Image: An experimental animal is created that has a defect in an innate gastrointestinal defense mechanism. Organism is found to have decreased HCl prod. After 2 months on biopsy gastric fundus and body show decreased mucosal thickness and hyperplasia of enterochromaffin like cells. This resembles?

56-year-old man comes with vague right-sided flank pain and 7-lb weight loss over past 6 weeks. No recent trauma or back strain. Urine is darker than usual over 2 weeks. Renal ultrasound shows 1.5-cm solid mass in upper portion of right kidney. Urine shows 50 erythrocytes, 6 leukocytes, no bacteria. Dx?

Renal Cell Carcinoma

hematuria, palpable masses, 2° polycythemia, flank pain, fever, weight loss

67-year-old woman with atrial fibrillation with sudden onset severe abdominal pain. Ex-lap shows embolus in superior mesenteric artery with complete occlusion of middle colic arteryIschemic changes where?

Small intestine, ascending colon, and part of the transverse colon

Midgut = major duodenal papilla (of the duodenum) to the proximal 2/3 of the transverse colon.

Image: 67-year-old woman with atrial fibrillation with sudden onset severe abdominal pain. Ex-lap shows embolus in superior mesenteric artery with complete occlusion of middle colic artery. Ischemic changes where?

Persons are at risk of influenza are vaccination every year because of antigenic variation, which can be drift (minor) or shift (major). Mechanism of antigenic shift?

Reassortment

viruses with segmented genomes (eg, influenza virus) exchange genetic material.

2 viruses making a combo virus – 2 genomes

Image: Persons are at risk of influenza are vaccination every year because of antigenic variation, which can be drift (minor) or shift (major). Mechanism of antigenic shift?

34-year-old man with a 3-month history of a progressive rash on his feet. Rash is not itchy or painful. A photograph of feet is shown = Kaposi Sarcoma. HIV+. In addition to HAART, which of the following pharmacotherapy is most appropriate?

Antineoplastic

Endothelial malignancy most commonly affecting the skin, mouth, GI tract, respiratory tract.

Associated with HHV-8 and HIV

Kaposi typically presents in HIV pts earlier, under a 500 CD4 tcell count,

bartonella angiomatosis presents <100

Image: 34-year-old man with a 3-month history of a progressive rash on his feet. Rash is not itchy or painful. A photograph of feet is shown = Kaposi Sarcoma. HIV+. In addition to HAART, which of the following pharmacotherapy is most appropriate?

70-year-old man with early morning awakening, decreased energy, difficulty concentrating, anhedonia, psychomotor retardation, depressed mood for 3 months. He had myocardial infarction and non sustained ventricular tachycardia 2 years ago. Tx?

Paroxetine

SSRIs are first line for depression.

Avoid TCAs in elderly = anticholinergic side effects.

29-year-old woman comes to the physician because of irregular menstrual periods since menarche at the age of 12. She is 5.3 Ft and weighs 86kgs. BMI 34. She is evaluated and a diagnosis of PCOS is made. After explaining the diagnosis, the physician discusses behavioral changes, including dietary modification and exercise as part of her treatment. Which of the following will ensure adherence?

Provide follow ups to monitor progress in attaining her goal

Improve Doctor patient relationship – add next appointment

52 yo man is brought to er 30min after the onset of chest pain and shortness of breath. He had played tennis all day and he does not remember how much fluid he had consumed. His temperature is 36.7 oC, pulse 122min, respirations 28min and BP 90/50 mmHg. PE shows dry skin and decreased capillary refill. An ECG and evaluation of cardiac enzymes show no abnormalities. Which of the following findings in the nephron best describes the tubular osmolarity, compared with serum in this patient?

PCT Isotonic,

Macula Densa hypotonic

Medullary Collecting Duct hypertonic

Image: 52 yo man is brought to er 30min after the onset of chest pain and shortness of breath. He had played tennis all day and he does not remember how much fluid he had consumed. His temperature is 36.7 oC, pulse 122min, respirations 28min and BP 90/50 mmHg. PE shows dry skin and decreased capillary refill. An ECG and evaluation of cardiac enzymes show no abnormalities. Which of the following findings in the nephron best describes the tubular osmolarity, compared with serum in this patient?

3-week-old male newborn with recurrent vomiting after feeding since birth, and eager to feed after vomiting. Abdominal exam shows firm, mobile mass in epigastrium to right of midline. Dx description?

Single Primary Development Defect

pyloric stenosis – olive shaped mass – stand-alone defect

VACTERL

vertebral defects, heart defects, renal and limb abnormalities

Image: VACTERL

65-year-old with sudden onset generalized tonic-clonic seizures. Personality change last 6 months; used to be mild mannered and now verbally abusive. CT shows single mass enhances with contrast in right frontal lobe and crosses to left hemisphere through corpus callosum. Dx?

High-grade fibrillary astrocytoma

Glioblastoma multiforme- “crosses hemisphere”

GFAP +

Image: 65-year-old with sudden onset generalized tonic-clonic seizures. Personality change last 6 months; used to be mild mannered and now verbally abusive. CT shows single mass enhances with contrast in right frontal lobe and crosses to left hemisphere through corpus callosum. Dx?

A new antiplatelet agent is developed for the prevention of recurrence of stroke. In a large randomized clinical trial with equal numbers of men and women, the rates of stroke are lower in patients receiving the new agent than in patients receiving the standard treatment. Results are shown:Recurrent Stroke Rates per 1000 Person-YearsStandard Treatment vs. New Antiplatelet Drug

Women .12 .04

Men .24 .08

Overall .18. .06

Based on these results, which of the following is the relative risk reduction in women?

67%

.12-.04= .08

.18-.06 = .12

.08/.12 = 67%

22-year-old man who is a professional cyclist undergoes extensive physiologic testing as part of his training regimen. His resting pulse is 33/min, and blood pressure is110/62 mm Hg. Echocardiography shows dilated ventricles with normal function and a left ventricular ejection fraction of 75%. Which of the following best describes the findings in this patient?

Eccentric Hypertrophy

ECCentric – ECCpands ventricle

CONcentric – CONstricts ventricle

68-year-old man with alcoholism comes for fever, chills and productive cough of purulent sputum for 3 days. Blood cultures positive. Gram stain: gram-positive, lancet-shaped diplococciVaccine is against which bacterial component?

Capsular polysaccharide

Strep pneumonia

Child with XXY karyotype, genetic studies showed he received the extra “x” from his father. An error of chromosome segregation occurred during anaphase at which of the following stages of spermatogenesis in the patient’s father?

Primary Spermatocyte

Most problems with cell division occur in anaphase of Meiosis I.

Meiosis 1⇒ from primary to secondary spermatocyte Meiosis 2 → Secondary to spermatid’

Image: Child with XXY karyotype, genetic studies showed he received the extra "x" from his father. An error of chromosome segregation occurred during anaphase at which of the following stages of spermatogenesis in the patient's father?

28-year-old man with ulcerative colitis, history of partial colectomy, and improved rectal bleeding and diarrhea since then. Meds include mesalamine, hydrocodone and acetaminophen. He says, “I’ve been taking so much of my opiate prescription that I ran out and stole some from my job.” Next step?

Discuss treatment options and a referral for detoxification program

67-year-old man with 1 year progressive difficulty writing and walking. Pt is stooped and talks slowly. PE shows bland facial expression, fine resting tremor in both hands, no tremor when moves, walks with difficulty starting and stopping, cogwheel rigidity. Brain tissue histology shown as well as gross cross sections of midbrain both normal and diseased. Substance referred to by arrow. Which Substance?

Alpha-synuclein

Image: 67-year-old man with 1 year progressive difficulty writing and walking. Pt is stooped and talks slowly. PE shows bland facial expression, fine resting tremor in both hands, no tremor when moves, walks with difficulty starting and stopping, cogwheel rigidity. Brain tissue histology shown as well as gross cross sections of midbrain both normal and diseased. Substance referred to by arrow. Which Substance?

8-year-old boy with pain on back and head since he fell off swing. Tender 2.5cm swollen mass over right occiput. CT shows osteolytic and soft-tissue mass in skull with inward displacement of dura. Biopsy shows sheet-like infiltrate of pale eosinophilic cells with bean-shaped nuclei. Cells positive for CD1a. Electron microscopy of cells shows Birbeck granules. Abnormal cell type?

Langerhans cells

Image: 8-year-old boy with pain on back and head since he fell off swing. Tender 2.5cm swollen mass over right occiput. CT shows osteolytic and soft-tissue mass in skull with inward displacement of dura. Biopsy shows sheet-like infiltrate of pale eosinophilic cells with bean-shaped nuclei. Cells positive for CD1a. Electron microscopy of cells shows Birbeck granules. Abnormal cell type?

30-year-old man in bicycle collision and hits right shoulder forcefully. Unable to flex right elbow with decreased sensation to pinprick over right lateral forearm. Brachial plexus lesion?

E (musculocutaneous nerve)

Image: 30-year-old man in bicycle collision and hits right shoulder forcefully. Unable to flex right elbow with decreased sensation to pinprick over right lateral forearm. Brachial plexus lesion?

8-year-old man comes with cracked lips and peeling sunburned skin. Works as lifeguard. PE shows desquamation of sunburned skin. Lips are dry and cracked. Petrolatum to lips may reduce lip symptoms by which of following effects of the compound?

Barrier

During a clinical study of calcium and phosphorus metabolism, a 50-year-old man undergoes series of lab studies. His serum Ca, PO4 and PTH are normal. He is given infusion 2 g Calcium chloride over 2 hours. His serum Ca concentration now is 11.5mg/dl. Compared with pre infusion levels, the serum concentration of which of the following substances is likely to be increased at this time?

24,25-Dihydroxycholecalciferol

inactive metabolite preferentially synthesized when normal 

Image: During a clinical study of calcium and phosphorus metabolism, a 50-year-old man undergoes series of lab studies. His serum Ca, PO4 and PTH are normal. He is given infusion 2 g Calcium chloride over 2 hours. His serum Ca concentration now is 11.5mg/dl. Compared with pre infusion levels, the serum concentration of which of the following substances is likely to be increased at this time?

30. 35-year-old woman with 2-day history of blistering lesions on sun-exposed face, arms and handsRecurrent episodes of skin lesions over several years. Taking oral contraceptives for 15y. PE shows fluid-filled vesicles and bullae. Labs: AST increased, ALT increased, total porphyrin increased, urine uroporphyrin III increased. Precursor to uroporphyrin?

Succinyl CoA

Image: 30. 35-year-old woman with 2-day history of blistering lesions on sun-exposed face, arms and hands. Recurrent episodes of skin lesions over several years. Taking oral contraceptives for 15y. PE shows fluid-filled vesicles and bullae. Labs: AST increased, ALT increased, total porphyrin increased, urine uroporphyrin III increased. Precursor to uroporphyrin?

32-year-old woman brought in after 10-foot fall from ladder. PE suggests severe hemorrhage and shock. Xray shows fracture of left ninth and tenth ribs. Organ injured?

Spleen

4-day-old boy with vomiting after breastfeeding. PE shows lethargy and dry mucous membranes. Labs: Na 139, Cl 90, K 7, HCO3 17, Glucose 42, BUN 25, Cr 0.4, 17-hydroxyprogesterone increased. Enzyme deficiency?

21-hydroxylase

infancy (salt wasting) or childhood(precocious puberty

Image: 4-day-old boy with vomiting after breastfeeding. PE shows lethargy and dry mucous membranes. Labs: Na 139, Cl 90, K 7, HCO3 17, Glucose 42, BUN 25, Cr 0.4, 17-hydroxyprogesterone increased. Enzyme deficiency?

70-year-old woman comes to the physician because of increasingly severe pain in the right knee over the past 3 months. She has fallen repeatedly. She has a 10-year history of symptomatic osteitis deformans. She has bowed tibia and tenderness of the proximal right tibia. an X ray shows a fracture of the proximal tibia with elevated periosteum and sunburst pattern. an x-ray of lungs shows pulmonary nodules of various sizes. which of the following findings is most likely on biopsy?

Pleomorphic neoplastic cells producing new woven bone

Osteosarcoma histological appearance. 

Paget disease is one of risk factor

Image: 70-year-old woman comes to the physician because of increasingly severe pain in the right knee over the past 3 months. She has fallen repeatedly. She has a 10-year history of symptomatic osteitis deformans. She has bowed tibia and tenderness of the proximal right tibia. an X ray shows a fracture of the proximal tibia with elevated periosteum and sunburst pattern. an x-ray of lungs shows pulmonary nodules of various sizes. which of the following findings is most likely on biopsy?

Cholera toxin catalyzes transfer of ADP-ribose to an arginine residue in stimulatory G protein resulting in inhibition of GTPase activity. Which increases in cells as a result?

Concentration of cAMP

Image: Cholera toxin catalyzes transfer of ADP-ribose to an arginine residue in stimulatory G protein resulting in inhibition of GTPase activity. Which increases in cells as a result?

10-year-old boy with 4 days nosebleeds and easy bruising. Three weeks ago, upper respiratory tract infection. PE ecchymoses on upper and lower extremities. Labs shows hemoglobin and leukocyte count normal, and decreased platelet count. Bone marrow smear shows increased megakaryocytes. Mechanism of dz?

Antibodies directed against glycoprotein (Gp) IIb/IIIa complex

Immune thrombocytopenia

60-year-old man 1 day of fever, chills, confusion and memory loss. Returned from Gulf coast where he walked barefoot. Hx of severe cirrhosis and portal hypertension. T 39C (102.2 F), RR 22, bp 90/48. Physical shows early blister formation on right lower extremity. Blood culture: gram-negative, lactose-fermenting organism. Bug?

Vibrio vulnificus

Image: 60-year-old man 1 day of fever, chills, confusion and memory loss. Returned from Gulf coast where he walked barefoot. Hx of severe cirrhosis and portal hypertension. T 39C (102.2 F), RR 22, bp 90/48. Physical shows early blister formation on right lower extremity. Blood culture: gram-negative, lactose-fermenting organism. Bug?

45-year-old man with hypertension not compliant with medications. bp 160/100. Cardiac exam shows apical impulse displaced laterally, loud S2 and S4 gallop. Echo shows thickening of left ventricular wall. Mechanism of change in cardiac muscle?

Transcription factor c-Jun: increased

beta-myosin heave chain: increased

endothelin: increased

Newborn is found to have a cervical rib. Transformation of seventh cervical segment to thoracic identity. Which is true of HOX gene alteration?

Expression of a HOX gene normally expressed only caudal to C7

Image: Newborn is found to have a cervical rib. Transformation of seventh cervical segment to thoracic identity. Which is true of HOX gene alteration?

30-year-old woman with a long-standing history of pelvic inflammatory disease has surgical resection of a scarred segment of a fallopian tube. Which of the following inflammatory cells is most likely to be found in the resected specimen?

Macrophages

39-year-old man with 1-week of red spots on shins, joint pain and fatigue. PE shows purpura over lower extremities. Liver palpated 4 cm below costal margin. Labs: WBC 10,000, AST 142, ALT 154, hepatitis C virus RNA positive, anti-hepatitis C virus antibody positive, cryoglobulins positive, C4 120 (N=350-600), urine protein 4+, urine RBC numerousHypersensitivity reaction?

Type III (immune complex-mediated)

25-year-old woman with polycystic kidneys and 3-month history of weakness, fatigue, headaches, hypertension, loss of appetite and itching. Cr 4. Labs

HCO3- decreased – no resorption

PO4 increased,

PTH increased

Image: 25-year-old woman with polycystic kidneys and 3-month history of weakness, fatigue, headaches, hypertension, loss of appetite and itching. Cr 4. Labs

5-year-old boy with motion sickness. Planning vacation to Australia, and wants diphenhydramine for motion sickness. Mechanism of action for motion sickness?

Antagonist at muscarinic-3 (M3) receptors

63-year-old woman with 2-week history of daily episodes of severe, lancinating, left-sided facial pain. Pain lasts 30 to 60 seconds and shoots down ear along jawline. Precipitated by chewing or brushing teeth. Dx?

Trigeminal neuralgia

Image: 63-year-old woman with 2-week history of daily episodes of severe, lancinating, left-sided facial pain. Pain lasts 30 to 60 seconds and shoots down ear along jawline. Precipitated by chewing or brushing teeth. Dx?

45-year-old homeless man found unconscious. Breath smells of alcohol. Vitals stable. PE shows bronzed skin and spider angiomata on chest. Labs: hemoglobin 10, hematocrit 30%, MCV 110, WBC 9000, platelets 160,000, ferritin 200, b12 500, folate 20. Blood smear shows hypersegmented neutrophils and 3+ oval macrocytes. Labs?

Methylmalonic acid: normal

Homocysteine: increased

55-year-old man with elevated LDL cholesterol is prescribed lovastatin. This treatment would result in which adaptive responses at the cellular level?

Increased transcription of HMG-CoA reductase

Image: 55-year-old man with elevated LDL cholesterol is prescribed lovastatin. This treatment would result in which adaptive responses at the cellular level?

63 yo man with mild emphysema has smoked 1 pack for 45 years sudden headache, myalgia and rising temperature. A dry cough develops, with chill and chest pain. Examination of gram stain of sputum disclose neutrophils, but bacteria are not evident. An X-ray of the chest is consistent with severe pneumonia. A culture of sputum is negative, but culture of bronchoscopy specimen on a highly specialized bacteriologic medium yields gram negative rods. The identity of these bacteria is confirmed by staining with specific fluorescent antibodies. The patient respond therapy with macrolide Ab. Mechanism?

Inhalation of aerosol from an environmental source

(Legionella) – Aerosol transmission 

50-year-old woman with COPD comes with 3 months of progressive shortness of breath. Physical shows JVD, loud pulmonary component of S2. Pulmonary function tests show FEV1:FVC ratio of 20% and decreased diffusing capacity for carbon monoxide. Which is decreased in pulmonary vascular smooth muscle?

Endothelial nitric oxide synthase production

63-year-old man with abnormal blood pressure. 6 months ago, bp was 135/85. Today bp 170/98. Bruit heard over left renal artery. CT angios hows left arterial stenosis. Labs?

Total peripheral resistance increased,

Plasma renin activity increased,

Serum aldosterone concentration increased

renal artery stenosis.

20-year-old woman at 27 weeks’ gestation is admitted to the hospital because of a 12-hour history of intense uterine contractions occurring every 8 minutes. Her membranes ruptured 32 hours ago. Her temperature is 39.1°C(102.4°F), and pulse is 115/min. Physical examination shows tenderness of the uterus. Pelvic examination shows a closed cervix that is not effaced. The fetal heart rate is 210/min. Which of the following is the primary stimulus for her uterine muscle contractions?

Inflamed maternal decidua release of prostaglandin

Preterm Premature Rupture Of Membrane

30-year-old woman with 2-years of numbness, blanching, and bluish color to ears, fingers and toes after emotional upset or cold exposure. Vitals and PE normal. Avoid taking which drug?

Phenylephrine

18-month-old girl with 2 day progressive cough and hoarseness. T 102.2F, HR 88, RR 24, bp 100/70. Oxygen saturation 95%. PE mild erythema of oropharyngeal and laryngeal mucosa no exudate, harsh, barking cough heard. Improves within 4 days. Cause?

Parainfluenza

Image: 18-month-old girl with 2 day progressive cough and hoarseness. T 102.2F, HR 88, RR 24, bp 100/70. Oxygen saturation 95%. PE mild erythema of oropharyngeal and laryngeal mucosa no exudate, harsh, barking cough heard. Improves within 4 days. Cause?

Patient with orthostatic hypotension, loose stools for 1 year, and history of type 1 DM. Stool studies are normal. What is the pathophysiological mechanism of the diarrhea?

Motility disorder

39-year-old man with polycystic kidney disease has 6-month history intermittent blood in urine. T 98.6 F HR 100 RR 24 BP 160/90. Urea nitrogen 100, creatinine 8. UA shows blood. Arterial blood gas shows?

pH 7.22

pCO2. 28

HCO3- 11

Uremia: part of MUDPILES

Bad kidneys = Can’t reabsorb HCO3 “base” > 

Metabolic Acidic with respiratory compensation

Image: 39-year-old man with polycystic kidney disease has 6-month history intermittent blood in urine. T 98.6 F HR 100 RR 24 BP 160/90. Urea nitrogen 100, creatinine 8. UA shows blood. Arterial blood gas shows?

20-year-old woman with 2-week fever, shaking chills, headaches, fatigue, and joint and muscle pain. Spent summer workin as lifeguard in Long Island, New York. Has splenectomy for motor vehicle collision 6 years ago. T 102.4 F. Peripheral blood smear shows small intraerythrocytic rings. Polymerase chain reaction for Plasmodium is negative. Causal organism from bite of?

Tick

Male newborn delivered at 28 weeks, neonatal respiratory distress syndrome, ABG shows decreased pH, decreased Po2 increased PCO2. A deficiency in which of the following most likely caused the disorder?

Dipalmitoylphosphatidylcholine

55-year-old woman who is a physician has fractured femur during motor vehicle collision. 2 days after admission, has tachycardia, restlessness, diaphoresis, anxiety, seeing “vague shapes” on walls. Dx?

Alcohol withdrawal

Full-term newborn in respiratory distress. Imaging shows abdominal contents in left pleural cavity. Maldevelopment of which structure led to diaphragm defect?

Left pleuroperitoneal membrane

Diaphragmatic hernia

To decrease risk for cv disease 24 yo man begins diet. 1.53 95 kg BMI 32, Intends to lose 16 kg by limiting caloric intake to 2000 cal. to maintain the recommended protein intake (56g day); a balanced decreased in carbs and fat is required (caloric radio of fat and carbs is 30:55). which best describes number of calories that should be provided by fat in this its diet each day?

630

0.3 fat x 2000 = 600 calories (fat)

(Fat 30 + Carbs 55+15 = 100)

35-year-old woman passed out while washing dishes. Began to feel weak and dizzy, husband shares with nursing staff history of bulimia nervosa, and has been binging more frequently. Suspected she has been vomiting. Labs of patient?

K+ decreased

HCO3- increased/

Anion Gap normal

pH increased’

Metabolic Alkalosis > loss of HCL

Anion Gap Normal = NOT a MUDPILE

HCL, NA+ = out the mouth

Retrospective study of incidence and outcome of subarachnoid hemorrhage. Incidence is 15 per 100,000 people, mortality is 6 per 100,000 people. Incidence remains constant, case fatality rate?

40%

16-year-old girl calls the physician on a Friday night 2 hours after a condom broke during sexual intercourse with her boyfriend. She asks the physician to prescribe an emergency oral contraceptive. The physician on call is not the patient’s regular physician and does not dispense emergency contraception for moral reasons. After the physician respectfully informs the patient that he does not prescribe this contraceptive, it is most appropriate for the physician to state which of the following?

“I can have one of my colleagues call you back to further discuss your concerns”

58-year-old man comes to the physician because of a 4-year history of recurrent cough productive of increased sputum. Use of over-the-counter cough suppressants has not resolved his symptoms. He has smoked 2 packs of cigarettes daily for 35 years. He has no family history of lung disease. His temperature is 37°C (98.6°F), pulse is 72/min, and respirations are 18/min. Physical examination shows cyanosis. Diffuse wheezing is heard on auscultation. Which of the following pulmonary cell types is most likely to be abnormal in this patient?

Pseudostratified columnar epithelial cells

Smoker, Cyanosis – chronic bronchitis

squamous metaplasia

64-year-old with non-Hodgkin lymphoma and 3-day history of abdominal pain and nausea. T 99.7F, HR 100, bp 130/80. Abdominal exam tenderness of flanks and lower quadrants. BUN 34 and creatinine 3.8. CT shows bilateral hydronephrosis and lymphadenopathy compressing uretersTx to improve renal function?

Bilateral stents in the ureters

34-year old man is brought to the ER semiconscious and combative. In addition to sedation, a short-acting neuromuscular blocking agent is administered for intubation to prevent aspiration. Within a few seconds after admin of the drug, he has transient muscle fasciculations in his face; he develops generalized paralysis within 1 minute. Forty five minutes after completion of the procedure, he is still paralyzed. A genetic abnormality of which of the following enzymes is the most likely cause of his unusually slow recovery from paralysis?

Pseudocholinesterase

Succinycholine – Acetylcholine Agonist

increased in system –> longer anesthesia recovery

Gy67-year-old man who eight months ago dx with primary lung carcinoma involving adrenal glands, liver, and bone. Had 17.6-lb weight loss during 3 months. PE shows cachexia and significant muscle wasting. Intracellular components increased in patient’s muscle cells?

Autophagic vacuoles

48-year-old man 2-month increasing abdominal girth and inability to achieve an erection. Smoked 1 pack cigarettes for 20 years and drank 1 pint of liquor daily. Vitals normal. PE shows scleral icterus, spider angiomata, gynecomastia, ascites, and prominent umbilical venous pattern. Testis small. Cause of gynecomastia?

Failure of liver to degrade estrogen

Alcoholic – cirrhosis – Liver injury

Investigator studying vancomycin-resistant strain of Enterococcus faecalis. Ten generations created. Culture inoculated and resulting bacterial colonies are screened for vancomycin resistance. Vancomycin-sensitive colonies observed at frequency of one per 100 cells. Mechanism of decreased vancomycin resistance?

Plasmid loss

20-year-old woman with palpable lump in right breast 4 months, no pain, swelling or nipple discharge. Lump is smooth, firm, round, mobile, nontender, well delineatedNo skin change. Dx?

Fibroadenoma

Researching new cancer drug, effective at killing rapidly dividing cells, in mice caused profound myelosuppression. 

In patients, most appropriate to follow which when at risk for infectious complications?

Neutrophil counts

35-year-old woman with fever and sharp chest pain for 3 days. T 101.3F. 

Friction rub heard. All causes of secondary pericarditis ruled out. Cause of primary pericarditis?

Virus

65-year-old woman comes to the physician because of a 1-week history of a swollen, painful left knee. Her temperature is 38°C (100.4°F), and blood pressure is 110/65 mm Hg. Examination of the left knee shows erythema and swelling of the joint and decreased range of motion. A photomicrograph of synovial fluid obtained by arthrocentesis is shown. This patient’s synovial fluid most likely contains which of the following?

Uric acid

needle shape right in the middle was a uric acid crystal

Image: 65-year-old woman comes to the physician because of a 1-week history of a swollen, painful left knee. Her temperature is 38°C (100.4°F), and blood pressure is 110/65 mm Hg. Examination of the left knee shows erythema and swelling of the joint and decreased range of motion. A photomicrograph of synovial fluid obtained by arthrocentesis is shown. This patient's synovial fluid most likely contains which of the following?

17-year-old boy brought by mother with concern that puberty is delayed. When mother leaves room, patient states, “I’m fine. I don’t know what’s the matter with her. She wants me to be tall like my dad.” Patient is 175 cm (5 ft 9 in) tall and weighs 70 kg (155 lb); BMI 23. Sex development is Tanner stage 4. After reassuring mom, Most appropriate next statement to mother?

“Tell me more about your concerns about your son’s height.”

\19-year-old man with gastrointestinal bleeding. Laparotomy done and 5-cm blind outpouching on antimesenteric side of terminal ileum 15 cm from ileocecal valve resected. 99mTc-pertechnetate scan Pathology shows?

Heterotopic gastric mucosa

99mTc-pertechnetate scan = Meckel Diverticula

Image: \19-year-old man with gastrointestinal bleeding. Laparotomy done and 5-cm blind outpouching on antimesenteric side of terminal ileum 15 cm from ileocecal valve resected. 99mTc-pertechnetate scan Pathology shows?

19-year-old woman is admitted to the hospital for antibiotic treatment of meningococcal meningitis. She is stabilized. Three days later, her pulse is 120/min, and blood pressure is 60/30 mm Hg. Physical examination shows bilateral flank tenderness. Serum studies show a sodium concentration of 128 mEq/L, potassium of 5.4 mEq/L, and bicarbonate of 20 mEq/L. Which of the following is the most appropriate next step to determine the cause of this patient’s hypotension?

Adrenocorticotropic hormone stimulation test

25-year-old woman with fatigue for 3 wks and intermittent fever for 7 days, had teeth cleaned a month ago, no abx for prophylaxis, and had rheumatic fever as child and endocarditis 4 y ago. PE shows 2/6 murmur, ultrasound shows abnormal mitral valve. Photo of growth from blood cultures shown (GP cocci in chains). Characteristic of causal organism?

]Greening reaction on blood agar

alpha hemolysis (green)

Image: 25-year-old woman with fatigue for 3 wks and intermittent fever for 7 days, had teeth cleaned a month ago, no abx for prophylaxis, and had rheumatic fever as child and endocarditis 4 y ago. PE shows 2/6 murmur, ultrasound shows abnormal mitral valve. Photo of growth from blood cultures shown (GP cocci in chains). Characteristic of causal organism?

83-year-old with arteriosclerosis undergoes repair of infrarenal abdominal aortic aneurysm. Graft extends just below the renal arteries to the bifurcation of the aorta. Which organ will lose primary blood supply and rely on collateral circulation?

Descending colon

Image: 83-year-old with arteriosclerosis undergoes repair of infrarenal abdominal aortic aneurysm. Graft extends just below the renal arteries to the bifurcation of the aorta. Which organ will lose primary blood supply and rely on collateral circulation?

In a 25-year-old patient who underwent splenectomy, target cells are seen on peripheral smear. These cells are seen due to loss of function of which portion of spleen?

Red pulp

Red pulp

the region of the spleen composed of blood-filled venous sinuses and thin plates of splenic tissue (cords), which consist of red blood cells and various white blood cells

Filter the blood of antigens, microorganisms, and defective or worn-out red blood cell

56-year-old exposed to possible chemical attack. Respirations labored, diaphoresis, excessive lacrimation, increased salivation, muscle strength 2/5, urinary and fecal incontinence. Besides atropine, another tx?

Pralidoxime

Studying epithelial repair of small intestine in experimental animal. Wants to identify most active cell division location. Where is this cell activity found?

Base of the crypt

Image: Studying epithelial repair of small intestine in experimental animal. Wants to identify most active cell division location. Where is this cell activity found?

48-year-old recently diagnosed with lupus and difficulty working as schoolteacher because of severe pain and swelling of joints. Current meds include prednisone, hydroxychloroquine, and oxycodone for pain. Best action to encourage patient to adapt to illness?

Encourage the patient to participate in a support group for persons with her condition

A 25-year-old man is admitted to the hospital because of severe crush injuries to the chest and extensive burns over 30% of his body surface area. Three hours later, he develops tachypnea and dyspnea. Arterial blood gas analysis on room air shows a decreased Po2 and Pco2. A chest x-ray shows bilateral interstitial and alveolar infiltrates. The patient is intubated and mechanically ventilated. Damage to what will preclude restoration?

Basement membranes

12-year-old boy with mother concerned about enlarged left breast and family history of breast cancer. Left breast is slightly larger than right, nipple mildly tender. Penis slightly enlarged and pubic hair curling and beginning to darken at base. Best next step?

Reassure the mother that physical findings are not uncommon for his age

17-year-old boy with syncopal episode, fever, nausea, muscle aches, progressive confusion. T 103.5F, palpable bp 80. PE rash on lower extremities. WBC 26,000. Blood and CSF cultures grow oxidase positive, gram negative diplococcus. Brother with similar infection at same age. Immune disorder?

Late component of complement deficiency

75-year-old man comes to the physician because of an enlarging face shoulders and trunk and thinning of his arms and legs. Physical examination shows a large plethoric face, fat pad over the upper thoracic spine and purple striae on the abdomen. Serum studies show undetectable ACTH and an increased cortisol concentration. Administration of low dose dexamethasone would most likely result in which of the following sets of serum findings?

ACTH no change

cortisol no change

Image: 75-year-old man comes to the physician because of an enlarging face shoulders and trunk and thinning of his arms and legs. Physical examination shows a large plethoric face, fat pad over the upper thoracic spine and purple striae on the abdomen. Serum studies show undetectable ACTH and an increased cortisol concentration. Administration of low dose dexamethasone would most likely result in which of the following sets of serum findings?

71-year old woman with coronary disease and well-controlled hypertension is brought to the physician 2 hours after sudden onset of weakness of her left leg. Her BP 145/85 mmHg. Neurologic exam: weakness and decreased sensation over the left lower extremity. There are no other sensory or motor deficits. Which labeled structure is site of injury?

A (Pre/postcentral gyrus; motor/sensory cortex)

Image: 71-year old woman with coronary disease and well-controlled hypertension is brought to the physician 2 hours after sudden onset of weakness of her left leg. Her BP 145/85 mmHg. Neurologic exam: weakness and decreased sensation over the left lower extremity. There are no other sensory or motor deficits. Which labeled structure is site of injury?

71-year-old man admitted for prostatic resection. Normal mental status. 2 days after surgeryconfused and restless. Cannot sleep and restless, seeing little men coming through window. Dx?

Delirium

2-month-old female with T 102F, vomiting, diarrhea, dehydration. Exam of stool shows viral particles with wheel-like shape. Properties of virus?

Type of nucleic acid/envelop/capsid symmetry

Double-stranded RNA, segmented/no/icosahedral

Double-stranded RNA, segmented/no/icosahedral

“wheel-like shape”

rotavirus

62-year-old with angina pectoris is referred for cardiac catheterization. Patient worried about hospital-associated infection. Studies show 30% of patients require admission after procedure, and 2 percent of admitted patients acquire hospital infection. Patient’s risk for infection overall?

6/1000

30% * 2% = .0006

Randomized controlled trial evaluating tx of acute otitis media. No statistically significant differences found between infants receiving the antibiotic and those with standard tx. Which aspect results in type II error?

Number of subjects in the study

.66-year-old woman is brought to the emergency department by her daughter because of a 2-day history of fever, flank pain, pain with urination, and nausea. Ten days ago, she was admitted to the hospital for similar symptoms and was diagnosed with acute pyelonephritis. She was discharged with instructions to take oral ciprofloxacin after a 3-day course of intravenous ciprofloxacin resulted in improvement. She also has hypertension, hyperlipidemia, and osteoporosis. Current medications also include alendronate, calcium carbonate, ezetimibe, hydrochlorothiazide, and simvastatin. Her temperature is 39.1°C (102.4°F), and blood pressure is 130/80 mm Hg. The most likely cause of this patient’s current condition is an interaction between her current oral antibiotic and which of the following medications?

Calcium carbonate

antacid – chelate tetracyclines and fluoroquinolones

35-year-old man with severe back pain, gained 8 kg in the past 6 months, decreased peripheral vision, compression fracture spine at T10 and L1, healing left rib fracture at T6, imaging shows pituitary macroadenoma. This tumor produces which?

ACTH

67-year-old man with 3 months fatigue and shortness of breath. Vitals HR 90, RR 15, bp 150/98. PE conjunctival pallor. Labs: hb 8.5, hct 26%, MCV 90, RDW 14.4% (N=13-15%), Cr 2.9, Ferritin 144, Iron 24, Transferrin saturation 23% (N=20-50%). Besides iron supplementation, most appropriate tx?

Erythropoietin

14-year-old girl with type 1 diabetes mellitus and 4-hour history of lethargy, confusion, disorientation. Symptoms gradually developed and she did not take her usual insulin dose during a sleepover. HR 110, RR 24 deep and rapid, bp 95/75. Labs: glucose 450, arterial pH 7.15. ABG?

pCO decreased

HCO3- decreased

Anion gap increase

DKA

Image: 14-year-old girl with type 1 diabetes mellitus and 4-hour history of lethargy, confusion, disorientation. Symptoms gradually developed and she did not take her usual insulin dose during a sleepover. HR 110, RR 24 deep and rapid, bp 95/75. Labs: glucose 450, arterial pH 7.15. ABG?

38-year-old man 1-week shortness of breath with exertion, light palpation of carotid artery shows upstroke is abnormally brisk and downstroke falls precipitously. Cause of finding?

Aortic Regurgitation

water hammer pulse = brisk upstroke and downstroke in the carotid

65-year-old man with 4-hour history of intermittent severe pain in flank area radiating to genital region. History of hypercalciuria and renal calculiDrug decrease the urinary excretion of calcium?

Hydrochlorothiazide

32-year-old woman has new neuro finding while being tx for acute infection of sphenoid sinus. Imaging shows cavernous sinus thrombosis on left. Additional finding most likely?

Inability to abduct the eye

Image: 32-year-old woman has new neuro finding while being tx for acute infection of sphenoid sinus. Imaging shows cavernous sinus thrombosis on left. Additional finding most likely?

4-year-old male with recurrent UTIs, left kidney found small and non functional; right is normal. Nephrectomy is done, and the picture shown (dilated ureter and renal calyx). Microscopic exam of kidney will show which?

Tubular atrophy

tubulointerstitial fibrosis of the kidney

Girl has chronic cough with thick sputum, abdominal cramps in RLQ, and frequent resp infections. Clubbing of fingers, hyperresonance on chest percussion. Diffuse crackles and scattered wheezes. On xray – diffuse hyperinflation of the right upper lobe. She has a healthy brother. What’s the likelihood he is a carrier for the condition?

2 of 3

15-year-old girl comes with lack of improvement of facial acne. Tried topical clindamycin and benzoyl peroxide PE severe acne vulgaris. Says, “Please help me.” She has never been sexually active. Tx?

Isotretinoin

40 year old brought to ER after wife found him unresponsibve- PCO2 60, PO2 50 HCO3 12 – acid base status?

Metabolic acidosis and respiratory acidosis

Check pH – NONE

Check Bicarb – low – metabolic acidosis

Check CO2- high – respiratory acidosis (hypoventilating)

35 year old woman – undergoes flexible nasopharyngoscopy – Purulent discharge from the right sphenoethmodial recess. Location of structure?

superior to superior concha

Image: 35 year old woman - undergoes flexible nasopharyngoscopy - Purulent discharge from the right sphenoethmodial recess. Location of structure?

18 year old on tennis team –after 5 hours or practicing – tenderness over lateral elbow pain –inflammation at the origin of what muscle?

Extensor Carpi Radialis Brevis

Image: 18 year old on tennis team -after 5 hours or practicing - tenderness over lateral elbow pain -inflammation at the origin of what muscle?

47 y/o woman patient with psoriasis – comes to physician for follow-up- previously given several topical creams used in specific sequence – no improvement of symptoms – what is appropriate way to beginning discussion of compliance?

Most people find it difficult to stick to routine. How did you do?

44 year old – severe pain and swelling in knuckles and knees – increased erythrocyte count and Rheumatoid Factor – Treatment with Ibuprofen for 2 week. Plasma after 2 weeks (Leukotriene, Arachidonic Acid, PGE2, PGH2)?

Leukotriene Increased

Arachidonic acid Increased

PGE2 decreased

PGH2 decreased

Image: 44 year old - severe pain and swelling in knuckles and knees - increased erythrocyte count and Rheumatoid Factor - Treatment with Ibuprofen for 2 week. Plasma after 2 weeks (Leukotriene, Arachidonic Acid, PGE2, PGH2)?

26 year old man – HIV+ , 3 days of reddening skin involving the palms, arms, legs, and soles. RPR test is positiveTreatment to eradicate the causal organism?

Penicillin

6 year old – firm, smooth, umbilicated papules 2 to 4 mm in diameter- causal organisms?

Poxvirus

Image: 6 year old - firm, smooth, umbilicated papules 2 to 4 mm in diameter- causal organisms?

28-year-old man –struck in right eye by a baseball. He has diplopia, periorbital swelling, and enophthalmos, Upwards gaze impaired- fracture involving the orbital-floor– Entrapment of which muscles?

inferior rectus and inferior oblique

fracture involving the orbital-floor

Image: 28-year-old man -struck in right eye by a baseball. He has diplopia, periorbital swelling, and enophthalmos, Upwards gaze impaired- fracture involving the orbital-floor- Entrapment of which muscles?

Acetaminophen and nonsteroidal anti-inflammatory agents – reduce fever by decreasing the synthesis of which of the following factors in the hypothalamus?

Prostaglandin E2

75-year-old – sudden onset of partial loss of vision – Visual fields show R- lower Quadrantopia – pie on the ground – a lesion in which of the following?

Parietal Lobe – left

Pie on the ground – Contralertal lesion in parietal lobe – baums loop -bombs on ground

Image: 75-year-old - sudden onset of partial loss of vision - Visual fields show R- lower Quadrantopia - pie on the ground - a lesion in which of the following?

53-year-old – GERD – dietary changes and stress reduction have no effect. What drug both relieves symptoms and promotes healing?

Omeprazole

Irreversibly inhibit H+/K+ ATPASE stomach parietal p. 399-2

22yo woman, g1p1, 2-day hx of fever, severe vaginal bleedingfour days ago delivered healthy male newborn. Temp 38.1 C (100.6 F). Pelvic exam: open cervix, heavy vaginal bleeding. US shows uterus with no placental tissue or thrombi. If operation required to control bleeding, ligation of branch of which artery required?

internal iliac

Image: 22yo woman, g1p1, 2-day hx of fever, severe vaginal bleeding. four days ago delivered healthy male newborn. Temp 38.1 C (100.6 F). Pelvic exam: open cervix, heavy vaginal bleeding. US shows uterus with no placental tissue or thrombi. If operation required to control bleeding, ligation of branch of which artery required?

42yo F with 3-year hx of an intermittent facial rash, including the forehead, eyelids, nose, and cheeks. Rash seems to be getting worse since she moved from New York to Florida last year. Spicy foods precipitate a flushing reaction that seems to exacerbate the rash. PE shows erythema over the nose and cheeks, with scattered telangiectasias and a few papules. Dx?

Rosacea

Image: 42yo F with 3-year hx of an intermittent facial rash, including the forehead, eyelids, nose, and cheeks. Rash seems to be getting worse since she moved from New York to Florida last year. Spicy foods precipitate a flushing reaction that seems to exacerbate the rash. PE shows erythema over the nose and cheeks, with scattered telangiectasias and a few papules. Dx?

17 year old – evaluated for growth delay – X-ray shows suprasellar calcifications- most likely explanation for this condition?

Craniopharyngioma

Most common childhood suprasellar tumors

Remnants of Rathke Pouch – derived from oral/surface ectoderm of oral cavity

Image: 17 year old - evaluated for growth delay - X-ray shows suprasellar calcifications- most likely explanation for this condition?

64 y/o man – bronchospastic pulmonary disease – wheezing in spite increasing on b-adrenergic agonist inhaler– hospitalized and systemic prednisone is added – which explains the action of prednisone in enhancing response on B-adrenergic agonist??

Enhanced action of the agonist at B-adrenergic receptors

Corticosteroids– upregulates B2 receptors + inhibit b2

Cortisol – permissive effect on catecholamines – 336-1

2-week-old male with Patent Ductus Arteriosus – which finding is likely?

Higher than normal Left ventricular cardiac output 

Image: 2-week-old male with Patent Ductus Arteriosus - which finding is likely?

33-year-old woman – MVC – splenic laceration – 3 units of RBC administered – 5 hours later – develops Shortness of Breath – crackles heard through lung fields – Chest X-ray bilateral infiltrates – what causes patient’s condition?

Transfusion-related acute lung injury

Image: 33-year-old woman - MVC - splenic laceration - 3 units of RBC administered - 5 hours later - develops Shortness of Breath - crackles heard through lung fields - Chest X-ray bilateral infiltrates - what causes patient's condition?

8-year-old girl is brought to the physician for a well-child examination. Her mother says that she has been well except for an occasional cold. Her immunizations are up to date. She is at the 50th percentile for height and 60th percentile for weight. Physical examination shows breast bud development and a few pubic hairs. The mother asks whether her daughter’s development is normal. Which of the following is the most appropriate initial response by the physician?

Your daughter development is normal

Precocious puberty before age 8 in girlsPrecocious puberty before age 9 in boys

Image: 8-year-old girl is brought to the physician for a well-child examination. Her mother says that she has been well except for an occasional cold. Her immunizations are up to date. She is at the 50th percentile for height and 60th percentile for weight. Physical examination shows breast bud development and a few pubic hairs. The mother asks whether her daughter's development is normal. Which of the following is the most appropriate initial response by the physician?

A 63 y/o – bladder cancer – starts treatment – significant decreased in tumor size. Paclitaxel mechanism of action?

Tubulin Polymerization Stabilization

M Phase 

Image: A 63 y/o - bladder cancer - starts treatment - significant decreased in tumor size. Paclitaxel mechanism of action?

25 year-old – participating in a study of blood glucose – During study, found to glucose of 100mg/dl after 20 hours of fasting– which would contribute to normal glycemia?

Alanine

Muscle breakdown – alanine – enter cahill cycle to form glucose

Image: 25 year-old - participating in a study of blood glucose - During study, found to glucose of 100mg/dl after 20 hours of fasting- which would contribute to normal glycemia?

3 y/o boy – recently immigrated to USA – skeletal deformities– frontal bossing, flattening of back of the skull, defects in enamel of the teeth, chest wall grooves- legs are bowed – Serum calcium and phosphorus low – if bone were examined, what finding likely?

increased portion of osteoid

rickets due to osteoid deposition

Vitamin D Decreased, Calcium Decreased, increased PTH

Image: 3 y/o boy - recently immigrated to USA - skeletal deformities- frontal bossing, flattening of back of the skull, defects in enamel of the teeth, chest wall grooves- legs are bowed - Serum calcium and phosphorus low - if bone were examined, what finding likely?

17-year-old boy – acute lethargy, sore throat, fever – positive Epstein Barr Viral Capsid antigen – what increased in peripheral blood?

Lymphocytes

a. P 164 – atypical lymphocytes on peripheral blood smear

Image: 17-year-old boy - acute lethargy, sore throat, fever - positive Epstein Barr Viral Capsid antigen - what increased in peripheral blood?

50 year old with RA – decreased iron, decreased TIBC, increased Ferritin, MCV 77– What DX?

Anemia of Chronic Disease

Image: 50 year old with RA - decreased iron, decreased TIBC, increased Ferritin, MCV 77- What DX?

38 year old – epigastric pain and frequent stools – 2 episodes of renal calculi –endoscopy shows non bleeding duodenal ulcers – PPI therapy is begun – next step in management?

serum calcium concentrations

PPI – reduce Mg+ and Ca-+ need to check – leading to fracture risk – p. 399-2

HIV Aids antiviral agent inhibits entry of the HIV virus into CD4+ T cells– which of the following antiretroviral agents?

Enfuvirtide

Binds gp41 – inhibiting viral entry p. 201

MO Entry – maraviroc and enfuviritide

Image: HIV Aids antiviral agent inhibits entry of the HIV virus into CD4+ T cells- which of the following antiretroviral agents?

Scientists – identified the flawed gene causes 1-6 colon cancer – when gene is working – gene repairs discriminates between the template and the newly synthesized DNA strand, correcting mistakes in the newly synthesized gene?

Mismatch repair

Colon Cancer – Lynch HNPCC

Image: Scientists - identified the flawed gene causes 1-6 colon cancer - when gene is working - gene repairs discriminates between the template and the newly synthesized DNA strand, correcting mistakes in the newly synthesized gene?

55 y/o– with renal artery stenosis – 50% reduction in renal blood flow – which part of kidney has the highest concentration of renin in response to reduction in arcuate pressure

Cortex

Renin is produced in JG Cells – located in cortex

P. 589 – located in modified smooth muscle of afferent

Image: 55 y/o- with renal artery stenosis - 50% reduction in renal blood flow - which part of kidney has the highest concentration of renin in response to reduction in arcuate pressure

14 year old – 2 month history of hair loss – PT sad since grandmother died unexpectedly 3 months ago. Underlining skin is normal. Explanation for hair loss

Trichotillmania

Compulsively pulling out one hair – causes significant distress despite attempts to stop – p. 563

Image: 14 year old - 2 month history of hair loss - PT sad since grandmother died unexpectedly 3 months ago. Underlining skin is normal. Explanation for hair loss

34 year old – 6 week history pf fatigue and shortness of breath – fixed split S2 – midsystolic pulmonary ejection murmur – A congenital heart defect – patient undergoes surgical repair – greatest risk for intraoperative injury?

Atrioventricular Bundle

Image: 34 year old - 6 week history pf fatigue and shortness of breath - fixed split S2 - midsystolic pulmonary ejection murmur - A congenital heart defect - patient undergoes surgical repair - greatest risk for intraoperative injury?

66 year old – pain, fever, chills, – left lower abdominal pain – severe constipation over 5 years -a barium of lower GI – 3 separate area of narrowing of the lumen of the distal sigmoid – Photograph of lesion – DX?

Diverticulitis

LLQ pain, fever, leukocytosis, wall thickening p. 383-3

> 60 years old –

Image: 66 year old - pain, fever, chills, - left lower abdominal pain - severe constipation over 5 years -a barium of lower GI - 3 separate area of narrowing of the lumen of the distal sigmoid - Photograph of lesion - DX?

64 year old women – T2DM and HTN – soreness and muscle Pain in large muscles – She began rosubastatin therapy – her medicatioons include gemfibrozil, HCTZ, losrtan, and metformin, elevated serum creatine kinase – what interaction between rosuvastatin and which of the following?

Gemfibrozil

Statins – 320- cause Rhabdomyolysis

Myopathy when used with fibrates

48-year-old man – ER – 1-hour history of vomiting blood – Physical exam shows ascites. A surgical shunt between which of the following veins – release esophageal bleeding?

Splenic and left renal vein

Image: 48-year-old man - ER - 1-hour history of vomiting blood - Physical exam shows ascites. A surgical shunt between which of the following veins - release esophageal bleeding?

37-year-old – MVC – severe, sharp, chest pain – hyperresonance- decreased tactile fremitus – adventitious sounds are absent- multiple rib fractures – pathway of chest pain?

Intercostal nerve

Multiple rib fractures and pneumothorax – COPD

costal and cervical pleura innervated by intercostal nerve

Image: 37-year-old - MVC - severe, sharp, chest pain - hyperresonance- decreased tactile fremitus - adventitious sounds are absent- multiple rib fractures - pathway of chest pain?

78 yeasr old women – paroxysomal embolus in left leg – left lower extremity cool and pale, absence pulses– ECG shows irregularly irregular rhythm – which treatment is most appropriate?

Embolectomy

Originated in heart – sudden acute of pain in the leg

Paroxysmal embolus

Image: 78 yeasr old women - paroxysomal embolus in left leg - left lower extremity cool and pale, absence pulses- ECG shows irregularly irregular rhythm - which treatment is most appropriate?

75 year old man – fever, back pain, pain on urination – long standing prostatitis- gram statin – gram positive cocci in chains – cause of illness?

Enterococcus Faecalis

Gram + cocci in chains + UTI = enterococcus

Image: 75 year old man - fever, back pain, pain on urination - long standing prostatitis- gram statin - gram positive cocci in chains - cause of illness?

65-year-old – vomiting blood – Hypovolemic shock– placement of catheter in internal jugular vein for blood transfusions – improper insertion damage to which of sets of underlying structures?

Lung and Common Carotid Artery

b. Internal jugular vein – next to common carotid – and lung apex extend above 1st rib

Image: 65-year-old - vomiting blood - Hypovolemic shock- placement of catheter in internal jugular vein for blood transfusions - improper insertion damage to which of sets of underlying structures?

32-year-old woman at 35 weeks’ gestation – during her labor – increased oxytocin concentrations – most likely cause of finding?

Birth Canal Reflex

Fergusson Reflex – positive feedback loop

Image: 32-year-old woman at 35 weeks' gestation - during her labor - increased oxytocin concentrations - most likely cause of finding?

75 year old woman – sudden Horner syndrome, dysphagia, difficulty speaking, which occluded?

Posterior Inferior Cerebellar

Lateral medullary syndrome – nucleus ambigious (IX, X, XI)

Image: 75 year old woman - sudden Horner syndrome, dysphagia, difficulty speaking, which occluded?

65-year-old man in MVC – photograph of lung at autopsy (Centrally located lesion) – Dx?

Squamous Cell Carcinoma

Centrally located– arising or extending into bronchus

Hilar mass arising from bronchus – p. 684

Hypercalcemia – PTHrp- source from cancer

Image: 65-year-old man in MVC - photograph of lung at autopsy (Centrally located lesion) - Dx?

54yo man for exam. P80, R14, BP140/95. Physical exam normal. ACE inhibitor givenWhere is MOA in kidney?

F – ACE preferentially vasoldiate the efferent arteriole

Preserves GFR – by back pressure

vasodilation of efferent

Image: 54yo man for exam. P80, R14, BP140/95. Physical exam normal. ACE inhibitor given. Where is MOA in kidney?

68 year old man – 1 month history of not being able to sustain an erection – wife died 2 years ago – able to masturbate – testosterone within reference range –no symptoms of depression- which pairs of finding in history?

Libido Normal

Nocturnal Erection Normal

A 30 year with hypothyroidism – receiving T4 – fetus at greatest risk if TSH concentrations increase?

Abnormal Brain Development

Image: A 30 year with hypothyroidism - receiving T4 - fetus at greatest risk if TSH concentrations increase?

55-year-old – 6 month history of bone pain – 20 year history of T2DM – receiving hemodialysis – can’t recall meds – Increased phosphate, increased ALP, increased PTH, decreased calcium

bone pain is caused by which of the following in serum concentrations?

Increased parathyroid hormone

Image: 55-year-old - 6 month history of bone pain - 20 year history of T2DM - receiving hemodialysis - can't recall meds - Increased phosphate, increased ALP, increased PTH, decreased calcium
bone pain is caused by which of the following in serum concentrations?

4 -month-old – brought in because she has seizures – diarrhea for 3 days – consumed only water for 24 hours because parents ran out of formula – what is likely electrolyte abnormalities

Hyponatremia

17-year-old boy returns to locker room after football practice – soaked in sweat- characteristic odor associated with seat – result of secretion of which glands?

Apocrine

Apocrine – develop in areas of hair follicles

Your armpit smell like an APE- body odor

Image: 17-year-old boy returns to locker room after football practice - soaked in sweat- characteristic odor associated with seat - result of secretion of which glands?

34 year old women – postpartum depression – I feel guilty nit enjoying my baby more – physician most appropriate step?

I’m Concerned about bad you are feeling lately. Have you had any thoughts about death or wanting to be dead?

4 year old boy with Chronic Granulomatous Disease – TX with interferon gammas. Which is increased after exposure to the drug?

macrophages producing interleukin-1 (Il-1)

Image: 4 year old boy with Chronic Granulomatous Disease - TX with interferon gammas. Which is increased after exposure to the drug?

4-year-old-girl conscious but unable to breathe under general anesthesia with sevoflurane – succinylcholine administered for intubation – cause prolonged apnea

Pseudocholinesterase Deficiency

3 year old boy brought to ER because of nosebleed – can’t stop bleeding – PT and PTT are prolonged – platelet count and thrombin time are within reference ranges – which of coagulation process if affected?

Synthesis of Factor X (stuart Factor)

Image: 3 year old boy brought to ER because of nosebleed - can't stop bleeding - PT and PTT are prolonged - platelet count and thrombin time are within reference ranges - which of coagulation process if affected?

27 year old man – no history of medical illness – never sexually active – minimal contact with his parents and siblings – not depressed – no hobbies – flat affect – personality disorder?

Schizoid

80yo F in ED for 2-day hx of “feeling funny.” “Lost my pep.” Hx of poorly controlled hypertension. Just started medication 2 weeks ago- doesn’t know the name. BP 130/85. Pe normal. Serum potassium is 3. Which drug?

Hydrochlorothiazide

Diuretic TX- HTN, heart failure pg. 609-1

Potassium is low

Hypokalemia metabolic acidosis, hyponatremia, nephrogenic diabetes insipidus

Image: 80yo F in ED for 2-day hx of "feeling funny." "Lost my pep." Hx of poorly controlled hypertension. Just started medication 2 weeks ago- doesn't know the name. BP 130/85. Pe normal. Serum potassium is 3. Which drug?

61 year old woman – 9 month history increasing forgetfulness – Mini Mental Status 22/30- most appropriate pharmacotherapy?

Donepezil

anticholinesterase

Dona, Riva, at gala – increased ACH

51-year-old woman – burning abdominal pain, sweats profusely, 2 cm- mass in proximal duodenum. Gastrin releasing tumor stimulates which labeled cells in picture?

B- parietal Cells

Stain Eosinophilic – fried egg appearance- PINK EGGS

Parietal cells – secrete gastric acid

Image: 51-year-old woman - burning abdominal pain, sweats profusely, 2 cm- mass in proximal duodenum. Gastrin releasing tumor stimulates which labeled cells in picture?

25-year old develops shortness of breath – after moving from sea level to a mining town at 3350. At rest, pulmonary hypertension with normal cardiac output – pulmonary function tests are normal – Which explains pulmonary hypertension?

Decreased Alveolar PO2

Image: 25-year old develops shortness of breath - after moving from sea level to a mining town at 3350. At rest, pulmonary hypertension with normal cardiac output - pulmonary function tests are normal - Which explains pulmonary hypertension?

30 year -old comes to the office – 20lb weight loss after initiating a low-carbohydrate diet – low fat diet -it is most important to counsel the patient to include which of the following to maintain a good nutrition?

Linoleic Acid

Essential fatty acids – OMEGA 6 and Omega 3

Omega 6= linoleic and Omega 3 = alpha linoleic

Must be obtained through diet – vegetables, oils, meats, nuts, and egg

Deficiency – lead to poor growth, skin lesions, reproductive failure

Image: 30 year -old comes to the office - 20lb weight loss after initiating a low-carbohydrate diet - low fat diet -it is most important to counsel the patient to include which of the following to maintain a good nutrition?

44-year-old man –altered mental status HTN (210/145)– fundoscopic exam shows papilledema and retinal hemorrhages – A drug with which of the following effects on vascular smooth muscle is likely to decrease blood pressure?

Increasing cGMP

A test has been developed to screen for HPV infections – If the prevalence increases by 50%– which of the following features of the test will increase?

Predictive value of a positive test

Image: A test has been developed to screen for HPV infections - If the prevalence increases by 50%- which of the following features of the test will increase?

. A newborn female external genitalia – 46 XY karotype – shows male genital ducts – the most likely cause of this anomaly is a mutations resulting in the absence expression of which of the following hormones or factors?

Dihydrotestosterone

Image: . A newborn female external genitalia - 46 XY karotype - shows male genital ducts - the most likely cause of this anomaly is a mutations resulting in the absence expression of which of the following hormones or factors?

36-year-old with AIDS – tx with CCR5 receptor inhibitors – which is natural ligand for HIV co-receptor?

Chemokine

Maraviroc binds to CCR5

Cysteine-Cysteine Chemokine receptors – found on surface of CD4+

Image: 36-year-old with AIDS - tx with CCR5 receptor inhibitors - which is natural ligand for HIV co-receptor?

7 year old – difficulty concentrating in school – “feels tired” – hypochromic anemia – fatigue – increased serum creatinine – additional abnormal concentrations in this patient?

Increased Blood Lead

Physician prescribes a recently marked drug for 20 patients – After several months, 5 patients develop increased serum AST and ALT – Physician discontinues drug and reports adverse effects to the FDA. Physician participated in what phase of clinical trial?

Phase 4

SWIM

Post marketing surveillance – detects rare or long term adverse effexts

Image: Physician prescribes a recently marked drug for 20 patients - After several months, 5 patients develop increased serum AST and ALT - Physician discontinues drug and reports adverse effects to the FDA. Physician participated in what phase of clinical trial?

6-month-old – exaggerated reaction to loud noises- Exam shows muscle weakness, increased start reflex, cherry red spots on retina– most likely cause of this disorder is accumulation of which of the following lipids in the patients lysosomes?

GM2

Tay-Sachs – cherry red spot

Gm2 gangliosides accumulate

Deficiency in Hexoaminidase A

Image: 6-month-old - exaggerated reaction to loud noises- Exam shows muscle weakness, increased start reflex, cherry red spots on retina- most likely cause of this disorder is accumulation of which of the following lipids in the patients lysosomes?

60 year old – ER -thrown from care in high speed automobile accident and struck guardrail– blood in urethral meatus and fractures of the pelvis and left femur. A Foley catheter is unable to be inserted– which is most likely explanation?

Urethral Disruption

Suspect urethral injury if blood is seen at the urethral meatus

Image: 60 year old - ER -thrown from care in high speed automobile accident and struck guardrail- blood in urethral meatus and fractures of the pelvis and left femur. A Foley catheter is unable to be inserted- which is most likely explanation?

30 year old – pruritic rash on his lower back – underwent liver transplantation 1year ago and maintained on immunosuppressive drugs –PE shows vesicles containing clear fluid associated with ulcerated and crusted lesions– likely diagnosis?

Varicella-zoster virus infection

Image: 30 year old - pruritic rash on his lower back - underwent liver transplantation 1year ago and maintained on immunosuppressive drugs -PE shows vesicles containing clear fluid associated with ulcerated and crusted lesions- likely diagnosis?

50-year-old man progressive dyspnea on exertion. Dyspnea at rest. Exam of lung tissue biopsy shows chronic inflammation and fibrous thickening of alveolar septa. Dx?

Usual Interstitial Pneumonitis

Image: 50-year-old man progressive dyspnea on exertion. Dyspnea at rest. Exam of lung tissue biopsy shows chronic inflammation and fibrous thickening of alveolar septa. Dx?

52-year-old woman admitted to hospital because of breath die to malignant ascites and pleural effuision– two gynecologists disagree about treatment –– pt comes to primary care pysician to help address conflicty – what do internal medicine physicians do to address concern?

Speak with two gynecologists to attempt to coordinate

36-year-old woman undergoes a total hysterectomy and bilateral salpingo-oophorectomy for extensive endometriosis of the pelvis. After operation, she refuses hormone therapy. Which of the following – most likely occur in gonadotrophs if this patient refuses hormone therapy

Hyperplasia

Gonadotrophs are the FSH/LH producing pituitary cells. No ovaries, no hormones, no feedback inhibition.

Image: 36-year-old woman undergoes a total hysterectomy and bilateral salpingo-oophorectomy for extensive endometriosis of the pelvis. After operation, she refuses hormone therapy. Which of the following - most likely occur in gonadotrophs if this patient refuses hormone therapy

An investigator – study efficacy of capsular polysaccharides – Neisseria Meningitidis – Which of the following compounds is most likely to induce increased titers of these antibodies when conjugated to polysaccharides?

Flagellin

35-year-old woman with blistering lesions on sun-exposed area of face, arms, hands – increased AST, ALT, Total Porphyrin, Urine uroporphyrin III, deficiency in what enzyme?

Uroporphyrinogen Decarboxylase

Porphyria cutanea tardia – vampire blistering lesion on sun-exposed

European Vampire – hide in cardboard

Image: 35-year-old woman with blistering lesions on sun-exposed area of face, arms, hands - increased AST, ALT, Total Porphyrin, Urine uroporphyrin III, deficiency in what enzyme?

. 23-year-old drinks alcohol heavily on weekend – which cellular changes most likely occurs in his liver?

Fatty Change

Acute alcohol consumption – increases fatty change

Cellular Swelling – indicates alcoholic hepatitis

Hepatic steatosis – alcoholics hepatitis –cirrihosis

Image: . 23-year-old drinks alcohol heavily on weekend - which cellular changes most likely occurs in his liver?

30 year old man – urinary incontinence for 2 weeks after treatment for pelvic fracture- pinned against loading dock- distended bladder- absence of micturition- overflow urine occurs – injury to which?

Pelvic Nerves

Image: 30 year old man - urinary incontinence for 2 weeks after treatment for pelvic fracture- pinned against loading dock- distended bladder- absence of micturition- overflow urine occurs - injury to which?

35-year-old – pain and swelling of his right knee and great toe – needle like crystals – Exam of tiw shows swelling and warmth. Aspirate of effected joint shows numerous neutrophils and needle like crystals what is causes of patients swelling?

Increased vascular permeability

25 year – nulligravid and infertile woman – irregular menses -– hirsutism – ultrasound shows ovaries with numerous cysts beneath the capsule. Photograph shown – patient at greatest risk for developing which of the following?

Endometrial Hyperplasia

Chronically elevated levels of estrogen can cause endometrial hyperplasia.

Image: 25 year - nulligravid and infertile woman - irregular menses -- hirsutism - ultrasound shows ovaries with numerous cysts beneath the capsule. Photograph shown - patient at greatest risk for developing which of the following?

. 10 year old – fever, malaise, loss pf appeptide followed be excess salivation 6 weeks after exploring a cave with her friends. Over the next few weeks, she develops delirium, seizures, paralysis, and hyperexcitability- virus reached brain by which of the following?

Retrograde transport through the nerves

21-year-old – comes to ED after MVC – laceration of spleen and undergoes splenectomy –Splenectomy most likely predisposes patient to develop future infections – causal organism?

Streptococcus Pneumonia

No Spleen Here

Neisseria meningitidis, Streptococcus Pneumoniae, Haemophilus influenza

Image: 21-year-old - comes to ED after MVC - laceration of spleen and undergoes splenectomy -Splenectomy most likely predisposes patient to develop future infections - causal organism?

62 year old – farmer – 8 yeas history of lesions indicated by arrow (photo) – picks lesions causes bleeding – numerous similar rough plaques on the scalp, face, ears, and forearms. Likely Diagnosis?

Actinic Keratosis

Farmer – Picks at it and bleeds –

Premalignant lesions – caused by sun exposure

Increased risk of skin cancer – keratin pearls

Image: 62 year old - farmer - 8 yeas history of lesions indicated by arrow (photo) - picks lesions causes bleeding - numerous similar rough plaques on the scalp, face, ears, and forearms. Likely Diagnosis?

A 40 year old – T2DM – TX with pioglitazone – mechanism of action?

Stimulation of peroxisome proliferator-activated receptor

Image: A 40 year old - T2DM - TX with pioglitazone - mechanism of action?

47-year-old – severe chest pain – Oxygen Saturation of 85%, CO low, ST elevation, which is decreased?

Mixed Venous Oxygen Tension

Decreased oxygen delivery to tissues – decreased cardiac output

CO=rate of O2 consumption/(arterial O2 content-venous O2 content

53-year-old – breast cancer – taking tamoxifen – what is effect on tamoxifen on breast and Uterus?

Breast Antagonist

Bone Agonist

Uterus partial agonist.

Image: 53-year-old - breast cancer - taking tamoxifen - what is effect on tamoxifen on breast and Uterus?

26-year-old – counseling prior to conception – pedigree – fatal to males not born = which is most likely risk to live born?

50% in females, but near 0 in males

Fatal to males in utero

X linked dominant – males have 50% of getting from mom

Image: 26-year-old - counseling prior to conception - pedigree - fatal to males not born = which is most likely risk to live born?

43 year old – follow-up exam –underwent cardiac valve replacement – Chest X-Ray shown – which of the following valves most likely replaced in this patient?

Aortic

Image: 43 year old - follow-up exam -underwent cardiac valve replacement - Chest X-Ray shown - which of the following valves most likely replaced in this patient?

30 year old – pregnant + gram positive rods – pregnant women – fever, chills, muscles aches – which causal organism?

Listeria Monocytogenes

Image: 30 year old - pregnant + gram positive rods - pregnant women - fever, chills, muscles aches - which causal organism?

Normal appearing 17-year-old – normal appearing – vagina ends blinding – no uterus or ovaries to palpate – chromatin negative nuclei – which is likely diagnosis?

Androgen Insensitivity Syndrome

27 year-old – 34 weeks pregnant – nausea, vomiting, abdominal pain for 12 hours –HTN 164/104, Elevated Liver Enzymes. platelet count low eve what do you see on peripheral blood smear?

Schistocytes

Image: 27 year-old - 34 weeks pregnant - nausea, vomiting, abdominal pain for 12 hours -HTN 164/104, Elevated Liver Enzymes. platelet count low eve what do you see on peripheral blood smear?

52 year – ER – sudden chest pain – IV morphine – generalized pruritus and warmth in 1 minute – which substance most likely cause new symptoms?

Histamine

Pruritus and warmth in 1 minute- Hypersensitivity

59-year-old man – 3-month history of progressive difficulty swallowing – 15lb weight loss – smoked 1 pack of cigs daily for 40 years – cachectic – mass ulcerated with elevation surrounding mucosal rim- 6 cm mass in mid esophagus – centrally located mass – examination of the biopsy specimen of the mass is likely to show neoplastic cells with which of the following?

Foci of Keratinization

Squamous Cell Carcinoma occurs in upper 2/3 –

Adenovrius occurs in distal 1/3

Image: 59-year-old man - 3-month history of progressive difficulty swallowing - 15lb weight loss - smoked 1 pack of cigs daily for 40 years - cachectic - mass ulcerated with elevation surrounding mucosal rim- 6 cm mass in mid esophagus - centrally located mass - examination of the biopsy specimen of the mass is likely to show neoplastic cells with which of the following?

34-year-old – periorbital headaches – last 60- 90 minutes – lacrimation and nasal stuffiness – whichh of the following is most likely diagnosis?

Cluster Headache

Image: 34-year-old - periorbital headaches - last 60- 90 minutes - lacrimation and nasal stuffiness - whichh of the following is most likely diagnosis?

7-year-old – lack of adequate quality protein identified as critical issue– which of the following is most critical in analysis of child’s diet?

Methionine

33-year-old women – fever, malaise, burning on urination, and scant vaginal discharge extremely tender vesicles and ulcers – DX?

Genital Herpes

Image: 33-year-old women - fever, malaise, burning on urination, and scant vaginal discharge extremely tender vesicles and ulcers - DX?

28-year-old – sudden onset of 30 of shortness of breath – history of cocaine abuse – diminished pulses in left upper extremity – widened aortic arch – dx?

Dissecting Aneurysm

Image: 28-year-old - sudden onset of 30 of shortness of breath - history of cocaine abuse - diminished pulses in left upper extremity - widened aortic arch - dx?

24 year old – sickle cell disease – generalized fatigue –MCV 105– Hematocrit dropped. Which diagnostic test most likely explain the patients symptoms and decreased hematocrit?

Serum Parvovirus B19 IgM antibody test

Image: 24 year old - sickle cell disease - generalized fatigue -MCV 105- Hematocrit dropped. Which diagnostic test most likely explain the patients symptoms and decreased hematocrit?

66-year-old – 1 day history of fever – underwent transurethral resection for prostate 1 week ago – BP 70/50 Blood cultures grow, lactose-positive, gram negative rods. Which describes the initial event of hypotension in the patient?

Lipopolysaccharide stimulation of toll like receptors

Image: 66-year-old - 1 day history of fever - underwent transurethral resection for prostate 1 week ago - BP 70/50 Blood cultures grow, lactose-positive, gram negative rods. Which describes the initial event of hypotension in the patient?

A photograph shows spinal cord injury after MVC – image shows damage to fasciculus gracilis?

Loss of sensation to vibration of both feet

Image: A photograph shows spinal cord injury after MVC - image shows damage to fasciculus gracilis?

47-year-old – aplastic anemia – TX for disseminated aspergillosis with antifungal drug binds to specific sterols in plasma membrane – which drug?

Amphotericin B

Image: 47-year-old - aplastic anemia - TX for disseminated aspergillosis with antifungal drug binds to specific sterols in plasma membrane - which drug?

34-year-old – ER – unconscious after alcohol intoxication – which is interpretation of arterial blood gas. Ph 7.25 Pco2 23 Po2 78 – Na+ 140, Cl – 105, HCO3 10?

Primary Metabolic Acidosis with increased anion gap

Ph low, Bicard low, CO2 low

Na+ – (HCO3+ + Cl-) 140 – (10 + 105) = 25

Image: 34-year-old - ER - unconscious after alcohol intoxication - which is interpretation of arterial blood gas. Ph 7.25 Pco2 23 Po2 78 - Na+ 140, Cl - 105, HCO3 10?

18-year-old – 2-week history of fatigue – PE shows scleral icterus – Lab shows – Hemogloblin Low– high bilirubin – increased 2,3 BPG, increased reticulocytes – mostly likely deficiency in enzyme in erythrocytes?

Pyruvate Kinase

22 year old – frequent episodes of cutaneous urticaria and laryngospsm – allergic cause of his urticaria. Which of the following complement proteins is most likely deficient?

C1 esterase inhibitor (binds activated C1r, C1s)

45 year – ER – MVC – patients’ ventilator – adjusted to tidal volume- application of PEEP will most likely prevent which of the following complications?

Absorption Atelectasis

Image: 45 year - ER - MVC - patients' ventilator - adjusted to tidal volume- application of PEEP will most likely prevent which of the following complications?

35-year-old –DKA – treated with insulin – decreased plasma glucose 900mg/dl to 270. which additional lab finding will likely decreases with insulin therapy?

Serum potassium concentrations

67 year sore – fever, sore throat, difficulty swallowing- Corynebacterium diphtheria – the pathogen produced diphtheria toxin – blocks which of the following processes?

Translation

Image: 67 year sore - fever, sore throat, difficulty swallowing- Corynebacterium diphtheria - the pathogen produced diphtheria toxin - blocks which of the following processes?

40-year-old woman – high triglycerides – 380 mg/dl – taking fish oil and a supplement to treat dislipidemia – MOA of most appropriate vitamin?

Antagonize VLDL-cholesterol Secretion

Niacin (vitamin B3) antagonizes VLDL

Image: 40-year-old woman - high triglycerides - 380 mg/dl - taking fish oil and a supplement to treat dislipidemia - MOA of most appropriate vitamin?

55-year-old woman – broad bait gait, nystagmus, confusion – confabulations – BMI 21 what is most likely to see on MRI of the brain shows?

Mammillary Bodies

Image: 55-year-old woman - broad bait gait, nystagmus, confusion - confabulations - BMI 21 what is most likely to see on MRI of the brain shows?

66-year-old – pancreatic cancer – comes to office because of 2 day history of black tarry stools – Upper endscopy shows gastric varices – A CT shows thrombosis of branch of splenic vein – which is most likely source of bleeding in the patient?

Short Gastric

Image: 66-year-old - pancreatic cancer - comes to office because of 2 day history of black tarry stools - Upper endscopy shows gastric varices - A CT shows thrombosis of branch of splenic vein - which is most likely source of bleeding in the patient?

29 year old woman – 10 day history of difficulty walking – broad based gait and spasticity – lesion in white matter of the cerebellum- TX with which of the following agents most likely improve spasticity?

Baclofen

Gaba B agonist

30 year old – dimness of vision and loss of color vison – 1 month after treatment for TB with RIPE – Which drug responsible for adverse effect treatments?

Ethambutol

75-year-old – stroke – 16 hours after onset of left arm and leg weakness – CT scan shows an edematous right hemisphere – No evidence of hemorrhage- but sparing of the right occipital and midline frontal corticies – one hour later dies At autopsy – microscopic examination of right lateral lobe is most likely to show which?

Shrunken Eosinophilic Red Neurons

within 12-24hours

Image: 75-year-old - stroke - 16 hours after onset of left arm and leg weakness - CT scan shows an edematous right hemisphere - No evidence of hemorrhage- but sparing of the right occipital and midline frontal corticies - one hour later dies At autopsy - microscopic examination of right lateral lobe is most likely to show which?

43 year old woman – irregular jerking movements – profound dementia – her father had similar disorder and died at age 65 year – huntingtin disease- autopsy of the patient is most likely to show severe atrophy of which of the labeled structure?

C = Caudate Nucleus

Image: 43 year old woman - irregular jerking movements - profound dementia - her father had similar disorder and died at age 65 year - huntingtin disease- autopsy of the patient is most likely to show severe atrophy of which of the labeled structure?

30-year-old man – intermittent severe lower abdominal pain, right flank tenderness, hematuria, brother has kidney stones. Patient urine turns cherry red with nitroprusside. Hexagonal crystals are present in the urinary sediment– Which of substances is most likely increased in patient urine?

Cysteine

Image: 30-year-old man - intermittent severe lower abdominal pain, right flank tenderness, hematuria, brother has kidney stones. Patient urine turns cherry red with nitroprusside. Hexagonal crystals are present in the urinary sediment- Which of substances is most likely increased in patient urine?

65 year – prostate cancer – metastasis to L1 underwent orchiectomy – high PSA – Which is most likely increased secondary to orchiectomy?

DNA fragmentation

23-year-old develops persistent sneezing each year – sneezing worsens when in garden – A drug with which of the following mechanisms is most likely treating patients?

Stabilization of Mast Cells Membranes

55 year old – with a benign nodule in left lobe of thyroid – undergoing a partial thyroidectomy – A branch of thyrocervical trunk is ligated to interrupt the blood supply of the resected specimen. It is most appropriate to ligate a direct branch of which of the following arteries– which of the following arteries?

External Carotid

Superior thyroid comes off external carotid

Inferior thyroid comes of thyrocervical trunk

Image: 55 year old - with a benign nodule in left lobe of thyroid - undergoing a partial thyroidectomy - A branch of thyrocervical trunk is ligated to interrupt the blood supply of the resected specimen. It is most appropriate to ligate a direct branch of which of the following arteries- which of the following arteries?

. 75-year-old woman – poor concentration and decreased energy. For 6 months, she is waking up at 4am and been unable to get to sleep. Physical examinations shows no abnormalities. What is most likely sleep disturbance?

Major Depressive Disorder

Insomnia – unable to get back to sleep = depression

39 year old woman with Rheumatoid Arthritis – develops proteinuria following treatment with penicillamine. Renal biopsy shows IgG and C3 in the glomerular basement membrane. Dx?

Membranous Glomerulonephritis

Image: 39 year old woman with Rheumatoid Arthritis - develops proteinuria following treatment with penicillamine. Renal biopsy shows IgG and C3 in the glomerular basement membrane. Dx?

28 year old man – evaluation of infertility – normal physical exam – normal serum concentration of testosterone, FSH, LH, and prolactin – what is abnormal?

Fructose

Image: 28 year old man - evaluation of infertility - normal physical exam - normal serum concentration of testosterone, FSH, LH, and prolactin - what is abnormal?

A screening program of Chlamydia – C trachomatis – initial screening is 500/2500 – one year later – an additional 200 – which is annual incidence of C trachomatis?

10%

=200 (new cases)/2000 #at risk

# at risk = 2500-500

. 5 year old homocystunuria – improves with high does oral pyridoxine therapy – cystathionine synthase is deficient but increases to normal when incubated with higher concentrations of pyridoxal phosphate. When cystathionine synthase is measured in the presence of pyridoxal phosphate – which lineweaver -burk plot would be most consitent with the enzyme?

Area labeled B

Competitive inhibitor

Image: . 5 year old homocystunuria - improves with high does oral pyridoxine therapy - cystathionine synthase is deficient but increases to normal when incubated with higher concentrations of pyridoxal phosphate. When cystathionine synthase is measured in the presence of pyridoxal phosphate - which lineweaver -burk plot would be most consitent with the enzyme?

28 year old woman – 18 weeks’ gestation has palpations – increased serum thyroxine (T4) concentration in pregnancy – Which test used to diagnose hyperthyroidism in the patient?

Free T4

28 year – acute leukemia – receiving high dose chemotherapy – Pancytopenia– leukoocyte count low, MCV low, Platelet low – antibiotic therapy is started. Which of the following is most appropriate?

Granulocyte Colony-Stimulating Factor

50 year old man is found dead at home – he has chest pain 3 weeks ago prior to death –. At autopsy – severe atherosclerosis of 3 major coronary vessels – myocardial softening and mottling involving the anterolateral wall – Photomicrograph of a section is shown. which is most likely mechanism of death

Arrhythmia

Ventricular Arrythmia – most common after 24 hours of MI

68 year old – fever, shortness of breath – Xray shows right lower lobe infiltrates – leukocytes 38,000 – peripheral blood smear is shown- which of the following best describes blood smear?

Reactive Granulocytosis

Image: 68 year old - fever, shortness of breath - Xray shows right lower lobe infiltrates - leukocytes 38,000 - peripheral blood smear is shown- which of the following best describes blood smear?

19 year old women –1 month of abdominal pain and severe diarrhea – 5.4 KG weight loss colonoscopy shows non-bloody diarrhea and cobblestone appearance of the colonic mucosa – stricture in retroperitoneal portion of the bowel – which is the most likely location of the stricture

Descending Colon

Image: 19 year old women -1 month of abdominal pain and severe diarrhea - 5.4 KG weight loss colonoscopy shows non-bloody diarrhea and cobblestone appearance of the colonic mucosa - stricture in retroperitoneal portion of the bowel - which is the most likely location of the stricture

Which of the following best explain why use of anti-idiotypic antibody is inappropriate therapeutic reagent for treatment?

Myeloma Cells do not have membrane surface immunoglobulin

a. Plasma cells produce secretory Ab’s (aka produce Ab’s) — plasma cells do not have membrane-bound Ig (aka do not have BCR’).

Image: Which of the following best explain why use of anti-idiotypic antibody is inappropriate therapeutic reagent for treatment?

9 year-old – tumor in heart – what is the diagnosis?

Rhabdomyoma

Tumor heart in child = rhabdomyoma

Image: 9 year-old - tumor in heart - what is the diagnosis?

32-year-old woman – sister recently died of melanoma – Other family members similarly affected. Which lesion is most likely on both sun exposed and nonsun exposed ares on patient skin?

Dysplastic Nevi

Image: 32-year-old woman - sister recently died of melanoma - Other family members similarly affected. Which lesion is most likely on both sun exposed and nonsun exposed ares on patient skin?

Diastolic blood pressure obtained from two groups of 100 asymptomatic – picture of Diastolic average?

Median Group Y = Higher

Mode Group Y = Higher

Image: Diastolic blood pressure obtained from two groups of 100 asymptomatic - picture of Diastolic average?

72 year old man – history of nonproductive cough –non hodgkin lymphoma– treated with antineoplastic drugs – chest xray shows diffuse infiltrates– which of the drugs most likely cause of patient symptoms?

Bleomycin

Image: 72 year old man - history of nonproductive cough -non hodgkin lymphoma- treated with antineoplastic drugs - chest xray shows diffuse infiltrates- which of the drugs most likely cause of patient symptoms?

47 y/o – jaundice – increased serum bilirubin and positive urine bilirubin test –Which is most likely associated with the positive urine bilirubin test

Obstruction of the Bile Duct

Image: 47 y/o - jaundice - increased serum bilirubin and positive urine bilirubin test -Which is most likely associated with the positive urine bilirubin test

70-year-old – persistent fever despite intravenous broad-spectrum antibiotic therapy 3 days after operative incision for cervical carcinoma. PE shows placement of central venous catheter – Blood cultures from cather grow – Gram stain colonies, 4um, elliptical, purple budding organisms– which is the causal organisms?

Candida Albicans

Image: 70-year-old - persistent fever despite intravenous broad-spectrum antibiotic therapy 3 days after operative incision for cervical carcinoma. PE shows placement of central venous catheter - Blood cultures from cather grow - Gram stain colonies, 4um, elliptical, purple budding organisms- which is the causal organisms?

A 74-year-old with mild dementia is admitted to the hospital because of congestive heart failure. Physician recommends coronary angiopgraphy and angioplasty. The patient states that she wants to go homeIt would be appropriate to question patient capacity if?

Inability to understand the severity and prognosis of her medical condition

informed consent process

A 74-year-old with mild dementia is admitted to the hospital because of congestive heart failure. Physician recommends coronary angiopgraphy and angioplasty. “On questioning, the patient does not know the date [time], the name of the hospital [place], or the name of her nurse who had just introduced himself [person].” The patient states that she wants to go homeIt would be appropriate to question patient capacity if?

inability to understand the severity and prognosis of her medical condition

The gene that codes for a protein normally found in the ER is mutatedWhich of the following mutations is most likely to impair the transport of the protein in the endoplasmic reticulum?

Deletion of the hydrophobic amino acid sequence on N Terminus

Image: The gene that codes for a protein normally found in the ER is mutated. Which of the following mutations is most likely to impair the transport of the protein in the endoplasmic reticulum?

25-year-old –increased SOB – exertion 2 weeks after birth of first child – JVD – evidence of edema at lung base, hepatomegaly, and mild pitting edema in lower extremities – which is the following cardiac abnormalities is abnormality is most likely present?

Four Chamber Dilation

Peripartum

Right heart failure – caused by left heart failure

Image: 25-year-old -increased SOB - exertion 2 weeks after birth of first child - JVD - evidence of edema at lung base, hepatomegaly, and mild pitting edema in lower extremities - which is the following cardiac abnormalities is abnormality is most likely present?

27-year-old – 39 weeks gestation is admitted to the hospital in labor – 5-year history of T2DM– currently treated with insulin – Difficulty controlling serum glucose – the cervix is 100% effaced and 7 cm dilated – babies weight is 9lb – what is the following obstetric complications

Shoulder Dystocia

Image: 27-year-old - 39 weeks gestation is admitted to the hospital in labor - 5-year history of T2DM- currently treated with insulin - Difficulty controlling serum glucose - the cervix is 100% effaced and 7 cm dilated - babies weight is 9lb - what is the following obstetric complications

38 year old woman –swollwen painful calf for 2 days– gave birth 5 days ago – one day later she has a cerebral infarction with hemiplegia. Passage of embolus into systemic circulation is most likely occur in which of the following locations?

Patent Foramen Ovale

Image: 38 year old woman -swollwen painful calf for 2 days- gave birth 5 days ago - one day later she has a cerebral infarction with hemiplegia. Passage of embolus into systemic circulation is most likely occur in which of the following locations?

35-year-old – 2 day history of fatigue and dizziness – He has profuse watery diarrhea for 8 hours despite lack of oral intake. He returned from remote village in Honduras. PE shows dry skin, decreased capillary refill – A gram stain = gram negative, comma shaped bacteria – which described the mechanism of the toxin?

Activation of Adenyl Cyclase

enterotoxin that permanently activates Gs, cAMP

Image: 35-year-old - 2 day history of fatigue and dizziness - He has profuse watery diarrhea for 8 hours despite lack of oral intake. He returned from remote village in Honduras. PE shows dry skin, decreased capillary refill - A gram stain = gram negative, comma shaped bacteria - which described the mechanism of the toxin?

40 y/o man with interstitial pulmonary fibrosis – had a greater expiratory flow rate than predicted– which explains this finding?

Increased radial traction on airways

Fibrosis – increased traction

Image: 40 y/o man with interstitial pulmonary fibrosis - had a greater expiratory flow rate than predicted- which explains this finding?

Clinical trial of 800 – 400 in surgery and 400 in medical management – 75 drop out of surgery procedure, 50 undergo operation during the study – Intention to treat analysis – which accurately represents how many patients would be analyzed in surgical procedures and medical management?

Surgical Procedure group = 400

Medical Management Group = 400

Intention to treat = Include all patients in the groups they were originally in

41-year-old numbness of his right hand – He works as a construction. Sensation to pinprick is decreased in the thumb and over a portion of the anterior forearm. The physician suspects compression of C5 and C6 spinals nerves as they exit the spinal canal – weakness of which of the following movements on the right is most likely to confirm diagnosis?

Abduction of the upper extremity

Image: 41-year-old numbness of his right hand - He works as a construction. Sensation to pinprick is decreased in the thumb and over a portion of the anterior forearm. The physician suspects compression of C5 and C6 spinals nerves as they exit the spinal canal - weakness of which of the following movements on the right is most likely to confirm diagnosis?

. Left radial arterial and venous blood samples are drawn from 10 chimps while resting quietly. Results indicate that the total carbon dioxide concentration average in the arterial samples and 24.3 mM in the venous samples. Which accounts for large carbons dioxide in the venous sample?

HCO3- transported into plasma

a. CO2 is primarily transported as HCO3 – in plasma – then converted back to CO2 lung

65 year old with – undergoes surgical repair for right internal carotid artery in the cavernous sinus. 3 days later right pupil is larger than left pupil. Weakness in movement of the eye. CN III injury – location in the cavernous sinus (photo)?

area labeled A – cranial nerve 3

Image: 65 year old with - undergoes surgical repair for right internal carotid artery in the cavernous sinus. 3 days later right pupil is larger than left pupil. Weakness in movement of the eye. CN III injury - location in the cavernous sinus (photo)?

83-year-old man – 3 day history of painful blisters on his torso – PE shows numerous .5 to 1cm – clear fluid filled tense blisters- subepidermal blisters– Production of autoantibodies directed against which of the following?

Bullous Pemphigoid

Image: 83-year-old man - 3 day history of painful blisters on his torso - PE shows numerous .5 to 1cm - clear fluid filled tense blisters- subepidermal blisters- Production of autoantibodies directed against which of the following?

32 year old woman – ER – fever, intense headaches, excruciating pain in joints and muscles of the arm, took a trip to Caribbean and Cancun the Yucatan region – Temperature in 104. physical examination shows a mild macular rash on the trunk- which of the following is the route of transmission of causal organism?

Arthropod Vector

Image: 32 year old woman - ER - fever, intense headaches, excruciating pain in joints and muscles of the arm, took a trip to Caribbean and Cancun the Yucatan region - Temperature in 104. physical examination shows a mild macular rash on the trunk- which of the following is the route of transmission of causal organism?

. 1 y/o girl– multiples fractures since birth – gray translucent, pointed teeth– Xray of long bones shows osteopenia– substitution of alanine for glycine residue in type 1 collagen – which occurred in the patient as a result of this mutation?

Disruption of the secondary structure of collagen molecules

Bones Fractures

I (eyes)– blue scelera

Teeth Imperfections

E (ears) Hearing Loss

Image: . 1 y/o girl- multiples fractures since birth - gray translucent, pointed teeth- Xray of long bones shows osteopenia- substitution of alanine for glycine residue in type 1 collagen - which occurred in the patient as a result of this mutation?

55 year old – African American – asked by public relations to be filmed – patient consents to filming and broadcasting – after filming, patient states he doesn’t want it shared?

Tell the patient that he can change his mind about participating

55-year-old man with HTN comes in with severe headaches and confusion– 190/110 – faint bruit is heard over the left abdomen – Smoker for 30 years – captopril renal radionucleotide scans delayed function in kidney – Which of the following is most likely on abdominal aortography?

Left Renal Artery Atherosclerosis

55-year-old man with HTN comes in with severe headaches and confusion– 190/110 – faint bruit is heard over the left abdomen – Smoker for 30 years – captopril renal radionucleotide scans delayed function in kidney – Which of the following is most likely on abdominal aortography

Left Renal Artery Atherosclerosis

Image: 55-year-old man with HTN comes in with severe headaches and confusion- 190/110 - faint bruit is heard over the left abdomen - Smoker for 30 years - captopril renal radionucleotide scans delayed function in kidney - Which of the following is most likely on abdominal aortography

48-year-old man – increasing bronzing of skin, hepatomegaly, fatigue, and small testes. Lab findinds AST, ALT, IRON, Transferrin Increased – Testosterone, LH, FSH decreased which explains findings?

increased intestinal iron absorption

Image: 48-year-old man - increasing bronzing of skin, hepatomegaly, fatigue, and small testes. Lab findinds AST, ALT, IRON, Transferrin Increased - Testosterone, LH, FSH decreased which explains findings?

34-year-old man- evaluated for light headedness after 12 miles of marathon on hot day – blood pressure is 80/60 – what changes in autonomic nervous system?

# Impulses Carotid Baroreceptor – decreased

sympathetic efferent – INCREASED 

parasympathetic – decreased

Baroreceptor – low – low blood pressure – increased sympathetic response

70 y/o old man – fever, night sweats, productive cough, X-ray xhows cavitary lesion in the right lung– which of the cytokines play the dominant role in inflammatory response to patients lung?

interferon-y

Th1 – secrete IFN-y – macrophages

Image: 70 y/o old man - fever, night sweats, productive cough, X-ray xhows cavitary lesion in the right lung- which of the cytokines play the dominant role in inflammatory response to patients lung?

63-year-old woman – 5 day history of SOBs and swollen legs – her respirations are labored, and blood pressure is 130/50. She had large subclavian AV fistula caused by a stab wound to the left supraclavicular area 15 years ago. PE shows 2+ edema of the lower extremities – likely finding?

increased resting cardiac output

Image: 63-year-old woman - 5 day history of SOBs and swollen legs - her respirations are labored, and blood pressure is 130/50. She had large subclavian AV fistula caused by a stab wound to the left supraclavicular area 15 years ago. PE shows 2+ edema of the lower extremities - likely finding?

A female newborn – 26 weeks gestation found to have hyaline membrane disease – inspired oxygen maintain 92% and 95% – primary goal of the treatment to protect following structures?

Retina

Image: A female newborn - 26 weeks gestation found to have hyaline membrane disease - inspired oxygen maintain 92% and 95% - primary goal of the treatment to protect following structures?

A study is conducted with asthma to compare a new asthma treatment with placebo;Sample size is 100 subjects and 80%power detecting a mean difference of 0.4 in at a significance level of 5%. When planning a study, what is the meaning of 80% power

If the treatment really changes the mean asthma score 0.4, there is an 80% chance that a study of this size will find a p-value <.05

Power = When a difference exists and the null hypothesis is rejected is power.

Rx detects a mean difference of 0.4 in asthma in the patients in the treatment group, then that data falls in the 80% power range, and its significance is 95% (p<0.05

5 year old boy brought to physician for a well-child exam. He recently emigrated from Nigeria – Skin red-bronze, red hair, continual horizontal nystagmus – Genetic testing shows compound heterozygoisty of TYRP1 gene – frame-shift and nonsense mutation are detected. These genetic changes caused the following?

Lack of Choroidal pigment in macula

Image: 5 year old boy brought to physician for a well-child exam. He recently emigrated from Nigeria - Skin red-bronze, red hair, continual horizontal nystagmus - Genetic testing shows compound heterozygoisty of TYRP1 gene - frame-shift and nonsense mutation are detected. These genetic changes caused the following?

A study evaluated relationship between TV exposure and learning disability – Two groups – one with learning disability and one with the absence of learning disability (case control) – investigate using?

Odds/Ratio

used to evaluate cAse cOntrol studies A/O = odds ratio

A study evaluated relationship between TV exposure and learning disability – Two groups – one with learning disability and one with the absence of learning disability (case control) – investigate using?

Odds/Ratio

sed to evaluate cAse cOntrol studies A/O = odds ratio

Image: A study evaluated relationship between TV exposure and learning disability - Two groups - one with learning disability and one with the absence of learning disability (case control) - investigate using?

8 y/o – with ADHD – a drug with the following action is most likely to be useful for his patient?

Stimulation of the release of biogenic amine neurotransmitter

Image: 8 y/o - with ADHD - a drug with the following action is most likely to be useful for his patient?

66-year-old right-handed woman develops acute onset of spastic left hemiparesis and weakness of the lower two thirds of the face on the left- the most likely cause of the condition is a lesion at the following labeled area?

Area Labeled B

internal capsule on Right

Stroke = opposite – can wrinkle forehead

Bells Palsy= Same Side – can’t wrinkle forehead

Image: 66-year-old right-handed woman develops acute onset of spastic left hemiparesis and weakness of the lower two thirds of the face on the left- the most likely cause of the condition is a lesion at the following labeled area?

76 year old 2 week history of constant, intense pain of his right arm – 7 months ago he had cerebral infarction that resulted in lack of sensation of right side of body – Muscle strength is normal – Decreased sensation to light touch, vibration, pain, temperature on right side of the body. What is cause of the patients pain is damage to

Left Thalamus

Neuropathic Pain after stroke is central Post stroke pain Syndrome caused by contralateral thalamic lesions 

Image: 76 year old 2 week history of constant, intense pain of his right arm - 7 months ago he had cerebral infarction that resulted in lack of sensation of right side of body - Muscle strength is normal - Decreased sensation to light touch, vibration, pain, temperature on right side of the body. What is cause of the patients pain is damage to

45 year old comes to physician – progressively worsening, constant pain in left thigh – over 3 month – X ray shows thickening of the diaphysis and disruption of the cortex – focal areas of increased calcification? Photomicrograph – Which is the DX

Chondrosarcoma

Hyaline Cartilage

alignancy – pleomorphic changes + Neoplastic chondrocytes

(malignant) mainly affects the axial skeleton than the appendicular skeleton

Image: 45 year old comes to physician - progressively worsening, constant pain in left thigh - over 3 month - X ray shows thickening of the diaphysis and disruption of the cortex - focal areas of increased calcification? Photomicrograph - Which is the DX

14-year-old – regularly undergoing imaging – family history of early onset of cancers – lots of death in family from cancer – a mutation in which of the following genes is the most likely cause of this family predisposition?

TP53

60 year old with psoriasis – comes top physician because of progressive joint swelling and pain of her hands – joint tenderness and effusion – treatment with methoxsalen and ultravioletA light (PUVA) – therapy would be contraindicated in the patient if she had the following?

Porphyria Cutanea Tarda

Image: 60 year old with psoriasis - comes top physician because of progressive joint swelling and pain of her hands - joint tenderness and effusion - treatment with methoxsalen and ultravioletA light (PUVA) - therapy would be contraindicated in the patient if she had the following?

14 y/o old boy – brought to physician – daily headaches – bilateral aching of temples – mother states he hasn’t been himself. He is clumsy with frequent falls and school grades have declined.n. PE, shows Broad based + Ataxic gait. He alert and oriented to person place and time, but slow to answer questions – Chronic abuse of which substances causes patient condition?

Inhaled Glue

Image: 14 y/o old boy - brought to physician - daily headaches - bilateral aching of temples - mother states he hasn't been himself. He is clumsy with frequent falls and school grades have declined.n. PE, shows Broad based + Ataxic gait. He alert and oriented to person place and time, but slow to answer questions - Chronic abuse of which substances causes patient condition?

A 8 y/o – with ADHD – a drug with the following action is most likely to be useful for his patient?

Stimulation of the release of biogenic amine neurotransmitter

5 year old boy brought to physician for a well-child exam. He recently emigrated from Nigeria – Skin red-bronze, red hair, continual horizontal nystagmus – Genetic testing shows compound heterozygoisty of TYRP1 gene – frame-shift and nonsense mutation are detected. These genetic changes caused the following?

Lack of Choroidal pigment in macula

Eumelanin = real

Pheomelonin = Fake

Image: 5 year old boy brought to physician for a well-child exam. He recently emigrated from Nigeria - Skin red-bronze, red hair, continual horizontal nystagmus - Genetic testing shows compound heterozygoisty of TYRP1 gene - frame-shift and nonsense mutation are detected. These genetic changes caused the following?

A 47 y/o TX with Colchicine for acute gouty arthritis – drug acts by inhibiting the following in the patient’s leukocytes?

Tubulin Polymerization

Image: A 47 y/o TX with Colchicine for acute gouty arthritis - drug acts by inhibiting the following in the patient's leukocytes?

46 y/o old 2 month history of prolonged bleeding from minor cuts – mild anemia, thrombocytopenia, leukopenia – bone marrow shows lipid laden macrophages – if incidence of disease among Ashkenazi Jews in 1/900 – approximate carrier frequency of mendelin trait in this population

1/15

q^2 = 1/900 –> q = 1/30

Carrier frequency = 2pq.

rare autosomal recessive diseases the carrier frequency of 2pq ≈ 2q.

2pq = 2q = 2(1/30) = 2/30 = 1/15

48 y/o woman – fatigue, weakness, loss of appetite, weight lossParathyroid hormone concentrations are increased – X-ray shows osteopenia – subperiosteal resorption within the phalanges – which mechanism is most likely to cause the skeletal changes

Paracrine Stimulation of osteoclasts by osteoblasts

Hyperparathyroidism – bone lesions

Osteoblasts increasing RANK -L expression to bind to RANK on Osteoclasts and stimulating them —> inc Bone Resorption

Image: 48 y/o woman - fatigue, weakness, loss of appetite, weight loss. Parathyroid hormone concentrations are increased - X-ray shows osteopenia - subperiosteal resorption within the phalanges - which mechanism is most likely to cause the skeletal changes

50-year-old woman – 3 days of nausea, vomiting, upper abdominal pain radiates to back – DX with HIV infection adherent to her antiretroviral medication regimen – low CD4+. PE shows epigastric tenderness, abdominal distention, absent bowel sounds – lab studies show serum amylase activity of 450 u/L. Most likely cause of findings is adverse effect of which drug?

Didanosine

“DIDanosine causes pancreaDIDis”

23 yo woman w/ vagina sore. Sexually active, inconsistent condom use. Pelvic exam nontender ulcer on vulva. Spirochetes positive on dark field. What is biopsy histology

Obliterative Endarteritis with lymphocytes and plasma cells

Image: 23 yo woman w/ vagina sore. Sexually active, inconsistent condom use. Pelvic exam nontender ulcer on vulva. Spirochetes positive on dark field. What is biopsy histology

25-year-old woman 3-week history of bleeding gums while brushing teeth and easy bruising – Pt. delivered healthy child 9 months ago – nomedications – except for oral contraceptive – Irregular bruises are noted on legs, thighs, and arms. Numerous petechiae. Lab studies show low platelet count. Cause of her condition?

Autoantibodies against platelet glycoproteins

Immune Thrombocytopenia

Image: 25-year-old woman 3-week history of bleeding gums while brushing teeth and easy bruising - Pt. delivered healthy child 9 months ago - nomedications - except for oral contraceptive - Irregular bruises are noted on legs, thighs, and arms. Numerous petechiae. Lab studies show low platelet count. Cause of her condition?

56-year-old man – surgical resection of duodenum after injuring his small intestine in MVC– which is most likely expected to decrease after procedure?

Release of Cholecystokinin

Image: 56-year-old man - surgical resection of duodenum after injuring his small intestine in MVC- which is most likely expected to decrease after procedure?

28 year old – chronic renal failure (creatine clearance less than 20 ml/min) – requires treatment for volume overload and pulmonary edema–- Which is the most appropriate pharmacotherapy?

Furosemide

Loop diuretics – work with GFR < 30

Thiazides do not work with GFR > 30

19 year old college student – “hear voices” – which drug is most appropriate treatment of patient”

Risperidone

66-year-old – colon cancer – receiving chemo –severe diarrhea – Physician prescribes an opioid antidiarrheal agent – which opioid prescribed as antidiarrheal agent?

Loperamide

19-year-old woman ED 30 mins after falling on outstretched hand – physical tenderness of anatomical snuff-box. X-ray of wrist shows fracture of which carpal bone?

Scaphoid

Fall with outstretched hand – fracture = scaphoid

Dislocation = lunate

Image: 19-year-old woman ED 30 mins after falling on outstretched hand - physical tenderness of anatomical snuff-box. X-ray of wrist shows fracture of which carpal bone?

56-year-old man renal transplant – five minutes after graft blood vessels are anastomosed to host– he develops hemorrhage and thrombotic occlusion of the graft vessels – which is most likely mediator of the process?

IgG plus Complement

Image: 56-year-old man renal transplant - five minutes after graft blood vessels are anastomosed to host- he develops hemorrhage and thrombotic occlusion of the graft vessels - which is most likely mediator of the process?

4-year-old boy falls and lacerates lower leg to subdermal connective tissue – first step in tissue repair?

Formation of Fibrin Clot

Image: 4-year-old boy falls and lacerates lower leg to subdermal connective tissue - first step in tissue repair?

57 y/o with SOBS– Thoracentesis planned with patient seated in -– needle placed in which of the following intercostal spaces in midaxillary line?

Ninth

Image: 57 y/o with SOBS- Thoracentesis planned with patient seated in -- needle placed in which of the following intercostal spaces in midaxillary line?

While lifting weights, 24yo M develops painfuswelling in right inguinal region that cannot be reducedPhotograph shown of small intestine resected at exploratory laparotomy. Dx?

Strangulation

Indirect inguinal hernia – possible complication is strangulation

ischemia and necrosis

Image: While lifting weights, 24yo M develops painful swelling in right inguinal region that cannot be reduced. Photograph shown of small intestine resected at exploratory laparotomy. Dx?

4-week-old – male – newborn – persistent nonbilious projectile vomiting after eating – visible peristalsis – firm 2 -cm ovoid mass in right upper quadrant – likely DX?

Pyloric Stenosis

Image: 4-week-old - male - newborn - persistent nonbilious projectile vomiting after eating - visible peristalsis - firm 2 -cm ovoid mass in right upper quadrant - likely DX?

During experiment, investigator observes – Glutamate depolarizes cells through both the NMDA and nonNMDA receptros . She finds the initial depolarization phased of glutamate -induced fast excitatory post synaptic potential is generally mediated by the activation of non-NMDA type glutamate receptors – which of the following is best explanation?

NMDA receptors are blocked by Mg2+ at the resting membrane potential

45-year-old F– 1 week severe abdominal pain – history of gallstones – increased amylase and lipase – CT scan of abdomen shows enlargement of pancreas and dilation of pancreatic ducts – gallstones lodged in which of the following locations?

hepatopancreatic ampulla

ampulla of vater

Image: 45-year-old F- 1 week severe abdominal pain - history of gallstones - increased amylase and lipase - CT scan of abdomen shows enlargement of pancreas and dilation of pancreatic ducts - gallstones lodged in which of the following locations?

Investigator studying – pulmonary lymphatic flow – Which increases pulmonary lymph flow?

intravenous infusion of 0.9% saline

Image: Investigator studying - pulmonary lymphatic flow - Which increases pulmonary lymph flow?

48-year-old man with possible hypertension. On basis of ten measurements, the patient’s average diastolic blood pressure is 113, and standard deviation is 8. If four rather than ten measurements are made, which is the expected impact on 95% confidence interval?

increase in width

50-year-old – azotemia – renal ultrasound shows bilateral hydroureters and hydronephrosis – most likely cause is primary carcinoma of which of the following?

Uterine Cervix

cervical carcinoma – associated with hydronephrosis 

Image: 50-year-old - azotemia - renal ultrasound shows bilateral hydroureters and hydronephrosis - most likely cause is primary carcinoma of which of the following?

11 year old boy – persistent pain in right knee – tender distal fermur- x-ray shows osteolytic mass that he eroded through the cortex – biopsy shows hyperchromatic pleomorphic nuclei – the cells are surrounded by an eosinophilic matric – tumor most likely spreads to?

Lung

osteosarcoma.

Image: 11 year old boy - persistent pain in right knee - tender distal fermur- x-ray shows osteolytic mass that he eroded through the cortex - biopsy shows hyperchromatic pleomorphic nuclei - the cells are surrounded by an eosinophilic matric - tumor most likely spreads to?

Immunosuppressive – agent cyclosporin A – potent T lymphocyte – successfully prevent graft rejection by action of which steps in T-lymphocyte recognition/activation?

interleukin 2 gene transcription

Image: Immunosuppressive - agent cyclosporin A - potent T lymphocyte - successfully prevent graft rejection by action of which steps in T-lymphocyte recognition/activation?

FOXO transcription factor responds to insulin signaling by altering the transcription of several genes– which describes the reversible ways in which insulin signaling regulates FOXO activity

Nuclear:Cytoplasmic Shutting YES

Serine Phosphorylation: Yes.

Ubiquitin Proteolysis: NO

65 year old – TX for metastic breast cancer unresponsive to chemo – never married and has no children. Throughout her illness, another woman – introduced as “my close fiend” has been with her during hospitalization. Pt is moved to inpatient hospice – sign states only family members – Her fiends states “we can’t bear to be apart. It would be cruel to separate us now”. Response from physician?

The two of you seem to have a very important relationship. Of course you may stay together

62 year old – early Alzheimer’s – retired and lives with husbsnd – what is the mostly likely way to help patient maintain her present functional level?

instruct the patient to keep notes and lists to help her memory

40-year-old women – Streptococcus pneumonia – which of the following is produced by the cell is responsible for the restitution of normal lung function?

Surfactant

Image: 40-year-old women - Streptococcus pneumonia - which of the following is produced by the cell is responsible for the restitution of normal lung function?

55 y/o man – 2 month history of fatigue and a 3 day history of nausea and diarrhea – 5.4 kg weight loss Physical exam shows dry skin with hyperpigmentation and delayed cap refill. Lab show Na+ low. Which serum hormone decreased in patient?

Cortisol

. A study to assess normal mean serum urea nitrogen in mean ages of 65 years. Which measurement will give precise but inaccurate estimate of mean serum urea nitrogen in this group?

500 Men from a list of patients scheduled to be examined by the urologist

Image: . A study to assess normal mean serum urea nitrogen in mean ages of 65 years. Which measurement will give precise but inaccurate estimate of mean serum urea nitrogen in this group?

4 moth baby –rare autosomal recessive skeletal dysplasia involving endochondral bone. Genetic analysis shows null mutations in gene for a protein that controls traffic of vesicles into golgi complex. Which is the most likely show on findings?

dilated Rough Endoplasmic Reticulum

Image: 4 moth baby -rare autosomal recessive skeletal dysplasia involving endochondral bone. Genetic analysis shows null mutations in gene for a protein that controls traffic of vesicles into golgi complex. Which is the most likely show on findings?

64 year – cough, dyspnea, chest pain – x-ray – shows density in hilar region– biopsy shows small round cells with little cytoplasm arranged in infiltrating sheets that have neither glandular nor squamous organizations. – Which of the following abnormalities in serum?a. Hyponatremia

Hyponatremia

Image: 64 year - cough, dyspnea, chest pain - x-ray - shows density in hilar region- biopsy shows small round cells with little cytoplasm arranged in infiltrating sheets that have neither glandular nor squamous organizations. - Which of the following abnormalities in serum?a. Hyponatremia

Pt with 2 month history of foul smelling, watery diarrhea, flatulence – 10lb weight loss – after trip top Indonesia – no access to clean water –Photomicrograph of stool specimen– what is MOA of drug

Formation of Destructive Free Radicals

Metronidazole – MOA Formation of Destructive Free Radicals

Image: Pt with 2 month history of foul smelling, watery diarrhea, flatulence - 10lb weight loss - after trip top Indonesia - no access to clean water -Photomicrograph of stool specimen- what is MOA of drug

80-year-old woman cannot concentrate urine above 450 mOsmol/kg – she accumulates 450 mOsmol/day – She loses 900 mL of water and 100 in sweat and feces – minimum volume of water to prevent increase in plasma osmolality?

2.0

1 L losses from kidney + 900 mL insensible + 100 mL in sweat and feces = 2L losses -> need to ingest 2 L of water to replace.

75-year-old man dementia, Alzheimer type in nursing facility – wife and children visit weekly -trouble eating – in speaking with patients family- most appropriate for physician to address issues of feeding tube in which manners?

encourage the family to come to a consensus based on their perception of the patient’s wishes

20-year-old woman 12 weeks gestation – genetic counseling – she has a reapir for cleft lip, other members of family have cleft lip– which is mostly mode of inheritance of this disorder?

Multifactorial

Only abnormality = Cleft lip

Image: 20-year-old woman 12 weeks gestation - genetic counseling - she has a reapir for cleft lip, other members of family have cleft lip- which is mostly mode of inheritance of this disorder?

24-year-old – history of drug abuse has nausea, rhinorrhea, hypertension, and tachycardia – The most likely cause is withdrawal from which of the following agents?

Heroin

Image: 24-year-old - history of drug abuse has nausea, rhinorrhea, hypertension, and tachycardia - The most likely cause is withdrawal from which of the following agents?

35 y/o veterinarian – headache, cough, fatigue, and myalgia – after delivery of lamb – Temp 102F – Leukocyte count is 12K – X ray shows patchy infiltrates – gram stain of blood and sputum are negative for pathogens. After 2 days started on doxycycline and symptoms alleviate– Which is causal organism?

Coxiella Burnetti

Image: 35 y/o veterinarian - headache, cough, fatigue, and myalgia - after delivery of lamb - Temp 102F - Leukocyte count is 12K - X ray shows patchy infiltrates - gram stain of blood and sputum are negative for pathogens. After 2 days started on doxycycline and symptoms alleviate- Which is causal organism?

3-day old female newborn – 4 hour history of yellowing of eyes. Jaunice and mildly increased total bilirubin (14.5 n <12). New born placed on phototherapy – Which describes mechanism?

Increased conversion of bilirubin to water soluble isomers

Phototherapy = bilirubin is converted to water soluble isomers that are then able to be excreted by the kidney.

Image: 3-day old female newborn - 4 hour history of yellowing of eyes. Jaunice and mildly increased total bilirubin (14.5 n <12). New born placed on phototherapy - Which describes mechanism?

2 week male – yellow skin for 7 days – mother is obese – Newborn severely lethargic and hypoglycemic – fed with soybean formula until 4 days ago. He has been breast fed-since then. PE shows jaundice. No glucosuria – Increased serum direct bilirubin concentration-

Galactose – 1-phosphate uridyltransferase

Image: 2 week male - yellow skin for 7 days - mother is obese - Newborn severely lethargic and hypoglycemic - fed with soybean formula until 4 days ago. He has been breast fed-since then. PE shows jaundice. No glucosuria - Increased serum direct bilirubin concentration-

17-year-old T1DM – multiple admissions to hospital during past 5 months for DKA – Appears well She tells the physician that she does not take her insulin regularly – forgets or too busy. Which is most appropriate initial step?

Acknowledge patient reason for missing insulin

1. acknowledge resistance to change

2. address discrepancies between behavior and long-term goals

3. enhance motivation to change

4. nonjudgmental

A 48-year-old – complete loss of hearing in his right earWhich CN on the photograph of ventral surface of brain stem is most likely damaged?

C – right vestibulocochlear nerve – ipsilateral

Loss of hearing in right ear = right vestibulocochlear nerve – ipsilateral

Feeling and hearing loss

Start at the pontomedullary junction and count from superior to inferiorly (or medially to laterally): VI, VII, VIII, IX.

Image: A 48-year-old - complete loss of hearing in his right ear. Which CN on the photograph of ventral surface of brain stem is most likely damaged?

In a study of ultrasonography – early detection of prostate cancer – 35/50 men with prostate cancer have abnormal test results and 20 of 200 men without prostate cancer have abnormal results. What represents specificity?

80%

specificity = TN/(TN+FP) = 80/100 = 0.8 (in % will be 80%)

Image: In a study of ultrasonography - early detection of prostate cancer - 35/50 men with prostate cancer have abnormal test results and 20 of 200 men without prostate cancer have abnormal results. What represents specificity?

A 24-year-old – 3/6 systolic murmur – Aorta pressure< Left Ventricle. DX?

Aortic Stenosis

Image: A 24-year-old - 3/6 systolic murmur - Aorta pressure< Left Ventricle. DX?

Transmission of bacterial agent person-to-person contact – most likely to occur in which infectious disease?

Meningococcal Meningitis

sharing respiratory and throat secretions (saliva or spit).

Cholera = Fecal oral

Image: Transmission of bacterial agent person-to-person contact - most likely to occur in which infectious disease?

16 year old girl – ER – 30 minutes after MVC – BP 70/40 0. PE shows cold, clammy, skin. Chext Xray shows fracture of 9th rib – rupture of spleenLigation of blood supply to spleen – most likely to damage which additional structure?

Tail of Pancreas

Image: 16 year old girl - ER - 30 minutes after MVC - BP 70/40 0. PE shows cold, clammy, skin. Chext Xray shows fracture of 9th rib - rupture of spleen. Ligation of blood supply to spleen - most likely to damage which additional structure?

. A study – mean age of muscular dystrophy – Pt divide into 2 group- Normal and subnormal intelligence – Normal Age of onset 36 – Mean 3.8 – STD 1.8. If age of onset for normal (gaussian) distribution – the % of normal intelligence who have onset at less than 9 years of age is closest to which of the following?

99%

Therefore, since we know +/- 2 SD covers 95% of the bell curve, it must be higher than that. The only option higher than 95% is 99%

Image: . A study - mean age of muscular dystrophy - Pt divide into 2 group- Normal and subnormal intelligence - Normal Age of onset 36 - Mean 3.8 - STD 1.8. If age of onset for normal (gaussian) distribution - the % of normal intelligence who have onset at less than 9 years of age is closest to which of the following?

A 2 week old F – Mexican – well child exam – baby is inconsolable after feeding with formula – Mother states “I am sure she has the evil eye”. Mother has been passing an egg over the baby’s body– PE shows no abnormalities – Which is most appropriate physician statement?

Why don’t you continue with what you are doing, and let’s try changing her to a different formula.

12 y/o F – tonic-clonic seizure. Pt has mild mental retardation, had surgery for lens dislocationCystathione concentration decreased. A CT scan shows a small infarction of right cerebral hemisphere. Which serum amino acid concentration is increased?

Methionine

HOMOCYstinuria

Homocysteine in urine,

Osteoporosis,

Marfanoid habitus,

Ocular changes (lens subluxation),

Cardiovascular effects (thrombosis and atherosclerosis stroke and MI),

kYphosis,

intellectualdisability, fair complexion.

Image: 12 y/o F - tonic-clonic seizure. Pt has mild mental retardation, had surgery for lens dislocation. Cystathione concentration decreased. A CT scan shows a small infarction of right cerebral hemisphere. Which serum amino acid concentration is increased?

43 y/o woman – fainted 3x in past 5 months. Weakness in face and right arm. Vital signs in normal limits. Cardiac exam – mild – grade 2/6 diastolic murmur over the apex – murmur louder when patient rises from prone position. Cause of patient syncope?

Left Atrial Myxoma

Left atrial myxoma can mimic mitral stenosis diastolic murmur with multiple syncopal episodes

triad of

1) mitral valve obstruction (i.e. malaise, symptoms of cardiac failure, syncope, etc.),

2) symptoms of embolism (i.e. facial and right arm hemiparesis in patient),

3) constitutional symptoms (fever, weight loss)

Image: 43 y/o woman - fainted 3x in past 5 months. Weakness in face and right arm. Vital signs in normal limits. Cardiac exam - mild - grade 2/6 diastolic murmur over the apex - murmur louder when patient rises from prone position. Cause of patient syncope?

36 y/o – genetic counseling with husband. No family history of genetic syndromes – They have read that risk of being carriers of autosomal recessive neurodegenerative disorder that occurs in 1/10,000. Which represents carrier frequency?

1 of 50

Hardy-Weinberg equation = 1= P^2 + 2pq + q^2

Carriers q^2 = 1/10,000

Square root. = 1/10,0000 = 1/100

Then remember P + q = 1 ——> 1/100 + p = 1 (p = 99/100)

Lastly plug back into Hardy-Weinberg Equation as:

2pq = Heterozygote carrier

(2 x 1/100 x 99/100 = 2/100 = 1/50)

Image: 36 y/o - genetic counseling with husband. No family history of genetic syndromes - They have read that risk of being carriers of autosomal recessive neurodegenerative disorder that occurs in 1/10,000. Which represents carrier frequency?

32 y/o man – lives at sea level – travels to mountains –develops SOBS om arrival – then develops exercise tolerance increases after 2 weeks. If this patient’s bone marrow were examined, it would most likely show increased numbers of which cells?

Normoblasts

Image: 32 y/o man - lives at sea level - travels to mountains -develops SOBS om arrival - then develops exercise tolerance increases after 2 weeks. If this patient's bone marrow were examined, it would most likely show increased numbers of which cells?

32 year old –Diastolic pressure between 100 and 110 mm Hg. Healthy and is not taking any meds. Serum concentrations are normal. Urine contains only trace amounts of protein and no albumin. He is unlikely to develop peripheral tissue edema because of regulatory adjustments in which of the following?

Precapillary Resistance

Systolic BP is determined by Cardiac Output

Diastolic BP is determined by arterioles 

Image: 32 year old -Diastolic pressure between 100 and 110 mm Hg. Healthy and is not taking any meds. Serum concentrations are normal. Urine contains only trace amounts of protein and no albumin. He is unlikely to develop peripheral tissue edema because of regulatory adjustments in which of the following?

A 28 yesr old man – High triglycerides – responds to gemfibrozil – Nicotinic acid therapy started. Which of the drugs administered along with Nicotinic Acid to prevent most common adverse effect of therapy?

Aspirin

A 45 y/o woman – undergo a lymph biopsy – Surgery resident asks 3rd to get a signed consent before the procedure is preformed – Which action by student is most appropriate?

Accompany resident while the resident obtains signed consent

35 y/0 F – undergoes vaginal delivery – A median episiotomy is done to hasten delivery. During procedure, an incision is made posteriorly from the posterior vaginal commissure. Which muscle is greatest risk of damage if the incision is torn?

External Anal Sphincter

Perineum – Anal sphincter – Rectum

Image: 35 y/0 F - undergoes vaginal delivery - A median episiotomy is done to hasten delivery. During procedure, an incision is made posteriorly from the posterior vaginal commissure. Which muscle is greatest risk of damage if the incision is torn?

A 74 y/o with COPD -breath sounds are decreased and distant auscultation of chest. In addition to albuterol, which is the most appropriate pharmacotherapy?

Ipratropium

(“I pray I can breathe soon!”)

Muscarinic antagonist – 241-1

A 65 y/o with erectile dysfunction. Most appropriate for physician to prescribe a drug that increases the actions of cGMP in which of the following cells types in the penis?

Smooth muscle of deep artery

Image: A 65 y/o with erectile dysfunction. Most appropriate for physician to prescribe a drug that increases the actions of cGMP in which of the following cells types in the penis?

A 40 y/o with pins and needles sensation starts in his left hand and then spreads to his left arm and face. Episodes last for 1 minute. PE show no neuro abnormalities. The most likely cause of these symptoms is a lesion in which of the following gyri of the cerebral cortex?

Postcentral

primary somatosensory cortex, the main sensory receptive area for the sense of touch

Image: A 40 y/o with pins and needles sensation starts in his left hand and then spreads to his left arm and face. Episodes last for 1 minute. PE show no neuro abnormalities. The most likely cause of these symptoms is a lesion in which of the following gyri of the cerebral cortex?

33-year-old woman – she has celiac sprue and fat malabsorption. PE shows asthenia. Lab shows serum calcium concentration of 7.8 mg/dl. Which additional sets of serum findings is most likely in this this patient?

Phosphate Decreased

PTH Increased

Calcitriol decreased

celiac sprue = fat malabsorption = Vit KADE

Vitamin D malabsorption = decreased serum calcium—decreased

Vit D Deficiency = ca decreased, Phosphate decreased, PTH increased

Image: 33-year-old woman - she has celiac sprue and fat malabsorption. PE shows asthenia. Lab shows serum calcium concentration of 7.8 mg/dl. Which additional sets of serum findings is most likely in this this patient?

A 40 y/o comes to physician for health maintenance exam. Concerned about male pattern baldness. Which of the following treatments is most appropriate medication for patient?

Finasteride

5α-reductase inhibitor (conversion of testosterone to DHT).

Image: A 40 y/o comes to physician for health maintenance exam. Concerned about male pattern baldness. Which of the following treatments is most appropriate medication for patient?

A 6-month-old – failure to thrive and intractable diarrhea that hes worse when he feeds. PE shows eczematous dermatitis and mild lymphadenopathy. Mutation is FoxP3, a transcriptional protein. Most likely cause of patients condition is a decrease in which T-lymphocyte?

Regulatory T Lymphocyte

IPEX

Image: A 6-month-old - failure to thrive and intractable diarrhea that hes worse when he feeds. PE shows eczematous dermatitis and mild lymphadenopathy. Mutation is FoxP3, a transcriptional protein. Most likely cause of patients condition is a decrease in which T-lymphocyte?

35 y/o comes to 4-month history persistent cough and intermittent nasal discharge.. He often awakened from sleep with shortness of breath, wheezing, dry cough, chest tightness. Vital signs are within normal limits. PE shows no abnormalities. The most appropriate drug blocks the receptors for endogenous substances that is a metabolite of which of the following?

Arachidonic Acid

precursor of leukotrienes = arachidonic acid.

Image: 35 y/o comes to 4-month history persistent cough and intermittent nasal discharge.. He often awakened from sleep with shortness of breath, wheezing, dry cough, chest tightness. Vital signs are within normal limits. PE shows no abnormalities. The most appropriate drug blocks the receptors for endogenous substances that is a metabolite of which of the following?

A 44 y/o female – steatorrhea, flatulence, and a 22lb weight loss over the past 6t months. MCV low. Negative stool occult blood. Stool cultures and exam are negativge. A lateral x-ray shows mild osteopenia. Which is most likely explanation?

Celiac Sprue

decreased absorption in distal duodenum and proximal jejunum – 381-1

iron deficiency anemia

Image: A 44 y/o female - steatorrhea, flatulence, and a 22lb weight loss over the past 6t months. MCV low. Negative stool occult blood. Stool cultures and exam are negativge. A lateral x-ray shows mild osteopenia. Which is most likely explanation?

44 y/o man – farmer – spraying insecticides – brought to ER – because of vomiting, diarrhea, wheezing, and sweating for 12 hours. He has excess salivation and muscles weakness. Confused and pupils are small. A drug acting at which of the following labeled sites is most appropriate?

Area labeled D – atropine (antimuscarininic)

Organophosphates – insecticides, and function by binding acetylcholinesterase and “deactivating”

Image: 44 y/o man - farmer - spraying insecticides - brought to ER - because of vomiting, diarrhea, wheezing, and sweating for 12 hours. He has excess salivation and muscles weakness. Confused and pupils are small. A drug acting at which of the following labeled sites is most appropriate?

44-year-old man – sustained injury in a bar fight. Multiple facial fractures and contusions, including several swollen right eye. Lots of edemas. Pt refuses to open eye and has double vision. CT scan shown. Which is most likely explanation of patients visual complaints?

Entrapment of the inferior rectus muscle

Image: 44-year-old man - sustained injury in a bar fight. Multiple facial fractures and contusions, including several swollen right eye. Lots of edemas. Pt refuses to open eye and has double vision. CT scan shown. Which is most likely explanation of patients visual complaints?

65 year old man – ER because of substernal chest discomfortWhich of the following eicosasnoids is most likely contributing to symptoms?

Thromboxane A2

Image: 65 year old man - ER because of substernal chest discomfort. Which of the following eicosasnoids is most likely contributing to symptoms?

A man touch surface of hot stove –20 minutes later. A blister develops at the site. Biochemical analysis of the blister fluid at this time is most likely to show the presence of which?

Histamine

Immediate Type 1 Hypersensitive – 20 minutes

Sattar – increase capillary permeability in the postcapillary venules and vasodilation in the arterioles as actions of histamine.

Image: A man touch surface of hot stove -20 minutes later. A blister develops at the site. Biochemical analysis of the blister fluid at this time is most likely to show the presence of which?

A 40 y/o m an with terminal small cell carcinoma – Na+ 120 mEq. Which is the most appropriate therapy?

Demeclocycline

Image: A 40 y/o m an with terminal small cell carcinoma - Na+ 120 mEq. Which is the most appropriate therapy?

61 y/o woman – health maintenance exam. She has split S1 over left sternal border. Which explains first component of this sound?

Closure of the mitral valve

First heart sound (S1) = two heart valves: the mitral valve and tricuspid valve.

Image: 61 y/o woman - health maintenance exam. She has split S1 over left sternal border. Which explains first component of this sound?

65 year old – progressive ataxia with gait unsteadiness. There is an intention tremor on right side. Most likely cause is damage to which of the following areas?

Right Cerebral Hemisphere

Patient presents with both ataxia and an intention tremor on the right-hand side

cerebellar lesions like we see here always present IPSILATERALLY as opposed to many other CNS lesions. – Double crosser

Image: 65 year old - progressive ataxia with gait unsteadiness. There is an intention tremor on right side. Most likely cause is damage to which of the following areas?

7 year old – 2 day history of itchy red rash on right arm and trunk after a hike in woods. Rash started 1 day after trip. PE shows 2-cm patches of raised, red, elongated blisters that ooze clear fluid on the upper extremities and trunk. The blisters are linear with patches of erythema. Which describes the mechanism of patients reaction?

Release of interferon gamma Th1 Cells

Image: 7 year old - 2 day history of itchy red rash on right arm and trunk after a hike in woods. Rash started 1 day after trip. PE shows 2-cm patches of raised, red, elongated blisters that ooze clear fluid on the upper extremities and trunk. The blisters are linear with patches of erythema. Which describes the mechanism of patients reaction?

60 y/o man – 1-month history of headaches and dizziness. His blood pressure is 160/85. PE exam shows moderate splenomegaly. Lab shows – MCV low, Lymphocytes 10%. Red Cell distribution Width increased. Which is most likely decreased in patient?

Erythropoietin’a. 

Low EPO in Polycythemia Vera due to Negative feedback on EPO release by kidney.

Image: 60 y/o man - 1-month history of headaches and dizziness. His blood pressure is 160/85. PE exam shows moderate splenomegaly. Lab shows - MCV low, Lymphocytes 10%. Red Cell distribution Width increased. Which is most likely decreased in patient?

A female neonate is delivered 38 weeks gestation. PE shows macrocephaly and shortening of the proximal segments of all limbs. A DX of achondroplasia. DNA sequence of sows a G-A mutations in the fibroblast growth factor gene on chromosome 4. Biosynthesis of this protein requires functional signal recognition particle receptor. This receptor is most likely found at the cellular location?

Rough Endoplasmic Reticulum

Image: A female neonate is delivered 38 weeks gestation. PE shows macrocephaly and shortening of the proximal segments of all limbs. A DX of achondroplasia. DNA sequence of sows a G-A mutations in the fibroblast growth factor gene on chromosome 4. Biosynthesis of this protein requires functional signal recognition particle receptor. This receptor is most likely found at the cellular location?

60 y/o T2 DM – 6 week history of right shoulder pain. The pain occurs when attempts to raise her arm when lying on right. No weakness, tingling, or numbness. PE shows tenderness to in the region of deltoid muscle just lateral to acromioclavicular joint. While sitting upright, she is able to abduct her arm to nearly 90 degrees, but additional abduction is prevented by pain. Muscle strength is 5/5 in right upper extremity, sensation intact, there is no impingement sign. DX?

Subacromial Bursitis

Image: 60 y/o T2 DM - 6 week history of right shoulder pain. The pain occurs when attempts to raise her arm when lying on right. No weakness, tingling, or numbness. PE shows tenderness to in the region of deltoid muscle just lateral to acromioclavicular joint. While sitting upright, she is able to abduct her arm to nearly 90 degrees, but additional abduction is prevented by pain. Muscle strength is 5/5 in right upper extremity, sensation intact, there is no impingement sign. DX?

15 year old boy – well child exam – if patient develops severe acne vulgaris that resolves over the next 3 years, a rapid increase in the secretion of which of the following substances will have been the most likely cause of acne?

Testosterone

Androgens cause acne

Image: 15 year old boy - well child exam - if patient develops severe acne vulgaris that resolves over the next 3 years, a rapid increase in the secretion of which of the following substances will have been the most likely cause of acne?

An 85 y/o man – progressive forgetfulness for 3 years. Mental status exam shows difficulty learning and recalling new verbal and nonverbal material. Which is appropriate pharmacotehrapy?

Donepezil

ACh-esterase inhibitor.

Alzheimer disease (DonaRiva dancesat the gala)

Investigators conduct a prospective, community based study to assess the effectiveness of a intervention. Which sets of characteristics best supports causal relationship between the intervention and child language scores?

Strength of association, temporal relationship. Dose-response gradient

An otherwise healthy 26 y/o scuba diver – difficulty breathing after ascending from a 30 minute dive at 15m. Which physiological signs is most consistent with the x-ray of chest scan?

Hyperresonance over most of the left side of the chest

a. Decompression sickness (DCS, or “the bends“) involves gases … Pulmonary barotrauma may result in a collapsed lung (pneumothorax).

Image: An otherwise healthy 26 y/o scuba diver - difficulty breathing after ascending from a 30 minute dive at 15m. Which physiological signs is most consistent with the x-ray of chest scan?

Investigator studying outbreak of virus in skilled nursing home. Virus infectivity is destroyed by heating to 60 C for 30 minutes. Based on this find, the virus has which characteristics?

Enveloped

76 y/o woman – brought to physician by her son who is concerned that she is forgetful. He thinks she should be placed in a long term care facility. She appears cheerful. Which is most crucial in determining her decision-making capacity?

She expresses clear and consistent values

An 18 month -old girl – failure to thrive and frequent wet diapers. 3rd% for length and weight. PE shows dehydration, decreased muscle tone, and bowing of legs. Urine studies down ph of 5, glucosuria, phosphaturia, and generalized aminoaciduria. Which is cause of metabolic acidosis in this patient?

Decreased sodium bicarbonate reabsorption in the proximal tubule

A 42 y/o comes to the office because of a 1-year history of an enlarging, painless mass on her right wrist. Exam of the right show in photograph. The mass is firm nontender and transilluminates with the application of a pen light. Which of the following is most likely to occur?

Spontaneous Regression

Image: A 42 y/o comes to the office because of a 1-year history of an enlarging, painless mass on her right wrist. Exam of the right show in photograph. The mass is firm nontender and transilluminates with the application of a pen light. Which of the following is most likely to occur?

ECG of heart study in anesthetized experimental animal. ECG shows (bradycardia). Which of the following procedures most likely to produce ECG shown?

Ablation of the atrioventricular node

Image: ECG of heart study in anesthetized experimental animal. ECG shows (bradycardia). Which of the following procedures most likely to produce ECG shown?

A study is planned to assesses HTN control between patients in the USA and Western Europe. Study population includes 21,053 patients with HTN receiving 291 cardiologist and 1284 primary care physicians. The main outcome measure is HTN control defined as a most recent blood pressure reading less than 140/90. Which describes study design?

Cross-sectional study

Image: A study is planned to assesses HTN control between patients in the USA and Western Europe. Study population includes 21,053 patients with HTN receiving 291 cardiologist and 1284 primary care physicians. The main outcome measure is HTN control defined as a most recent blood pressure reading less than 140/90. Which describes study design?

A 2-year-old boy – ER department – severe abdominal pain and vomiting for 3 hours – PE shows yellowish white papules on back, abdominal tendernes, and hepatosplenomegaly. Serum studies show an increased in amylase activity and increased total cholesterol, triglycerides, and chylomicrons. This patient most likely has a deficiency in which activity

Lipoprotein Lipase

Image: A 2-year-old boy - ER department - severe abdominal pain and vomiting for 3 hours - PE shows yellowish white papules on back, abdominal tendernes, and hepatosplenomegaly. Serum studies show an increased in amylase activity and increased total cholesterol, triglycerides, and chylomicrons. This patient most likely has a deficiency in which activity

A 16-year-old – 4-month history of nonbloody diarrhea. Mother concerned she is abusing laxatives to maintain a low weight. BME is 19. BP 89/61. Which is serum electrolyte concentration?

Potassium decreased

Chloride increased

Bicarbonate decreased

Diarrhea causes non-anion gap (i.e. hyperchloremic) metabolic acidosis

excessive diarrhea = excessive loss of HCO3- and K+.

‘Chloride levels in the serum will be increased due to the normal HCO3-/Cl- equilibrium, so as

42-/o history of recurrent pyelonephritis – 2-day history of left flank pain, fever, and chills. Temp is 101.8F. He appears anxious and in moderate distress/. PE sows left costovertebral tenderness. Urine is cloudy with a pH of 7.3. CT shows a large renal calculus (photograph). What type of renal calculus?

Magnesium Ammonium Phosphate

Staghorn Calculus in adults

Image: 42-/o history of recurrent pyelonephritis - 2-day history of left flank pain, fever, and chills. Temp is 101.8F. He appears anxious and in moderate distress/. PE sows left costovertebral tenderness. Urine is cloudy with a pH of 7.3. CT shows a large renal calculus (photograph). What type of renal calculus?

63-year-old woman – 2-month history of progressive shortness of breath with exertion. A chest x-ray shows a mass pressing against the outside of the trachea. Pulmonary function is most likely to show which of the following?

FVC decreased

FEV1/FVC decreased

Peak Expiratory Flow Rate decreased

Peak Inspiratory Flow Rate decreased

Image: 63-year-old woman - 2-month history of progressive shortness of breath with exertion. A chest x-ray shows a mass pressing against the outside of the trachea. Pulmonary function is most likely to show which of the following?

A cohort study to examine the association between cigarette smoking an lung cancer in 4000 people. 1000 smoke cigs and 3000 do not smoke. After 10 years, 30 cases of lung cancer in smokes and 30 cases of lung cancer among nonsmokers. According to results, how many lung cancers attributable to smoking would be expected in a group of 10,000 smokers over the same period?

200

Attributable risk = incidence in exposed – incidence in unexposed

=30/1,000 (smokers) -30/3,000 (nonsmokers)=0.03-0.01= 0.02 (so the attributable risk is about 2%)

Applying it to a population of 10,000:

=0.02*10,000= 200

Image: A cohort study to examine the association between cigarette smoking an lung cancer in 4000 people. 1000 smoke cigs and 3000 do not smoke. After 10 years, 30 cases of lung cancer in smokes and 30 cases of lung cancer among nonsmokers. According to results, how many lung cancers attributable to smoking would be expected in a group of 10,000 smokers over the same period?

An 8 y/o boy brought to physician by his mother -noticed a lump on left side of neck– Exam shows pectus excavatum. 2 mm subcutaneous nodules on lips, and irregular 2cm mass on neck, anterior cervical lymphadenopathy. 14 year sisyer – similar findings at age 10. Which studies support diagnosis?

Serum Calcitonin Concentration

Marfanoid habitus + Mucosal neuromas + Neck mass =

MEN 2B (PMM) Pheochromocytoma

Medullary thyroid CA (Calcitonin secreting), mucosal neuromas

Image: An 8 y/o boy brought to physician by his mother -noticed a lump on left side of neck- Exam shows pectus excavatum. 2 mm subcutaneous nodules on lips, and irregular 2cm mass on neck, anterior cervical lymphadenopathy. 14 year sisyer - similar findings at age 10. Which studies support diagnosis?

Which of the following pairs of drugs interferes with DNA synthesis by crosslinking?

Lomustine and cyclophosphamide

Image: Which of the following pairs of drugs interferes with DNA synthesis by crosslinking?

A 75 year old woman – stress urinary incontinence with sphincteric incompetence. Pelvic floor muscle exercise and the use of vaginal pessary provide inadequate improvement. Administration of which of the following mechanism of action is most appropriate?

Stimulation of a-adrenoreceptors

α1 stimulation (via α1 agonist) constricts the bladder sphincter thereby, preventing sudden bouts of micturition during coughing/sneezing

35-year-old woman – 6 month history of fever, fatigue, and unexplained weight loss. She has had 40lb weight loss. Lab show CD4 T lymphocytes 60. The most appropriate treatment includes a drug that targets which of the following steps of viral replication?

Synthesis of viral DNA

Image: 35-year-old woman - 6 month history of fever, fatigue, and unexplained weight loss. She has had 40lb weight loss. Lab show CD4 T lymphocytes 60. The most appropriate treatment includes a drug that targets which of the following steps of viral replication?

A female newborn – respiratory distress – PE shows intercostal retractions. A chest x-ray shows multiple rib fractures. Skeletal x-rays show multiple fractures of the long bones. There is no family of history of recurrent fractures. Most likely diagnosis?

Osteogenesis Imperfecta, type I

Image: A female newborn - respiratory distress - PE shows intercostal retractions. A chest x-ray shows multiple rib fractures. Skeletal x-rays show multiple fractures of the long bones. There is no family of history of recurrent fractures. Most likely diagnosis?

A 42 y/o man – intermittent high-grade fever, dizziness, diarrhea, and fatigue. 5kg weight loss. During trip to India, bitten by an insect. He states sore developed at the site of bite. Temperature is 100.4. PE shows splenomegaly and muscle wasting. Lab studies show pancytopenia. Which vector of patient’s infection?

Sand Fly

Visceral Leishmaniasis

Image: A 42 y/o man - intermittent high-grade fever, dizziness, diarrhea, and fatigue. 5kg weight loss. During trip to India, bitten by an insect. He states sore developed at the site of bite. Temperature is 100.4. PE shows splenomegaly and muscle wasting. Lab studies show pancytopenia. Which vector of patient's infection?

Which of the following types of vaccine is most appropriate to prevent mumps?

Live attenuated virus

Killed = Rest In Peace Always

RabiesInfluenzaPolio(Salk)Hepatitis A

Image: Which of the following types of vaccine is most appropriate to prevent mumps?

45-year-old – right sided abdominal pain. She pain after high-fat meals. Which is most likely on abdominal exam in patient?

Inspiratory pause on right upper quadrant palpation

Image: 45-year-old - right sided abdominal pain. She pain after high-fat meals. Which is most likely on abdominal exam in patient?

A 21 year old – 1 month history of fever and abdominal pain; he also had a 6.8 kg (15-lb) weight loss during this periodHe returned from Middle East 3 weeks ago. PE shows generalized lymphadenopathy and hepatosplenomegaly. Lab studies show pancytopenia. Cause?

Leishmania

Image: A 21 year old - 1 month history of fever and abdominal pain; he also had a 6.8 kg (15-lb) weight loss during this period. He returned from Middle East 3 weeks ago. PE shows generalized lymphadenopathy and hepatosplenomegaly. Lab studies show pancytopenia. Cause?

. A 50-year-old from Minnesota – 3 day history of fever and malaise. Test for Anaplasma phagocytophilum is positive. Causal organism is transmitted via the same vector as which of the following?

Babesia microti and Borrelia Burgdorferi

Image: . A 50-year-old from Minnesota - 3 day history of fever and malaise. Test for Anaplasma phagocytophilum is positive. Causal organism is transmitted via the same vector as which of the following?

55-year-old – construction worker – bilateral basilar, fine, end-inspiratory crackles. A chest X-ray shows reticular pattern and bilateral diaphragmatic pleural plaques. No atypical cells. Pulmonary function test is likely to show?

FEV1/FVC normal

FVC decreased

DLCO decreased

Image: 55-year-old - construction worker - bilateral basilar, fine, end-inspiratory crackles. A chest X-ray shows reticular pattern and bilateral diaphragmatic pleural plaques. No atypical cells. Pulmonary function test is likely to show?

3-month-old girl – 2 month history of enlarging red spot on her abdomen – She appears well nourished– A photograph of the lesion is shown. Microscopic exam of the lesion is most likely to show which of the following?

Densely packed capillaries

Strawberry hemangioma

Image: 3-month-old girl - 2 month history of enlarging red spot on her abdomen - She appears well nourished- A photograph of the lesion is shown. Microscopic exam of the lesion is most likely to show which of the following?

A 44 y/o woman – Pap smear – dysplastic columnar ciliated cells. If the patient were healthy. Columnar cells would most likely originate from which of the following areas?

Cervical Canal

Transformation Zone

Endocervix= simple columnar epithelium

Vagina = Stratified squamous

Image: A 44 y/o woman - Pap smear - dysplastic columnar ciliated cells. If the patient were healthy. Columnar cells would most likely originate from which of the following areas?

65 year old with DKA – fever and proptosis of left eye for 3 days- A black eschar is present of nasal mucosa. Which is likely DX?

Mucormycosis

Image: 65 year old with DKA - fever and proptosis of left eye for 3 days- A black eschar is present of nasal mucosa. Which is likely DX?

36 y/o woman- BP 180/120 – CT scan shows renal artery aneurysm. PE shows no other abnormalities. A CT scan of abdomen renal angiogram obtained from femoral approach. (String of beads) Which is most likely cause of findings?

Fibromuscular Dysplasia

‘string-of-beads’

Image: 36 y/o woman- BP 180/120 - CT scan shows renal artery aneurysm. PE shows no other abnormalities. A CT scan of abdomen renal angiogram obtained from femoral approach. (String of beads) Which is most likely cause of findings?

A 2-week-old girl – asymmetric left pupil Ophthalmologic exam shows coloboma. Incomplete closure of the embryonic fissure is limited to which of the following structures of the left eye most likely to spare this patient vision?

Iris

“keyhole” appearance of the pupil, generally do not lead to vision loss.

Image: A 2-week-old girl - asymmetric left pupil Ophthalmologic exam shows coloboma. Incomplete closure of the embryonic fissure is limited to which of the following structures of the left eye most likely to spare this patient vision?

A 44 year old – invasive pancreatic cancer – 2 week history of severe right side back and abdominal pain. 6.8 weight loss. PE shows no abnormalities. An operation is scheduled to relive pain. Most likely target of pain relief is which of the following labeled sites in cross section of spinal cord?

Area Labeled H

Pain & temperature fibers for the right side come in on the dorsal right side, cross at the anterior white commissure, and travel up in the Spinothalamic tract.

Image: A 44 year old - invasive pancreatic cancer - 2 week history of severe right side back and abdominal pain. 6.8 weight loss. PE shows no abnormalities. An operation is scheduled to relive pain. Most likely target of pain relief is which of the following labeled sites in cross section of spinal cord?

62-year-old woman – brought to physician because her left foot has become cool and discolored. She has had difficulty walking – Symptoms began 4 hours ago with several acute episodes of palpitations followed by tingling and numbness of the left foot several hours later. Which is the most likely explanation for findings?

Embolus to the femoral artery with left leg ischemia

Cool, discolored foot with tingling and numbness = blood clot. Cardiac symptoms may indicate that she has some sort of thrombotic disease

. An antiserum is elicited by immunization with protein X and anti-X antibodies are absorbed to microtiter wells. Graph shows the amount of radiolabeled protein X bound by antiserum of varying concentrations by protein YWith respect to ant-X serum – which of the following interpretations of the epitopes expressed by the proteins?

Proteins x and Y have no epitopes in common

Image: . An antiserum is elicited by immunization with protein X and anti-X antibodies are absorbed to microtiter wells. Graph shows the amount of radiolabeled protein X bound by antiserum of varying concentrations by protein Y. With respect to ant-X serum - which of the following interpretations of the epitopes expressed by the proteins?

18 year old woman – progressive muscle weakness, poor exercise tolerance, shortness of breath. She has bilateral ptosis – Examination shows abnormal accumulations of mitochondria in subsarcolemmal regions. Which additional set of findings is most likely?

Max Oxygen Consumption decreased

Venous Blood Lactate increased

Energy Production by Glycolysis increased

21 year old man ER – 30 minutes after sustaining knife wound in abdomen – He is in shock. He undergoes immediate operative repair of small intestine and bleeding wound to the liver. 6 months later he is admitted to hospital for treatment of intestinal obstruction. During the corrective abdominal operation, it is observed that his previous injury has regenerated without any evidence of fibrous scarring. Which of the following sites is most likely being described?

Liver

70 y/o man with severe congestive heart failure and orthopnea. During this time, urine output is decreased to 300 ml/dayWhich of the following sets of laboratory finds is most likely?

BUN 40

Creatinine 2

Urine Na 5+

Specific Gravity 1.025

Prerenal azotemia, BUN:Cr ratio is >= 20;

Activation of the RAAS system

spec grav is high – holding onto sodium so urinary sodium will be low (<20, FENa <1%).

Image: 70 y/o man with severe congestive heart failure and orthopnea. During this time, urine output is decreased to 300 ml/day. Which of the following sets of laboratory finds is most likely?

A 26-year-old woman comes to the physician because of progressive itchy red rash on her arms. She works at an animal clinic. She was scratched on both arms while caring for litter of kittens. PE shows annular erythematous plaques with central clearing and advancing scaly borders. There is no regional lymphadenopathy. Most likely diagnosis?

Tinea Corporis

ring worm from cat

Image: A 26-year-old woman comes to the physician because of progressive itchy red rash on her arms. She works at an animal clinic. She was scratched on both arms while caring for litter of kittens. PE shows annular erythematous plaques with central clearing and advancing scaly borders. There is no regional lymphadenopathy. Most likely diagnosis?

56-year-old man – severe pain in great toe – He is allergic to aspirin. Microscopic exam shows joint fluid shows negatively birefringent crystals. Which is most appropriate acute treatment?

Colchicine

A prospective study is done to assess two different antihypertensive meds. In the study, patients were randomly assigned to two groups. At end of study, some participants reported inconsistent adherence. In their primary analysis, which of the following methods should the investigators use to analyze data from patients who were not adherent to regime?

Include the outcome for each participant in the group to which he or she was randomized
intention to treat

A 2-year-old girl – cross eyed – PE shows moderate strabismus – if not treated, she will most have deficits in depth perception due to lack of appropriate competitive interactions in visual cortex. Calcium entry – mediates outcomes of this competitive process?

N-Methyl-D-aspartate (NMDA)

A 61-year-old man – prescribed fluoxetine for depression. This drug has its initial effects on neurons in which of the following structures?

Raphe Nuclei
a string of nuclei in the midline of the midbrain and brainstem that contain most of the serotonergic neurons of the brain

Image: A 61-year-old man - prescribed fluoxetine for depression. This drug has its initial effects on neurons in which of the following structures?

45 year old man – found unconscious in the desert – 24 hours ago – he had been hiking and lost consciousness. Pulse is 124 min and BP 80/40. PE shows multiple ecchymoses on the upper and lower extremities. Which explains hypotension?

Hypovolemic Shock
dehydration since – in the desert.

A 22 year old – brought to ED 30 minutes after a friend found him unconscious. Drug overdose is suspected. He is intubated using PEEP – which of the following sets of findings best describes effects of PEEP?

Peep Inspiratory Pa. Positive
End-Tidal Pa Positive
Peak Inspiratory Positive
End tidal Volume Positive

Image: A 22 year old - brought to ED 30 minutes after a friend found him unconscious. Drug overdose is suspected. He is intubated using PEEP - which of the following sets of findings best describes effects of PEEP?

40-year-old – enlarging breast tissue – He is taking a diuretic – Which drug is most likely causes adverse effect?

a. Spironolactone

Image: 40-year-old - enlarging breast tissue - He is taking a diuretic - Which drug is most likely causes adverse effect?

A 68-year-old man – treated with NSAIDs for RA – has severe gastric burning and discomfort. A second drug decrease these adverse effects, but the patient develops severe diarrhea. The second drug most likely acts at which site?

Area Labeled B
PGE analog -misoprostol – create a barrier

A 65 year old – scheduled for physical therapy 3 days following shoulder operation. As part of regime – he begins to strengthen the subscapularis muscle. Which of the following movements of the arm should this patient perform against resistance?

Medial (internal) rotation
subscapularis muscle is very important for the Internal rotation of the humerus. The internal rotation supports the upper arm during abduction and adduction.

Image: A 65 year old - scheduled for physical therapy 3 days following shoulder operation. As part of regime - he begins to strengthen the subscapularis muscle. Which of the following movements of the arm should this patient perform against resistance?

A 65 y/o recovering from cerebral infarction due to atherosclerotic vascular disease. Clopidogrel is added to medication regime because of intolerance to aspirin. Which is most like mechanism of decreased platelet aggregation due to clopidogrel?

Preventing ADP-stimulated platelet activation

Image: A 65 y/o recovering from cerebral infarction due to atherosclerotic vascular disease. Clopidogrel is added to medication regime because of intolerance to aspirin. Which is most like mechanism of decreased platelet aggregation due to clopidogrel?

A child with septiciemia has an antibiotic clearance (CL) of .09 L/hr/kg, the steady state concentration is 12ug/Ml. Which of the following is maintenance dose (in mg/kg/day)?

28.8
Maintenance dose formula is (Css × Cl × tau) ÷ F

=(12 ug/mL × 1 mg/1000 ug) × (0.09 L/hr/kg × 1000 mL/1 L × 24 hr/1 day)= 25.92 mg/kg/day

Which of the following best explains impaired action potentials in the affected axons of demyelinating diseases?

Increase in axonal capacitance
“myelin speeds the transmission of electrical impulses called action potentials along myelinated axons by insulating the axon and reducing axonal membrane capacitance”

Image: Which of the following best explains impaired action potentials in the affected axons of demyelinating diseases?

47-year-old with a goiter for 10 months. Adequate iodine in diet. Antithyroid peroxidases antibodies are identified in her serum. Serum concentrations for T3 and T4 are decreased, and serum TSH concentrations is increased. Which of the following is most likely to be involved in pathogenesis

Replacement of thyroid parenchyma by lymphoid cells
Hashimoto’s thyroiditis
Hürthle cells
lymphoid aggregates with germinal centers.

Image: 47-year-old with a goiter for 10 months. Adequate iodine in diet. Antithyroid peroxidases antibodies are identified in her serum. Serum concentrations for T3 and T4 are decreased, and serum TSH concentrations is increased. Which of the following is most likely to be involved in pathogenesis

Cyclophosphamide and some other chemo agents kill mitotic cells. Which of the following cellular compartments is most rapidly depleted during cyclophosphamide therapy?

Granulocytes
Granulocytes (Mainly neutrophils) are particularly very sensitive to chemotherapeutic alkylating agents

4 year old – marked yellowing skin. Boys family is vegan. Examinatio shows yellow-tinged skin, but his sclerae are nonicteric. Which interventions will most likely improve the child’s appearance?

Dietary Change
excess carotene due to his diet

Image: 4 year old - marked yellowing skin. Boys family is vegan. Examinatio shows yellow-tinged skin, but his sclerae are nonicteric. Which interventions will most likely improve the child's appearance?

A newborn has external genital appear to be male. Chromosome analysis shows 46 XX Karyotype. Which is most likely explanation for physician findings?

Hyperplastic fetal adrenal glands

Image: A newborn has external genital appear to be male. Chromosome analysis shows 46 XX Karyotype. Which is most likely explanation for physician findings?

A 48 y/o man = 24 hour history of abdominal pain and blood tinged vomiting. Heavy smoker and drinker. Abdominal exam shows absent bowel sounds, involuntary guarding, and epigastric tenderness. An x-ray shows free air in the abdominal cavity. DX?

Perforated peptic ulcer

Image: A 48 y/o man = 24 hour history of abdominal pain and blood tinged vomiting. Heavy smoker and drinker. Abdominal exam shows absent bowel sounds, involuntary guarding, and epigastric tenderness. An x-ray shows free air in the abdominal cavity. DX?

78 year old – found unresponsive in backyard – After patient is admitted to hospital, neighbor says.
“he always said he wouldn’t want to be kept on life support if there was no hope of recovery”. Wife died and no children. Friend has been helping with errands. Physician believes prognosis is poor. It is most appropriate for the physician to state to the neighbor which of the following?

“Thank you for telling me this. Your friend’s previous statements to you can be helpful in making decisions for him”

6-month-old girl – brought to office for well child exam. Patient and moth emigrated from China- No regular health care. Screening for which is most appropriate next step in management?

Hepatitis B

A 46 y/o woman – 1 day history of high grade fever, productive cough, and pleuritic chest pain. Leukocyte Alkaline Phosphatase elevated. Which findings most reliably distinguishes leukemoid reaction from CML in this patient?

Leukocyte alkaline phosphatase activity greater than 250 U/L

Image: A 46 y/o woman - 1 day history of high grade fever, productive cough, and pleuritic chest pain. Leukocyte Alkaline Phosphatase elevated. Which findings most reliably distinguishes leukemoid reaction from CML in this patient?

. A 75 y/o with prostate cancer has metastases in spine near L2. Which is most likely route of metastatic spread?

Prostatic venous plexus to the vertebral venous plexus

Image: . A 75 y/o with prostate cancer has metastases in spine near L2. Which is most likely route of metastatic spread?

A 19 y/o woman – burned over 35% of her total body surface in fire. Which of the following is most likely to occur in the patient over the next 7 days?

Increased metabolic rate

A 48 y/o referred for evaluation of possible HTN. Average BP is 113 mm Hg with a 95% confidence interval of 110 to 116mm Hg. Which most like represents the 99% confidence interval for this data?

108 to 118
Confidence Interval (CI)- range of values w/in which the true mean of the population is expected fall.
CI of 95% is 110 to 116 then a CI of 99% has to be a range that is wider… 108 to 118

Image: A 48 y/o referred for evaluation of possible HTN. Average BP is 113 mm Hg with a 95% confidence interval of 110 to 116mm Hg. Which most like represents the 99% confidence interval for this data?

A 45 y/o – 3 days of left arm pain and weakness. Use of Ibuprofen has provided no relief. Exam shows weakness with external rotation of the shoulder but full strength with elbow in extension. The most likely cause of the patient’s condition is injury to

Infraspinatus

Image: A 45 y/o - 3 days of left arm pain and weakness. Use of Ibuprofen has provided no relief. Exam shows weakness with external rotation of the shoulder but full strength with elbow in extension. The most likely cause of the patient's condition is injury to

teres minor action

laterally rotates arm

Image: teres minor action

A 27 y/0 – headaches and blurred vision. BP 196/142 mm Hg. PE shows papilledema and bilateral costovertebral angle bruits. Which serum concentrations is most likely increased?

Angiotensin

Image: A 27 y/0 - headaches and blurred vision. BP 196/142 mm Hg. PE shows papilledema and bilateral costovertebral angle bruits. Which serum concentrations is most likely increased?

A 38 y/o woman with hypercalcemia, hypophosphatemia, and phosphaturia is diagnosed with primary hyperparathyroidism. Which of labeled segments of the nephron most likely involved in the development of phosphaturia?

B. PCT
PTH acts on the PCT to dec Phosphate reabsorption–> more Phosphate in the urine. PTH also acts at DCT to inc Ca reabsorption

Image: A 38 y/o woman with hypercalcemia, hypophosphatemia, and phosphaturia is diagnosed with primary hyperparathyroidism. Which of labeled segments of the nephron most likely involved in the development of phosphaturia?

A 20 y/o woman – 10 day history of vaginal discharge, itching, soreness. Pelvic exam shows vulvar erythema, edema, and thick white vaginal discharge. Vaginal ph 4.2. Photomicrograph of vaginal discharge is show. Most appropriate pharmacotherapy is an oral agent with which mechanism of action?

Inhibition of the cytochrome P450-dependent demethylation reaction
-Azoles (fluconazole) (inhibit CYP450 demethylation)

Image: A 20 y/o woman - 10 day history of vaginal discharge, itching, soreness. Pelvic exam shows vulvar erythema, edema, and thick white vaginal discharge. Vaginal ph 4.2. Photomicrograph of vaginal discharge is show. Most appropriate pharmacotherapy is an oral agent with which mechanism of action?

A previously healthy 32 y/p – sudden onset of severe headache, seizures, left side weakness, and nausea that began while attending a party. Friends say there may have been drugs at the party. Pulse is 120/min, BP 160/100. Neuro exam shows bilateral dilated pupils, hyperreflexia, and mild hemiparesis. What is the most appropriate next step in diagnosis?

Toxicology Screening
picks up any other drugs that the patient might have been using.
So even though the pre-test probability is high for amphetamine use ,the tox screen would pick that up as well.

A 14 year old boy – tibial fracture – leg cast and progresses satisfactorily for 6 weeks. One week after cast replaced, he has pain and numbness in the foot of the casted extremity. Decreased sensation to pin prick on the anterolateral aspect of the leg and the dorsum of the foot. Cause of nerve compression by a tight cast at which site?

Neck of the Fibula

Image: A 14 year old boy - tibial fracture - leg cast and progresses satisfactorily for 6 weeks. One week after cast replaced, he has pain and numbness in the foot of the casted extremity. Decreased sensation to pin prick on the anterolateral aspect of the leg and the dorsum of the foot. Cause of nerve compression by a tight cast at which site?

A pregnant – 28 y/o – symmetric enlargement of pituitary gland – Which of the cells types accounts for pituitary enlargement?

Lactotrope

Image: A pregnant - 28 y/o - symmetric enlargement of pituitary gland - Which of the cells types accounts for pituitary enlargement?

An investigator studying two drugs, Drugs X and Y to determine which has greater bioavailability in a healthy human volunteer. The drug has similar mechanism of actions. If Drug X has a greater bioavailability compared with Drug Y, which parameters is most likely to be greater for Drug X than Drug Y if both are administered at the same dose?

Area under the curve
Represents overall drug exposure

Image: An investigator studying two drugs, Drugs X and Y to determine which has greater bioavailability in a healthy human volunteer. The drug has similar mechanism of actions. If Drug X has a greater bioavailability compared with Drug Y, which parameters is most likely to be greater for Drug X than Drug Y if both are administered at the same dose?

A newborn – 24 in long and 12lb – mother has T1DM. Serum glucose is 20. Which of the following fetal conditions immediately prior to birth most likely precipitated the newborn’s postnatal hypoglycemia

Decreased gluconeogenesis
baby’s mother has Type 1 Diabetes mellitus, it is plausible that they had elevated blood glucose levels during or shortly before birth.

Insulin does not cross the placenta, but glucose does, so during birth the neonate would have been hyperglycemic.

This would lead to the neonatal pancreas releasing insulin, driving glucose into cells and turning down gluconeogenesis; this is why the baby is hypoglycemic right now.

31-year-old – woman – ER because of headache. Exam shows multiple contusions on the face and torso. Pt states husband assaulted her and she is afraid for her life. She pleads with physician not to allow her husband to enter the treatment area. What is the physicians most appropriate next step

Tell the patient’s husband that he is not allowed in the treatment area, and that he must remain in the waiting area.
wife did not grant him the permission to see him.

A 65 year old with gout – 3 days history of pain and swelling in knee. He is having trouble walking. Asks physician for disabled parking permit. Most appropriate response by physician?

I will authorize the permit, but I recommend that you continue with regular exercise and only use the permit when you are having severe pain.

DOCTOR decides the eligibility then sends it to the DMV disability

1 y/o – bilateral cleft palate. Both parents have family history of cleft palate. Pattern of inheritance?

Multifactorial
Cleft lip and cleft palate have distinct, multifactorial etiologies, but often occur together.”

Image: 1 y/o - bilateral cleft palate. Both parents have family history of cleft palate. Pattern of inheritance?

Genetic analyses – obtained on amniocentesis confirm that a fetus has a familial loss of function mutation that leads to impaired of G-coupled receptors signal transduction. This defect would most directly impair transcription of genes who expression is regulated by which of the following hormones?

Epinephrine
epinephrine acts on alpha or beta adrenergic receptors which are all G protien receptors.

Image: Genetic analyses - obtained on amniocentesis confirm that a fetus has a familial loss of function mutation that leads to impaired of G-coupled receptors signal transduction. This defect would most directly impair transcription of genes who expression is regulated by which of the following hormones?

A 32 y/o positive home pregnancy test. A single transaxial ultrasound image of the uterus shown. The arrow indicates a cystic structure. Which best describes this structure?

Yolk Sac

Image: A 32 y/o positive home pregnancy test. A single transaxial ultrasound image of the uterus shown. The arrow indicates a cystic structure. Which best describes this structure?

A 35 y/o man – alcoholic – PE shows gynecomastia, hypogonadism, spider angiomata. Metabolic dysfunction of which organ?

Liver

Image: A 35 y/o man - alcoholic - PE shows gynecomastia, hypogonadism, spider angiomata. Metabolic dysfunction of which organ?

A 25 y/o man – brough to ER for 30 minutes after MVC. Abdominal tenderness and swelling. Treated for spleen rupture. He develops hypotension during operation. Over next 24 hours, he develops azotemia. Urine volume remains less than 20 mL/h. Urine studies are most likely to show which of the following?

Fractional excretion of sodium greater than 2%

A 34 y/o woman – long standing history of alcoholism- frequent foul-smelling stools for 2 weeks. Admitted for alcohol -induced pancreatitis. Treatment with a pancreatic enzyme replacement. For dosage adjustments, it most appropriate to monitor which of the following in the patient’s stool?

Fat
Supplementing with pancreatic enzymes means that you should NOT see for fat in the stool

A 44 y/o woman – 2-week history of episodes of shortness of breath, perioral tingling, vague chest tightness. Episodes last 20-30 minutes – and relived by alcohol and lorazepam. Pt is mildly anxious. An ECG shows sinus tachycardia. Lab studies show?

Decreased arterial PCo2
panic attack.
Hyperventilation drops pCO2 leading to a respiratory alkalosis. po2 is relatively unaffected

A 40 y/o woman – 1 year history of facial puffiness and weight gain. She has URI and HTN. PE shows increased fat around neck, facial plethora, moon facies, red striae. Serum studies – increased ACTH – suppressed after an oral administration of dexamethasone. The most likely cause of findings in this patient is hypersecretion of a hormone from a tumor located in which organ?

Pituitary Gland
b. Dexamethosone suppresses ACTH = Pituitary Adenoma

Image: A 40 y/o woman - 1 year history of facial puffiness and weight gain. She has URI and HTN. PE shows increased fat around neck, facial plethora, moon facies, red striae. Serum studies - increased ACTH - suppressed after an oral administration of dexamethasone. The most likely cause of findings in this patient is hypersecretion of a hormone from a tumor located in which organ?

A male newborn – delivered 39 weeks gestation – PE shows protrusion of short segment of small intestine through the abdominal to the right of umbilicus. No layer covering intestine. DX?

Gastroschisis

Image: A male newborn - delivered 39 weeks gestation - PE shows protrusion of short segment of small intestine through the abdominal to the right of umbilicus. No layer covering intestine. DX?

A 68 y/o woman – weakness, fatigue, and weight loss associated with widespread autoimmune destruction of both adrenal glands.

Inhibition of H+/K+ ATPase
PPIs, which inhibit gastric H K ATPase

Image: A 68 y/o woman - weakness, fatigue, and weight loss associated with widespread autoimmune destruction of both adrenal glands.

A 68 y/o woman – weakness, fatigue, and weight loss associated with widespread autoimmune destruction of both adrenal glands.

Na decreased
K+ increased
pH decreased

widespread destruction adrenal glands,
loss of aldosterone – decreased Na+ reabsorption;
increased serum potassium.
not expressing H+ ATPases will lead to H+ not being secreted, creating a more acidic environment (pH will decrease).

A 31 y/o pregnant woman – 32 weeks gestation- 3 day of fever and chills. Recently immigrated from Ghana. Photomicrograph of peripheral blood smear shown. Pt is greatest risk of which complications?

Hypoglycemia
impair hepatic gluconeogenesis and can also consume glucose for its own metabolic demands.

Image: A 31 y/o pregnant woman - 32 weeks gestation- 3 day of fever and chills. Recently immigrated from Ghana. Photomicrograph of peripheral blood smear shown. Pt is greatest risk of which complications?

A 10 year old boy – abrasions on left knee. 1 week later – epidermal cells cover abraded skin. This stage of wound healing is best explained by the fact that epidermal cells belong to which of the following classes?

Labile Cells
regularly regenerate

Image: A 10 year old boy - abrasions on left knee. 1 week later - epidermal cells cover abraded skin. This stage of wound healing is best explained by the fact that epidermal cells belong to which of the following classes?

A 20 y/o woman- 1 year history of low grade fever, fatigue, and joint pain. She develops a red rash – even on areas not exposed to sun. Malar rash on trunk and upper extremities. Elevated antinuclear antibody. Studies most likely show a mutation of a gene encoding which?

C1q
SLE is associated with deficiency of early complement protein

Image: A 20 y/o woman- 1 year history of low grade fever, fatigue, and joint pain. She develops a red rash - even on areas not exposed to sun. Malar rash on trunk and upper extremities. Elevated antinuclear antibody. Studies most likely show a mutation of a gene encoding which?

54 y/o F – with rheumatic heart disease – severe shortness of breath. Crackles are heard over lung fields. Cardiac exam shows 2/6 systolic ejection murmur. An ECG shows afib and left atrial enlargement. Which of the following pulmonary mechanisms explains this patient’s dyspnea?

Intra-alveolar transudates
HF d/t a. fib and left atrial enlargement-> inc hydrostatic pressure->

Image: 54 y/o F - with rheumatic heart disease - severe shortness of breath. Crackles are heard over lung fields. Cardiac exam shows 2/6 systolic ejection murmur. An ECG shows afib and left atrial enlargement. Which of the following pulmonary mechanisms explains this patient's dyspnea?

A 45 y/o – BMI 36 – hypercholesterolemia – Pravastatin is prescribed. Patient is increased risk of which of the following conditions?

Myopathy

A 44 year old woman – 3 week history of severe headache – Papilledema – Lumbar puncture shows an high opening pressure. Treatment with acetazolamide is begun. This drug will most likely improve patients symptoms by which mechanism?

Decreased CSF production by the Choroid Plexus
Carbonic anhydrase inhibitors – treat ICP by reducing cerebrospinal fluid (CSF) production at the choroid plexus.

Image: A 44 year old woman - 3 week history of severe headache - Papilledema - Lumbar puncture shows an high opening pressure. Treatment with acetazolamide is begun. This drug will most likely improve patients symptoms by which mechanism?

A 3 day old male – Down Syndrome – repeated episodes of vomiting bile-stained fluid. Stomach is distended. Pregnancy complicated by polyhydramnios. DX?

Duodenal Atresia
double bubble

Image: A 3 day old male - Down Syndrome - repeated episodes of vomiting bile-stained fluid. Stomach is distended. Pregnancy complicated by polyhydramnios. DX?

43-year-old woman – undergoing chemo for Acute Myelogenous Leukemia – given blood transfusion for anemia. Shortly after transfusion is started she develops chest pain and shortness of breath. Results show patient’s blood group is O and transfused blood group A. Which effectors cause of symptoms?

Preformed IgM antibodies

Image: 43-year-old woman - undergoing chemo for Acute Myelogenous Leukemia - given blood transfusion for anemia. Shortly after transfusion is started she develops chest pain and shortness of breath. Results show patient's blood group is O and transfused blood group A. Which effectors cause of symptoms?

60 year old woman – lump in breast – fluorescent in situ hybridization shows HER2/neu receptor is overexpressed with greater than 10 receptors per cell. Which mechanism cause overexpression of patients tumor?

Amplification

Image: 60 year old woman - lump in breast - fluorescent in situ hybridization shows HER2/neu receptor is overexpressed with greater than 10 receptors per cell. Which mechanism cause overexpression of patients tumor?

30 y/o man – increasing fatigue and dyspnea. Fixed wide split S2 and a grade 2/6 murmur heard nest at the left upper sternal border. DX?

Atrial Septal Defect

Image: 30 y/o man - increasing fatigue and dyspnea. Fixed wide split S2 and a grade 2/6 murmur heard nest at the left upper sternal border. DX?

A pt with T1DM – prolonged episodes of hypoglycemia following injections of insulin. Cause of prolonged hypoglycemia?

Impaired release of glucagon
glucagon secretion is inhibited by hyperglycemia, somatostatin and insulin

Image: A pt with T1DM - prolonged episodes of hypoglycemia following injections of insulin. Cause of prolonged hypoglycemia?

A 70-year-old woman – 3-month history of severe pain in joints. PE shows rheumatoid nodules. The immunologic mechanism of this patient is most similar to which?

Serum Sickness

Image: A 70-year-old woman - 3-month history of severe pain in joints. PE shows rheumatoid nodules. The immunologic mechanism of this patient is most similar to which?

A 78 y/o progressive loss of hearing in right ear. Bone conduction is greater than air conduction. Cause of hearing loss?

Otosclerosis
BC>AC = conductive hearing loss

AC>BC = sensoneurial hearing loss

Image: A 78 y/o progressive loss of hearing in right ear. Bone conduction is greater than air conduction. Cause of hearing loss?

82-year-old – fecal incontinence for 2 months. Underlying cause of patient’s condition?

Decreased function of puborectalis muscle

Image: 82-year-old - fecal incontinence for 2 months. Underlying cause of patient's condition?

A 23 year old – woman – family history of breast cancer- concerned about consuming irradiated food. Initial response by physician?

Because irradiated food is not radioactive, you will not be exposed to ionizing radiation by consuming it.

Image: A 23 year old - woman - family history of breast cancer- concerned about consuming irradiated food. Initial response by physician?

A 24 y/o woman – weeks purple spots on legs. Underwent splenectomy – for immune thrombocytopenia. Platelet count was low and increased after surgery. 6 weeks ago peripheral blood smear shows presence of Howell-jolly bodies. Today no howell-jolly bodies. Which explains current lab findings?

Accessory spleen

Image: A 24 y/o woman - weeks purple spots on legs. Underwent splenectomy - for immune thrombocytopenia. Platelet count was low and increased after surgery. 6 weeks ago peripheral blood smear shows presence of Howell-jolly bodies. Today no howell-jolly bodies. Which explains current lab findings?

32-year-old – pregnant – in labor – membranes ruptured spontaneously. Epidural analgesia in initiated. Fetal heart rate shows spontaneous decelerations. Umbilical cord compression is suspected. Which is best explains the decrease in fetal heart rate because of compression?

Increased fetal systemic vascular resistance
blocking umbilical veins- reduces the flow in parallel circulation; thus increases the TPR

Image: 32-year-old - pregnant - in labor - membranes ruptured spontaneously. Epidural analgesia in initiated. Fetal heart rate shows spontaneous decelerations. Umbilical cord compression is suspected. Which is best explains the decrease in fetal heart rate because of compression?

70-year-old man – decreased urinary output and fatigue during past 2 days. 6 month history of joint pain – Treatment with Ibuprofen. Lab show increased BUN/Cr. Cause of increase in BUN/CR

Inhibition of prostaglandin synthesis

43. A 50-kg experimental animal receives 1L of 5% saline intravenously. Which of the following changes in body fluid is most likely in response to 5% Saline

Intracellular Volume decreased
Intracellular Osmolality increased

Extracellular Volume increased
Extracellular Osmolality increased

A 50-kg experimental animal receives 1L of 5% saline intravenously. Which of the following changes in body fluid is most likely in response to 5% Saline?

Intracellular Volume decreased
Intracellular Osmolality increased

Extracellular Volume increased
Extracellular Osmolality increased

Image: A 50-kg experimental animal receives 1L of 5% saline intravenously. Which of the following changes in body fluid is most likely in response to 5% Saline?

50 y/o woman – immigrated from the Dominican Republic 20 years ago. Positive Strongyloidiasis antibody titer. Infection has following modes of transmission?

Penetration from the larvae from the soil to the skin

Image: 50 y/o woman - immigrated from the Dominican Republic 20 years ago. Positive Strongyloidiasis antibody titer. Infection has following modes of transmission?

A male newborn with malformed extremity. Woman’s husband, mother, and sister also are in delivery room. Most appropriate initial action?

Ask the mother and sister to leave the room and inform the parents of the abnormality.
patient privacy

A 25 y/o man – orchiectomy for testicular cancer. A biopsy shown – which is most likely cells of origin of neoplasms?

Germ Cells
Seminoma is the most common testicular tumor. It’s a germ cell tumor. Commonly see “fried egg cells”.

Image: A 25 y/o man - orchiectomy for testicular cancer. A biopsy shown - which is most likely cells of origin of neoplasms?

A 63 y/o – BP 220/160. Required a drug that decreases heart rate and systemic vascular resistance. Which drug?

Labetalol
Nonselective alpha & beta antagonists end
– carvedilol
– labetalol

A 65 year old – increasing shortness of breath – low back pain. High output cardiac failure. Radiologic exam showed intermixed osteosclerotic and osteolytic areas in pelvic bones. Elevated Alk phos. DX?

Osteitis Deformans (Paget Disease)

Image: A 65 year old - increasing shortness of breath - low back pain. High output cardiac failure. Radiologic exam showed intermixed osteosclerotic and osteolytic areas in pelvic bones. Elevated Alk phos. DX?

. A 5 y/o boy – URI – develops easy bruising- Platelet count low. Photomicrograph of tissue. Mechanism of thrombocytopenia

Immune Destruction of Platelets

Image: . A 5 y/o boy - URI - develops easy bruising- Platelet count low. Photomicrograph of tissue. Mechanism of thrombocytopenia

39 y/o with systolic ejection murmur heard at the apex and does not radiate. Murmur is louder when standing. ECG shows left ventricular hypertrophy. Most likely mutation?

B-Myosin

Image: 39 y/o with systolic ejection murmur heard at the apex and does not radiate. Murmur is louder when standing. ECG shows left ventricular hypertrophy. Most likely mutation?

A 56 year old F – 3 week history of increasing weakness of her arms and legs. 30 year smoking history – PE shows pronounced weakness of the hip girdle muscles and lesser weakness of the shoulder girdle. Chest X-ray shows hilar lung mass. ECG shows malfunction of the NMJ. Impairment of which of the following is most likely cause of these findings?

Pre-synaptic release of acetylcholine

Image: A 56 year old F - 3 week history of increasing weakness of her arms and legs. 30 year smoking history - PE shows pronounced weakness of the hip girdle muscles and lesser weakness of the shoulder girdle. Chest X-ray shows hilar lung mass. ECG shows malfunction of the NMJ. Impairment of which of the following is most likely cause of these findings?

A 26 y/o man – Peace Corps Volunteer – nausea after eating, profuse watery diarrhea – Returned from 1-year assignment in Haiti. PE shows dry mucous membranes. Stool culture grows oxidase-positive, curved, gram-negative bacillus. Which is the most appropriate immediate treatment for this patient?

0.9 Saline

A 52 y/o man – inability to achieve an erection 6 weeks after undergoing removal of a tumor on the distal portion of the rectum – attached to posterior aspect of prostate. Damage to which nerve?

Pelvic Splanchnic
Point and Shoot
Parasympathetic – Point

An asymptomatic 30-year-old F – PE shows nontender adnexal mass on left. Ultrasonography confirms the presence of a single 6-cm fluid filled cyst. Most likely cause of this ovarian cyst?

Anovulation
when a follicle doesn’t rupture (aka anovulation) then it will form a cyst.

Image: An asymptomatic 30-year-old F - PE shows nontender adnexal mass on left. Ultrasonography confirms the presence of a single 6-cm fluid filled cyst. Most likely cause of this ovarian cyst?

A 30 y/o man with HIV infection comes to office – PE shows no abnormalities – CD4 T lymphocyte count is 158 and PE shows no abnormalities – Prescribed ritonavir and another protease inhibitor – Why is the combination effective?

It increases plasma concentration of other Pls also metabolized

Image: A 30 y/o man with HIV infection comes to office - PE shows no abnormalities - CD4 T lymphocyte count is 158 and PE shows no abnormalities - Prescribed ritonavir and another protease inhibitor - Why is the combination effective?

56 y/o woman – PE shows no abnormalities – Pap smear show high grade squamous intraepithelial lesions. Cause of the lesions in the patient?

Microbial oncoprotein that interferes with tumor suppressor genes

Image: 56 y/o woman - PE shows no abnormalities - Pap smear show high grade squamous intraepithelial lesions. Cause of the lesions in the patient?

47 y/o – MS patient – bilateral LE numbness – Proprioception and sensation to vibration are diminished – Symptoms are caused by damage to which of the following?

Fasciculus gracilis

Image: 47 y/o - MS patient - bilateral LE numbness - Proprioception and sensation to vibration are diminished - Symptoms are caused by damage to which of the following?

A 31 y/o – 28 week gestation – CXR shows evidence of interstitial pneumonia. Tetracycline is contraindicated because of what toxicity to the fetus?

Bone
Teeth discoloration and inhibition of bone growth – chelating agent + photosensitivity

A 61 y/o with severe abdominal pain – Polycythemia Vera – PE shows mild scleral icterus. Abdominal exam shows tender, enlarged liver, enlarged spleen, ascites- DX?

Budd-Chiari Syndrome
widened hepatic venules – central lobular congestion

Image: A 61 y/o with severe abdominal pain - Polycythemia Vera - PE shows mild scleral icterus. Abdominal exam shows tender, enlarged liver, enlarged spleen, ascites- DX?

5-year-old boy, immunosuppressed because of chemotherapy for ALL, 2 day history of fever, cough, shortness of breath, febrile (101.8 F), respirations 46/min, with cyanosis. Generalized vesicular rash – Chest Xray shows nodular infiltration. Most likely cause by which virus?

Varicella-zoster virus

Image: 5-year-old boy, immunosuppressed because of chemotherapy for ALL, 2 day history of fever, cough, shortness of breath, febrile (101.8 F), respirations 46/min, with cyanosis. Generalized vesicular rash - Chest Xray shows nodular infiltration. Most likely cause by which virus?

43 y/o woman – multiple fractures and lacerations in an automobile collision – develops tachypnea, tachycardia, and cyanosis – Despite intensive therapy – died in the hospital. Which is most likely on exam of the lungs at autopsy?

Hyaline membranes
Trauma – ARDS

Image: 43 y/o woman - multiple fractures and lacerations in an automobile collision - develops tachypnea, tachycardia, and cyanosis - Despite intensive therapy - died in the hospital. Which is most likely on exam of the lungs at autopsy?

An otherwise healthy 82 y/o man – 6-month history of numbness and tingling in his toes. Sensation to pinprick is decreased in lower extremities below the level of the knee. A photomicrograph of a peripheral blood smear is shown – Which is most likely diagnosis?

Posterior cord syndrome
Hyper segmented neutrophil – Marcocytic Anemia – B12 deficiency – Posterior Column

Image: An otherwise healthy 82 y/o man - 6-month history of numbness and tingling in his toes. Sensation to pinprick is decreased in lower extremities below the level of the knee. A photomicrograph of a peripheral blood smear is shown - Which is most likely diagnosis?

A research investing prevalence and incidence of ulcers in 100 residents in nursing care – At the beginning of the study 10 residents had ulcers – During the first year, 5 additional residents develop ulcers, and another 10 develop in the second year. National standards for nursing care mandate that the prevelance be less than 20% and the new incidence should be less than 50 per 1000 patient years. Which is the most appropriate conclusion regarding the prevalence and incidence if decubitus ulcers in this population at the end of the 2 year period?

Prevalance Above the standard
Incidence Above the standard
100 residents
prevalence after 2 years is =10 at the beginning, +5 in the first year, +10
second year,
-3 that healed, for a total prevalence of 22 residents or 22/100=22 percent.
Incidence = 15 new cases / 90 residents over the 2 years
(100 total residents – 10 that already had ulcers),
15 new ulcers per 180 patient⋅years.
This would be 83.3 new ulcers per 1000 patient⋅years if you extrapolated it out — basically (1000/180) * 15 — thus, incidence = above the standard.

Electric stimulation of the stellate ganglion – most likely result in an increase in which?

Heart Rate
sympathetic stimulation of a GANGLION

Image: Electric stimulation of the stellate ganglion - most likely result in an increase in which?

A male newborn of has a myelomneninigocele – involves verterbral arches L2-l5 – PE shows bilateral clubfeet. Which development process is affected?

Closure of the caudal neuropore

Image: A male newborn of has a myelomneninigocele - involves verterbral arches L2-l5 - PE shows bilateral clubfeet. Which development process is affected?

A 45 y/o man – reclusive and weird ideas – I believe people can read minds. I carry these crystals to help self-actualize. No delusions or hallucinations. Which personality disorder?

Schizotypal

Image: A 45 y/o man - reclusive and weird ideas - I believe people can read minds. I carry these crystals to help self-actualize. No delusions or hallucinations. Which personality disorder?

Which integral membrane proteins – results in anchored within the membrane?

Extensive hydrophobic interactions between the amino acid chains of the protein and the membrane phospholipid tails

Image: Which integral membrane proteins - results in anchored within the membrane?

25 y/o MCV < 80. Hemoglobin electrophoresis showed no abnormalities. Couples offspring is at greatest risk for which?

a-thalassemia trait

Image: 25 y/o MCV < 80. Hemoglobin electrophoresis showed no abnormalities. Couples offspring is at greatest risk for which?

13 year old – itchy red spots on face and neck – Went on a trip for swim meet. Several teammates noted small bed bugs (photograph). The patient is at risk which infections?

Staphylococcus aureus
scratch bites – increase the risk of a staph skin infection (since staph aureus is the most common skin infection pathogen)

Image: 13 year old - itchy red spots on face and neck - Went on a trip for swim meet. Several teammates noted small bed bugs (photograph). The patient is at risk which infections?

48 year old – farmer – develops muscle spasms, trismus, and seizures – after a puncture wound to his hand – Mechanism of toxin?

Blockade of postsynaptic inhibition of spinal motor reflexes
Tetanospasmin is a neurotoxin that inhibits the release of γ-aminobutyric acid (GABA

Image: 48 year old - farmer - develops muscle spasms, trismus, and seizures - after a puncture wound to his hand - Mechanism of toxin?

Botulism

inhibits ACh release at the neuromuscular junction

floppy baby

Image: Botulism

A 62 y/o woman – BP 173/95 – HTN, Hyperlipidemia – Speech is dysarthric. Horizontal eye movements are impaired bilaterally. Vertical movement intact. Exam shows quadriplegia. MRI shows a lesion at which location?

Pons
Locked in syndrome – loss of horizontal eye movement and vertical movement intact
Quadriopelgia
basilar artery runs right on top of the pons

Image: A 62 y/o woman - BP 173/95 - HTN, Hyperlipidemia - Speech is dysarthric. Horizontal eye movements are impaired bilaterally. Vertical movement intact. Exam shows quadriplegia. MRI shows a lesion at which location?

An 11-year-old – found him naked in bed with 4 y/o male cousin. Cousin was crying. 11-year-old school performance has declined. Most appropriate to evaluate for which of the following?

Sexual Abuse

A couple – genetic counseling – two siblings with Cystic Fibrosis – 1/25 carrier frequency – Which approximates recurrence risk that the individual identified by the arrow in the pedigree will have a child with CF?

1/150
carrier status to 2/3
Chance of affected individual = chance of father passing allele * chance of mother passing allele
= 2/3 1/25 1/4 = 2/300 = 1/150
He has a 2/3 chance of being heterozygous (not 2/4 b/c we know he for sure doesn’t have CF)
1/25 chance in the population
1/4 chance of a heterozygous couple having a child with CF

Image: A couple - genetic counseling - two siblings with Cystic Fibrosis - 1/25 carrier frequency - Which approximates recurrence risk that the individual identified by the arrow in the pedigree will have a child with CF?

An investigator – adverse effects of proteasome inhibitor – bortzeomib. Bortzeomib administered to group of patients with multiple myeloma – Which process is affected by this drugs?

Presentation of antigens to CD8+ T Lymphocytes

An investigator – adverse effects of proteasome inhibitor – bortzeomib. Bortzeomib administered to group of patients with multiple myeloma – Which process is affected by this drugs?

Presentation of antigens to CD8+ T Lymphocytes
proteasome= degradation of malformed proteins
MHC class I is present on all nucleated cells
Failure to display MHC I – triggers a cellular immune response, leading to destruction of the cell.

A 45 y/o man with coronary artery disease and anemia – assess cardiac perfusion- Doppler echocardiography shows blood flow in LAD is 30 ml/min. Arterial blood oxygen is .15 ml (O2/mL) Which approximates myocardial oxygen supply in left ventricle in this patient (in mL O2/min)?

4.5
30* 0.15. Think about it, there is x flow with an oxygen concentration of y–so to find out the delivery you just multiply them together.

64 year – found dead – photograph of the liver obtained on autopsy. Most likely DX?

Metastatic Breast Cancer
Multiple lesions = metastasis

Image: 64 year - found dead - photograph of the liver obtained on autopsy. Most likely DX?

A 34 y/o woman – in ED 30 mins- after MVC – Sensation to pain is decreased over left lower extremity- Which of the following labeled regions of spinal cord is damaged?

Area Labeled D
spinothalamic tract is responsible for contralateral pain and temperature sensation. Because our patient has lost sensation on the left, the lesion is in the right.

Image: A 34 y/o woman - in ED 30 mins- after MVC - Sensation to pain is decreased over left lower extremity- Which of the following labeled regions of spinal cord is damaged?

42 y/o – fatigue and abnormally heavy menses – No abdominal or pelvic pain. Exam of endometrial biopsy shows glandular hyperplasia without atypia – Which describes pathogenesis of endometrial disorder?

Binding of permeable ligand to nuclear transcription factor
endometrial hyperplasia – excess estrogen, a steroid hormone that translocate to the nucleus and binds its transcription factor
Sex + Thyroid – nuclear transcription
ALL the rest are peptide.

Which of present in integral membrane-associated proteins in absent from secreted proteins?

Transmembrane region
Integral membrane proteins are found within the plasma membrane and span the whole length across.

65 y/o – DVT – 10 days after hip replacement – receiving subcutaneous anticoagulant therapy. Platelet count low. Cause?

Heparin-Induced Thrombocytopenia
IgG antibodies against platelet factor 4

Unlike DNA polymerase – Ecoli, RNA polymerase from bacterium lacks proofreading activity. Which is most likely reason for lack of proofreading in RNA polymerase does not compromise the survival of the species?

Mistake in transcription are not transmitted to progeny

A 52 y/o man – severe recurrent abdominal pain for 6 years – bulky, foul smelling stools that float. 9 kg weight loss. 20 year history of alcoholism. Which lipids found in excessive amounts?

Triglyceride
Alcoholic = pancreatic insufficiency
No Lipases – unable to break down triglycerides

15 y/o boy – nasal polyps and bronchospasms taking a drug. The drug most likely inhibit which enzyme?

B Aspirin induced Asthma
5HPETE – peter is a leukotriene
Asprin works at Cyclic Endoperoxides – COX / Prostaglandins

Image: 15 y/o boy - nasal polyps and bronchospasms taking a drug. The drug most likely inhibit which enzyme?

55 y/o T2DM + fecal incontinence – Which additional symptoms in patient?

Impotence
fecal incontinence – damaged- pudendal n. (S2-S4).
pelvic splanchnic nerves – S2-S4
Point and Shoot
SLUDGE

Image: 55 y/o T2DM + fecal incontinence - Which additional symptoms in patient?

Ecoli – digested into numerous small fragments with a restriction endonuclease – Wild type is resistant to digestion with the restriction endonuclease. Mutation occurs in a gene encoding which of the following?

Methylase
mutates DNA

Image: Ecoli - digested into numerous small fragments with a restriction endonuclease - Wild type is resistant to digestion with the restriction endonuclease. Mutation occurs in a gene encoding which of the following?

65 y/o lesion at sit of peripheral scar on the lung. Site developed at site of pulmonary infarct – Smoker for 20 years. A lobectomy is done. Most likely type of malignancy?

Adenocarcinoma
Most likely cause of lung cancer
Peripheral lung cancer
more common in women.

Image: 65 y/o lesion at sit of peripheral scar on the lung. Site developed at site of pulmonary infarct - Smoker for 20 years. A lobectomy is done. Most likely type of malignancy?

A 35 y/o man – tremor and difficulty initiating body movements starting since starting haloperidol for schizophrenia. PE shows masked facies and resting tremor. In addition to decreasing dose of haloperidol – most appropriate for physician to prescribe which meds?

Anticholinergic agent
1st gen antipsychotics extra pyramidal effects
Low dopamine and high acetylcholine – tx with anticholinergic

54 y/o – 1 week shoulder pain – PE shows tenderness of right deltoid – Abduction and passive shoulder flexion produce pain. Pt has weakness with abduction and internal rotation. Likely affected?

supraspinatus
Pain is worst with internal rotation of the shoulder –
consistent with the findings of the empty-can test, which indicates a supraspinatus injury

Image: 54 y/o - 1 week shoulder pain - PE shows tenderness of right deltoid - Abduction and passive shoulder flexion produce pain. Pt has weakness with abduction and internal rotation. Likely affected?

A med student – caring for 72 year old with lung mass. Biopsy shows carcinoma. Resident instructs student not to inform patients of results. The morning patient asked “what did my biopsy show”. Most appropriate Response?

Let me get the resident physician so we can go over the results

A 34 y/o woman – chronic Hep C – progressive fatigue. – AFAP in reference range – A liver biopsy will likely show?

Infiltration of lymphocytes, plasma cells, and macrophages

A 68 y/o woman – 3 month history of nighttime urination and pain in lower back. She has cervical cancer 15 years ago and underwent hysterectomy – Received radiation to pelvis. Increased BUN and Creatine. Renal ultrasound shows bilateral hydronephrosis – with distal narrowing. Explain findings?

Retroperitoneal fibrosis
Bilateral Hydronephrosis
direct consequence of the external beam radiation
Carcinoma – unlikely to be bilateral

Image: A 68 y/o woman - 3 month history of nighttime urination and pain in lower back. She has cervical cancer 15 years ago and underwent hysterectomy - Received radiation to pelvis. Increased BUN and Creatine. Renal ultrasound shows bilateral hydronephrosis - with distal narrowing. Explain findings?

65 y/o man – farmer – skin lesions on his face, arms, and upper trunk for the past few years. Lesions to be actinic keratoses, and one is squamous cell carcinoma. Which is most likely responsible for the development of squamous cell carcinoma?

Disruption of DNA strands secondary to ionization effect
UVB light from sun

22 y/o – 12 weeks gestation comes to physician because of a 3 hour history of vaginal bleeding. PE shows uterus consistent in size with a 16 week gestation. Ultrasonography shows grapelike intrauterine structures and no fetus. Which of the following components correlates with grapelike structures?

Hydropic villi with central cisterns

Image: 22 y/o - 12 weeks gestation comes to physician because of a 3 hour history of vaginal bleeding. PE shows uterus consistent in size with a 16 week gestation. Ultrasonography shows grapelike intrauterine structures and no fetus. Which of the following components correlates with grapelike structures?

Which of the following sets of serum finding is most likely in a 50-year-old man with recently diagnosed chronic renal failure?

Phosphate increased
Ca 2+ decreased
Parathyroid Increased

Not able to trash phosphate
Hypocalcemia – CKD

Image: Which of the following sets of serum finding is most likely in a 50-year-old man with recently diagnosed chronic renal failure?

A study – examine effective of new antihypertensive drug – Subjects to be enrolled most have moderate systemic hypertension, – approx. 100-200 tested. Drug is most likely in what stage?

Phase 2
Working – involves up to several hundred affected patients.

Image: A study - examine effective of new antihypertensive drug - Subjects to be enrolled most have moderate systemic hypertension, - approx. 100-200 tested. Drug is most likely in what stage?

A 39 y/o man – darkening skin and fatigue – T1DM — Treated with insulin. PE shows hepatomegaly and testicular atrophy. Which serum concentrations abnormal?

Ferritin
Hemochromatosis, aka “bronze diabetes”.

A 40 y/o – low grade B Cell lymphoma – TX with rituximab. Increased risk for which of the following?

Bacterial pneumonia
B-cell deficiency can predispose to bacterial infections.

20 year old – ER – acute confusion – all night dance party – PE shows pale, cold extremities – An ECG shows sinus tachycardia. Which substance ingested?

MDMA

A 32-year-old – takes 40 alprazolam. Boyfriend threatened to leave. She drinks excessively and uses cocaine. She has outbursts whether or not intoxicated. Which personality disorder?

Borderline

16 month old boy – recurrent bacterial infections – since 8 months – Serum immunoglobulin concentrations are uniformly decreased. Which findings is most likely in this patient?

Absence of germinal centers in lymph node
X linked agammaglobulinemia
IG decreased in all classes

Image: 16 month old boy - recurrent bacterial infections - since 8 months - Serum immunoglobulin concentrations are uniformly decreased. Which findings is most likely in this patient?

A 4 year-old – two bacterial UTI – Radiology shows marked dilation of left ureter and renal pelvis, and minimal left-sided renal function. Left nephrectomy – Microscopic examination of renal parenchyma – most likely to show?

Interstitial Inflammation
Vesicoureteral Reflux from UTI

Image: A 4 year-old - two bacterial UTI - Radiology shows marked dilation of left ureter and renal pelvis, and minimal left-sided renal function. Left nephrectomy - Microscopic examination of renal parenchyma - most likely to show?

12 year old girl – Central Africa – 5 week abdominal pain. Fever, increased heart rate and blood pressure. PE shows hepatosplenomegaly and abdominal ascites. A photomicrograph of stool sample shows (schistosomiasis) Most appropriate pharmcotherapy?

Praziquantel
treat a fluke

Image: 12 year old girl - Central Africa - 5 week abdominal pain. Fever, increased heart rate and blood pressure. PE shows hepatosplenomegaly and abdominal ascites. A photomicrograph of stool sample shows (schistosomiasis) Most appropriate pharmcotherapy?

35-year-old itchy rash on his hands, waist, and feet 2 days. Wife and kids develop same symptoms. 2 mm erythematous papules in fingers webs, wrist, umbilicus. Scraping show mites and eggs. Most appropriate therapy?

Prescribe permethrin for the patient and his family

Image: 35-year-old itchy rash on his hands, waist, and feet 2 days. Wife and kids develop same symptoms. 2 mm erythematous papules in fingers webs, wrist, umbilicus. Scraping show mites and eggs. Most appropriate therapy?

45-year-old woman – ductal carcinoma of the breast – 8 weeks on tamoxifen – homozygous presence of Cytochrome P450 – likelihood of sister same alleles?

25%
homozygous + AR = 25%
Autosomal Recessive disorders – present as enzyme deficiencies.
Autosomal Dominant disorders – present as defects in structural genes

A 50 year old – Chronic gastritis is diagnosed with a marginal zone lymphoma of Stomach. Which organism?

Helicobacter Pylori

Image: A 50 year old - Chronic gastritis is diagnosed with a marginal zone lymphoma of Stomach. Which organism?

A 30-year-old man brought to the ER after being stabbed – diagnose of rupture of aorta- what is expected compensatory mechanism?

Increased serum angiotensin II concentration

A 56-year-old woman – headache, confusion, difficulty speaking + walking. Mental status shows confusion and memory loss. Labs show – opening press 260, Protein raised, Glucose Low. EEG abnormalities in right temporal lobe and periodic lateralized epileptiform discharges?

Herpes Encephalitis

Image: A 56-year-old woman - headache, confusion, difficulty speaking + walking. Mental status shows confusion and memory loss. Labs show - opening press 260, Protein raised, Glucose Low. EEG abnormalities in right temporal lobe and periodic lateralized epileptiform discharges?

An obese 45-year-old woman – T2DM, hypertension, dyslipidemia, osteoarthritis – starts orlistat, diet, and exercise program. During first month, increased risk of developing?

Diarrhea

A 38-year-old man – High Altitude Sickness – vacation ion Colorado Ski – Takes Carbonic Anhydrase Inhibitor to adjust to altitude. Changes to (urinary) pH, bicarbonate, and volume?

pH increased
HCO3 mEq/L – increased
Volume – increased

18-year-old – injured hand during football game – tenderness on palmar tip of ring, unable to flex DIP of right ring finger, able to flex PIP and metacarpophalangeal. Pt injured fibers from which nerve root?

C8

Image: 18-year-old - injured hand during football game - tenderness on palmar tip of ring, unable to flex DIP of right ring finger, able to flex PIP and metacarpophalangeal. Pt injured fibers from which nerve root?

A 27 year old- swelling in left arm after spontaneous abortion in second trimester- axillary vein thrombosis – Bleeding time normal, PT increased, PPT increased, Thrombin Time – Normal. Most likely cause?

Antiphospholipid antibodies
pregnancy + Sponataneous abortion + Thrombosis + SLE In pregnancy

Image: A 27 year old- swelling in left arm after spontaneous abortion in second trimester- axillary vein thrombosis - Bleeding time normal, PT increased, PPT increased, Thrombin Time - Normal. Most likely cause?

A 54 year-old woman – HTN and bilateral renal artery stenosis- taking NSAID- serum Cr concentrations increase – cause by drug’s ability to inhibit which?

Vasodilating prostaglandins at the afferent arteriole

Image: A 54 year-old woman - HTN and bilateral renal artery stenosis- taking NSAID- serum Cr concentrations increase - cause by drug's ability to inhibit which?

35 year old alcoholic man – admitted for vomiting, ascites, periumbilical vein, loss of consciousness, flapping tremor of the hands. Pathogenesis is most likely directly related to which?

Absorption of nitrogenous products from the gastrointestinal tract

Image: 35 year old alcoholic man - admitted for vomiting, ascites, periumbilical vein, loss of consciousness, flapping tremor of the hands. Pathogenesis is most likely directly related to which?

Lactulose – MOA

reduces intestinal ammonia production

The Risk of hemorrhagic stroke from drug X is investigated by a questionnaire about drug use in 702 patients with stroke and in 1376 control subjects contact by random-digit. Study Design?

Case-Control Study

A 28-year-old man (seropositive for HIV) – numerous non-ulcerated purple nodules on the skin- biopsy of the skin shows which?

Slit-like vascular spaces with plump spindle-shaped stromal cells

kaposi sarcoma – spindle cells

Image: A 28-year-old man (seropositive for HIV) - numerous non-ulcerated purple nodules on the skin- biopsy of the skin shows which?

A 22 y/o man – – infertility – Photomicrograph of biopsy of scrotal testes, Which genetic abnormality?

47, XXY

Image: A 22 y/o man - - infertility - Photomicrograph of biopsy of scrotal testes, Which genetic abnormality?

26 y/o – fever, chills, malaise, Cd4+ 700, HIV Viral Load 1 million copies. Serum HIV antibody test are negative. Which explains patient’s condition?

Acute retroviral infection

Image: 26 y/o - fever, chills, malaise, Cd4+ 700, HIV Viral Load 1 million copies. Serum HIV antibody test are negative. Which explains patient's condition?

72-year-old multiple myeloma. Peripheral blood and T lymphocytes are isolated. B lymphocyte DNA shows a 1.5kb band. T-lymphocytes J-region shows a 6-kb band. 6kb band signifies which

unrearranged immunoglobulin gene

A screening Test developed and applied to 500 subjects with cancer and 500 patients without. Sensitivity and specificity of each are plotted on a graph as a function of 1-specificity. Which will most likely rule in cancer?

A – (1- specificity) = specificity highest
(SP – IN) – most specific highest specificity

Image: A screening Test developed and applied to 500 subjects with cancer and 500 patients without. Sensitivity and specificity of each are plotted on a graph as a function of 1-specificity. Which will most likely rule in cancer?

A 56 y/o – brief ventricular tachycardia after MI. Arrythmia treated with loading dose of lidocaine – followed by continuous infusion of the same drug. To maintain drug concentration (Css), the most appropriate rate if infusion is determine by which of the following).
Cl = drug clearance and Vd = volume distribution.

CL X Css
Maintenance Dose = Css CL t) / F
t is elapsed time between doses (not relevant continuous infusion)
F is bioavailability (which is 100% or 1.0 here because it’s given IV)
Loading Dose (Css * Vd) / F

Image: A 56 y/o - brief ventricular tachycardia after MI. Arrythmia treated with loading dose of lidocaine - followed by continuous infusion of the same drug. To maintain drug concentration (Css), the most appropriate rate if infusion is determine by which of the following).
Cl = drug clearance and Vd = volume distribution.

Maintenance dose equation

Css * CL

Loading Dose

(Css * Vd) / F

A 35-year-old pain in right arm and tingling in fingers, weakness of extension + pronation, decreased triceps reflex. Which nervous structures most likely involved

C7 nerve root

Image: A 35-year-old pain in right arm and tingling in fingers, weakness of extension + pronation, decreased triceps reflex. Which nervous structures most likely involved

82 year old – surgical repair for hip fracture – Healing is slow. Physician concerned about a DVT. A drug that suppresses the action of which factors is most likely to decrease this patient’s risk of thrombosis?

Carboxylation of precursor proteins
warfarin inhibits the synthesis of factors II, VII, IX, X, C, and S by blocking reduction of oxidized vitamin K.
The enzyme Epoxide Reductase is inhibited by warfarin.
The reduced (active) form of vit. K is a cofactor for gamma-glutamyl carboxylase

Which of the following is required for synthesis of glucosamine from fructose -6-phosphate?

Glutamine
Hexoamine pathway – Fructose 6-phosphate to glucosamine requires (glutamine)

Physician receives $100 for referral of drug – should he tell the patient?

Yes; the patient may wish to consider the money’s influence on the physician’s recommendation


informed consent- includes disclosure, which includes the incentive that the company is offering

49-year-old gradual onset of numbness and pain in left index finger, finger mottled blue and pale –
Platelet count – 1,530,00. Bone marrow shows megakaryocytic hyperplasia. Which is most likely diagnosis?

Bicarbonate

42 year old man on MPTP – frozen as ice, muscle rigidity, cell bodies effected?

Dopaminergic neurons on the substantia nigra
b. TRAPS- Tremors, Rigidity, Akinesia, Postural Instability, Shuffling Gait
c. MPTP – toxic to substantia

A 62-year-old – excision for squamous carcinoma The physician explained the reason for the procedure, the technique to be used, possible complication, name of physician performing procedure, and which additional information?

Alternative treatments

A study to assess cardiac valvular abnormalities in ergot dopamine agonists. Initial study shows a significantly greater score for patient on dopamine agonist. Research double the patient size (increase sample size) and keep statistical significance P < .05. This will have which of following effects on Type I and Type II error?

Type I Error No change
Type II Error decreased

49-year-old gradual onset of numbness and pain in left index finger, finger mottled blue and pale –
Platelet count – 1,530,00. Bone marrow shows megakaryocytic hyperplasia. Which is most likely diagnosis?

Essential Thrombocythemia

High Platelet count – increased megakaryocytes

Normal RBC and WBC

Erythromelalgia = condition characterized by episodes of pain, redness, and swelling in various parts of the body, particularly the hands and feet

Image: 49-year-old gradual onset of numbness and pain in left index finger, finger mottled blue and pale -
Platelet count - 1,530,00. Bone marrow shows megakaryocytic hyperplasia. Which is most likely diagnosis?

A 53-year-old evidence of adrenal failure – On CT both adrenal glands decreased in size. Most likely diagnosis?

Autoimmune adrenalitis
Addisons disease – bilateral adrenal atrophy and destruction

Image: A 53-year-old evidence of adrenal failure - On CT both adrenal glands decreased in size. Most likely diagnosis?

85-year-old – femur fracture – begins TX with morphine controlled analgesic pump – 3 days later – respirations are 6/min. PE showed pinpoint pupils. Serum creatinine increased. What explains clinical deterioration after morphine therapy?

Morphine is metabolized to active metabolites

45 year old end stage renal failure – ER due to depression, bilateral crackles in chest, confusion, Mental status exam shows confusion. Serum studies show a decrease in which?

Bicarbonate

29 year old woman inflammatory disease involving joints, kidneys, serosal surfaces, anterior chamber of the eye, choroid plexus. Which mechanism most likely causing disorder?

immune complex-mediated cytotoxicity
SLE – Type III Hypersensitivity – Organ damage

Image: 29 year old woman inflammatory disease involving joints, kidneys, serosal surfaces, anterior chamber of the eye, choroid plexus. Which mechanism most likely causing disorder?

A 57 y/o woman – lump in her breast. Carcinoma suspected. X-ray show multiple lesions in vertebral column, and metastatic disease is suspected. Which of the following veins draining the breast provides the most direct pathway for malignant cells to the vertebral column?

Intercostal

38-year-old man – Down Syndrome – brought to physician by his nephew for a follow-up exam. Pt lives with sister and her children. He is unkempt, malodorous, and appears anxious, 6.4 weight loss. When asked to interview on patient on his own, the nephew refuses to leave the room. Most appropriate initial action?

contact adult protective services

2-year-old – fever, cough, 2 strep infections, multiple ear infections, 101.1, crackles in lower lobe. PE shows no tonsillar tissue. Crackles are heard in right lobe of lung. Which most likely immunodeficiency syndrome?

Bruton agammaglobulinemia

Image: 2-year-old - fever, cough, 2 strep infections, multiple ear infections, 101.1, crackles in lower lobe. PE shows no tonsillar tissue. Crackles are heard in right lobe of lung. Which most likely immunodeficiency syndrome?

43-year-old eversion fracture – contraction of which muscle caused pain from fracture?

Fibularis (peroneus) brevis –

Image: 43-year-old eversion fracture - contraction of which muscle caused pain from fracture?

50-year-old – severe chest pain – MI – ST elevation in anterior chest leads- troponin I increased- Which reflects electrolyte content for myocardiocytes that produced the increase enzymes in this patient?

Intracellular (Na+) Increased
Intracellular (K+) decreased
Intracellular Ca2+ Increased

Image: 50-year-old - severe chest pain - MI - ST elevation in anterior chest leads- troponin I increased- Which reflects electrolyte content for myocardiocytes that produced the increase enzymes in this patient?

A 45 y/o old – T2DM – low grade fever – catheter inserted below clavicle. Two blood grow Nonhemolytic, catalase negative – gram positive cocci in pairs and chains – organism?

Enterococcus Faecalis

42-year-old numbness and tingling vibration decreased in both legs. Sensation to vibration is decreased. Deep tendon reflexes. Romberg positive, cause of findings. Which is most likely cause?

Sensory neuropathy

. A 3 month girl – well child exam – able to life head, does not smile to human faces, has not started to coo or gurgle with attention. What is the development status of (Motor, Social and Verbal and Cognitive)?

Motor Normal
Social delayed
Verbal and Cognitive delayed

73 year old – obstructive sleep apnea -central obesity – CPAP will decrease which?

Blood pressure

62-year-old, HTN nephropathy, chronic renal insufficiency, likely serum concentrations?

Inorganic Phosphorus – Increased
Parathyroid Hormone – Increased
Calcitriol – Decreased

Image: 62-year-old, HTN nephropathy, chronic renal insufficiency, likely serum concentrations?

10-year-old renal transplant – from living donor – 3 months later there is tenderness at the site of the graft – serum creatinine increased. Which findings confirm diagnosis of transplant rejection?

Lymphocytes infiltrating tubular epithelium

A 52-year-old with stable angina – treated with med decrease preload and increase coronary artery flow – greatest risk of adverse effect of this medication?

Headache

25-year-old – 6 month irregular menstrual periods, acne, hair on lip, closed comodones, serum shows?

Fasting Insulin increased
Testosterone Increased
LH increased

Image: 25-year-old - 6 month irregular menstrual periods, acne, hair on lip, closed comodones, serum shows?

A study conducted to assess the prevalence of HTN in Caucasian vs African American. Determining statistical significance between Drug X and Drug Y. Which statistical test is most appropriate to determine the difference is statistical significant?

Chi Square test

An 84-year-old woman- 2-month history of sore left hand – red, scaly plaque on the dorsum of left middle finger- squamous cell carcinoma – invasion of basement membrane facilitated by which of the following?

Downregulation of e-cadherin

52-year-old with lump in inguinal region, (present for 4 months). Which of the following will be impaired if herniated gut compresses the nerve lies of external surface of spermatic cord at the superficial inguinal ring?

Sensation from the anterior surface of the scrotum

Image: 52-year-old with lump in inguinal region, (present for 4 months). Which of the following will be impaired if herniated gut compresses the nerve lies of external surface of spermatic cord at the superficial inguinal ring?

30 y/o M and 24 y/o F – preconceptional counseling – husband’s brother is albino – autosomal recessive that effects (1:40,000). Alleles in equilibrium, woman family is negative. A pedigree is shown. Which of the following is the best estimate of child will be affected with oculocutaneous albinism?

1/600

The probability of the father being a carrier is 2/3 – known doesn’t have the disease.

Then the probability of him passing it on to his kid is: 1/2 * 2/3 = 1/3

With the Hardy-Weinberg Principle, you can figure out the probability of the mother being a carrier:
q = sqrt(1/40,000) = 1/200
So, 2pq = 2 1/200 199/200, which approx is 1/100, and the probability of the child getting this allele is 1/100 * 1/2 = 1/200
Thus:
1/200 * 1/3 = 1/600

A 72-year-old on ketorolac after abdominal operation (Pt has 2- year history of HTN and T2DM), greatest risk drug induced adverse effects

Acute renal failure
NSAID – causes acute renal failure

A 79 y/o – brought to ED 30 minutes after he lost consciousness for 30 seconds. – ECG shows third degree AV block – next step?

Insertion of transvenous pacemaker

Image: A 79 y/o - brought to ED 30 minutes after he lost consciousness for 30 seconds. - ECG shows third degree AV block - next step?

26-year-old develops hypotension and hemoglobinuria after receiving packed red blood cells- Which of following is most likely to be involved in her condition?

Antibody, complement C5-9

40-year-old woman – dry mouth, itching, burning eyes. PE shows inflammation of both cornea and sclerae, enlarged salivary glands, and paucity if saliva. She is told she has risk of developing lymphoma. What is mode of inheritance of disorder

Multifactorial

. A randomized control trial to assess the effect of diuretic therapy on systolic hypertension. A total of 4736 patients – with systolic BP > 140 and diastolic pressure from 50mm HG to 85 mm Hg. Which represents the absolute risk reduction for cerebral infarction in patients receiving diruetic therapy:

Diuretic Therapy: 123 with CI / 2365 Total
Placebo: 194 with I / 2371 Total

(194/2371-123/2365)

Image: . A randomized control trial to assess the effect of diuretic therapy on systolic hypertension. A total of 4736 patients - with systolic BP > 140 and diastolic pressure from 50mm HG to 85 mm Hg. Which represents the absolute risk reduction for cerebral infarction in patients receiving diruetic therapy:

Diuretic Therapy: 123 with CI / 2365 Total
Placebo: 194 with I / 2371 Total

29-year-old MVA – MVC – Abdominal exam shows rebound tenderness- Ct shows small amount of intraperitoneal fluid – surgeon slides hand behind liver- sudden massive hemorrhage- blood pressure drops and becomes undectable within 1 minute- what lesion involved with hemorrhage?

avulsion of hepatic veins from the inferior vena cava
massive hemorrhage – behind liver – IVC – connects to liver via hepatic veins

Image: 29-year-old MVA - MVC - Abdominal exam shows rebound tenderness- Ct shows small amount of intraperitoneal fluid - surgeon slides hand behind liver- sudden massive hemorrhage- blood pressure drops and becomes undectable within 1 minute- what lesion involved with hemorrhage?

37-year-old – excision of 1cm, painless bump on back of neck – 7 days later incision is erythematous and warm, no pain or drainage- what is the histologic appearance of the incision?

Angiogenesis
Red in color, warm, no pain, no drainage

12-year-old boy is swimming in mountain cold stream – he is immersed in 60 F for 20 minutes. physiologic changes?

Central Blood Volume increased
ADH Decreased
Atrial Natural Peptide Increased

physiological response to hypothermia = vasoconstriction of peripheral vessels
in an effort to keep your core body temperature normal, and thus your organs functioning properly.
Peripheral Vasoconstriction will increase CBV. Increased CBV will cause an increase in preload, and thus cause an increase in ANP/BNP.
ANP/BNP has inhibitory effects on the Renin-Angiotensin-Aldosterone System, resulting in decreased ADH

Following a stroke, a patient is hoarse and cannot detect pinprick or cold on left side of face or right side of body- damage occurred where?

Lateral Medulla

Image: Following a stroke, a patient is hoarse and cannot detect pinprick or cold on left side of face or right side of body- damage occurred where?

54-year-old man- severed fingers slicing meat in deli 6 months ago- decreased sensation In fingers- finger was successfully reattached. Exam shows muscle wasting. Which cells is most likely blocking reinnervation of the muscles of this patient’s finger?

Schwann Cells

Image: 54-year-old man- severed fingers slicing meat in deli 6 months ago- decreased sensation In fingers- finger was successfully reattached. Exam shows muscle wasting. Which cells is most likely blocking reinnervation of the muscles of this patient's finger?

56-year-old man – radical prostatectomy – structures greatest risk of injury during removal of prostate?

Pelvic Parasympathetic nerves
S2-S4

Image: 56-year-old man - radical prostatectomy - structures greatest risk of injury during removal of prostate?

A previously health 28 year old – severe abdominal pain – nausea, vomiting, blood in urine, hypoactive bowel, right flank pain – no rebound, no hernia, testes are normal – Likely Diagnosis?

Nephrolithiasis
Kidney Stones – hypoactive bowel sounds, flank pain, unilateral, distress writhing in pain

Image: A previously health 28 year old - severe abdominal pain - nausea, vomiting, blood in urine, hypoactive bowel, right flank pain - no rebound, no hernia, testes are normal - Likely Diagnosis?

42-year-old woman heart burn – relief with ranitidine – The therapeutic effect of is most likely mediated by receptor who activation increases the parietal cells of the stomach?

cAMP
b. H2 receptors are Gs Receptors = increase cAMP
c. Gastrin released by G Cells

Image: 42-year-old woman heart burn - relief with ranitidine - The therapeutic effect of is most likely mediated by receptor who activation increases the parietal cells of the stomach?

A 45 year old with Li-Fraumeni syndrome – participates in study of tumor suppressor gene disorders. P53 mutation. Which is most likely result if mutation on the transcription of genes that inhibit cell division and contain the consensus sequence (TATA) at the transcription site?

Decreased binding of RNA Polymerase

Ten healthy adults given new oral drug – blood sample analyzed to determine pharmacokinetics for the first time – Stage of trial?

Phase 1
small number of healthy – safety studies

Image: Ten healthy adults given new oral drug - blood sample analyzed to determine pharmacokinetics for the first time - Stage of trial?

Previously healthy 60-year-old F- MI of right ventricle – hepatomegaly- ascites -accumulation of fluid in the peritoneal space. Which mechanism causes hepatomegaly and ascites?

Increased central venous pressure

fluid backup – right heart failure – leads to Elevated pressures in the liver – leads to hepatomegaly and free fluid accumulation

Image: Previously healthy 60-year-old F- MI of right ventricle - hepatomegaly- ascites -accumulation of fluid in the peritoneal space. Which mechanism causes hepatomegaly and ascites?

A 65 y/o – starts using topical fluoruracil for treatment of actinic keratosis. Fluorouracil is effective because it inhibits which enzyme?

Thymidylate synthase

Image: A 65 y/o - starts using topical fluoruracil for treatment of actinic keratosis. Fluorouracil is effective because it inhibits which enzyme?

22 y/o pregnant woman decreased Alpha Fetoprotein and unconjugated steroids and increased serum concentration of human chorionic gonadotropin. Which disorder?

Down Syndrome
decreased AFP and increased serum concentration of HcG

A newborn has cyanosis, tachypnea, and retractions, arterial blood gas values while she breathes room air: pH 7.04, PCO2 65, PO2 36. HCO3 15 – Acid Based Status?

Respiratory acidosis and metabolic acidosis
Ph low – acidosis
Bicarb low – Acidosis
pCO2 high – Respiratory response

A 60-year-old man brought to ER – sudden onset 0f right chest pain – fractured rib – multiple osteoblastic lesions in lumbar spine, pelvis, and ribs – what is most likely diagnosis?

Prostate adenocarcinoma

A 3 day old – full term newborn – yellow skin – PE shows jaundice. Hemoglobin concentrations of 17 – Total bilirubin 10mg/dl and indirect component of 8 mg/dl. Jaundice resolves in 5 days later. A deficiency in what causes the jaundice?

Hepatic UDP-glucuronosyltransferase activity

Image: A 3 day old - full term newborn - yellow skin - PE shows jaundice. Hemoglobin concentrations of 17 - Total bilirubin 10mg/dl and indirect component of 8 mg/dl. Jaundice resolves in 5 days later. A deficiency in what causes the jaundice?

63-year-old man – progressive stiffness – difficulty initiating movements – He has a resting tremor- which structure in the photograph undergoes degeneration in this condition?

D – Substantia Nigra

Image: 63-year-old man - progressive stiffness - difficulty initiating movements - He has a resting tremor- which structure in the photograph undergoes degeneration in this condition?

Which of the following drug effects is the most common reason for non compliance with cyclic antidepressant?

Anticholinergic

5-year-old – on a farm – diarrhea, cramping, abdominal pain, fever, drinks unpasteurized milk – Am organism is isolated in stool. which is most likely organism?

Yersinia enterocolitica

Image: 5-year-old - on a farm - diarrhea, cramping, abdominal pain, fever, drinks unpasteurized milk - Am organism is isolated in stool. which is most likely organism?

20-year-old – on crutches -tingling and numbness on dorsum of right hand – PE shows weak extension of the right wrist and difficulty raising the right arm above the shoulder. Most likely nerve injury?

C – Radial and Axillary nerve impacted

Image: 20-year-old - on crutches -tingling and numbness on dorsum of right hand - PE shows weak extension of the right wrist and difficulty raising the right arm above the shoulder. Most likely nerve injury?

62 y/o F – discovers painless mass in left breast-menopause occurred at 50 – 2cm firm, non- tender mass in upper quadrant, irregular microcalcifications, extremely radio dense. No weight loss, axillary lymphadenopathy or discharge. Mammography shows a radiodense mass with irregular margins. It contains clustered irregular microcalcifications – DX

Carcinoma of the breast

73-y/o – difficulty urinating and frequent urination – drugs block following labeled sites

Blocker of alpha 1
Tamsulosin- difficulty urinating

Image: 73-y/o - difficulty urinating and frequent urination - drugs block following labeled sites

A 50 year old man – 3 day history of tonic-clonic seizures. One-month episodes of pins and needles around the mouth and in the hands and feet. involuntary contraction of the muscles of the hands and feet. PE shows diffuse hyperreflexia – An abnormality of which electrolyte concentrations

Calcium

32 y/o – receiving zidovudine – mutation in zidovudine causing resistance. Which mutation?

Reverse Transcriptase

52-year-old man – brought to ER – confused – he had a faulty space heater – Reddish tinged skin – Carbon Monoxide poisoning, supplemented oxygen, best estimate of time for removal of CO-carrying erythrocytes takes how long?

4 Months

Image: 52-year-old man - brought to ER - confused - he had a faulty space heater - Reddish tinged skin - Carbon Monoxide poisoning, supplemented oxygen, best estimate of time for removal of CO-carrying erythrocytes takes how long?

35-year-old enlarging nose, facial features, muscle weakness, increased hand and foot size. MRI shows pituitary adenoma. Further studies show Gas subnit of G portiens lack GTPase activity- tumor cells have increased activity of the following?

Adenylyl cyclase

Image: 35-year-old enlarging nose, facial features, muscle weakness, increased hand and foot size. MRI shows pituitary adenoma. Further studies show Gas subnit of G portiens lack GTPase activity- tumor cells have increased activity of the following?

A 62-year-old man – evaluated for rectal bleeding – Xray of GI track shown. Which explains feathery appearance of X compared with the portion of Y?

Greater mucosal surface area

Image: A 62-year-old man - evaluated for rectal bleeding - Xray of GI track shown. Which explains feathery appearance of X compared with the portion of Y?

32-year-old 1 day history of chest pain radiates to her left arm, returned from a trip to the amazon rain forest. Image of blood smear. What is vector?

Reduviid bug –
Amazon, Chest Pain, Dilated Cardiomegaly, Cardiac Arrythmia, treponema cruzi

Image: 32-year-old 1 day history of chest pain radiates to her left arm, returned from a trip to the amazon rain forest. Image of blood smear. What is vector?

22 year old woman 3 day history of nonproductive cough. 1-week history of fatigue, SOB, and swelling of her legs. She delivered a male newborn uncomplicated vaginal delivery. Bilateral basilar crackles are heard. There is 1+ edema in lower extremity: diagnosis

Cardiomyopathy
Pregnancy can cause dilated cardiomyopathy causing systolic dysfunction.

Image: 22 year old woman 3 day history of nonproductive cough. 1-week history of fatigue, SOB, and swelling of her legs. She delivered a male newborn uncomplicated vaginal delivery. Bilateral basilar crackles are heard. There is 1+ edema in lower extremity: diagnosis

Clostridium perfrigens A-toxin affects cells and facilitates gas gangrene by which of the following mechanisms?

Splitting lecithin to phosphorylcholine and diglyceride

Image: Clostridium perfrigens A-toxin affects cells and facilitates gas gangrene by which of the following mechanisms?

Physician unable to communicate bad news to a patient because the patient reminds him of his older brother who intimidated him. Which of the following terms describes reaction?

Countertransference

12-year-old boy. 6-hour history of severe diarrhea. He appears dehydraterd. Oral hydration contains which of the following sets of ingredients?

Glucose and Sodium

Image: 12-year-old boy. 6-hour history of severe diarrhea. He appears dehydraterd. Oral hydration contains which of the following sets of ingredients?

86-year-old with three-week history of chronic nonproductive cough. He is tired in the mornings because the cough interrupts his sleep. In addition to further evaluation to determine the cough, which of the drugs is most appropriate treatment the patient while avoiding adverse effects of constipation?

Dextromethorphan

74-year-old emphysema and lung cancer – wasted appearance -He is impoverished – cereal and toast diet- He had lost 13kg. He has muscle wasting, sunken eyes, loose skin, and edema. He has a large mass in hilum of lung – edema – What causes the edema?

Negative Nitrogen Balance

Image: 74-year-old emphysema and lung cancer - wasted appearance -He is impoverished - cereal and toast diet- He had lost 13kg. He has muscle wasting, sunken eyes, loose skin, and edema. He has a large mass in hilum of lung - edema - What causes the edema?

65-year-old man (T2DM and peripheral neuropathies) wasting of interosseous muscle of left land, inability to abduct the fingers of the hand; and plantar flexion of the right foot is absent – Which pairs of nerve are impaired?

Ulnar and Tibial

7-year-old – excessive urination- weight loss. PE shows poor skin tugor and fruity odor on breath – HIGH BLOOD GLUCOSE 612 mg/dl – lab studies show increase in:

Serum potassium concentration
DKA – decrease insulin – increased serum potassium

29-year-old – 5 week history of fatigue- 4 day history of heart palpitations. She has Primary Hypothyroid – and treated with Triidothyronine. She says she has a double dose because of fatigue, fine tremor, deep tendon reflexes are brisk: Serum Concentrations- TSH, Free Thryoxine, Free Triiodfothryronine?

TSH Decreased
Free Thyroxine Decreased
Free Triiodothyronine Increased

An unimmunized 1-year old boy is admitted to hospital fever, irritability, stiff neck, cultures of the nasopharynx and cerebrospinal fluid grow gram negative coccobacilli – pili removed from nasopharynx but absence from CSF – Which process shuts off expression of pili?

Phase variation

Image: An unimmunized 1-year old boy is admitted to hospital fever, irritability, stiff neck, cultures of the nasopharynx and cerebrospinal fluid grow gram negative coccobacilli - pili removed from nasopharynx but absence from CSF - Which process shuts off expression of pili?

50-year-old – smoker, increase SOB, chronic cough productive, thick sputum and wheezing. Anterior and Posterior Chest Wall Increased. Diminished breath sounds and scattered rhonchi. Which lab abnormality?

Increased blood HCO3-
emphysema leads to CO2 trapping leading to increase paCO2 in the blood, which gives you a respiratory acidosis

Image: 50-year-old - smoker, increase SOB, chronic cough productive, thick sputum and wheezing. Anterior and Posterior Chest Wall Increased. Diminished breath sounds and scattered rhonchi. Which lab abnormality?

18-month old boy – admitted with tonic clonic seizure, – He has mild development delays – 25th% on growth charts, six hypopigmented macules, flesh colored lesion on back. An MRI of brain shows multiple lesions in cerebral cortex. Which DX?

Tuberous sclerosis complex

Image: 18-month old boy - admitted with tonic clonic seizure, - He has mild development delays - 25th% on growth charts, six hypopigmented macules, flesh colored lesion on back. An MRI of brain shows multiple lesions in cerebral cortex. Which DX?

A 66-year-old – 2 month erectile dysfunction. He has fatigue and difficulty sleeping. 3 months ago cerebral infarction 2 month prior – PE shows no abnormalities – Which additional pairs of findings (Libido and Nocturnal erection?)

Libido Decreased
Nocturnal Erection Normal

A 66-year-old – 2 month erectile dysfunction. He has fatigue and difficulty sleeping. 3 months ago cerebral infarction 2 month prior – PE shows no abnormalities – Which additional pairs of findings (Libido and Nocturnal erection?)

Libido Decreased
Nocturnal Erection Normal

31-year-old – concerns about hair loss. Family History of alopecia. If pharma treatment is elected – use a drug to block synthesis of which of the following:

Dihydrotestosterone

56-year-old recovering from pneumonia – No X-ray abnormalities. Which allowed resolution to occur?

Maintenance of basement membrane integrity

65 y/o osteoarthritis, pain radiating down distal anterior thigh, knee, medial leg, and foot. Bony outgrowth of vertebrae compressing one of the spinal nerves. Compression of nerve root in which of the intervertebral foramina most likely cause her symptoms?

L-3 to 4

Image: 65 y/o osteoarthritis, pain radiating down distal anterior thigh, knee, medial leg, and foot. Bony outgrowth of vertebrae compressing one of the spinal nerves. Compression of nerve root in which of the intervertebral foramina most likely cause her symptoms?

. 17 y/o boy – 1 week of fever, fatigue, sore throat, red oropharynx, bilateral cervical lymphadenopathy – heterophil antibody positive – 12 month after complete recovery, which cells still contain virus that caused his sore throat?

B Lymphocyte

Image: . 17 y/o boy - 1 week of fever, fatigue, sore throat, red oropharynx, bilateral cervical lymphadenopathy - heterophil antibody positive - 12 month after complete recovery, which cells still contain virus that caused his sore throat?

10-year-old girl lacerated her chin. Painful subcutaneous nodule on chin. Sutured Ten weeks ago later image of biopsy- which describes the pathology features?

Granuloma

Image: 10-year-old girl lacerated her chin. Painful subcutaneous nodule on chin. Sutured Ten weeks ago later image of biopsy- which describes the pathology features?

Pts with mucolipidosis II (I Cell Disease) lack of phosphotransferase required to form mannose 6-phosphate that attach to destined enzymes – mannose-6-phosphate is normally added to proteins in the cis-Golgi. Which occurs to lysosomal enzymes in these patients?

Secreted from the cells

A case control assess exposure to environmental tobacco smoke and sinusitis. Exposed to Smoke Cases and Not Exposed Cases Shown – What is the prevalence of sinusitis in the general population?

Cannot be determined from the data given

Image: A case control assess exposure to environmental tobacco smoke and sinusitis. Exposed to Smoke Cases and Not Exposed Cases Shown - What is the prevalence of sinusitis in the general population?

A case control assess exposure to environmental tobacco smoke and sinusitis. Exposed to Smoke Cases and Not Exposed Cases Shown – What is prevalence of sinusitis in the general population?

Cannot be determined from the data given

Study on symptomatic proximal DVT, patients randomly assigns (2 groups) to receive either dalteparin subcutaneous and then oral warfarin, or dalteparin alone for 6 months- Probability of recurrent thromboembolism 6 months is 10% in daltepain and 19% in warfarin group. Which is the study design?

Open-label clinical trial
a. Clinical trial – compares therapeutic benefits of 2+ treatments (warfarin vs. dalteparin)
b. Open-label – both the health providers and the pt are aware of the drug being given

blood pressure, systolic pulsation of the liver. Liver is percussed to 12cm. Which is the diagnosis?

tricuspid insufficiency

78 year old severe back pain, worse at night, serum ALK phosphatase, increased lytic lesions in vertebral columns. Diagnosis?

Metastatic Carcinoma

31-year-old woman – 2nd trimester of pregnancy- vagina bleeding. BP 140/95 HTN- Ultrasound shows conceptus with patches of tissue – no apparent embryo. Biopsy shows 69 XXY. The presence of what would indicate that an embryo had formed within the fetus?

Hematopoietic stem cells

24-year-old woman, post pregnancy hyperthyroid, nervousness and tremor – 5lb weight loss – thyroid 2X as large. Free Thyroxine 2.4, TSH low, thyroid iodine uptake 1%- which likely explanation for symptoms?

Release of stored thyroid hormone from a thyroid gland infiltrated by lymphocytes

A 27-year-old man – smoker- wants to quit, but not sure ready for change. Contemplation. Which is the next step for the physician?

Ask the patient to identify the pros and cons of smoking cessation

A 36 year-old woman- SLE – Treatment with prednisone – drug binds to receptor in following site?

Cytosol with translocation into the nucleus

Image: A 36 year-old woman- SLE - Treatment with prednisone - drug binds to receptor in following site?

27-year-old man construction worker fell from a ladder – wood splinter grazed inferior pole of left kidney and splinter now in structure immediately anterior? What structure?

Splenic Flexure

Image: 27-year-old man construction worker fell from a ladder - wood splinter grazed inferior pole of left kidney and splinter now in structure immediately anterior? What structure?

A 2 day newborn develops bilious emesis, abdominal tenderness, hemodynamic deterioration, Upper GI- X-ray – shows ligament of Treitz on right of abdomen and duodenal obstruction. Pt greatest risk for occlusion of following arteries?

Superior mesenteric

Image: A 2 day newborn develops bilious emesis, abdominal tenderness, hemodynamic deterioration, Upper GI- X-ray - shows ligament of Treitz on right of abdomen and duodenal obstruction. Pt greatest risk for occlusion of following arteries?

A 60 year-old- man – SOB+ worked in shipyard. X-ray show reticulonodular pulmonary infiltrates – Microscopic exam shows elongated structures (Image Ferruginous body) – The fibrosis was most likely initiated by interaction of these structures with which of the following?

Alveolar Macrophage

Image: A 60 year-old- man - SOB+ worked in shipyard. X-ray show reticulonodular pulmonary infiltrates - Microscopic exam shows elongated structures (Image Ferruginous body) - The fibrosis was most likely initiated by interaction of these structures with which of the following?

Previously heathy 45-year-old woman fever, confusion, decreased urine for past 2 days. Platelet count low. Peripheral blood smear. Which is most like cause?

Thrombotic Thrombocytopenia purpura

Image: Previously heathy 45-year-old woman fever, confusion, decreased urine for past 2 days. Platelet count low. Peripheral blood smear. Which is most like cause?

73-year-old woman dies 7 years after onset of progressive memory loss. Severe intellectual deterioration. A coronal section of the brain at autopsy (Shown). Numerous neuritic plaque and neurofibrillary tangles. Cause of ventricular enlargement?

Hydrocephalus ex Vacuo

Image: 73-year-old woman dies 7 years after onset of progressive memory loss. Severe intellectual deterioration. A coronal section of the brain at autopsy (Shown). Numerous neuritic plaque and neurofibrillary tangles. Cause of ventricular enlargement?

76-year-old – 1 month history of a pulsatile abdominal mass is diagnosed with an abdominal aortic aneurysm. CT scan shows horseshoe kidney. Surgical repair is most likely to be further complicated by presence of?

Anomalous origins of multiple renal arteries to each kidney

Image: 76-year-old - 1 month history of a pulsatile abdominal mass is diagnosed with an abdominal aortic aneurysm. CT scan shows horseshoe kidney. Surgical repair is most likely to be further complicated by presence of?

Healthy 25-year-old – eats a meal. serum insulin increases after a meal – mechanism of pancreatic circulation of insulin?

Fusion of an intracellular vesicle with the plasma membrane

26 year old new mother – constant worry – excessive concern for safety and people breaking into her house- most appropriate pharmacotherapy

Sertraline

48-year-old – vomiting blood- severe alcoholic pancreatitis – pseudocyst of pancreatic tail requires surgical drainage. Exam shows splenomegaly – which is vessels are most likely thrombosed?

Splenic Vein

52-year-old -metastatic oat cell carcinoma- receives hospice- severe bone pain—tx narcotic started- Which treatment is most appropriate?

Ensure that the patient receives enough medication to control his pain

38-year-old – chronic headaches- comes to physician for a follow-up exam – She has 3 sons kids sickle cell disease- She tells physician she used to feel angry and abandoned – started kickboxing- feels happier and more relaxed? Coping mechanism?

Displacement

6-year-old emigrated from Russia. Unstable gait and incoordination. He has frequent pale, bulky stool. Neuro exam shows ataxia, absence of DTR, fatty stools, loss of proprioception. Stool analysis shows an increased fat concentration. Which vitamin deficient?

Vitamin E

45 y/o p- HIV positive -1-week history of headaches and blurred vision- a lumbar puncture is done- CSF – positive for fungal yeast – Most appropriate treatment is a drug with which of the following mechanisms?

Disruption of the pathogen cell membrane

Image: 45 y/o p- HIV positive -1-week history of headaches and blurred vision- a lumbar puncture is done- CSF - positive for fungal yeast - Most appropriate treatment is a drug with which of the following mechanisms?

60-year-old woman – 1 year of involuntary rhythmic jerking, increase jerk reflex, Babinski sign present. An MRI shows a 2 -cm, round, enhancing lesion with interhemispheric fissure in the region of the central sulcus. Most likely DX?

Meningioma

Image: 60-year-old woman - 1 year of involuntary rhythmic jerking, increase jerk reflex, Babinski sign present. An MRI shows a 2 -cm, round, enhancing lesion with interhemispheric fissure in the region of the central sulcus. Most likely DX?

New Virus – causes encephalitis – The viral particle contains and RNA-dependent DNA polymerase – which of the following types of RNA most likely to be genome of the virus?

Single-stranded positive-sense

5-year-old – pain in right eye at night for the past week- Exam shows strabismus and tenderness in the eye. Examination of the retina shows presence of a mass. Physician explains that boy is unlikely to develop other neoplasms- mutation started in which of following?

Retinal Cells

Image: 5-year-old - pain in right eye at night for the past week- Exam shows strabismus and tenderness in the eye. Examination of the retina shows presence of a mass. Physician explains that boy is unlikely to develop other neoplasms- mutation started in which of following?

62-year-old man with dyslipidemia-6 hours of severe muscle aches, generalized weakness, and brown urine – he recently started on lipid lowering medication, increased serum creatinine, blood on urinalysis -what medication was added?

Gemfibrozil

5-year-old boy – red cheeks+ rash over arms and lefs. 1 week history of fever – PE shows red, lacy rash over the upper extremity and lower extremity. Low hemoglobin. Cause of anemia?

Interruption of erythrocyte production
Parvovirus infection, which infects erythroid precursors

Image: 5-year-old boy - red cheeks+ rash over arms and lefs. 1 week history of fever - PE shows red, lacy rash over the upper extremity and lower extremity. Low hemoglobin. Cause of anemia?

57-year-old woman – progressive weakness in arms and legs. She says states that weakness is most apparent when she tries to lift dishes and when she walks down a flight of stairs. Muscles strength improves on repetitive manual motor testing.

Sensation intact in all extremities. Which of the following mechanisms explains weakness?

Autoimmune downregulation of Ca2+ channels of the presynaptic terminal

10-year-old boy bruises easily since swallowing grandfather pill. Aspirin and Dicumarol are found. How do we confirm patient took dicumarol rather than aspirin?

Prolonged prothrombin time
Dicumarol – Depletes vitamin K stores
factors 2, 7, 9, & 10

39-year-old -woman with Rheumatoid arthritis – TX with multiple medications – including prednisone and methotrexate – NO progress. The next step pharmacotherapy includes drugs that block the effects of?

Tumor necrosis factor-a

Image: 39-year-old -woman with Rheumatoid arthritis - TX with multiple medications - including prednisone and methotrexate - NO progress. The next step pharmacotherapy includes drugs that block the effects of?

17-year-old -taking 2 aspirin tablets for tension headache- 30 minutes after she has difficulty breathing – what meds in future should she use to treat headaches?

Acetaminophen

53-year-old woman – fibromuscular dysplasia involving renal artery-develops HTN – evaluated for revascularization procedure. Which of the following histologic findings most likely?

Tubular atrophy

Image: 53-year-old woman - fibromuscular dysplasia involving renal artery-develops HTN - evaluated for revascularization procedure. Which of the following histologic findings most likely?

81-year-old – ongoing management of HTN for 20 years, high blood pressure (200/110), To reconfirm the BP, physician puts index finger over the radial artery. At this point the radial artery becomes non-pulsatile but remains easily palpable even as cuff is further inflated. Which explains phenomenon?

Atherosclerosis

An 1814-g (4-lb) male – delivered in hospital at 39 weeks – Develops respiratory distress and dies 8 hours later. Congenital abnormality- congenital absence of pancreatic islet cell. Most likely cause of this congenital abnormality is s defect in the initial differentiation from which of the following precursors?

Endodermal cells

65-year-old ER 1 day of SOBs – He has a history of hypertension, T2DM, 2/6 systolic murmur. Pulse Ox is 90% – Cardiac exam shows S3 – The point of maxillary impulse in the axillary line. Which finding is most likely on pulmonary auscultation?

Crackles
S3–> dilated cardiomyopathy (eccentric)

Image: 65-year-old ER 1 day of SOBs - He has a history of hypertension, T2DM, 2/6 systolic murmur. Pulse Ox is 90% - Cardiac exam shows S3 - The point of maxillary impulse in the axillary line. Which finding is most likely on pulmonary auscultation?

8-year-old boy – parents concerned about his weight – BMI 25- 75th height and 95th percentile weight + BMI – most likely explanation for obesity?

Calorie consumption that excess energy expenditure

. 5-year-old with mental retardation- grossly obese and facial features of Prader Willi – Karotyping and fluorescent in situ hybridization do not show deletion in the usual site of chromosome 15. Which of the following is most likely to confirms Prader Willi?

maternal origin of both chromosomes 15

Prader Willi = Paternal deletion (partial or full). Noted for imprinting.

25% of cases are due to maternal uniparental disomy

Nicotinic Acid acts at which of the following labeled sites in the diagram?

C. Cholesterol – VLDL

Image: Nicotinic Acid acts at which of the following labeled sites in the diagram?

In pt’s with Adenosine deaminase deficiency – 50-100 fold increase in dATP concentrations in T lymphocytes- the increased dATP inhibits which enzyme – compromising DNA synthesis?

Ribonucleotide reductase

RNR converts ribonucleotides to deoxyribonucleotides,- then – dATP (a deoxyribonucleotide) would inhibit the enzyme

Image: In pt's with Adenosine deaminase deficiency - 50-100 fold increase in dATP concentrations in T lymphocytes- the increased dATP inhibits which enzyme - compromising DNA synthesis?

37-year-old – 1-year HTN -high BP despite TX (190/135) – Fundoscopic exam shows – multiple flame hemorrhages – severe arteriolar narrowing. What disorder of blood vessels is the cause?

Hyperplastic arteriolosclerosis

Image: 37-year-old - 1-year HTN -high BP despite TX (190/135) - Fundoscopic exam shows - multiple flame hemorrhages - severe arteriolar narrowing. What disorder of blood vessels is the cause?

A 4 month old boy small size, difficulty feeding from a bottle, recently adopted from developing country (unknown medical history), (Photograph of baby cleft lip) Most likely cause of facial finding (INVOLVING LIP) is a failure of normal fusion of the following pairs of structures?

maxillary and medial nasal prominence

Image: A 4 month old boy small size, difficulty feeding from a bottle, recently adopted from developing country (unknown medical history), (Photograph of baby cleft lip) Most likely cause of facial finding (INVOLVING LIP) is a failure of normal fusion of the following pairs of structures?

38-year-old – admitted after multiple injuries in farm implement accident- The day after admission. what causes increase in his plasma fibrinogen?

Acute Phase Response
Acute phase reactants up-regulated during systemic manifestations: “More FFiSH in the C” Ferritin, Fibrinogen, Serum amyloid A, Hepcidin, C-reactive protein.

A 13-year-old with teased for “having girl like breasts”. Tanner Stage 3 – no health issues- average height and weight – PE shows soft mound of tissue bilaterally beneath the areola – Most appropriate next step?

reassurance

Image: A 13-year-old with teased for "having girl like breasts". Tanner Stage 3 - no health issues- average height and weight - PE shows soft mound of tissue bilaterally beneath the areola - Most appropriate next step?

A 48 -year-old – begins Furosemide therapy for pedal edema – associated with biventricular failure and HTN- 5 days later decreases potassium – what actions should be added to the regime?

Decrease the luminal permeability to Na+ in the collecting duct
Amiloride and Triamatrene block EnAC channels on luminal membrane- K Sparing
Keep your SEAT

Image: A 48 -year-old - begins Furosemide therapy for pedal edema - associated with biventricular failure and HTN- 5 days later decreases potassium - what actions should be added to the regime?

Pt early stage of hemorrhagic shock – most likely to have what symptom?

A weak pulse due to decreased stroke volume
(hypovolemic shock)-
early in shock not enough time for RAAS activation

During first week of life – Newborn has vomiting, severe dehydration, hyponatremia, salt wasting. Serum 17-hydroxyprogesterone increased. Most likely cause is a defect in which disorder?

21- Hydroxylase
Congenital Adrenal Hyperplasia

Image: During first week of life - Newborn has vomiting, severe dehydration, hyponatremia, salt wasting. Serum 17-hydroxyprogesterone increased. Most likely cause is a defect in which disorder?

28-year old – recurrent pancreatitis with familial hypertriglyceridemia. Administration of a drug with which of the following effects?

Decreasing VLDL

. A newborn at 26-weeks gestation- Pt has respiratory Distress and requires intubation and maximal oxygen support. A chext X-ray shows ground glass appearance in both lungs. Deficient synthesis of which is most likely cause of respiratory difficulty in the newborn?

Dipalmitoyl Lecithin
Dipalmitoyl Lecithin = Lung Surfactant = dipalitoylphosphstidylcholine

Screening tests for fetal lung maturity (Lecthin/Sphingomyelin) > 2 healthy, < 1.5 NRDS

Image: . A newborn at 26-weeks gestation- Pt has respiratory Distress and requires intubation and maximal oxygen support. A chext X-ray shows ground glass appearance in both lungs. Deficient synthesis of which is most likely cause of respiratory difficulty in the newborn?

A 6-month-old – development delay- difficulty feeding and recurrent vomiting. He has hepatosplenomegaly and lymphadenopathy. Examination of a biopsy of bone marrow shows foam cells. A defect in metabolic pathways is most likely to cause this diorder?

Sphingomyelin degradation
Niemann Picks Disease- mental retardation, foam cells, cherry red spots in macula

Image: A 6-month-old - development delay- difficulty feeding and recurrent vomiting. He has hepatosplenomegaly and lymphadenopathy. Examination of a biopsy of bone marrow shows foam cells. A defect in metabolic pathways is most likely to cause this diorder?

A 35 year-old-man – (diving accident). Evaluation shows transection of spinal cord superior to the level of the sympathetic nervous outflow – Which responses will occurs if he gets a systemic infection?

Alternation of the thermostatic set point
Hypothalamus controls temperature

A 17-year-old boy (wrestler) – collapsed while competing in wrestling competition. PE shows pallor and diaphoresis. He is losing weight to compete in his weight class. He is using a long steam baths and laxatives – Lab show?

Hypokalemia

A 1-year-old boy impaired respiratory burst of phagocytes – Family history of immunodeficiency – Patient has greatest risk of infection by which organism?

Staphylococcus aureus
Chronic Granulomatous Disease -NADPH deficiency phagocytes need to produce hydrogen peroxide to undergo oxidative burst-

CGD patients can’t kill catalase + organisms (Staph Aureus, Aspergillus) – catalase enzymes neutralize superoxide

An 11-year-old CC: pain in right thigh for 2 weeks. PE shows warmth tenderness, ill-defined mass. , solitary mid-diaphyseal —–osteolytic lesions (small, uniform, hyperchromatic cells: high nuclear: cytoplasmic ratio). – DX?

Ewing Sarcoma

Image: An 11-year-old CC: pain in right thigh for 2 weeks. PE shows warmth tenderness, ill-defined mass. , solitary mid-diaphyseal -----osteolytic lesions (small, uniform, hyperchromatic cells: high nuclear: cytoplasmic ratio). - DX?

A 45-year-old man- alcoholic- chronic pancreatitis + diarrhea – PE shows (decreased fecal elastase) – what meds for diarrhea?

Pancrelipase
Pancrelipase = pancreatic enzymes

Image: A 45-year-old man- alcoholic- chronic pancreatitis + diarrhea - PE shows (decreased fecal elastase) - what meds for diarrhea?

A 30-year-old man – 2 day history of blood in urine, dull flank pain, voiding large urine- BP is 150/100. Pt father died of unknown kidney disease. CT showed cystic masses in each kidney – surgical specimen shown (polycystic kidney disease)- Underlying defect – involves which molecule?

Polycystin

Image: A 30-year-old man - 2 day history of blood in urine, dull flank pain, voiding large urine- BP is 150/100. Pt father died of unknown kidney disease. CT showed cystic masses in each kidney - surgical specimen shown (polycystic kidney disease)- Underlying defect - involves which molecule?

A 42 year-old-woman – tonic clonic seizures- She tells physician that she smelled a foul odor before seizure- where did seizure arise?

Temporal Lobe

A 9-year-old boy with signs of Cystic Fibrosis (cough, recurrent upper respiratory tract and sinus infection)- PE shows clubbing of fingers – increased sweat chloride and sodium. A defect in patient’s bronchial epithelium is most likley causing?

Protein Structure
mis-folding –> less CF receptors on cell surface –> phenotypic CF.

During an experiment, 22 y/o receives an infusion of Histamine in Left Brachial Artery – what changes?

Arteriolar Resistance decreased
Capillary Hydrostatic Pressure Increased
Capillary Filtration Rate Increased

Histamine causes arteriolar dilation, – increased blood flow into capillaries – increased capillary permeability in venules -increased capillary filtration

Image: During an experiment, 22 y/o receives an infusion of Histamine in Left Brachial Artery - what changes?

67-year- old woman non-productive cough, crackles on lung bases, NO fever – CT scan shows bibasilar reticulonodular markings with interlobar septal thickening; Pulmonary function shows Fev1 (normal), FVC 48% of predicted – DLCO – low- Which is likely DX?

Idiopathic pulmonary fibrosis

Image: 67-year- old woman non-productive cough, crackles on lung bases, NO fever - CT scan shows bibasilar reticulonodular markings with interlobar septal thickening; Pulmonary function shows Fev1 (normal), FVC 48% of predicted - DLCO - low- Which is likely DX?

Two patients, 54 year old man and 76 year old man – receive vancomycin – both are same weight – GFR > in Patient X than Y. Which parameter is the same in patient X and Y?

Loading Dose
independent of drug clearance- unchanged in liver or renal dx

Image: Two patients, 54 year old man and 76 year old man - receive vancomycin - both are same weight - GFR > in Patient X than Y. Which parameter is the same in patient X and Y?

A 35-year-old -weakness and fatigue – When 25, she was treated for Hodgkin Disease with Chemo Drugs- 10 years ago- no evidence of lymphadenopathy- A peripheral blood smear shows numerous blasts. Which most likely explanation of findings?

Acute Myelocytic leukemia
Chemo agents – increase risk for AML
ALL = 0-14
AML= 15-39; 40-59 – Characteristics of Numerous Blasts
CML = 40-59
CLL = 60+

Aurer rods

Image: A 35-year-old -weakness and fatigue - When 25, she was treated for Hodgkin Disease with Chemo Drugs- 10 years ago- no evidence of lymphadenopathy- A peripheral blood smear shows numerous blasts. Which most likely explanation of findings?

A 15-year-old – redness from, sunbathing – use sunblock- no blisters – condition?

First-degree burn

A 15-year-old – redness from, sunbathing – use sunblock- no blisters – condition?

First-degree burn

49-year-old severe hip pain – alcoholic – increasing severe pain in his left hip – He has pain with weight bearing- Coronal T1 is shown- Most likely cause of patients pain

Avascular Necrosis
CASTS Bend LEGS = Corticosteroids + Alchoholism+ Sickle Cell + Trauma + SLE
The Bends + Legg Calves

Image: 49-year-old severe hip pain - alcoholic - increasing severe pain in his left hip - He has pain with weight bearing- Coronal T1 is shown- Most likely cause of patients pain

A 48 year- old renal artery stenosis- undergoes stent placement- femoral access stent – After entrance into aorta, guidewire should be advanced superiorly just beyond which structure to approach right renal artery

Testicular Artery

Image: A 48 year- old renal artery stenosis- undergoes stent placement- femoral access stent - After entrance into aorta, guidewire should be advanced superiorly just beyond which structure to approach right renal artery

A 32-year old woman -begins to hyperventilates + blurry vision+ numbness around mouth – after friend dies – which of the following is most likely decreased in this woman?

Cerebral blood flow
vasovagal sympathy – hyperventilation decreased pH, PaCO2,
Bicarb- central chemoreceptors respond to low PaO2 by vasoconstricting cerebral blood flow

A 12-yeasr-old girl – unable to walk for 5 days- her parents are anxious – but she seems unconcerned about her symptoms. DTR are equally bilaterally. DX?

Conversion disorder

60-year-old – SOB and cough – 2 week history of of progressive swelling of her face – Productive cough with blood tinged sputum -Smoker for 45 years – 2.3 kg weight loss – hyponatremia- Which is cause of facial edema?

Lung cancer

42-year-old fatigue, excessive thirst, frequent urination with large volume – He is 5’10 – weighs 61kg – BMI 19 – high ADH- increase serum OsM, decreased Urine OsM. In addition to low-sodium diet, most appropriate drugs for this patient?

Hydrochlorothiazide

Parenteral cholera vaccine (killed) whole bacterial cells have limited efficacy- which is the primary reason for his failure?

Inability of the vaccine to elicit secretory antibody at the epithelial surface

During a clinical study, 15 patients with renal allografts volunteer to undergo treatment with DrugX – The drug is converted to a purine antagonist that interferes with synthesis of nucleic acids and toxic to dividing cells. Drug X is most likely?

Azathioprine

25-year-old -admitted with septic shock – splenectomy from MVA – Coarse tubular breath sounds CXR shows bilateral pulmonary opacities, dense consolidation, and right lung effusin – causal organism

Streptococcus pneumoniae
Splenectomy – vulnerable against encapsulated organisms

Pt early stage of hemorrhagic shock – most likely to have what symptom?

A weak pulse due to decreased stroke volume

A 48 -year-old – begins Furosemide therapy for pedal edema – associated with biventricular failure and HTN- 5 days later decreases potassium – what actions should be added to the regime?

Decrease the luminal permeability to Na+ in the collecting duct

A 13-year-old with teased for “having girl like breasts”. Tanner Stage 3 – no health issues- average height and weight – PE shows soft mound of tissue bilaterally beneath the areola – Most appropriate next step?

reassurance

23-year-old – multiple, red, papulovesicular lesions following poison IVY – Which is mechanism of these lesion??

Processing of the antigen by Langerhans cells, leading to activation of CD4+ T cells

A 48-year-old – fatigue, SOB on exertion, decreased exercise tolerance – X-ray shows decreased cardiac enlargement and prominent pulmonary vasculature- glucose intolerance -liver dysfunction- An older brother died at age 52 years with similar symptoms. Which is most important screening test?

Transferrin saturation and serum ferritin measurements
triad – cirrhosis, diabetes, skin pigment
glucose intolerance (post pharm) + Familial + Presenting after 40- Total Body Iron over 20g
Bronze Diabetes

66 y/o – SOB+, swelling of the legs. HE MI 6 months ago. Exam shows distended jugular veins, bilateral crackles, pitting edema in the lower extremities. Serum sodium concentrations 129. – cause of hyponatremia?

Dilution of serum sodium due to ADH (vasopressin) secretion

Heart Failure – increased vasopressin secretion

A 2-month old with sore throat because of a 7 day history of hoarseness- mother has AIDS and used cocaine through pregnancy – PE shows no abnormalities. Laryngoscope shows a nodule on the left vocal cord. Which viruses is most likely cause of nodule?

Human papillomavirus

Image: A 2-month old with sore throat because of a 7 day history of hoarseness- mother has AIDS and used cocaine through pregnancy - PE shows no abnormalities. Laryngoscope shows a nodule on the left vocal cord. Which viruses is most likely cause of nodule?

A 6-year-old- itchy rash in his armpits, both hands, feet, and groin. The itching is most intense at night. Vitals are within normal limits. Exam of skin shows multiple erythematous papules, burrows and many excoriated (photograph). Which most helpful in establising diagnosis?

Does anyone else in the family have an itchy rash like this

Image: A 6-year-old- itchy rash in his armpits, both hands, feet, and groin. The itching is most intense at night. Vitals are within normal limits. Exam of skin shows multiple erythematous papules, burrows and many excoriated (photograph). Which most helpful in establising diagnosis?

A 13-year-old – routine child exam – Neither mother or child have health concerns – The physicians asks questions on sexual activity – Which of the following actions by the physician is most appropriate?

Ask the mother to leave the room before asking the patient any questions

A 28-year – old woman – 3 week double vision – blurry vision in left eye – right eye does not adduct beyond midline – right horizontal gaze – normal – pt has lesion in which structure labeled structure?

Area labeled C
Intranuclear ophthalmalgia – lesion of ipsilateral MLF

Image: A 28-year - old woman - 3 week double vision - blurry vision in left eye - right eye does not adduct beyond midline - right horizontal gaze - normal - pt has lesion in which structure labeled structure?

10-month-old – pale and unresponsive – mother gave her 3 does of loperamide – for diarrhea – marked distention – If medication is required, most appropriate therapy?

Naloxone

A 28-year-old man – 6-hour history of severe left midabdominal pain – abdominal mass in mid- abdomen – CT scan of abdomen shown – Abnormalities indicted by arrows – intussusception of which of the following portions of intestinal tract is most likely cause of patients pain?

Jejunum

Image: A 28-year-old man - 6-hour history of severe left midabdominal pain - abdominal mass in mid- abdomen - CT scan of abdomen shown - Abnormalities indicted by arrows - intussusception of which of the following portions of intestinal tract is most likely cause of patients pain?

27 year old – ER after injury to ankle while rocking climbing – swelling on medial lateral angle of left ankle – X-ray of left ankle is shown. The patient injury most likely occurred when his left foot was forcibly moved into the extreme of which position?

Eversion

Image: 27 year old - ER after injury to ankle while rocking climbing - swelling on medial lateral angle of left ankle - X-ray of left ankle is shown. The patient injury most likely occurred when his left foot was forcibly moved into the extreme of which position?

A 17-year-old – 16 weeks gestations has a mucopurulent vaginal discharge. Culture of the discharge grows Chlamydia – most appropriate pharmacotherapy?

Azithromycin

. 13-year-old girl brought to ER after sudden onset of SOB- soft tissue swelling- Some episodes accompanied by acute abdominal pain with diarrhea – marked facial edema- the most likely condition- is a deficiency of which substances?

C1 inhibitor

Image: . 13-year-old girl brought to ER after sudden onset of SOB- soft tissue swelling- Some episodes accompanied by acute abdominal pain with diarrhea - marked facial edema- the most likely condition- is a deficiency of which substances?

A 57-yearold woman – burning shock like pain on left of face – pain last 5-10 seconds- No neuro abnormalities – most likely diagnosis?

Trigeminal Neuralgia
facial pain, often triggered by chewing, speaking, or brushing the teeth.

Image: A 57-yearold woman - burning shock like pain on left of face - pain last 5-10 seconds- No neuro abnormalities - most likely diagnosis?

An Rh negative woman, gravida 3, para 2 – increasing bilirubin concentration in amniotic fluid- umbilical cord hematocrit 6%- transfusion of the following types of packed red cells to the fetus in utero should correct the anemia?

O, Rh-negative

16 year old has pain and tingling – while backpacking- symptoms resolve after she removes the back pack- what are the abnormalities

Cervical Rib

Image: 16 year old has pain and tingling - while backpacking- symptoms resolve after she removes the back pack- what are the abnormalities

16-year-old – excessive dieting, constant studying, and social withdrawl. BMI 17. PE shows skin and fine hair growth – Labs shows metabolic alkalosis – The patient is most likely abusing which?

Diuretics
Diuretics: [K dec] [Cl dec] [HCO3 inc] [pH inc]
Vomiting: [K dec] [Cl dec] [HCO3 inc] [pH inc]
Laxatives: [K dec] [Cl inc or dec] [HCO3 dec or inc] [pH dec or inc]

Diuretics:

[K dec] [Cl dec] [HCO3 inc] [pH inc]

Vomiting:

[K dec] [Cl dec] [HCO3 inc] [pH inc]

Laxatives:

[K dec] [Cl inc or dec] [HCO3 dec or inc] [pH dec or inc]

54-year-old man has abdominal aneurysm. The estimated cross-sectional areas is 2 cm^2 and the mean velocity if blood flow is 20cm/sec. Which represents flow rate (L/min

2.4
2cm^2 20 cm/sec 60sec/1 min * L/min

56 y/o – brought to ER – Chest Pain mowing the lawn – Hypotension, PE shows cool, clammy, diaphoretic skin. 2 hours after admission, pulmonary catherization shows pulmonary artery pressure 40/25. Left Atrial Pressure 25 g (pulmonary emboli). Compared with a healthy person- sets of pulmonary pressure changes?

Hydrostatic Pressure High,
Capillary hydrostatic pressure high
Interstitial Oncotic pressure Low

cardiogenic shock, the extra blood increases capillary hydrostatic pressure, driving fluid into the interstitial space.

67 y/o – brother has history of colon cancer. Physician recommends colonoscopy, but the patient says she would prefer stool culture instead. Physician explains that testing the stool for occult blood is not appropriate. Physician concerned about?

Low Sensitivity

A 62 y/o – 1 week of muscle cramps – trouble opening jars and using silverware. PE shows muscle weakness and exaggerated deep tendon reflexes. What is appropriate systemic treatment for muscle cramps and spasms?

agonist at GABAb Receptors

41 y/o schedule for dialysis catheters. Surgeon asks the resident to get the consent. The resident has never seen this procedure performed and has read little about it. Which of the following actions by the resident is most appropriate?

Ask attending to obtain informed consent

A 2 year old – undergoes resection of right kidney – chronic infection secondary to influx. The left kidney is grossly normal. Which structural adaptations will eventually occur in left kidney?

increase in glomerular size
Congenital solitary functioning kidney
compensatory hypertrophy of contralateral kidney

A 54 y/o woman – terminal metastatic pancreatic cancer – She asks physician to prescribe sufficient meds to commit suicide due to unbearable pain. Physicians refuses – but will do anything to help pain. Physician decision is most consistent with which?

Nonmaleficence

22 y/o woman – recently emigrated from rural Mexico – 20 minutes after generalized tonic-clonic seizures. 6 week history of headaches, nausea, vomiting. Funduscopic exam shows bilateral papilledema. An MRI of the brain shows enlargement of right ventricle consistent with Cysticercosis. Which is mostly likely mechanism of hyderocephalus?

Obstruction of right interventricular foramen (foramen of Monroe)

22 y/o woman – recently emigrated from rural Mexico – 20 minutes after generalized tonic-clonic seizures. 6 week history of headaches, nausea, vomiting. Funduscopic exam shows bilateral papilledema. An MRI of the brain shows enlargement of right ventricle consistent with Cysticercosis. Which is mostly likely mechanism of hyderocephalus?

Obstruction of right interventricular foramen (foramen of Monro)

Image: 22 y/o woman - recently emigrated from rural Mexico - 20 minutes after generalized tonic-clonic seizures. 6 week history of headaches, nausea, vomiting. Funduscopic exam shows bilateral papilledema. An MRI of the brain shows enlargement of right ventricle consistent with Cysticercosis. Which is mostly likely mechanism of hyderocephalus?

59 y/o – develops fatigue and decreased appetite 3 months after receive mechanical aortic valve replacement. PE shows scleral icterus and elevated JVP. Lab studies show total bilirubin 4 elevated and direct bilirubin 1. Which is the cause of the jaundice in this patient?

Hemolysis
Prosthetic heart valves / aortic stenosis- hemolytic anemia secondary to mechanical destruction of RBCs.

71 y/o man – 6 month history of calf pain and discomfort when he walks. He has HTN, T2DM, weak dorsalis pedis, and ankle brachial index. The physician knows a drug that would improve symptoms by altering platelet function and providing direct arterial vasodilation- Which drug?

Cilostazol
phosphodiesterase inhibitor; leads to increased cAMP which inhibits platelet degranulation/activation while also causing vasodilation

A 3 month-old boy – facial abnormalities and weakness since birth. PE shows coarse facial features and hypotonia. Mucopolysaccharides within reference range. Concentrations of lysosomal enzymes are increased in serum and decreased in cultured skin in fibroblasts. Which of the following is decreased in patient’s lysosomal enzymes?

Decreased Mannose-6-phosphate

A Newborn with lethal chromosome defect. He is unable to suck sufficiently. He has episodes of crying that occur every 2.25 hours and relieved by tube feeding. Which best describes the most appropriate clinical approach?

feed by whatever necessary to maintain comfort

An 18 y/o woman – fever, sore throat, fatigue for 1 week. Temp 100.6. PE shows pharyngeal erythema, generalized tender lymphadenopathy, splenomegaly. Lab show 20% atypical lymphocytes. Which accounts for atypical lymphocytes in patients peripheral blood smear?

Reactive T – Lymphocytes

Image: An 18 y/o woman - fever, sore throat, fatigue for 1 week. Temp 100.6. PE shows pharyngeal erythema, generalized tender lymphadenopathy, splenomegaly. Lab show 20% atypical lymphocytes. Which accounts for atypical lymphocytes in patients peripheral blood smear?

51-year-old man – acute onset of fever and respiratory failure 6 weeks after cadaveric renal transplant. He currently takes cyclosporine, bactrim, prednisone, and TMP_SMX. Temp high – Respirations increased. Diffuse inspiratory and expiratory crackles. Creatinine increased. X-ray of chest shown. Cause?

Cytomegalovirus
Organ transplant patients are at an increased risk of CMV pneumonia.

Image: 51-year-old man - acute onset of fever and respiratory failure 6 weeks after cadaveric renal transplant. He currently takes cyclosporine, bactrim, prednisone, and TMP_SMX. Temp high - Respirations increased. Diffuse inspiratory and expiratory crackles. Creatinine increased. X-ray of chest shown. Cause?

A 2 month old – 6 week history of constipation. She has poor feeding, sleepiness, and yellow skin. Her parents are migrant workers and have not received routine medical care. PE shows lethargy, hypotonia, jaundice, large fontanelles, macroglossia, and umbilical hernia. Most appropriate next step to establish diagnosis?

Thyroid Function Test
CRETINISM

A 39 y/o reports bright red spots on toilet paper after defecating. Rectal examination shows large swollen rectal veins. Which is predisposing cause of these lesions?

constipation
Large swollen rectal veins –> patient has external hemorrhoids.
Swollen and inflamed veins in the rectum and anus that cause discomfort and bleeding.
The most common cause of external hemorrhoids is repeated straining while having a bowel movement.

Image: A 39 y/o reports bright red spots on toilet paper after defecating. Rectal examination shows large swollen rectal veins. Which is predisposing cause of these lesions?

A 45 y/o develops proteinuria and hematuria. She has taken 3g of ibuprofen for headaches daily for the past 2 years. Which of the following is most likely obtained on renal biopsy?

Tubulointerstitial nephritis
Acute interstitial renal inflammation. Pyuria (classically eosinophils) and azotemia occurring after administration of drugs that act as haptens, inducing hypersensitivity (eg, diuretics, NSAIDs, penicillin derivatives, proton pump inhibitors, rifampin, quinolones, sulfonamides).

A 92 y/o – recently admitted to a nursing home because of progressive dementia has large purpuric lesions over the dorsa of both forearms and hands. Family states shew had similar lesions over the years. Lesions occur in absence of trauma. Platelet count is normal. Which is most likely explanation for purpura?

atrophy of dermal collagen

Image: A 92 y/o - recently admitted to a nursing home because of progressive dementia has large purpuric lesions over the dorsa of both forearms and hands. Family states shew had similar lesions over the years. Lesions occur in absence of trauma. Platelet count is normal. Which is most likely explanation for purpura?

10-year-old boy has mass in ventral midline of the neck just inferior to hyoid bone. This structure is developed from which of the following embryonic tissues?

Endoderm of Foramen caecumThe frequency of autosomal recessive disease in a population is 1/1600. A deletion has been identified that accounts for 80% of mutations at this locus. 20% of mutations result in point mutations. Frequency of deletion carriers in the population is closest which of the following?

A 62-year-old woman – progressive bilateral lower abdominal pain and distention. Pelvic exam shows adnexal mass. Malignant cells. Paclitaxel is initiated. Treatment targets which in tumor?

B- Tubulin
hyperstabilizes polymerized microtubules (made up of alpha- and beta- tubulin)

Taxes Stabilize society

The frequency of autosomal recessive disease in a population is 1/1600. A deletion has been identified that accounts for 80% of mutations at this locus. 20% of mutations result in point mutations. Frequency of deletion carriers in the population is closest which of the following?

1/25
=sqrt(1/1600) = 1/40
(1/40)*80% = .02
2 1 .02 = .04 = 1/25

A 46 y/o has dyspnea, orthopnea, midsystolic murmur best heard over the cardiac apex. An ECG shows a left atrial abnormality. Echo shows enlarged left atrium and normal sized ventricles. Which findings?

Mitral Regurgitation

Image: A 46 y/o has dyspnea, orthopnea, midsystolic murmur best heard over the cardiac apex. An ECG shows a left atrial abnormality. Echo shows enlarged left atrium and normal sized ventricles. Which findings?

A 19-year-old woman – refractory pustular acne – unresponsive to several topical and systemic therapies. Treatment with isotretinoin. Which of the following pharmacologic effects is dependent on binding to the RXR receptor?

Teratogenic effects in the embryo
hormone superfamily of nuclear receptors (NRs) that predominately function as transcription factors with roles in development, cell differentiation, metabolism, and cell death

A 35 y/o man comes to physician because of a 2 month history of inflamed thickened silvery scales on scalp, trunk, elbows, and knees. This condition not responsive to coal tar, calcipotriene, and triamcinolone. Which drug is most appropriate?

Methotrexate

A 55-year-old with T2DM, HTN, hyperlipidemia develops myalgias with weakness. Serum CK increased. Urine is positive for myoglobin. Which drugs responsible for symptoms?

Pravastatin
statin-induced myopathy

In patients with breast cancer, metabolically stable agonists of gonadotropin-releasing hormone are effective because they inhibit release of which?

Gondatropin by the pituitary Gland
GnRH agonists like Leuprolide – continuous fashion, they downregulate the GnRH receptor in the pituitary and ultimately decrease FSH and LH.

83 y/o home bed-ridden and confused. Temp 96F. Hypotensive (BP 85/50). Given 1 liter of saline hypotension. Pulmonary catheter inserted (Cardiac Output high, Pulmonary Cap Wedge Pressure Low, Systemic Vascular Resistance low) – cause of hypotension?

Early Septic Shock
distributive shock which is marked by massive vasodilation causing decreased SVR, decreased preload / PCWP, and increased CO

Image: 83 y/o home bed-ridden and confused. Temp 96F. Hypotensive (BP 85/50). Given 1 liter of saline hypotension. Pulmonary catheter inserted (Cardiac Output high, Pulmonary Cap Wedge Pressure Low, Systemic Vascular Resistance low) - cause of hypotension?

23-year old – 1-month history of weakness and muscle pain after vigorous exercise. (Family history similar). A muscle biopsy specimen shows ragged red fibers. Mutation in a gene located in which structure on photomicrograph?

Area labeled B
Mitochondrial myopathies show “ragged red fibers” on muscle biopsy due to the accumulation of diseased mitochondria in the subsarcolemma of the muscle fiber

Image: 23-year old - 1-month history of weakness and muscle pain after vigorous exercise. (Family history similar). A muscle biopsy specimen shows ragged red fibers. Mutation in a gene located in which structure on photomicrograph?

A 32 y/o woman – receives prescription for tetracycline and is instructed to not to take tetracycline with milk. If tetracycline is taken with milk, its absorption is decreased due to which?

Formation of complexes with mineral ions

A 23 year old F – Sjogren syndrome 6 years – she has burning pain in her toes for past month. Which neurotransmitters is most likely mediating patients pain?

Substance P
undecapeptide present in the CNS and the peripheral nervous system.
involved in the synaptic transmission of pain and other nerve impulses.

22 y/o fever, chills, muscle aches. 6 hours ago – injection with Penicillin G for syphilis. Temp is 102. PE shows no abnormalities. Which explains patient’s symptoms?

release of bacterial products
Administration of Penicillin for Syphilis may lead to the Jarisch-Herxheimer reaction

Herxheimer reaction is that treatment results in the sudden death and destruction of large numbers of treponemes, with the liberation of protein products and toxins

Image: 22 y/o fever, chills, muscle aches. 6 hours ago - injection with Penicillin G for syphilis. Temp is 102. PE shows no abnormalities. Which explains patient's symptoms?

A 48 year old – 2 hours of dizziness and palpitations with sustained, rapid, irregular heartbeat. ECG shows AFIB. If Amiodarone administered, the patient is at greatest risk of which adverse effects?

Hypothyroidism
amiodarone – associated with a number of side effects, including thyroid dysfunction (both hypo- and hyperthyroidism

Removal of Thymus at birth results in severely impaired immune responses. However, if the thymus is removed from adults, little if any deficit in immune responsiveness occurs because of which of the following reasons?

Thymic lymphocytes produce before thymectomy are long lived
Thymus produces all of your T cells by the time you reach puberty.
By age 75, the thymus is little more than fatty tissue.
thymus produces all of your T cells by the time you reach puberty.
Thymocytes are long-lived and that’s why you can lose your thymus without impairment of your immune system.

A 4-week infant – vomited after feeding for past 2 days. Recent episode was very forceful regurgitation through the nostrils. Physical exam shows mild abdominal distention with visible peristalsis. Explanation of these findings?

Hypertrophic Pyloric Stenosis
olive-shaped mass in epigastric region,

A 45-year old woman – asymmetric enlargement of the thyroid gland during PE 6 weeks ago. She underwent adrenalectomy for pheochromocytoma 3 years ago. Thyroid lesions are composed of spindle cells arranged in small clusters. Deposits of amyloid are present between neoplastic cells. Foci of C-Cell Hyperplasia are also present. The lesions are confirmed as malignant. Which of the following marked is most appropriate for the development of the recurrence of the thyroid neoplasm?

Calcitonin
Patient has medullary carcinoma. Malignant proliferation of parafollicular “C” cells that produce calcitonin and have sheets of cells in an amyloid stroma.
Patient likely has MEN 2A or 2B with the presence of medullary thyroid cancer and pheochromocytoma

62 year old man – pain in left hip for 3 weeks. An x-ray of the hip shows a 4.5cm, destructive, osteoblastic lesion with associsted fracture in the proximal femur and two separate smaller lesions in the bony pelvis. A needle core biopsy of the femur shows metastic carcinoma. Which is the most likley primary site of the cancer?

Prostate
Osteoblastic lesions are associated with prostatic adenocarcinoma mets.
Increased serum ALP
OB = osteoblastic
OC = osteoclastic/osteolytic
P=prostate, B=breast, K=kidney, T=thyroid, L=lung
P–B–K–T–L

P = OB
B = OB/OC
K = OC
T = OC
L = OB/OC

During a clinical study, investigator tests a new drug for treatment of breast cancer. A population of 10,000 patients with breast cancer is recruited and randomized into a treatment group and control group. Analysis shows a p-value of 0.1 with no significant difference in the treatment outcomes between the experimental and control. When repeating, which is most likely to decrease making a beta error?

Increase sample size
You are aiming to increase power and you can do so by increasing sample size (reduce B error). “power in numbers

Image: During a clinical study, investigator tests a new drug for treatment of breast cancer. A population of 10,000 patients with breast cancer is recruited and randomized into a treatment group and control group. Analysis shows a p-value of 0.1 with no significant difference in the treatment outcomes between the experimental and control. When repeating, which is most likely to decrease making a beta error?

23-year-old man – 3-year history of schizophrenia. He has been taking an antipsychotic medication for hallucinations, paranoia, and disorganized thoughts. Pt does not make eye contact. Lab show serum sodium of 120 mEq/l, urine sodium of 8, and urine osm of 80. Which is most likely cause of lab finding?

Psychogenic Polydipsia
hyponatremia + a low ADH = psychogenic polydipsia

A 12 year old – girl – 3 months ago, she sustained a complex fracture of the left tibia, which required cast immobilization. The cast is removed. Examination of the left calf shows that it is smaller than the right calf. Which of processes in the patient’s myocytes is most likely cause of findings?

polyubiquitnation
atrophy is decrease in tissue mass due to decrease in size (increased cytoskeleton degradation via ubiquitin-proteasome pathway and autophagy

A 45-year-old man – tooth abscess – He has not seen a physician or a dentist in over 10 years. He lives alone, unemployed. He has no friends, maintains minimal contact with relatives. No thought disorder. Which is most likely DX?

Schizoid personality disorder

60 y/o prolonged apnea – after general anesthesia. Plasma cholinesterase is abnormal. Which agent is most likely responsible for prolonged apnea?

succinylcholine
Strong AcH Receptor Agonist
cause respiratory depression
duration of action of Succinylcholine is determined by its metabolism by plasma. cholinesterase.
abnormal plasma cholinesterase (=pseudocholinesterase), it will lead to delayed metabolism
complications = hypercalcemia, hyperkalemia, and malignant hypertension

Two days after admission to hospital for drug induced thrombocytopenia, a 37 year old woman develops severe headache and becomes comatose. She has few petechia and low platelets. A CT scan of the head shows intracranial hemorrhage. A platelet transfusion is recommended, but the patient is a Jehovah’s witness and stated she did not want a transfusion of any blood products. Husband wants her to have a transfusion. Most appropriate management?

Do not proceed with transfusion
Autonomy = always, always follow the patient’s wishes (in this case you are aware that the patient did not want a transfusion

13 y/o girl has episode of severe cellular rejection necessitating maximal immunosuppression 6 months after a cadaveric renal transplantation. Two weeks later, she develops lymphadenopathy and hepatosplenomegaly. Exam of lymph nodes shows monomorphous population of B lymphocytes. Which finding is most likely on evaluation of these cells?

Epstein Barr Virus Genome

A new drug has the following effects on the activity of the primary arachidonic acid metabolites:

neutrophils, Leukocyte chemotaxis – inhibition
Platetlets – no effect
Mast Cell – no effect
Endothelium – no effect

Which of the following is the most likely site of drug action (image shown)?

Leukotriene B4
Leukotriene B4 is a potent chemotactic molecule for neutrophils. Selective loss of this would have no effect on platelets, mast cells, or endothelium
C5a, IL-8, LTB4

Image: A new drug has the following effects on the activity of the primary arachidonic acid metabolites:

neutrophils, Leukocyte chemotaxis - inhibition
Platetlets - no effect
Mast Cell - no effect
Endothelium - no effect

Which of the following is the most likely site of drug action (image shown)?

4 y/o has a a fever, abdominal cramping, and repeated bloody mucoid stools with tenesmus- Several other children in the daycare have a similar illness. Direct microscopic exam of methylene blue-stained fecal smear shows numerous neutrophils. Which is causal organism?

Shigella Sonnei

Image: 4 y/o has a a fever, abdominal cramping, and repeated bloody mucoid stools with tenesmus- Several other children in the daycare have a similar illness. Direct microscopic exam of methylene blue-stained fecal smear shows numerous neutrophils. Which is causal organism?

A 3y/o boy is brought to physician because of 1 month history of pale skin. PE shows pallor. Lab studies show: Hemoglobin and hematocrit low. A photomicrograph of a peripheral blood smear is shown. Genetic testing is most likely to show which of the following?

Heterozygoues mutation of Ankyrin Gene
Hereditary spherocytosis – defect in proteins interacting with RBC membrane skeleton and plasma membrane (ankyrin, band 3, protein 4.2, spectrin).
Mostly autosomal dominant inheritance (so heterozygous mutation since you only need one mutant allele to get the disease).

Image: A 3y/o boy is brought to physician because of 1 month history of pale skin. PE shows pallor. Lab studies show: Hemoglobin and hematocrit low. A photomicrograph of a peripheral blood smear is shown. Genetic testing is most likely to show which of the following?

A 50-year-old woman – brought to ED by her husband – found her in semicomatose state with an empty bottle next to her. The bottle had contained a sodium salt drug with CNS effects. Alkalization of her urine will cause greatest absolute increase in elimination of the unmetabolized drug if it has which of the following pharmacokinetic profiles?

A 50-year-old woman – brought to ED by her husband – found her in semicomatose state with an empty bottle next to her. The bottle had contained a sodium salt drug with CNS effects. Alkalization of her urine will cause greatest absolute increase in elimination of the unmetabolized drug if it has which of the following pharmacokinetic profiles?

pKA 6.0
protein binding in plasma = 20%
Volume Distribution = 1 L/kg

drug to be cleared by the kidney, it must first be filtered in the glomeruli.
Drugs with a high VD have more of the drug in tissue – not available to filtered by kidney
Drugs with high protein binding won’t be filtered either.
We want a drug with low Vd and low binding if you want it cleared via the kidneys/urine
pKa is pH at which any drug is at its 50% ionized stat
if pKa of drug is 6—at pH 7 we will start eliminating

.18 y/o women screening for colon cancer – Her father and grandfather died of colon cancer at ages of 36 and 45 years, respectively. PE shows no abnormalities. Colonoscopy shows approx. 200 lesions throughout the colon. Colectomy is done (Photograph) Which is the chance that the offspring will develop adenocarcinoma?

50% inherited
Familial adenomatous polyposis is an autosomal dominant mutation.
Thousands of polyps arise starting after puberty; pancolonic; always involves rectum. Prophylactic colectomy or else 100% progress to CRC.
Autosomal dominant diseases have, on average, 50% chance of being passed down to offspring.

Image: .18 y/o women screening for colon cancer - Her father and grandfather died of colon cancer at ages of 36 and 45 years, respectively. PE shows no abnormalities. Colonoscopy shows approx. 200 lesions throughout the colon. Colectomy is done (Photograph) Which is the chance that the offspring will develop adenocarcinoma?

23y/o excessive bleeding while brushing teeth – PE shows multiple ecchymoses. Intensive chemotherapy is planned. Before this treatment is initiated to decrease this patients risk for kidney failure, it is most appropriate to administer a drug that inhibits or binds to which of the following sites?

Xanthine Oxidase
Allopurinol inhibits xanthine oxidase. It is used for chronic gout as well as prevention of tumor-lysis associated urate nephropathy

A 17-year-old comes – swollen tender ankle. X-ray strongly suggestive of osteosarcoma. She and her parents are told only that “further evaluation is needed.” Next Steps?

Speak to both girl and patient and family

A 17-year-old comes – swollen tender ankle. X-ray strongly suggestive of osteosarcoma. She and her parents are told only that “further evaluation is needed.” Next Steps?

Speak to patient and her parents about findings
Sex Drugs or Psych – speak to patient alone

A 25 year old woman – mass in right axilla. 1 week history of malaise, headaches, and night sweats. Pt adopted a kitten and has sustained several bite and scratch marks. Tenderness in right axiallry lymph node. The skin over the node is erythematous, tough, and warm. Bartonella Henselae antibody test is positive. A biopsy of this lymph node is most likely to show which histological pattern?

Granulomas containing stellate micro abscesses + Axillary mass
catch scratch
competent immune system – stellate necrotizing granulomas.

Image: A 25 year old woman - mass in right axilla. 1 week history of malaise, headaches, and night sweats. Pt adopted a kitten and has sustained several bite and scratch marks. Tenderness in right axiallry lymph node. The skin over the node is erythematous, tough, and warm. Bartonella Henselae antibody test is positive. A biopsy of this lymph node is most likely to show which histological pattern?

A 35-year-old man – comes to physician to discuss donating kidney to his sister who has T1DM. Which describes likelihood that this donors HLA type will match his sister?

1:4
You have a 25% chance of inheriting the same HLA markers as your siblings.
Two siblings have a 25% chance of being genotypically HLA identical,
a 50% chance of being HLA haploidentical (sharing one haplotype), and a 25% chance that they share no HLA haplotypes.

A 50-year-old woman – restrictive pulmonary disorder with no obstructive component is most likely to have which of the following sets of pulmonary volumes?

FRC – Decreased
Residual Volumes – Decreased
Vital Capacity – Decreased

restrictive lung disease- all lung values are DECREASED (“restricted”) except for the FEV1/FVC ratio.
FEV1/FVC ratio may be normal or increased

29 y/o – 1 week history of fever, muscle aches, sore throat, and non-productive cough. Rhonchi
Heard on auscultation of chest. CXR shows bronchopneumonia of right lower lobe. Symptoms persist after 7 days of amoxicillin therapy. Cold Agglutinins positive. Which of the following features of the causal organism best explains the ineffectiveness of the pharmacotherapy?

Absence of peptidoglycan
Mycobacterium – does not CELL WALL

A 16 year old – 2 month history of wheezing and shortness of breath while participating in track meets. Diagnosed with exercised-induced asthma. Exposure to which is most likely to precipitate a similar episode in this patient in the future?

Cold, Dry Air
exercise induced asthma.
when exercising, breathing through the nose is decreased and replaced by more vigorous mouth breathing. This means the air is not humidified and warmed up.
The air is less dense and the asthma ensues.

A 43 y/o female -initial prenatal visit. PE shows uterus consistent in size with a 10-week gestation. This patient is at increased risk for a child with Down Syndrome if she has which of the following sets of ultrasound and serum findings?

Fetal Ultrasound Translucency – Increased
Pregnancy Associated Plasma Protein- Decreased
Human Chorionic Gonadotropin – Increased

A male newborn – decreased blood thyroxine on newborn screening. PE shows no abnormalities. Serum T3 and T4 decreased – Serum free T4 and TSH within reference ranges. Which best explains findings?

Thyroid Hormone Binding Globulin deficiency
Decreased total, normal free (unbound) = Thyroid hormone-binding globulin deficiency
Thyroxine-binding globulin (TBG) deficiency is characterized by low serum total T4 but normal free T4 and TSH;

the diagnosis is confirmed by measuring TBG concentrations

Image: A male newborn - decreased blood thyroxine on newborn screening. PE shows no abnormalities. Serum T3 and T4 decreased - Serum free T4 and TSH within reference ranges. Which best explains findings?

A study conducted to assess the effect of a new screening test on the mortality of prostate cancer. Prior to implementation, the overall 5-year survival was 70%. After screening of random population, 5-year survival rate increases to 85%. No changes in treatment. The decrease in mortality could be due to which type of bias?

Lead Time
caused by early detection being confused with increased survival. early detection makes it seem as though survival has increased, but the natural history of the disease has not been impacted.

50 y/o come to physician because of a 6-week history of depression, anxiety, difficulty sleeping, decreased appetite, and poor memory and concentration. He has an MI 3 months ago. He has one glass of wine each evening. Which of the following most likely decrease this patient’s risk of mortality over the next 2 years?

Antidepressant Therapy
6-week history (e.g. >2 weeks) of depression
(1), difficulty sleeping (2), fatigue (3), decreased appetite (4), and poor memory/concentration (5)

Image: 50 y/o come to physician because of a 6-week history of depression, anxiety, difficulty sleeping, decreased appetite, and poor memory and concentration. He has an MI 3 months ago. He has one glass of wine each evening. Which of the following most likely decrease this patient's risk of mortality over the next 2 years?

At a postnatal checkup – 6-week female newborn weighs 20 ounces less than delivery. Vomits and regurgitates bile-tinged fluid after every feeding. Which of the following is the most likely cause of abnormalities?

Annular Pancreas
bile in the vomit;
abnormal rotation of ventral pancreatic bud forms a ring of pancreatic tissue encircles 2nd part of duodenum; may cause duodenal narrowing (arrows in) and vomiting

Image: At a postnatal checkup - 6-week female newborn weighs 20 ounces less than delivery. Vomits and regurgitates bile-tinged fluid after every feeding. Which of the following is the most likely cause of abnormalities?

An investigator compares the DNA sequence of a newborns with achondroplasia and healthy newborns. DNA sequence shows a G-C mutation in the FGF3 gene on chromone 4.

Achnodroplasia 5’TACCGGGT-3′
Healthy 5’TACGGGGT-3′

As a result of this mutation an MspL restriction enzyme site is created that allows for a convenient diagnostic test. Which of the following sequences most likely represents the substrate specificity for Mspl?

5’CCGG
restriction enzymes bind palindromes.
both 5’CCGG or 3’GGCC would have been acceptable in this scenario.

18 y/o F – never had a menstrual period. PE shows a 1 -cm invagination where vagina would normally be present. No uterus palpable. Lab show FSH, LH, estradiol, and testosterone in normal range. Which of the following structures is most likely normal?

Ovaries
mullerian agenesis. Normal ovaries but absent uterus.
Underdevelopment of the Mullerian system leading to congential absence of the vagina. Usually no cervix or uterus.

Image: 18 y/o F - never had a menstrual period. PE shows a 1 -cm invagination where vagina would normally be present. No uterus palpable. Lab show FSH, LH, estradiol, and testosterone in normal range. Which of the following structures is most likely normal?

25 y/o man – severe Pain in low back radiates down his left leg- he started a weight-lifting – tried to lift a 200lb bar form ground over his head. Pain began immediately after this attempt. Cause of this patient’s pain?

Rupture of an intervertebral disc
radiculopathy – disc herniation (radiates to the leg aka lasegue sign),
degenerative osteoarthritis (positional relieved with rest),
spinal stenossi(pain with standing),
spondylorpathy(relieved with exercise,dominant at rest),
spinal metastasis (constant pain, worse at night, not relieved by positional changes
vertebral osteomyelitis(focal tenderness, acute back pain, and fever), aortic dissection (severe restrosternal pain, radiating to the back)

Image: 25 y/o man - severe Pain in low back radiates down his left leg- he started a weight-lifting - tried to lift a 200lb bar form ground over his head. Pain began immediately after this attempt. Cause of this patient's pain?

A 28-year-old man – excessive thirst and polyuria. Labs

Baseline Serum Glucose 75, Serum Osm 310, Urine Osm 175
Water deprivation – no change in serum glucose, serum Osm, or urine OsM
With ADH added – Urine OsM icnrease

Which is the primary site of the pathophysiolohic process in this patient?

Hypothalmus
Central DI= responds to vasopressin, high serum Na
Nephrogenic = responds to nothing, normal serum Na

80 y/o – T2DM – 2 month history of severe constipation. Laxatives provide no relief. Distention of abdomen. Patient most likely has dysfunction of the following nerves?

pelvic splanchnic
Parasympathetic- no rest and digest

80 y/o – T2DM – 2 month history of severe constipation. Laxatives provide no relief. Distention of abdomen. Patient most likely has dysfunction of the following nerves?

pelvic splanchnic
a. Constipated – Parasympathetic- no rest and digest

25 y/o – F – 2 day history of fever, malaise, and a rash. Single painless lesion on labia which resolves spontaneously. Sexually active and does not use condoms. PE shows diffuse rash on palms and soles. A sensitive but non-specific rapid serologic test uses which reagents is most likely to support diagnosis?

A phospholipid (Cardiolipin) antibodies on charcoal particles
Single painless lesion on labia
Visualized by immunofluorescence or dark-field microscopy; serology is important – two types of antibodies:
Cardiolipin sensitive but not specific for treponoma

A 44-year-old woman – pap smear for atypical squamous cells. Protein is known to promote cell growth and malignancy by causing p53 protein degradation. This degradation begins when the p53 protein is targeted by which of the following types of cellular enzymes?

Ubiquitin Ligase
starts degradation cycle
p53 targeted degradation cycle

A 28 year old woman – Eastern European Jewish Descent – lump in her left breast. Maternal grandmother was diagnosed with ovarian cancer, her maternal aunt was diagnosed with breast cancer at age 36. Pt concerned about genetic mutation. Genetic testing most likely to show a mutation in a component of which of the following pathways?

Recombinational Double Stranded DNA breaks
Defective homologous recombination is seen in breast/ovarian cancers with the BRCA1 gene mutation.

Graph (PICTURE) shows radiolabeled protein X that is bound by anti-X antibodies in the presence of varying concentrations of protein Y. With respect to anti-X. which of the following interpretations regarding the epitopes expressed by proteins X and Y is correct?

Proteins X and Y express the same epitopes If they share the same epitopes, it will have a downward slope.

If they share none of the same epitopes, the line will be horizontal across the graph

Image: Graph (PICTURE) shows radiolabeled protein X that is bound by anti-X antibodies in the presence of varying concentrations of protein Y. With respect to anti-X. which of the following interpretations regarding the epitopes expressed by proteins X and Y is correct?

A previously healthy 32 year old – 2 hours of severe epigastric pain, nausea, and vomiting blood. Pulse is 125/min, respirations are 18/min, and blood pressure is 85/45 mm Hg. PE shows cool skin and tender abdomen. A decrease in which hemodynamic parameters is most likely to cause patients hypotension?

Preload
Vomiting blood and cool skin indicates this is a type of hypovolemic shock

Decreased EDV means that the “filling volume” is decreased, which also means the preload will be decreased

Which of the following changes in the cardiovascular system occurs with normal Aging?

Increased Basal Systolic blood pressure
vascular scleorsis and stiffening- changes with aging- leads to high sys BP

A 55-year-old – constipation since starting an OTC medication 2 weeks ago for chronic, persistent cough, and sinus congestion. Which ingredients in this medication cause symptoms?

Dextromethorphan
antitussive – (antagonizes NMDA receptor)
mild opioid effects when used in excess
Mild abuse potential

19 y/o college student -sudden onset of right-sided chest pain and difficulty breathing after an accident in which he was thrown from his bicycle. He has difficulty walking and cannot climb stairs because of pain and shortness of breath. He is slightly cyanotic, afebrile, and tachypneic. Which of the following is most suggestive that fractured ribs caused the respiratory problem?

subcutaneous crepitus
pneumothorax – inspiratory stridor
When there is a fractured rib it will cause a trauma pneumothorax which can cause air to escape and become trapped under the skin leading to crepitus.

Subcutaneous crepitus is very specific sound referencing air finding its way into the skin which you can hear but also feel by rubbing your hand over the affected area

A physician prescribes newly marketed drug to 45 patients. Over the next several weeks, she notes good efficacy, but elevated enzymes in 5 patients. Physician unable to find any official data linking the new drug to liver dysfunction. Which is most appropriate action of physician?

Discontinue the new drug in people who have effects and report the cases

14 y/o girl – 1 month history of migraine-like headaches, vomiting, and multiple left sides seizures. She has hearing loss. Her mother and maternal grandmother have high tone deafness. Pt most likely has a mutation of which of the following?

Mitochondrial tRNALeu
It’s a mitochondrial disease. – present in successive generations, always with maternal transmission – damage to high-energy tissues – CSF lactic acidosis

An 18-y/o boy – diagnosed with Schizophrenia – Identical Twin brother is unaffected. Parents are concerned that twin will have the disorder. Which represents brother’s risk for developing schizophrenia?

50%
psychosis of about 50% in monozygotic twins seems to be a realistic estimate, which is significantly higher than that in dizygotic twins of about 10-19%

32 y/o fecal incontinence after pregnancy – what was damaged?

Damage to anal sphincter
A stretch injury during childbirth will result in damage to the external ureteral and anal sphincters and damage to the pudendal nerve (S2-S4). This can lead to decreased sensation in the perineal and genital area and fecal or urinary incontinence

78 y/o – 3 week history of severe weakness and fatigue. 16lb weight Loss – Daughter usually bring groceries twice per week. Daughter on vacation. He is only eating crackers and black coffee. PE shows generalized muscle weakness. Serum studies most likely to show an increased concentration of which hormones?

Cortisol
b. cortisol’s functions is to increase gluconeogenesis, lipolysis, and proteolysis.
c. helps maintain blood pressure even in the setting where he is malnourished.

66 y/o old man – comes to ED after MVC. Pronounced dead on arrival. Photomicrograph of a section of distal left ureter taken at autopsy are shown. Which of the following was the most likely predisposing factor in the development of the ureteral lesions in this patient?

Cigarette Smoking

transitional cell carcinoma, which smoking is a common risk factor for; it can involve the renal pelvis/calyces.

histo image shows the papillary nature of the tumor

Image: 66 y/o old man - comes to ED after MVC. Pronounced dead on arrival. Photomicrograph of a section of distal left ureter taken at autopsy are shown. Which of the following was the most likely predisposing factor in the development of the ureteral lesions in this patient?

A 36 y/o woman – 1-month history of joint pain, frequent headaches, and fatigue. PE shows edema of the upper and lower extremities. An abdominal CT scan is shown. Which findings most likely seen in this patient?

Splenomegaly

Image: A 36 y/o woman - 1-month history of joint pain, frequent headaches, and fatigue. PE shows edema of the upper and lower extremities. An abdominal CT scan is shown. Which findings most likely seen in this patient?

56 y/o -frequently burned herself while cooking over the past months. Exam shows loss of pain and temperature sensation in both upper extremities. Touch, vibration, and proprioception are normal. Some wasting of her hands. Which of the following findings on MRI of the head and spine?

Syrinx of the central region of the spine C4 – T5

Image: 56 y/o -frequently burned herself while cooking over the past months. Exam shows loss of pain and temperature sensation in both upper extremities. Touch, vibration, and proprioception are normal. Some wasting of her hands. Which of the following findings on MRI of the head and spine?

70 y/o man – 3-month history of weakness and a 22lb weight loss. 2 pack of cigarettes daily for 50 years. An x-ray shows a 4-cm lesion in right upper lobe. Serum calcium concentration is 13.5 mg/dl. A malignant lesion arising in a segmental bronchus is seen during lobectomy. DX?

squamous cell carcinoma
ARISE FROM BRONCHUS , Cavitation, Cigarattes, HyperCalcemia (produce PTHrp)

23 y/o woman – fever, hypotension, and DIC for 24 hours. She splenectomy following trauma. Which bacteria is cause of sepsis?

Streptococcus pneumonia
Encapsulated organisms run rampant in patients who have no spleen, whether physically or functionally. (Recall the wide-array of sequalae sickle cell patients experience thanks to their functional autosplenectomy.)

Image: 23 y/o woman - fever, hypotension, and DIC for 24 hours. She splenectomy following trauma. Which bacteria is cause of sepsis?

Female Newborn – Respiratory Distress, Cyanosis, Nasogastric tube placed – what embryologic events cause issue?

incomplete formation of Pleuroperitoneal membrane
Diaphragmatic hernia

Image: Female Newborn - Respiratory Distress, Cyanosis, Nasogastric tube placed - what embryologic events cause issue?

A 70-year-old – woman – transferred to rehab service 2 days after operative repair of a fracture of femur. She weighs 6Kg (143 lb). PE exam in unremarkable. Most important predictor of success in rehab

Activity level before the fracture
Activity level increase in Bone density(Specially Weigh bearing) and OsteoBlastic activity

27 y/o woman – 3-month history of intermittent headaches, palpitations. Blood pressure 125/80 in a chair. When asked to sit on the exam table, she has flushing, and her blood pressure is 185/115. PE shows no abnormalities. Most likely diagnosis?

Pheochromocytoma
When standing up, the body normally activates sympathetic system to avoid orthostatic hypotension.

But since there is now an additive effect of the pheochromocytoma adrenergics, it will lead to a hypertension

Pt with 1-week history of diarrhea has reduced tissue turgor. Arterial pH 7.3. Serum bicard is 15 mEq/L. Pt is most likely to have which of the following?

Decreased arterial PC02
chronic diarrhea leading to normal anion gap metabolic acidosis
pH = HCO3/CO2

12 y/o girl – dentist found many unerupted and supernumerary teeth. PE shows frontal bossing, hytpertelorism, and retained deciduous teeth. Decreased ALK phos. A CXR shows hypoplasia of clavicles. Skull X-rays show open sutures and Wormian bones. Mutation of CBFA1 gene. Which cell types is affected by the mutation?

Osteoblast
ALP is decreased. Osteoblast activity is measured by bone ALP

A 57-year-old man – hemoglobin concentration of 18.5 g/dl. A peripheral blood smear shown. Findings most consistent with which disorder?

Chronic Obstructive Pulmonary Disease
Absolute polycythemia vera, which is due to high altitude or lung disease.

Image: A 57-year-old man - hemoglobin concentration of 18.5 g/dl. A peripheral blood smear shown. Findings most consistent with which disorder?

38 y/o African American – recently diagnosed with hypertension – patient aggreed to initiate a regime including physical activity, low sodium diet, and pharmacotherapy. PE shows no abnormalities. What is the role of ethnicity when selecting appropriate medication?

It should play a role because the efficacy of certain classes of medications varies among different ethnicities

A new antiplatelet agent for prevention in stroke. In a large randomized trial, the rate of stroke are lower in patients receiving the new agent than in patients receiving standard treatment.

Results Stroke New Platelet Drug
Women =. .12 .04

Based on results which of the following is the absolute risk reduction in women?

8%
Absolute risk: the difference in risk (not the proportion) attributable to the intervention as compared to a control.
(.12) – (.04) = .08
ARR = 8%

65 y/o man – substernal chest pain, nausea, sweating for 1 hour. 10-year history of T2DM. Pulse is 120/min. BP 98/60. PE shows a JVD of 12cm. Crackles are heard halfway up lungs fields. ECG findings?

ST elevation
Continued or severe ischemia (>20 minutes) leads to transmural necrosis involving most of the myocardial wall (transmural infarction); EKG shows ST-segment elevation.
ST will first decrease; however, after 20 min it will increase (elevate)

30 y/o comes to physician – 3-month history of pain in his jaw and left arm. Intense thirst and urination. Found to have peptic ulcer for 2 month. Serum calcium 13.5. PTH 110 (N= 10-65). X-ray normal. Which explains impaired calcium homeostasis?

Increased Osteoclast and Maturation and activity hypercalcemia (stones, bones, groans, THRONES [increased diuresis}, and psychiatric overtones).

Image: 30 y/o comes to physician - 3-month history of pain in his jaw and left arm. Intense thirst and urination. Found to have peptic ulcer for 2 month. Serum calcium 13.5. PTH 110 (N= 10-65). X-ray normal. Which explains impaired calcium homeostasis?

Following a wedding reception – 25 adults develop low grade fever, abdominal cramping, vomiting, and diarrhea. No blood in stool, No pathogens on stool culture. Several family members exhibit similar symptoms 3 days after initial outbreak. Causal organisms?

Norovirus

Legionella Pneumophilia

gram negative rods

use silver stain.
Grow on charcoal
yeast extract medium with iron and
cysteine.


Aerosol transmission

Noperson-to-person transmission

31 y/o man – large, yellow, soft mass deeply infiltrating the gluteus maximus muscle. Microscopic exam shows irregular vacuolated cells and clear cells with frequent mitoses. DX?

Liposarcoma
malignant, increased mitotic activity
most common soft-tissue tumors in adult
high mitotic index & infiltrative nature indicate that the mass is malignant

Image: 31 y/o man - large, yellow, soft mass deeply infiltrating the gluteus maximus muscle. Microscopic exam shows irregular vacuolated cells and clear cells with frequent mitoses. DX?

An otherwise healthy 4-month-old girl is brought to physician because of lesion shown (strawberry hemangioma). The lesion has been present for 2 months and growing rapidly. Which is expected over the next 5 years?

spontaneously involution
strawberry hemangiomas tend to grow and then randomly involute.
appears early in life, grows rapidly and regresses spontaneously by 5-8 years old

Image: An otherwise healthy 4-month-old girl is brought to physician because of lesion shown (strawberry hemangioma). The lesion has been present for 2 months and growing rapidly. Which is expected over the next 5 years?

A 2-year old boy – increased thirst, urinary frequency, and failure to thrive. He is 5th % for height and 7% for weight. PE shows dehydration and decreased muscle tone. The diagnosis of Fanconi syndrome is made. Which sets of changes in fractional reabsorption in the kidneys is most likely in this patient?

All Decreased

Amino Acid Decreased
Glucose Decreased
Phosphate Decreased
HCO3 Decreased

Image: A 2-year old boy - increased thirst, urinary frequency, and failure to thrive. He is 5th % for height and 7% for weight. PE shows dehydration and decreased muscle tone. The diagnosis of Fanconi syndrome is made. Which sets of changes in fractional reabsorption in the kidneys is most likely in this patient?

A 28-year-old – man – fatigue and back pain for 6 weeks. 13-lb weight loss. DX of TB of lumbar spine. When placed in supine position, patient holds right lower extremity in rigid extension. Spread of the infection to the lumbar vertebrae is most likely causing the patient to guard which muscle in supine position?

Psoas Major
flexion of hip
“rigid extension”, hence he does not want to flex

Image: A 28-year-old - man - fatigue and back pain for 6 weeks. 13-lb weight loss. DX of TB of lumbar spine. When placed in supine position, patient holds right lower extremity in rigid extension. Spread of the infection to the lumbar vertebrae is most likely causing the patient to guard which muscle in supine position?

A 53-year-old woman – 1 year history of abdominal pain. Travelers diarrhea and UTI. Avid traveler to Middle East, South America, and Asia. Photomicrograph of stool sample. Which infectious agent?

Schistosoma Mansoni
lateral spine

Image: A 53-year-old woman - 1 year history of abdominal pain. Travelers diarrhea and UTI. Avid traveler to Middle East, South America, and Asia. Photomicrograph of stool sample. Which infectious agent?

55 y/o women follow-up examination – She has a family history of skin cancer and avoids sun. Takes Vit D. PE no abnormalities. Serum studies show decreased 25 -hydroxycholecalciferol concentration. Decreased production of which precursors in skin is most likely?

Cholecalciferol
D3 (cholecalciferol) from exposure of skin (stratum basale) to sun, ingestion of fish, milk, plants.
D2 (ergocalciferol) from ingestion of plants, fungi, yeasts

Image: 55 y/o women follow-up examination - She has a family history of skin cancer and avoids sun. Takes Vit D. PE no abnormalities. Serum studies show decreased 25 -hydroxycholecalciferol concentration. Decreased production of which precursors in skin is most likely?

A 45-year-old man – fever, chills, dysuria, and a tender, enlarged prostate. Which is most likely causal organism?

Escherichia Coli
Prostatitis is characterized by dysuria, frequency, urgency, low back pain. Warm, tender, enlarged prostate.
Acute bacterial prostatitis—in older men most common bacterium is E. coli.

33y/o Falling on outstretched right hand – PE shows swelling on palmar side of wrist. No pain in anatomical snuff box. Most likely cause of findings dislocation of which of the following bones?

Lunate
Dislocation of lunate may cause acute carpal tunnel syndrome

Image: 33y/o Falling on outstretched right hand - PE shows swelling on palmar side of wrist. No pain in anatomical snuff box. Most likely cause of findings dislocation of which of the following bones?

A mother who has toxoplasmosis gives birth to an asymptomatic female newborn. Which of the following tests on newborn will indicate that the newborn has congenital toxoplasma gondii infection?

Anti-toxoplasma IgM antibody concentrations

baby does not get any maternal IgM, IgA or IgE as they do not cross the placenta, so if IgM is found it may suggest the baby has encountered an infection in utero.

IgG is passed down to the baby as a means of passive immunity until the baby can form their own antibodies of different types. So, if you see anything other than IgG (e.g. IgM) you know it must be d/t an infection.

50 y/o man comes to the physician because of persistent cough for past 2 months. He is farmer and started itraconazole therapy for histoplasmosis acquired through chicken coops. Current medications include hydrochlorothiazide, enalapril, atenolol, omeprazole, and metoclopramide for HTN. An interaction between itraconazole and which following drugs most likely accounts for the lack of effect of itraconazole?

Omeprazole
CYP 450 inhibitor (SICKFACES.COM)

S odium valproate
I soniazid
C imetidine
K etoconazole
F luconazole
A cute alcohol abuse
C hloramphenicol
E rythromycin/clarithromycin
S ulfonamides
C iprofloxacin
O meprazole
M etronidazole

A miodarone
Grapefruit juice

A 75-year-old woman – burning, itching, and rash. Exam shows clustered skin lesions overlying two ribs near the right axillary line. Some of the lesions are fluid-filled blisters and have crusty scabs. Light microscope shows which findings?

Multinucleated giant cells and neutrophil infiltrates
Shingles. Herpes simplex and herpes zoster viruses cause abnormal cell division in epidermal cells, and this creates multinucleated giant cells.

A Tzank smear showing multinucleated giant cells is characteristic of Varicella Zoster Virus infections

Image: A 75-year-old woman - burning, itching, and rash. Exam shows clustered skin lesions overlying two ribs near the right axillary line. Some of the lesions are fluid-filled blisters and have crusty scabs. Light microscope shows which findings?

A 5-year old girl with AIDS develops vesicular rash. Antiviral therapy is ineffective and the lesions become hyperkeratotic. The genetic mutation responsible for the ineffectiveness involves the activity of which enzymes?

Thymidine-Kinase

60 y/op man – 2 pillow orthopnea – severe dyspnea, edema of lower extremities. Onset was preceded by an episode of prolonged substernal chest pain for 5 days. Which of the following is most consistent with these findings?

Jugular Venous Pressure 12 mm Hg
patient has heart failure.
Normal JVP is 6-8 mmHg.

A 10-year-old girl – not yet menstruated. PE shows absence of breast bud development and no pubic or axillary hair. The most asks what is the first objective sign of puberty?

Breast Bud Development first

Females: Thelarche -> Pubarche -> growth spurt -> Menarche

Males: Testicular growth -> Penile Growth -> Pubarche -> Growth Spurt -> Spermatogenesis

Monoclonality of Neoplastic Cells in endometrial carcinoma can be best determined by analysis of which of the following?

X chromosome-linked isoenzymes
Neoplasia is new tissue growth that is unregulated, irreversible, and monoclonal.
Clonality can be determined by glucose-6-phosphate dehydrogenase (G6PD) enzyme isoforms. G6PD is X-linked.

56 y/o man – worsening fatigue and lower extremity edema. poorly controlled HTN – BP 195/110. BUN is 70mg/dl, CR 7.0. Which is most likely pathologic finding in kidneys?

fibromuscular hyperplasia of arterioles

Image: 56 y/o man - worsening fatigue and lower extremity edema. poorly controlled HTN - BP 195/110. BUN is 70mg/dl, CR 7.0. Which is most likely pathologic finding in kidneys?

56 y/o woman – brought to ER – progressive SOBS. 2 weeks ago she underwent coronary artery bypass. Dullness to percussion. CXR shows fluid in left pleural cavity. Thoracentesis yield 1200 ml of pale, milky chyle. Which produced iatrogenic chylothorax?

Placement of the central line via the left jugular vein
nicking the thoracic duct, specifically at its inlet, the left subclavian.
Placement of the central line via left internal jugular vein can cause damage to thoracic duct
Placement of pulmonary artery can cause damage to right lymphatic duct.

Image: 56 y/o woman - brought to ER - progressive SOBS. 2 weeks ago she underwent coronary artery bypass. Dullness to percussion. CXR shows fluid in left pleural cavity. Thoracentesis yield 1200 ml of pale, milky chyle. Which produced iatrogenic chylothorax?

A 30 year old won with multiple sclerosis- 2 week history of shooting pains in cheek- lasts for less than 1 second. Cause of pain is plaque in which location?

pons
trigeminal neuralgia
MIDBRAIN – 1,2,3,4
PONS- 5,6,7,8
Medulla 9,10,11,12

Image: A 30 year old won with multiple sclerosis- 2 week history of shooting pains in cheek- lasts for less than 1 second. Cause of pain is plaque in which location?

Serum cholesterol study for women by age. Assuming gaussian distribution, what is probability that a woman between 50 and 54 has a serum cholesterol greater than 296?

Age Cholesterol
50-54 246+50

16%
68% of the data will fall within 1SD of the mean.
100%-68% = 34
16% will fall above and 16% will fall below 1 SD.
2STD = top 5%

Image: Serum cholesterol study for women by age. Assuming gaussian distribution, what is probability that a woman between 50 and 54 has a serum cholesterol greater than 296?

Age Cholesterol
50-54 246+50

36 y/o – not urinated for 24 hours. Intermittent renal calculi – Ultrasound shows bilateral hydronephrosis. Creatinine increased, K+ increased – Bilateral nephrostomy tubes are placed. Which predicts likely changes in urine and potassium excretion during next 24 hours?

Increased Urine Output
Increased Urine Potassium
urine output increased: pretty simply they are putting tubes in to increase the urine flow into the bag.
If you increase urine flow rate you increase K+ secretion. That’s why diuretics like thiazides and loops increase K+ secretion

A 26-year-old man – stab wound in LUQ – laceration of fundus of the stomach. During mobilization, he develops bleeding due to damage to short gastric arteries. Short gastric arteries are a branch of which arteries?

splenic
Short gastric a. branch from the splenic a.
Branches of the celiac trunk that constitute the blood supply to the stomach: common hepatic, splenic, and left gastric.

Image: A 26-year-old man - stab wound in LUQ - laceration of fundus of the stomach. During mobilization, he develops bleeding due to damage to short gastric arteries. Short gastric arteries are a branch of which arteries?

17-year-old girl – 4-day history of headache and vomiting. Right sided hemiparesis. Serum pyruvate and lactate increased. Deficiency in NADH dehydrogenase. Pedigree shown. Which is mode of inheritance of the disorder?

Mitochondrial
mitochondrial – passed by the mother
Mothers have diseased children; Fathers don’t

Image: 17-year-old girl - 4-day history of headache and vomiting. Right sided hemiparesis. Serum pyruvate and lactate increased. Deficiency in NADH dehydrogenase. Pedigree shown. Which is mode of inheritance of the disorder?

A 48 y/o – 6 month history of angina – Cholesterol 450, LDL increased. Treatment with statin is begun. Statins decrease serum cholesterol by which mechanisms?

Upregulate LDL receptors
Statins decrease cholesterol synthesis, which indirectly Statins indirectly cause increased LDL receptor expression on hepatocytes

A randomized clinical trial – compare difference between 2 surgical procedures. Procedure A has a lower rate wound infection Procedure B (relative risk of 0.66 with confidence interval of .30-1.45. Which statement represents the comparison between Procedure A and B in a clinical setting?

Neither procedure is superior
The CI value contained 1, which means that its insignificant
CI including 1 fails to reject the null hypothesis*
Odds ratio/relative risk, CI including 1 fails to reject the null hypothesis

Image: A randomized clinical trial - compare difference between 2 surgical procedures. Procedure A has a lower rate wound infection Procedure B (relative risk of 0.66 with confidence interval of .30-1.45. Which statement represents the comparison between Procedure A and B in a clinical setting?

During an experiment, an investigator isolates abnormal elastin from connective tissue of mice. Analysis of elastin shows a decreased number of desmosine cross links. Which amino acids in the abnormal protein would be most likely different compared with normal elastin?

Lysine

Lysine is used in elastin and collagen cross linking; it is cross linked by lysyl oxidase to make collagen fibers

Desmosine is an amino acid found uniquely in elastin, a protein found in connective tissue such as skin, lungs, and elastic arteries.

Desmosine is made up of four lysine residues. Therefore abnormal elastin is likely missing lysine necessary for the formation of these desmosine cross-links.

A 50-year-old man – 3 hour history of confusion and difficulty seeing. He sees white spots that interfere with vision. 3-year history of alcohol disorder. He has been drinking methanol. He is confused and incoherent. PE shows dilated pupils and diminished pupillary reflex. Cause of visual symptoms due to an excess of which of the following metabolites?

Formic Acid
Methanol is converted to formaldehyde via alcohol dehydrogenase (ADH) and formaldehyde is converted to formic acid (formate) via aldehyde dehydrogenase (ALDH).

Image: A 50-year-old man - 3 hour history of confusion and difficulty seeing. He sees white spots that interfere with vision. 3-year history of alcohol disorder. He has been drinking methanol. He is confused and incoherent. PE shows dilated pupils and diminished pupillary reflex. Cause of visual symptoms due to an excess of which of the following metabolites?

A 60-year-old woman – T2DM – substernal chest pain radiates to her left arm. She is diagnosed with MI. 2 days later she develops acute respiratory distress. PE shows elevated JVP. Crackles are heard over the lower half of her lungs. A grade 4/6 apical systolic murmur is heard. Mechanism of worsening pain?

Rupture of a papillary muscles
most important MI complications that occur within a 2-5 day span are papillary muscle rupture and interventricular septum rupture.

Papillary muscle rupture leads to severe mitral regurgitation, heard as a systolic murmur at the apex.

Image: A 60-year-old woman - T2DM - substernal chest pain radiates to her left arm. She is diagnosed with MI. 2 days later she develops acute respiratory distress. PE shows elevated JVP. Crackles are heard over the lower half of her lungs. A grade 4/6 apical systolic murmur is heard. Mechanism of worsening pain?

56 y/o – automobile crush injuries of both legs. 36 hours later he develops oliguria, hyperkalemia, and increased BUN. Which is most likely cause of findings?

Rhabdomyolysis
Rhabdomyolysis can present looking like a kidney injury
The electrolyte findings are just like renal failure (Inc. K+, inc. PO4-, dec. Ca)
To differentiate between rhabdomyolysis and kidney injury, you check the urine to see if there are any RBCs. In rhabdomyolysis there are no free RBCs in the urine

8-year-old boy – evaluated for ventricular systolic overload. ECG shows left axis deviation of the main QRS vector (upward in lead 1 and downward in lead II). Which is most likely explanation for patients condition?

Coarctation of the aorta
leads to increased LV overload causing LV hypertrophy and a L axis deviation.

Image: 8-year-old boy - evaluated for ventricular systolic overload. ECG shows left axis deviation of the main QRS vector (upward in lead 1 and downward in lead II). Which is most likely explanation for patients condition?

30-year male – completed Hodgkin Lymphoma 2 months ago-calls physician on an increasingly frequent basis. He takes temp several times per day. After evaluation he shows no evidence of recurrence, he is increasingly fearful that the disease many not be detected. DX?

Adjustment Disorder with anxiety
Emotional symptoms (eg, anxiety, depression) that occur within 3 months of an identifiable psychosocial stressor (eg, divorce, illness) lasting < 6 months once the stressor has ended.

A 3-month old boy – 1 month history of lump on right side of his groin. Bulge increases when patient crises. PE shows soft mass in the right groin that decreases easily. Which best describes the relationship to the hernia sac?

Lateral to the inferior epigastric and superior to inguinal ligament
b. Inguinal hernias are usually reducible
c. indirect inguinal hernia. It enters internal inguinal ring lateral to inferior epigastric vessels and is superior to the inguinal ligament.
d. failure of processus vaginalis to close (can form hydrocele).

Image: A 3-month old boy - 1 month history of lump on right side of his groin. Bulge increases when patient crises. PE shows soft mass in the right groin that decreases easily. Which best describes the relationship to the hernia sac?

A 72 y/o woman – dysphagia- surgical evaluation of a mass in posterior mediastinum. During procedure, the thoracic duct is inadvertently damaged near the mass. The injury is most likely to impair normal lymphatic return to which structures?

Right Kidney
The left upper extremity and breast are drained by the axillary lymph node.
The kidney is drained by the thoracic duct. The heart has its own lymph system going on surrounding the heart

Image: A 72 y/o woman - dysphagia- surgical evaluation of a mass in posterior mediastinum. During procedure, the thoracic duct is inadvertently damaged near the mass. The injury is most likely to impair normal lymphatic return to which structures?

The sequence surrounding the first 2 exons of the human B globin shown (exons in bold capital letters). The translation start codon is underlined. A mutation of G-A at position 355 is likely to lead to B-thalassemia by which mechanism?

Disruption of normal splicing creation of a new 3′ splice site
B-thalassemia is due to point mutations in splice sites and promoter sequences

mutation occurs within an intron (a gene segment which is transcribed [DNA->RNA] but not translated). RNA splicing enzyme(s) grab RNA and “loop it”; an intron is cut out and the exons on either side of the intron are adjoined, like this:

Image: The sequence surrounding the first 2 exons of the human B globin shown (exons in bold capital letters). The translation start codon is underlined. A mutation of G-A at position 355 is likely to lead to B-thalassemia by which mechanism?

A 77-year-old woman – difficulty fastening buttons. Weakness in intrinsic muscles of hands and loss of sensation in the little fingers. Causes of findings?

C7-T1 Foraminal Stenosis
Little finger = ulnar nerve
C8-T1 are the roots of the ulnar nerve, which is a branch of the medial cord. The ulnar nerve is not found in the carpal tunnel

A 9 year old – girl – poor growth during past year – 3rd percentile for height and 10th percentile for weight. PE shows no abnormalities. Visual field testing shows bitemporal hemianopia. Lab show a growth hormone deficiency. An MRI of the brain shows a calcified cystic mass in the suprasellar region. The tumor is most likely derived from?

Diverticulum of the roof of embryonic oral cavity
craniopharyngioma.

Most common childhood supratentorial tumor. Derived from remnants of Rathke pouch (oral ectoderm). Calcification is common. Cholesterol crystals found in “motor oil”-like fluid within tumor.

A cystic suprasellar mass with calcifications and enhancement of the wall or solid portions in a child or adolescent is almost always a craniopharyngioma.

Image: A 9 year old - girl - poor growth during past year - 3rd percentile for height and 10th percentile for weight. PE shows no abnormalities. Visual field testing shows bitemporal hemianopia. Lab show a growth hormone deficiency. An MRI of the brain shows a calcified cystic mass in the suprasellar region. The tumor is most likely derived from?

A 50-year-old man – brought in by wife because of personality change and cognitive decline. He used to be kind – now yelling all the time. He almost invested all family money in a startup / scam. He has inappropriate cursing. A CT scan shows atrophy of the frontal lobes bilaterally. DX?

Pick Disease
Fronto-temporal dementia characterized by personality change is usually Picks

A 1 -month old – 5 days of vomiting after feeding – vomiting is forceful – PE shows decreased skin turgor. Which set of serum findings – most likely in this newborn?

Na 132
K 3.2
Cl- 90
HCO3 37
hypochloremia, hypokalemia, hyponatremia, and metabolic alkalosis
chronic vomiting, you lose electrolytes and a lot of acid.
It triggers metabolic alkalosis which is why all the serum values are low (or on the lower end of the normal range) except for bicarbonate.

Image: A 1 -month old - 5 days of vomiting after feeding - vomiting is forceful - PE shows decreased skin turgor. Which set of serum findings - most likely in this newborn?

66-year-old fatigue, SOBs, temp. X-ray of chest shows cavitary fluid level in right lower lobe of lungs. Culture of sputum gram stain grows Klebsiella pneumonia. Symptoms resolve with antibiotic treatment. Which is found in right lower lobe 6 months later?

Focal pulmonary fibrosis
Visible air-fluid level = large lesion

Six months following resolution of symptoms you can expect healing in the form of a scar; that is, fibrosis but only in a single spot.’

Fibrosis seen in chronic diseases such as pulmonary fibrosis is often responsible for organ dysfunction and even organ failure

16 y/o – 4 day history of severe vaginal itching discharge and a yellow discharge. She uses condoms inconsistently. Saline wet mount shows motile organisms. Causal organisms?

Trichomonas vaginalis
thin, yellow-green, malodorous, frothy discharge and vaginal inflammation / itching.
Lab findings: pH >4.5 and motile trichomonas

Image: 16 y/o - 4 day history of severe vaginal itching discharge and a yellow discharge. She uses condoms inconsistently. Saline wet mount shows motile organisms. Causal organisms?

50 y/o discharge from both breast- History of HTN and T2DM – She takes hydrochlorothiazide. PE shows a thin, milky discharge from both breasts. Which is responsible for this patient’s condition?

Drug Effect
three major causes of galactorrhea as nipple stimulation, prolactinoma of anterior pituitary, and drugs (see 16.1 – Breast Pathology). Only drug effect is an answer choice for this question.

A 6-week-old girl – 6-day history of vomiting a small amount of milk 2 to 3 times daily. She is average weight and height. PE shows no abnormalities. Which is most likely cause of this patient’s findings?

Immature Lower Esophageal Sphincter
vomiting a small amount
No abnormalities, only some vomiting, looks well w/ no failure to thrive. Most likely immature LES.
esophageal spasm tends to present with pain and dysphagia.
common disease that impacts babies

Image: A 6-week-old girl - 6-day history of vomiting a small amount of milk 2 to 3 times daily. She is average weight and height. PE shows no abnormalities. Which is most likely cause of this patient's findings?

60 y/o – 6-week history of cough productive blood tinged sputum. He has a 6.6 lb weight gain. Smoker 30 years. BP 165/101. A CT scan shows pulmonary lesion. Serum Sodium 120. Which additional lab results most likely in this patient?

Urine NA: Increased
Serum Osm: decreased

Urine osmolality > serum osmolality
SIADH
Excessive free water retention
Euvolemic hyponatremia with continued urinary Na+ excretion

A 38-year-old woman – sore throat. During exam, physician say “ah” – causes elevation of area of tip of the arrow in photograph shown. Which nerve tested?

Vagus
uvular deviation would be testing for CN X palsy

CN X carries efferent to palate. CN IX carries afferent from palate.

Glossopharyngeal nerve function: – Taste and sensation from posterior 1/3 of tongue – Swallowing – Salivation (parotid gland) – Monitoring carotid body and sinus chemo- and baroreceptors – Elevation of pharynx/larynx (stylopharyngeus)

Image: A 38-year-old woman - sore throat. During exam, physician say "ah" - causes elevation of area of tip of the arrow in photograph shown. Which nerve tested?

African American – hyperpigmented papules, cheeks, jawline, neck – cause?

Pseudofolliculitis Barbae

Image: African American - hyperpigmented papules, cheeks, jawline, neck - cause?

34 years – HIV positive – severe abdominal pain for 12 hours. He has been receiving antiretroviral therapy for past 2 weeks. Prior to starting meds, CD4+ 35/min. A CT scan of the abdomen shows enlargement of lymph nodes. Improved function of which cells is required for the following?

CD4+ T Lymphocytes
CD4+ cells activate B-cells which form follicles and cause enlargement of lymph nodes. Therefore, in an AIDS patient, to enlarge the lymph nodes, the CD4+ dysfunction must be resolved.

Image: 34 years - HIV positive - severe abdominal pain for 12 hours. He has been receiving antiretroviral therapy for past 2 weeks. Prior to starting meds, CD4+ 35/min. A CT scan of the abdomen shows enlargement of lymph nodes. Improved function of which cells is required for the following?

25 y/o – 42 weeks’ gestation – admitted for failure to begin spontaneous labor. Infusion of oxytocin is initiated. Activation of which signaling pathway mediates therapeutic effect for this patient?

Phosphoinositide hydrolysis
Oxytocin uses IP3 signaling pathway.
GnRH, Oxytocin, ADH (V1-receptor), TRH, Histamine (H1-receptor), Angiotensin II, Gastrin.
FA mnemonic: “GOAT HAG”

A 35 year old woman – HTN – BP 153/106 mm Hg. A bruit is heard lateral to umbilicus on the right side. Which diagnostic test is most appropriate to evaluate HTN?

MR angiography of the renal arteries

Image: A 35 year old woman - HTN - BP 153/106 mm Hg. A bruit is heard lateral to umbilicus on the right side. Which diagnostic test is most appropriate to evaluate HTN?

A 37-year-old nurse stuck with a needle used to obtain blood for HIV positive. Receives antiretroviral therapy. 4 weeks later develop anemia and neutropenia. Which is the MOA of antiretroviral responsible for anemia and neutropenia?

reverse transcriptase
NRTI’s are associated with possible side effects of anemia, granulocytopenia, and myelosuppression.

A 70 y/o man dies of coronary artery disease – He had cerebral infarction 8 years ago. A photo of brain stem is shown. Which neuro deficits most likely present after his cerebral infarction?

Spastic Hemiparesis on the right
Left crus cerebri was damaged (see what it should normally look like below). This contains the corticospinal tract. Since the corticospinal tract decusates at the medulla, below the midbrain section we’re looking at, you would see Contralateral (Right) Spastic Hemiparesis

Image: A 70 y/o man dies of coronary artery disease - He had cerebral infarction 8 years ago. A photo of brain stem is shown. Which neuro deficits most likely present after his cerebral infarction?

A 16-year-old student – uncontrollable sleepiness, falling asleep several times a day while sitting in class. The slightest noise or touch is enough to awaken him. Which characterizes the onset of his sleep at night?term-163

Direct transition from wakefulness to REM sleep
“Narcolepsy can be conceptualized as a disorder of sleep-wake control in which elements of sleep

direct transition from wakefulness to REM sleep. Basically instead of going through the early stages and gradually falling into a deep sleep, you just suddenly go from being awake to being in a deep sleep.

A study is conducted to identify healthy women at risk of cardiovascular disease. Because atherosclerosis is inflammatory, the concentration of c-reactive protein is increased. Which is likely site of CRP biosynthesis?

Liver
CRP is an acute phase reactant, these are synthesized primarily by the liver
location (Acute Phase Reactant)

Investigator looking at effectiveness in TX (substance X) for Sjogren Syndrome. Substance X produces 2X saliva as placebo. Placebo group contains half as much sodium chloride as that from Substance X. Bicarb is the same in both groups. Which explains these findings?

Ductal ion reabsorption more efficient at low flow rates
Ductal cells reabsorption is hindered at high flow rates

At low flow = High concentration of potassium; low concentrations of sodium, bicarb, & chloride
at high flow = low concentration of potassium; high concentrations of sodium, bicarb, & chloride

20 y/o brought to office by mother. Pt is hearing voices and bizarre behavior for 6 months. He used to be a good student, and now he is failing his classes. Pt appears unkept, distracted. Mother thinks he is scaring the family because he is hearing voices. Which initial response is most appropriate?

How frightening- Do you have an idea what might be causing the problem?

A 53-year-old man – 4-hour history of severer headache, anxiety, sweating, palpitations. He has a 2 year history of HTN. PE shows diaphoresis. Which is the effects of propranolol in this patient?

Increase TPR and Decrease CO
non-selective Beta blocker
CO – HR decrease-
compensatory increase in TPR

Pilot crashed plane – 2 weeks ago in Alaska wilderness. Pilot has minor injuries – lost considerable weight. Pilot found good water supply but couldn’t find food. The pilot most likely has which condition?

Negative Nitrogen balance

Negative nitrogen balance is associated with burns, serious tissue injuries, fevers, hyperthyroidism, wasting diseases, and during periods of fasting

Positive nitrogen balance is associated with periods of growth, hypothyroidism, tissue repair, and pregnancy. This means that the intake of nitrogen into the body is greater than the loss of nitrogen from the body, so there is an increase in the total body pool of protein.

A 45-year-old man – intermittent bloody diarrhea and abdominal pain. Rectal biopsy shows inflammatory bowel disease. A PT with IBS (diarrhea and abdominal pain) Treated with monoclonal antibody is begun. The antibody is most likely directed against which component?

Tumor Necrosis Factor
infliximab and adalimumab-
Stop inflammatory events

37 y/o – HIV positive- needs Pain relief numbness in legs, pain, tingling in his feet. He is receiving antiretroviral therapy. On PE soles of the feet are tender to palpation. Sensation to pinprick is decreased over his calves and feet. Which drug is most appropriate for long term relief of his pain?

Nortriptyline
Inhibit 5-HT and NE reuptake
Peripheral neuropathy, chronic neuropathic pain, migraine prophylaxis

Image: 37 y/o - HIV positive- needs Pain relief numbness in legs, pain, tingling in his feet. He is receiving antiretroviral therapy. On PE soles of the feet are tender to palpation. Sensation to pinprick is decreased over his calves and feet. Which drug is most appropriate for long term relief of his pain?

A 35-year-old man – 6 month history of burning abdominal pain occurs 1 to 2 hours after he eats. He has black stools for 2 days. Antacids and H-2 blockers are not effective. He is sweating profusely and has lightheadedness when he stands. PE shows epigastric tenderness. A CT scan of the abdomen shows a 1-cm mass in the pancreas. Immunohistochemical labeling – most likely to involve the use of antibodies directed to which substance?

Gastrin
Zollinger Ellison Tumor – GASTRIN SECRETING

47 y/o female- 6-month history of severe cough and shortness of breath. She has no fever, chills, or muscle aches. Nonsmoker works as a librarian. She lives alone with 4 parakeets. She is a knitter and can spin wool. A CXR – shows reticulogranular changes. Examination shows interstitial infiltrates of lymphoocytes and granulomas. Cause?

Contact with Parakeets
Chlamydophila psittaci
Hypersensitivity pneumonitis—mixed type III/IV hypersensitivity

Image: 47 y/o female- 6-month history of severe cough and shortness of breath. She has no fever, chills, or muscle aches. Nonsmoker works as a librarian. She lives alone with 4 parakeets. She is a knitter and can spin wool. A CXR - shows reticulogranular changes. Examination shows interstitial infiltrates of lymphoocytes and granulomas. Cause?

56 y/o – long history of chronic back pain undergoes operative placement of electrode in his midbrain for pain management. When implant is activated, pain decreases. Administration of which substances would most likely result in the return of the patient’s pain?

Naloxone
opioid antagonist – block mu receptors would lead to increase in mediators that induce pain.

A 55-year-old man – severe emphysema comes to physician for evaluation of SOB- Findings on lung exam?

Percussion Hyperressonant
Tactile Fremitus Decreased
Expiratory Phase Prolonged

37 y/o healthy woman – 6 month history of fatigue – She has 18-kg weight gain during this period. Round face, truncal obesity, purple striae. Plasma ACTH low < 5 (normal < 120). Serum cortisol does not become suppressed at low or high dexamethasone doses. Which is is most likely cause of findings?

Tumor of adrenal Zona Fasciculate
i. SALT, SUGAR, SEX
ii. SUGAR = Cortisol

A previously healthy 16-year-old – 2 days history of thick, yellow vaginal discharge and burning with urination. She does not want her parents to know that she is sexually active. Gram stain shows gram-negative diplococci. Testing for Neisseria and Chlamydia is ordered. Which is the most appropriate course of action?

Prescribe antibiotics without notifying parents

Image: A previously healthy 16-year-old - 2 days history of thick, yellow vaginal discharge and burning with urination. She does not want her parents to know that she is sexually active. Gram stain shows gram-negative diplococci. Testing for Neisseria and Chlamydia is ordered. Which is the most appropriate course of action?

A 27 y/o man – Weakness in Left Hand for 3 weeks. Professional bicyclist. He rides bicycle 8-10 hours per day. PE shows mild weakness with abduction and adduction of fingers. He is unable to hold a piece of paper between two fingers. Sensation is intact. Cause of findings is compression of which structures?

Deep branch of ulnar nerve at the hook of hamate
Guyon Canal – associated with Ulnar Nerve – intrinsic hand muscles
handlebars
Thumb Opposition – MEDIAN NERVE

A 27 y/o man – Weakness in Left Hand for 3 weeks. Professional bicyclist. He rides bicycle 8-10 hours per day. PE shows mild weakness with abduction and adduction of fingers. He is unable to hold a piece of paper between two fingers. Sensation is intact. Cause of findings is compression of which structures?

Deep branch of ulnar nerve at the hook of hamate
intrinsic hand muscles
Guyon Canal – associated with Ulnar Nerve – handlebars
Thumb Opposition – MEDIAN NERVE

12. A 35-year-old man – given cyclosporine following a liver transplant. Which describes the utility of this drug in this situation?

Decreased synthesis of interleukins
calcineurin inhibitor, which aims to decrease IL-2

23 y/o – ASD repair 3 months ago – PE shows increase in size of scar. An abnormality is most likely cause of these findings?

Collagen Synthesis
keloid
type 3 collagen is first synthesized and then degraded by Matrix metalloproteinase

Image: 23 y/o - ASD repair 3 months ago - PE shows increase in size of scar. An abnormality is most likely cause of these findings?

A 60 year old man – semitransparent nodule on his nose. Exam of tissues shows small basophilic cells forming tumor islands by clear spaces and extending across the basement membrane. DX?

Basal Cell Carcinoma
peripheral palisading, pink pearly, waxy, pink

Image: A 60 year old man - semitransparent nodule on his nose. Exam of tissues shows small basophilic cells forming tumor islands by clear spaces and extending across the basement membrane. DX?

12. 55 y/o previously healthy man diagnosed with HTN. He has not received therapy. Lab studies show mild hypokalemia, metabolic Alkalosis, increased plasma renin and aldosterone. Following administration of ACE-inhibitor, plasma renin increases further. Most likely cause of HTN?

Renal Artery Stenosis
ACE-I likely Renal Artery Stenosis
Vasoconstriction of efferent

Image: 12. 55 y/o previously healthy man diagnosed with HTN. He has not received therapy. Lab studies show mild hypokalemia, metabolic Alkalosis, increased plasma renin and aldosterone. Following administration of ACE-inhibitor, plasma renin increases further. Most likely cause of HTN?

65-year-old dies 6 months after onset of headaches – Brain autopsy shown in the photograph. Which is likely cell origin of this neoplasm??

Meningeal
Meningioma- parasagittal region –

Image: 65-year-old dies 6 months after onset of headaches - Brain autopsy shown in the photograph. Which is likely cell origin of this neoplasm??

23-year-old with significant blood loss from MVC – She is given 3 units of blood and has an uneventful recovery. 4 weeks later she becomes jaundiced. Lab show anemia and reticulocytosis. Which is most useful in establishing the diagnosis?

Direct Antiglobulin Test
Hemolysis due to transfused blood
Delayed onset of hemolytic reaction

15 y/o – 1-day history of a rash on her face, arms, and legs. She noticed rash on sunny day at the beach. She is receiving ABX treatment for ACNE. PE shows diffuse erythema and mild edema over the face and extremities. Which ABX cause findings?

Doxycycline
Bind 30s subunit
BUY AT – 30S— CELLL @ 50S

5-year-old boy – DX with genetic defect – inability to reabsorb in the proximal tubule. Serum studies are likely to show which of the following findings at this time?

Hypophosphatemia
b. 85% of Phosphate absorbed in PCT
c. 25% Na reabsorbed in TAL

Image: 5-year-old boy - DX with genetic defect - inability to reabsorb in the proximal tubule. Serum studies are likely to show which of the following findings at this time?

78 y/o dies of CHF- Exam shows passive congestion of lungs, hypoxic changes, hydropic changes im the renal tubular epithelial cells. Which of the following cause hydropic change?

Failure of Na+/K+ pump
ATP STOPS WORKING
Hydropic Changes – one of the early signs of cellular degeneration in response to injury that results in accumulation of water in the cell.
Hypoxia decreased in aerobic respiration
Cellular injury causes swelling

25-year-old – – collapses in ICU where she works as a nurse. Coworkers report she felt fine all morning. Shortly after lunch she became pale and diaphoretic and then collapsed. She has a history of episodes of hypoglycemia without apparent precipitants. She is unresponsive and low blood 2- concentration, given bolus of 50% dextrose. Several episodes of confusion. Glucose is less than 20 – bolus of 50% dextrose is administered. Lab studies show.
Glucose 32
C-peptide <0.075
Proinsulin < 1 pmol/L

Which of the following is most likely diagnosis?

Factitious Disorder
EXOGENOUS INSULIN
Low C-peptide, low proinsulin, high plasma free insulin and low glucose, in a nurse
Endogenous Insulin – increased C-Peptide

An Investigator is studying patients with West Nile Virus infection. During a 5-year period, data are collected on 25 patients. Demographic information on the patients is reported (age, sex, ethnicity) as well as information on the source of infection. What is the study design?

Case Series
group or series of case reports involving patients who were given similar treatment
NO case controls

Double blind study – many adult participants non adherent- In accordance with intention to treat, how should data pertaining to non-adhering be treated?

Analyze all non-adherent participants as part of the group they were randomized too
Intention to treat – none of patients are excluded
Per Protocol – only those who complete the entire protocol are counted

Image: Double blind study - many adult participants non adherent- In accordance with intention to treat, how should data pertaining to non-adhering be treated?

45-year-old woman – intubated and mechanically ventilated develops fungemia with Candida albicans during a prolonged hospital stay. Treated with Caspsofungin. What features of the causal organism will most likely be targeted by this drug?

B-glucan Carbohydrates in Cell Walls
Echinocandins – cell wall synthesis

24 y/o man – 8-hour history of abdominal pain, nausea, and vomiting. Family history of pancreatitis and hyperlipidemia. Triglycerides 3500 mg/dl and lipase of 400 U/L (N<280). Serum appears milky. Most likely cause is a deficiency of which of the following?

Lipoprotein lipase deficiency
Pt dyslipidemia + increased chylomicrons

A 58-year-old man – severe Chest Pain Radiating to back for 30 minutes. X-rays shows do not show a widening Aorta. Aortic angiography shows an extra lumen in the media of the proximal aorta. What is most likely cause of finding?

Hypertension
cause aortic dissection
False lumen p. 301

62 y/o mild cough, purulent sputum for 2 weeks. She smoked 2 packs for 42 years. She drinks 3 to 6 beers. PE shows, swelling of the in fingers, arthritis of joints, and clubbing of the digits. What is the most likely underlying disease?

Bronchogenic carcinoma

Immediately after a cerebral infarction, a 62 y/o right handed man has decreased fluency but his comprehension is normal. Pt most likely has lesion in which of the following areas?

Area Labeled A – BROCAS areas
b. fluency decreased, comprehension intact
c. Wernicke area – poor comprehension, fluent speech

Image: Immediately after a cerebral infarction, a 62 y/o right handed man has decreased fluency but his comprehension is normal. Pt most likely has lesion in which of the following areas?

55 y/o – 2 month history of decreased appetite. 20lb weight loss and an intermittent rash. PE shows necrolytic migratory erythema over the axilla and groin. Glucose 280 and Glucagon 1500 (N- 20-100). A CT scan shows mass at head of pancreas. Which is most likely occurring in liver?

Partial Oxidation of Fatty acids
Glucagon upregulates – CAT1 and CAT2 are important enzymes of fatty acid beta oxidation.

Glucagonoma: migratory erythema

Dermatitis (necrolytic migratory erythema), Diabetes (hyperglycemia), DVT, Declining weight, Depression

28 y/o – 32 weeks gestation, 4 day history of fever and back pain. PE shows costophrenic tenderness, photomicrograph shown represents her disease. What mechanism is likely cause?

Obstructive nephropathy
pregnancy dilation of the upper urinary tract can occur due to compression of the ureters by the growing fetus/uterus and the linea terminalis
postrenal azotemia

Image: 28 y/o - 32 weeks gestation, 4 day history of fever and back pain. PE shows costophrenic tenderness, photomicrograph shown represents her disease. What mechanism is likely cause?

65 y/o woman – 25 year history of chronic alcoholism – vomiting blood—She eats very little, Exam of her legs show -perifollicular hyperkeratosis and hemorrhage. Abnormally decreased function of which metabolic pathways is most likely in this patient?

Hydroxylation of proline
Vit. C deficiency- curvy features swollen gums, easy bruising, petechiae, perifollicular and subperiosteal hemorrhages.

16 y/o girl with cystic fibrosus – 3-week history of numbness, tingling of her arms and legs, and difficulty walking. She has not adhered to her medication region for 6 months. PE shows bilateral weakness and decreased deep tendon reflexes in the upper and lower extremities. She walks with ataxic gait. Deficiency in which of the following?

Vitamin E
fat soluble (neurologic with ataxia)

A 33-year-old – HIV infection – generalized tonic-clonic seizures. 2 month history of headaches. CD4 count is 22. CT shows 3-cm lesion in right cerebral cortex. Serologic studies – positive for IgG antibody to Toxoplasmosis Gondii. Tx with pyrimethamine and sulfadiazine is initiated. During next 2 weeks, 3 more seizures. CT shows lesion increased to 3.5cm. Cause of mass?

Non-Hodgkin Lymphoma
Single enhancing lesion
Toxoplasmosis – multiple lesions – improved by TMP-xMX

A 45-year-old – man – surgical procedure – Which maneuver by the anesthesiologist would result in increased intracranial pressure?

Decreased respiratory rate
increased CO2 – cause vasodilation

62 y/o – physician follow-up – 2 weeks ago discharged after acute MI. Smoked 2 packs a day. Diet mostly cured meats and fast food. He does not exercise. I know I need to make some changes in how I live. I just don’t have willpower to quit smoking. Which best describes patients stage of behavioral change?

Contemplation –
patient acknowledges problem but unwilling to make change
Precontemplation – patient denies existence of problem

A 30-year-old woman – develop serum sickness following parenteral administration of penicillin. Plasma sample is found to contain a mediator that stimulates histamine release from mast cells. Which mediators most likely found in this patient’s plasma?

C3a
b. mast cells activated by C3a and C5a p. 400

45 year old man – counseling prior to bariatric Surgery – BMI 40- PE shows no abnormalities, In addition to recommending that the patient follow a balanced diet, it is most appropriate to avoid ingesting excessive amounts of which?

Starchy foods
cause dumping syndrome- avoid simple carbs
Have small portions

A 32-year old man – 3-month history of swelling and tenderness of both breasts. He is receiving thyroid hormone and corticosteroid therapy since surgical removal of pituitary adenoma 2 years ago. Began HCG injections for months ago. Which is most likely binding site of hCG causing gynecomastia?

Tissue: Testicle
Effect: estradiol production
B-HCG and LH,FSH,TSH share same alpha subunit, so HCG can activate those receptors if its in high enough quantity.
Activating LH receptor will lead to more Testosterone from the Leydig cells. More testosterone can lead to more estrogen formation via aromatase.

A 48-year-old man – overdose on ferrous sulfate. Na and K in normal range. What happens to bicarb and cl serum concentrations?

HCO3 8
Cl 96
IRON OVERDOSE- high metabolic acidosis
High anion gap metabolic acidosis due to Iron excess (the I in MUDPILES). Serum anion gap = (Na) – [(HCO3 + Cl)]. Chloride levels low;HCO3 low, which results in the high anion gap.

Image: A 48-year-old man - overdose on ferrous sulfate. Na and K in normal range. What happens to bicarb and cl serum concentrations?

A 61-year-old man – erectile dysfunction due to spinal cord injury at L-2. Sildenafil used to correct ED by acting at which of the following labeled structures (Image Shown)?

corpus cavernosum
Sildenafil increases blood flow to the penis by dilating the corpus cavernosum (increased NO via inhibition of PDE5 –> cGMP –> smooth muscle relaxation).

Image: A 61-year-old man - erectile dysfunction due to spinal cord injury at L-2. Sildenafil used to correct ED by acting at which of the following labeled structures (Image Shown)?

A 32-year-old woman — confused and lost consciousness – fell from ladder an struck the right side of her head on the edge of her desk. She felt well enough to continue working. PE shows edema on right anterior temple. Right pupil dilated. Left pupil normal. What is DX?

Arterial laceration
epidural hematoma Meningeal Artery

Pts bumps head, gets back up – severe issues/ dies 6 hours later — = epidural hematoma

middle meningeal artery laceration => epidural hematoma. Epidural hematomas are known to cause transtentorial herniations and CN III palsy
Branch of maxillary artery

Image: A 32-year-old woman -- confused and lost consciousness - fell from ladder an struck the right side of her head on the edge of her desk. She felt well enough to continue working. PE shows edema on right anterior temple. Right pupil dilated. Left pupil normal. What is DX?

A 12-year-old boy – 3 episodes of meningococcal bacteremia during past 4 years. He has had not other significant infections. The most likely cause of the recurrent meningococcal infections is a deficiency in which of the following?

Terminal Component of Complement
MAC Complex
(C5-C9) Deficiencies increase susceptibility to recurrent Neisseria bacteremia.

20 y/o man – states that food had lost its flavor ever since he fell down flight of stairs and hit his head. PE shows no abnormalities. Neuro exam is most likely to show damage to which cranial nerve?

Olfactory

A 33-year-old – 3 month history of pain in right thigh. No history of trauma. PE shows mild swelling but no erythema. MRI shows tumor in anterolateral muscle. Which muscle is infiltrated with the tumor?

Rectus Femoris

Image: A 33-year-old - 3 month history of pain in right thigh. No history of trauma. PE shows mild swelling but no erythema. MRI shows tumor in anterolateral muscle. Which muscle is infiltrated with the tumor?

51-year-old- develops diaphoresis, tachycardia – 24 hours after undergoing abdominal operation -BP 155/100 – 2 hours late he has generalized tonic-clonic seizures. Which is responsible for the adverse effect seen in the patient?

alcohol withdrawal

Image: 51-year-old- develops diaphoresis, tachycardia - 24 hours after undergoing abdominal operation -BP 155/100 - 2 hours late he has generalized tonic-clonic seizures. Which is responsible for the adverse effect seen in the patient?

64-year-old – surgical repair of abdominal aortic aneurysm. During the repair the testicular artery is ligated. Anastomotic supply from which of the following arteries will maintain adequate arterial supply to the left testis in this patient?

Artery of the ductus deferens

Image: 64-year-old - surgical repair of abdominal aortic aneurysm. During the repair the testicular artery is ligated. Anastomotic supply from which of the following arteries will maintain adequate arterial supply to the left testis in this patient?

A 33 year old woman – 34 weeks gestation – grade 2/6 systolic ejection murmur heard at the second intercostal space. The remainder of the PE is normal. Best explanation for this finding is an increase in which?

Stroke Volume
SV = EDV – ES
EDV increases in pregnancy due to an increase in plasma volume
“Blood flow to various organs increases during pregnancy to meet the increased metabolic needs of tissues. Thus, venous return and cardiac output increases dramatically during pregnancy. Cardiac output gradually increases during the first 2 trimesters with the largest increase occurring by 16 weeks of gestation

A clinical study – evaluate the association of caffeine consumption and pancreatic cancer. In the study alpha is set at 0.05 and beta at .10. Which of the following best describes the likelihood of missing associations between caffeine consumption and pancreatic cancer in this study?

10%
Type II beta error
“likelihood of missing an association”
represented by beta. =
Power = 1 – beta.
Type I = alpha error

A 10-year-old boy – adopted from Democratic Republic of Congo. PE shows 0.5-, firm, nontender nodules, two over the right iliac crest, one on the left thigh, and one on the left knee. Exam of the skin from one of the nodules shows microfilariae. Which is the likely vector in this patient?

Black Fly
Nodules show microfilariae
Black Flies, Black Skin Nodules, Black Sight
Onchocerca Volvulus via female blackfly. Black flies, black skin nodules, “black sight” (aka Blindness). Question stem here describes the black nodules seen in Onchocerca.

A new screening test for color cancer is done on 86 patients with colon cancer and 124 who do not:

Results are shown:
Present Absent
Positive: 67 6
Negative: 19 118

Specificity?

95%

Image: A new screening test for color cancer is done on 86 patients with colon cancer and 124 who do not:

Results are shown:
Present Absent
Positive: 67 6
Negative: 19 118

Specificity?

A 56 year old man – 3 year history of progressive memory loss followed by aphasia, visuospatial disorientation, and inappropriate behavior. Father died in nursing home at age 65 from progressive dementia. At autopsy, brain shows cerebral atrophy, neurofibrillary tangles, neuritic plaques, amd marked neuronal loss. Mutation in gene encoding for which of the following molecules?

Presenilin
early inherited cases of Alzheimer’s dementia are associated with mutations in presenilin-1/presenilin-2, as well as Down’s syndrome
associated with familial alzheimers disease

At 2:00am – 32 year old man with T2DM has a decrease in serum glucose concentration to 37 mg/dl. In response to the hypoglycemia, cortisol secreted by the adrenal cortex induces the synthesis of which of the following in the adrenal medulla?

Phenylethanolamine N- Methyltransferase 2:am cortisol secreted by adrenal cortex induces synthesis – catecholamine synthesis pathway from medulla

At 2:00am – 32 year old man with T2DM has a decrease in serum glucose concentration to 37 mg/dl. In response to the hypoglycemia, cortisol secreted by the adrenal cortex induces the synthesis of which of the following in the adrenal medulla?

Phenylethanolamine N- Methyltransferase
b. 2:am cortisol secreted by adrenal cortex induces synthesis – catecholamine synthesis pathway
c. NE is converted to EPI via PNMT, which is induced by cortisol

Image: At 2:00am - 32 year old man with T2DM has a decrease in serum glucose concentration to 37 mg/dl. In response to the hypoglycemia, cortisol secreted by the adrenal cortex induces the synthesis of which of the following in the adrenal medulla?

53-year-old woman – fainting precipitated by exercise and intermittent substernal chest pain – relieved by rest. Her blood pressure is 120/80. Prominent left ventricular impulse. A late peaking harsh, midsystolic murmur is heard at second right intercostal space with radiation to the carotid arteries. Which is best explanation of syncope?

Fixed Cardiac Output in spite of increased demand
Aortic Stenosis – unable to push through valve – Syncope, Angina, Left Heart Failure

12 year old boy- persistent rash around his nose and mouth for 4 months. Decreased appetite and unintentional weight loss. Rash began after he started a new school. Grades are falling. Rash mild redness and flaking. PE shows conjunctival injection, eczematous eruption on the nasolabial folds, friable with gold stippling. Cause of condition?

inhalant abuse
Intellectual decline

Image: 12 year old boy- persistent rash around his nose and mouth for 4 months. Decreased appetite and unintentional weight loss. Rash began after he started a new school. Grades are falling. Rash mild redness and flaking. PE shows conjunctival injection, eczematous eruption on the nasolabial folds, friable with gold stippling. Cause of condition?

A 25-year-old woman – overdose of barbiturates and is found unresponsive several hours later by her husband. PT is comatose and undergoes endotracheal intubation. 3 weeks later she has no purposeful response to stimuli. The patient signed a living will – consistent with its directives that mechanical ventilation be discontinued. Appropriate next step?

extubate the patient and discontinue mechanical ventilation, make no attempt at resuscitation

A 63-year-old woman – repair of leaking berry aneurysm in the circle of Willis. Two days later, a CT scan of the head shows widening of the subarachnoid space. What change in CSF causes the finding?

Decreased movement through arachnoid villi
arachnoid villi drains the CSF from the subarachnoid space to the venous system; if this part becomes defective then you can imagine all that CSF now building up in the subarachnoid space.

acute SAH can lead to decreased absorption and movement via arachnoid villi

Choroid plexus – make
Arachnoid – Absorbs

A 68 y/o – 9 month history of abdominal pain after meals. 24lb weight loss. She avoids eating because of cramping pain that lasts 1 hour after meals. PE shows thin body frame and a soft nontender abdomen. BP 160/90 A CT scan shows a 3-cm ectatic (dialated) aorta. Which pairs of arteries is most likely involved in this patient?

inferior mesenteric and superior mesenteric
Mesenteric cause post prandial pain – AAA
compression of duodenum at 3rd
Chronic Mesenteric Ischemia aka intestinal angina.
abdominal aortic aneurysm. Generally, they tend to occur below the renal branches so I went with the two closest branches- SMA

In a study of antibiotic resistance, strain of Escherichia Coli to ampicillin but sensitive to streptomycin is cultured with a strain of Salmonella Enteritidis sensitive to ampicillin but resistant to streptomycin. After 4 hours of cocultivation, Salmonella Enteritidis are resistant to both antibiotics. Mechanism of ampicillin resistance by the Salmonella Enteritidis?

only cookie are independently associated with E Coli
OR >1 indicates increased occurrence of event.

Image: In a study of antibiotic resistance, strain of Escherichia Coli to ampicillin but sensitive to streptomycin is cultured with a strain of Salmonella Enteritidis sensitive to ampicillin but resistant to streptomycin. After 4 hours of cocultivation, Salmonella Enteritidis are resistant to both antibiotics. Mechanism of ampicillin resistance by the Salmonella Enteritidis?

38 year old woman gravida 4m, para 4 – 6 month history of urinary incontinence when sneezing and coughing – Leakage occurs when patient performs valsalva – Instructed to do kegel exercises – which unaffected by kegel exercise to strengthen muscles of perineum. Which muscle are unaffected by the exercise?

Internal Anal Sphincter
Internal anal sphincter = parasympathetic/sympathetic control
external anal sphincter = skeletal muscle and controlled by pudendal nerve

Image: 38 year old woman gravida 4m, para 4 - 6 month history of urinary incontinence when sneezing and coughing - Leakage occurs when patient performs valsalva - Instructed to do kegel exercises - which unaffected by kegel exercise to strengthen muscles of perineum. Which muscle are unaffected by the exercise?

26-year-old man – 40 minutes after MVC – BP is 90/60. Systolic blood pressure 20 mm Hg with inspiration. PE shows JVD. There is distant heart sounds. DX?

Pericardial Tamponade
Hypotension Distant Heart Sounds, Distended Jugular Vein,
Electrical Alternans
pulsus paradoxus, a sign in which blood pressure decreases drastically during inspiration. Pulsus paradoxus is a classic sign of pericardial tamponade

Image: 26-year-old man - 40 minutes after MVC - BP is 90/60. Systolic blood pressure 20 mm Hg with inspiration. PE shows JVD. There is distant heart sounds. DX?

23-year-old woman – SOBs for 2 weeks, PE Shows no abnormalities – Na+ 135, HCO3 15, Cl 110- Most likely diagnosis?

Renal Tubular Acidosis
normal anion gap (10) Na- Cl – HCO3= 10
High Anion Gap needs to be greater than 12
HARDASS

Image: 23-year-old woman - SOBs for 2 weeks, PE Shows no abnormalities - Na+ 135, HCO3 15, Cl 110- Most likely diagnosis?

A 30-year-old woman – chronic renal failure and normocytic anemia – Following recombinant EPO, her hemoglobin concentration increases due to increased activity of 5-aminolevulinic acid synthase in erythrocytes. This enzyme regulates the reaction involving the condensation of which compounds?

Glycine and Succinyl CoA
substrates used the first step in heme synthesis

38 year old woman gravida 4m, para 4 – 6 month history of urinary incontinence when sneezing and coughing – Leakage occurs when patient performs valsalva – Instructed to do kegel exercises – which unaffected by kegel exercise to strengthen muscles of perineum. Which muscle are unaffected by the exercise?

Internal Anal Sphincter
Not under voluntary control

Image: 38 year old woman gravida 4m, para 4 - 6 month history of urinary incontinence when sneezing and coughing - Leakage occurs when patient performs valsalva - Instructed to do kegel exercises - which unaffected by kegel exercise to strengthen muscles of perineum. Which muscle are unaffected by the exercise?

26-year-old man – 40 minutes after MVC – BP is 90/60. Systolic blood pressure 20 mm Hg with inspiration. PE shows JVD. There is distant heart sounds. DX?

Pericardial Tamponade
Hypotension Distant Heart Sounds, Distended Jugular Vein

Image: 26-year-old man - 40 minutes after MVC - BP is 90/60. Systolic blood pressure 20 mm Hg with inspiration. PE shows JVD. There is distant heart sounds. DX?

A 45 year old man – hypercholesterolemia (LDL 260 mg/dl) – HMG CoA reductase inhibitor is prescribed. Mechanism of action of treatment?

Increased Synthesis of LDL Receptors
statins –
HMG CoA Reductase inhibitors prevent the liver from synthesizing its own cholesterol.
LDL receptor expression in order to take cholesterol from the blood

22-year-old woman – nausea and vomiting – 6-month history of headaches. Neuro exam shows marked vertical gaze palsy – impaired accommodation. Horizontal gaze is normal. MRI of the brain is likely to show a tumor at which location?

Pineal Gland
VERTICAL GAZE PALSY – PARINAUD Syndrome
– stroke hydrocephalus, pinelomas
Horizontal Gaze intact – Pontine Paramedian Reticular Formation

An investigator is conducting a study of a novel protein in an experimental animal model. The protein interferes with desmosomes. Which cell junctions of the skin are affected?

Basal Keratinocyte – Suprabasal Keratinocyte
Pemphigus Vulgaris. Separation of suprabasilar epidermis, intact basal keratinocytes
Bullous pemphigoid–> includes separation of basal layer

Image: An investigator is conducting a study of a novel protein in an experimental animal model. The protein interferes with desmosomes. Which cell junctions of the skin are affected?

19-year-old – weakness of the muscles of his neck and extremities – Bladder and bowel function is normal. Position sense is impaired. DX?

Polyneuropathy
A condition involves damage to multiple peripheral nerve fibers. Patients typically present with symmetric distal sensory loss or a burning sensation associated with motor weakness
Vitamin B6 deficiency
Charcot-Marie-Tooth
Guillain-Barré

Forty of 100 attendees at a company party develop explosive watery diarrhea followed by abdominal cramps and vomiting 12 to 48 hours after the party. All affected people consumed raw oysters – No other associations between the consumption of specific foods and symptoms. Causal organism?

Vibrio Parahaemolyticus
associated with consumption of undercooked seafood like shellfish.

30-year-old man – history of fever, chills, severe headache. He immigrated from Honduras. A photomicrograph of a peripheral blood smear is shown. (Schuffner Stipling) The patients overall condition is most likely caused by an infectious agent with which features?

Formation of Hypnozoites
plasmodium vivax- Schuffner Stipling
chloroquine = clear the infection in the bloodstream,
Treatment with primaquin to clear the hypnozoites,

Image: 30-year-old man - history of fever, chills, severe headache. He immigrated from Honduras. A photomicrograph of a peripheral blood smear is shown. (Schuffner Stipling) The patients overall condition is most likely caused by an infectious agent with which features?

63-year-old flank pain, tachycardia, hypotension while receiving a blood transfusion in the intensive care unit. Her urine is wine colored. Which is the cause these findings?

ABO Incompatibility
TYPE II hypersensitivity – delayed reaction
Rh Incompatibility – Jaundice
Extravascular Hemolysis –

55-year-old woman – change in behavior over last 2 days – Yesterday she didn’t recognize a picture of her own mother. Neuro exam shows inability to recognize objects unless she touches them or hears the sound that they make. Lesion in which of the following arteries?

Posterior Cerebral
PCA stroke can cause “prosopagnosia” which is the inability to recognize familiar faces.
Caused by bilateral lesions of visual association areas, which are situated in the inferior occipitotemporal cortex (fusiform gyrus).

Image: 55-year-old woman - change in behavior over last 2 days - Yesterday she didn't recognize a picture of her own mother. Neuro exam shows inability to recognize objects unless she touches them or hears the sound that they make. Lesion in which of the following arteries?

A 90-year-old man – 1-week history of constipation and abdominal swelling. He is afebrile. 4-cm mass is palpated in the right groin. The mass is not reducible, but moderately increased tympanic bowel sounds are audible over it. Diagnosis?

incarcerated inguinal hernia
protrusion of peritoneum through an opening
usually at a site of weakness
contents at risk of incarceration

Image: A 90-year-old man - 1-week history of constipation and abdominal swelling. He is afebrile. 4-cm mass is palpated in the right groin. The mass is not reducible, but moderately increased tympanic bowel sounds are audible over it. Diagnosis?

A 27 year old man – spinal cord transection at C8 in a skiing accident. In which of the following situations is he most likely to achieve and sustain an erection?

Penile Stimulation
S2-S4

Image: A 27 year old man - spinal cord transection at C8 in a skiing accident. In which of the following situations is he most likely to achieve and sustain an erection?

13-year-old with Type 1 Diabetes – has poor glucose control for past 2 months. Previously the T1DM was controlled. Now family is fighting at meals. Parents argue in physician’s office. Which is the most appropriate next step?

Discuss further the impact of the patient’s illness on the family
getting heated and therefore doc should try to calm things down before proceeding

ARBS

-sartan

Renin Increased
ANG I Increased
ANG II Increased
Aldosterone Decreased

Image: ARBS

A 71-year old woman with non-hodgkin lymphoma is diagnosed with lymphomatous meningitis. Serum concentrations is 2.3. A course of methotrexate is planned. IV administration of which adjuvant agent is most appropriate?

Leucovorin
common chemoprotectant used to avoid Methotrexate toxicities.

A 40-year-old man – 6 months of constipation and weakness. Family history of colorectal carcinoma in multiple members of both sexes. MCV is 73. Photograph of colon. Analysis shows mutations in the MHS2 gene. Diagnosis?

HNPCC syndrome
MHS2 gene mutation
Lynch syndrome AKA hereditary non-polyposis colorectal carcinoma (HNPCC),

Image: A 40-year-old man - 6 months of constipation and weakness. Family history of colorectal carcinoma in multiple members of both sexes. MCV is 73. Photograph of colon. Analysis shows mutations in the MHS2 gene. Diagnosis?

30-year-old woman – 2 week history of right flank pain – BP is 168/98. PE shows a 7-cm mass in right side of the abdomen. CT shows mass in the retroperitoneal space compressing the renal artery. If perfusion pressure is decreased, but her GFR and RBF remain unchanged, which mechanism mediates the autoregulation of the renal arterioles in this patient?

Decreased afferent arteriolar resistance
myogenic mechanism
autoregulation- afferent arteriole controls blood flow

45-year-old man -picture with 10 years of painful ulcers that recur every few months. No other problems. Photograph shown. Which is likely cause of patient condition?

Aphthous Ulcers
caused by stress
canker sores

Image: 45-year-old man -picture with 10 years of painful ulcers that recur every few months. No other problems. Photograph shown. Which is likely cause of patient condition?

40-year-old woman – 25-year history of T1DM has serum concentration of 2.5 mg/dl. – A creatine clearance test is ordered to determine the GFR. This test is limited to Inulin Clearance Test, because of the following properties of creatinine?

Secreted by the proximal Tubule
Creatinine clearance slightly overestimates GFR because creatinine is moderately secreted by PCT

Image: 40-year-old woman - 25-year history of T1DM has serum concentration of 2.5 mg/dl. - A creatine clearance test is ordered to determine the GFR. This test is limited to Inulin Clearance Test, because of the following properties of creatinine?

40 from Kenya – 3 week history of diarrhea. He recalls zero of three objects after 5 minutes. – Photomicrograph of the head shown (dermatitis) – DX of vitamin deficiency is made. Vit deficiency is made. The deficient cofactor contains which components?

Adenine
Pellagra Diarrhea, Dementia, Dermatitis
Vitamin B3 deficiency – Co-Factor for NAD
Niacin combines with Adenine to form B3

Image: 40 from Kenya - 3 week history of diarrhea. He recalls zero of three objects after 5 minutes. - Photomicrograph of the head shown (dermatitis) - DX of vitamin deficiency is made. Vit deficiency is made. The deficient cofactor contains which components?

47-year-old – abdominal enlargement and rectal bleeding. PE shows internal hemorrhoids. Increased pressure in which of the following is most likely to cause the hemorrhoids?

Superior Rectal Vein
Superior rectal offer IMA
External Hemorroid – Inferior Rectal of Internal pudendal

Image: 47-year-old - abdominal enlargement and rectal bleeding. PE shows internal hemorrhoids. Increased pressure in which of the following is most likely to cause the hemorrhoids?

A 24 y/o woman – diffuse toxic goiter (graves disease) – partial thyroidectomy – months later she develops muscle cramps, tetany, hypocalcemia. Which labs findings?

phosphate increased
parathyroid decreased
25- Hydroxyvitamin – normal
accidental removal of the PTH glands during thyroidectomy ⇒ ↓ PTH
↑ removal of Ca²⁺ and Phophate from bone –in kidneys: ↑ Ca²⁺ reabsorption and ↓ PO₄³⁻ reabsorption –↑ conversion of 25, Hydroxyvitamin D to 1,25 Hydroxyvitamin D (Calcitriol – active form) via ↑ activity of 1-a Hydroxylase deficiency

Image: A 24 y/o woman - diffuse toxic goiter (graves disease) - partial thyroidectomy - months later she develops muscle cramps, tetany, hypocalcemia. Which labs findings?

. 72-year-old – coronary artery disease – 2 -month history of angina – 5 months ago underwent – stent placement for significant stenoses – symptom free for 3 months – ECG at rest shows no abnormalities – ST segment changes in II, III, AvF (inferior leads). Cause of recurrent angina?

Neointima Formation in the right coronary stent
Stents cause neointima growth – restenosis
Stent thrombosis vs re-stenosis. Stent thrombosis is an acute occlusion
of a coronary artery stent, which often results in acute coronary
syndrome.

Image: . 72-year-old - coronary artery disease - 2 -month history of angina - 5 months ago underwent - stent placement for significant stenoses - symptom free for 3 months - ECG at rest shows no abnormalities - ST segment changes in II, III, AvF (inferior leads). Cause of recurrent angina?

A 45 year old – sudden onset of severe pain in the left flank – awakened from sleep. Reports that the pain – now principally located in the scrotum. PE shows no palpable masses in abdomen. No rebound tenderness – Cause of pain?

Renal Calculi in the Left Ureter
kidney stone referred to the testicles.
no mass in the scrotum- testicular torsion will have that “bag of worms”

60 year old – cisplatin therapy for bladder cancer – develops paresthesia – Medication damaged region labeled B – Damage to these cells would most likely cause which motor sign?

Hyporeflexia
afferent arc of the muscle stretch reflex has to go through the dorsal rami and dorsal root ganglia.

Image: 60 year old - cisplatin therapy for bladder cancer - develops paresthesia - Medication damaged region labeled B - Damage to these cells would most likely cause which motor sign?

10-year-old girl – hair growth on face and increased muscle mass. PE shows clitoromegaly. Pelvic exam shows a normal appearing vagina. Serum Studies:
LH Low
Testosterone High

An unregulated increase in hormone production is most likely to cause patient hirsutism?

Ovarian Sertoli -Leydig Cell
increased testosterone
Ladies dig testosterone

A 25 year old woman – lost consciousness at home – fever (103) and history of recurrent upper urinary tract infections. BP is 82/48 – Diffuse crackles are heard over both lung fields. ABG shows:
pH. 7.18
PCo2 32
Po2 64

Blood culture grows gram negative rods – CXR – shows diffuse infiltrates- Most likely cause of pulmonary symptoms?

Pulmonary Capillary Leakage
Fluid in lungs
Urosepsis- ARDS – diffuse bilateral infiltrates on chest radiography…
pneumonia/sepsis (ARDS?)
PO2 is low at 64.
pneumonia there is increased capillary leakage leading to pulmonary edema.

45-year-old – 3 day history of fever and muscle pain – He went hunting – ate polar bear meat. Temp is 103. PE shows periorbital edema after and muscle tenderness. Which tests most likely to establish DX?

Muscle Biopsy
Trichinella – larvae deposit in muscle
Muscle pain + periorbital edema is a classic presentation for trichonella spiralis. Best diagnosis for this is a muscle biopsy, as the wormy likes to hangout within the muscles.

3-year-old – well child exam- flat purple lesions that measures 2.1 cm on her face – Lesions show cavernous channels. Which is most likely outcome of the lesion?

Spontaneous Regression
CAVERNOUS VASCULAR CHANNELS
nevus simplex lesions regress within the first 2 years of life

Image: 3-year-old - well child exam- flat purple lesions that measures 2.1 cm on her face - Lesions show cavernous channels. Which is most likely outcome of the lesion?

1 week old newborn – poor feeding, vomiting, progressive lethargy – No congenital anomalies. PE shows decreased muscle tone and poor responsiveness – reflexes are normal. pH 7.15, bicarb low, high ammonia (10X) – Cause?

Organic Acid Metabolism disorder
Organic acidemias most commonly present in infancy with poor feeding, vomiting, hypotonia, a high anion gap metabolic acidosis, hepatomegaly, and seizures.

Male newborn – Baby is small for weight and height – head circumference is 31cm Eye exam shows choriorentinitis – CT scan of the head shows Hydrocephalus and intracranial calcifications. The newborn congenital infection was most likely acquired by mother vis which mode of transmission?

Ingestion of Undercooked meat
Congenital Toxoplasmosis:
Triad intracranial calcifications, Chorioretinitis, hydrocephalus

Image: Male newborn - Baby is small for weight and height - head circumference is 31cm Eye exam shows choriorentinitis - CT scan of the head shows Hydrocephalus and intracranial calcifications. The newborn congenital infection was most likely acquired by mother vis which mode of transmission?

68 year old woman – 2 hours after vomiting blood – 3 weeks ago began meds for vertebral compression fracture with osteoporosis. PE shows epigastric tenderness – Endoscopy shows esophageal erosions. Which medication cause hematemesis?

Aldendronate
bisphosphonates – causes esophagitis and osteonecrosis

In a study of antibiotic resistance, strain of Escherichia Coli to ampicillin but sensitive to streptomycin is cultured with a strain of Salmonella Enteritidis sensitive to ampicillin but resistant to streptomycin. After 4 hours of cocultivation, Salmonella Enteritidis are resistant to both antibiotics. Mechanism of ampicillin resistance by the Salmonella Enteritidis?

Conjugation
plasmid gets transferred from conjugal bridge from one bacteria to another

Image: In a study of antibiotic resistance, strain of Escherichia Coli to ampicillin but sensitive to streptomycin is cultured with a strain of Salmonella Enteritidis sensitive to ampicillin but resistant to streptomycin. After 4 hours of cocultivation, Salmonella Enteritidis are resistant to both antibiotics. Mechanism of ampicillin resistance by the Salmonella Enteritidis?

55-year-old smoke – comes to physician for a routine health maintenance – She never smoked cigs. CXR shows 2-cm left pulmonary nodule (coin lesion). Microscopic exam shows nests of cords of regular uniform round nuclei and rare mitoses. Cells are arranged in trabeculae, and scattered rosettes. Electron shows dense-core, membrane-bound granules in the cytoplasm. These cells resemble which type of respiratory tract cells?

Neuroendocrine Cell
nests of well-differentiated, “regular” cells is a carcinoid tumor. Additionally, rosettes are histological features of carcinoid tumors

Kulchitsky cells = pulmonary neuroendocrine cells (PNEC).

Image: 55-year-old smoke - comes to physician for a routine health maintenance - She never smoked cigs. CXR shows 2-cm left pulmonary nodule (coin lesion). Microscopic exam shows nests of cords of regular uniform round nuclei and rare mitoses. Cells are arranged in trabeculae, and scattered rosettes. Electron shows dense-core, membrane-bound granules in the cytoplasm. These cells resemble which type of respiratory tract cells?

A 63-year-old woman – repair of leaking berry aneurysm in the circle of Willis. Two days later, a CT scan of the head shows widening of the subarachnoid space. What change in CSF causes the finding?

Decreased movement through arachnoid villi
SAH – communicating hydrocephalus
acute SAH can lead to decreased absorption and movement via arachnoid villi

Image: A 63-year-old woman - repair of leaking berry aneurysm in the circle of Willis. Two days later, a CT scan of the head shows widening of the subarachnoid space. What change in CSF causes the finding?

A 68 y/o – 9 month history of abdominal pain after meals. 24lb weight loss. She avoids eating because of cramping pain that lasts 1 hour after meals. PE shows thin body frame and a soft nontender abdomen. BP 160/90 A CT scan shows a 3-cm ectatic (dialated) aorta. Which pairs of arteries is most likely involved in this patient?

inferior mesenteric and superior mesenteric
Mesenteric cause post prandial pain – AAA
compression of duodenum at 3rd

A full-term male newborn has lethargy, poor feeding, and vomiting after delivery. He appears flaccid. Serum studies show increased concentration of ammonia and orotic acid. Which labeled steps in the metabolic pathway is the most likely cause of these findings?

Area labeled C.
Ornithine to Citrulline
Ornithine transcarbamolase deficiency.
Carbomoyl phosphate (CP) is shunted to pyrimidine synthesis –> increased orotic acid production (CPS you would not see orotic acidemia).

Image: A full-term male newborn has lethargy, poor feeding, and vomiting after delivery. He appears flaccid. Serum studies show increased concentration of ammonia and orotic acid. Which labeled steps in the metabolic pathway is the most likely cause of these findings?

A 15 month old – 9 month history of recurrent bacterial infections – Multiple areas of honey crusted lesions over the lower extremities. Gram stain shows gram positive cocci – no leukocytes – Leukocyte count very high. Pt most likely has a rare autosomal recessive disease leading to a lack of CD18 expression. Leukocytes would be deficient in which characteristic?

Migration
CD double digits B CD single digits T cells
MR ATM
Margination, Rolling, Adhesion, Transmigration

Image: A 15 month old - 9 month history of recurrent bacterial infections - Multiple areas of honey crusted lesions over the lower extremities. Gram stain shows gram positive cocci - no leukocytes - Leukocyte count very high. Pt most likely has a rare autosomal recessive disease leading to a lack of CD18 expression. Leukocytes would be deficient in which characteristic?

A 24-year-old woman – 28 weeks gestation. She has asthma – Diffuse wheezes – An inhaled B2 adrenergic agonist is administered. Most likely effect in this patient after this therapy?

Tremor
Albuterol – relaxes bronchial smooth muscle (short acting β2-agonist).
c. For acute exacerbations. Can cause tremor, arrhythmia.

A 70-year-old man – 1 week SOB with mild exertion, cough, fatigue, and weakness. The shortness of breath frequently awakens him from sleep and is partly relieved by sitting or standing. No Medications. 20 year history of hypertension. BP 180/110. Crackles heard on auscultation. A chest x-ray shows cardiomegaly. Decreased left ventricular systolic function. What is SV, EDV, CO?

Stroke volume Decreased
Left EDV Increased
Cardiac Output Decreased
Decompensated left heart failure

A 32-year woman – 7 day of history of sneezing, nasal stuffiness, and water eye. She has a history of similar symptoms each spring while gardening. Which cells will be increased in her nasal secretions as a result of this reaction?

Eosinophils
Mast cells degranulate, producing histamine which attracts eosinophils.
The early stage of an allergic reaction is mast cell mediated, but the late
stage (including mucus production) is mediated by eosinophils.
Charcot Leyden crystals: are made up of Eosinophils

Newborn with ARDS – intubation and 100% inspired oxygen with continuous PEEP – One month later – a CXR shows increased densities in all lung fields. He develops pneumonia and dies. Microscopic exam shows prominent interstitial fibrosis surrounding dilated airspaces with bronchiolar metaplasia. Most likely diagnosis?

Bronchopulmonary Dysplasia
Tissue damage to lunch with supplemental oxygen
oxygen toxicity due to free radical generation.

Image: Newborn with ARDS - intubation and 100% inspired oxygen with continuous PEEP - One month later - a CXR shows increased densities in all lung fields. He develops pneumonia and dies. Microscopic exam shows prominent interstitial fibrosis surrounding dilated airspaces with bronchiolar metaplasia. Most likely diagnosis?

A 32-year-old man – deep laceration that injured nerve supply to flexor Digitorum muscles index fingers. Which of the following movements can be used to specifically test for the function of the muscle?

Flexion in distal interphalangeal joint
responsible for flexion of DIP. Medial aspect of the muscle (which flexes the 4th and 5th digit)

Image: A 32-year-old man - deep laceration that injured nerve supply to flexor Digitorum muscles index fingers. Which of the following movements can be used to specifically test for the function of the muscle?

An experimental study to examine the function of the pro-opiomelanocortin gene. The gene is found to encode for mRNA. This protein is a precursor for ACTH and B-lipotropin. Which processes generate these hormones?

Post-Translational Modification
PROTEIN serves as a precursor for making the hormones thus it has already undergone translation

POMC is a prohormone peptide chain.

It is translated and later enzymes cut this peptide in the subpeptides.

A 6 year old – 2 day history of itchy scalp. 10 children have same symptoms. Vitals normal. Exam shows multiple hair shafts 1-2mm white, globular protuberances. Which cause of patients condition?

Pediculus Humanus Capitis
Blood-sucking lice that cause intense pruritus with associated excoriations, “white, globular protuberances”.

Image: A 6 year old - 2 day history of itchy scalp. 10 children have same symptoms. Vitals normal. Exam shows multiple hair shafts 1-2mm white, globular protuberances. Which cause of patients condition?

A thing 66-year old – confusion. Show severe hyponatremia. SIADH (vasopressin) secretion is suspected. Which supports diagnosis?

Urine Osomolality > Plasma Osmolality

Excessive ADH causes the collecting duct of the kidney to reabsorb huge amounts of water that it should normally excrete. That means that the plasma will now have much more water relative to solute

A 40-year-old man with AIDS develops severe diarrhea that is refractory to all standard oral therapies. Diarrhea resolves after administration of a stable analogue of naturally occurring hormone. Which hormone?

Somatostatin
Octreotide TX = refractory diarrhea
D cells in the pancreatic islets and GI mucosablocks everything GI-related

(“encourages somato-stasis”): decreased gastric acid & pepsinogen secretion, decreased pancreatic and small intestine fluid secretion, decreased gallbladder contraction, decreased insulin & glucagon release.’

A 24 year old man – receives injection in the brachial artery. Which changes occur as a result in response to histamine?

Cigarette Smoking
transitional cell carcinoma = painless hematuria + papillary growth

Image: A 24 year old man - receives injection in the brachial artery. Which changes occur as a result in response to histamine?

A 24-year-old man – receives injection in the brachial artery. Which changes occur as a result in response to histamine?

Increased lymph Flow

Histamine increases microvascular permeability, i.e. Fluid + Proteins, this increases pressure in the interstitium > lymph flow increases

direct action of histamine on the lymphatic smooth muscle via stimulation of H1 (and in some vessels H2) receptors.

A 37-year-old – drank bottle of whiskey- great despair – broke up with boyfriend. She describes mood as depressed. She says there is no one I can trust but you. Everyone I work with is a jerk. The patient has which of the following personality disorders?

Borderline

unstable mood and a crazy relationship. She’s also splitting

35 year old man – HTN – BP 180/100. TX started with losartan. Which is likely to occur?

Increased Serum Angiotensin II Concentration
increased renin, increased Ang I, increased Ang II, decreased aldosterone and unchanged bradykinin

man and wife – 25-year-old- come for genetic counseling. Both had microcytic anemia and increased fraction of hemoglobin. Heterozygous null mutation of the B-globin gene, and the heterozygous mutation known to cause a 50% decrease in B-globin gene function in one allele. Which is the B-globin function possibilities for this couple’s offspring?

One in four will have 25% B Globin Function may require occasional Transfusion
Thalassemia – recessive
Make a punnett square with a cross
B B+ and B B0;
B+ represents 50% function
B0 represents 0% (null) function.
the husband = B B0 genotype
wife has a B B+ genotype.
Cross of these two will result in the following genotypes; BB, BB0, BB+, B+B0 BB = 100% function, BB+ = 75% function, BB0 = 50% function, B+B0 = 25% function

A 62-year-old man – DX with atrial and Ventricular Arrythmia – appropriate pharmacotherapy was initiated at the time. BP 136/88. PE shows no abnormalities. An ECG shows normal sinus rhythm with prolonged QT interval. Which drug is most likely cause of these findings

Solatol
type III + (Type II antiarrhythmic).
(K+ channel-blocking)
beta-blocker activity
decreased heartrate and blood pressure (beta-1 blocking activity), with the QT prolongation –
all type III AADs cause QT prolongation.

55 year old with T!DM – intermittent burning pain in his feet during the past 4 months. Exam shows allodynia bilaterally. Decreased sensation to pinprick. Motor, DTR, and vibration sense normal. Which is most likely cause of pain in this patient?

Persistent activation of voltage-gated Na+ Channels in Nociceptors
allodynia. = intact sensation, it’s just “turned up” way too much.
Diabetes causes non-enzymatic glycosylation – damage neuron

After completing 6 courses of chemotherapy for cancer of the breast, a 45 year old woman – SOBs, dyspnea, orthopnea. Basilar crackles heard bilaterally. S3 gallop. X rays show an enlarged heart, bilateral interstitial and alveolar edema, and bilateral pleural effusions. Which chemotherapy drug causes these findings?

Doxorubicin
CHEMOMAN
anthracyclines is dilated cardiomyopathy.
Doxorubicin = Cardiac problems
Bleomycin = Pulmonary problems
Cardiac = BILATERAL crackles/effusions since blood is backing up it would affect both lungs while Bleomycin could present as unilateral. Main though thing is the S3 which makes it automatically cardiac-related

Image: After completing 6 courses of chemotherapy for cancer of the breast, a 45 year old woman - SOBs, dyspnea, orthopnea. Basilar crackles heard bilaterally. S3 gallop. X rays show an enlarged heart, bilateral interstitial and alveolar edema, and bilateral pleural effusions. Which chemotherapy drug causes these findings?

A 50 year-old man – diarrhea and 13.6 kg (30lb ) weight loss over the past 2 months. He has pale, foul smelling stools with oil droplets. History of alcohol dependence and abdominal pain. X-ray shows calcifications in the mid-upper abdomen. Which pathophysiologic mechanisms most likely cause this condition?

Generalized Malabsorption
“calcifications in the mid-upper abdomen” is most likely referring to a chronic pancreatitis.
lack of lipase= pale, foul-smelling stools with oil droplets per pt Hx
no amylases, proteases, nor trypsinogen (to activate other enzymes

A 32 year old woman – 20 weeks pregnant – severe heartburn for 3 weeks. Epigastric tenderness of palpation f the abdomen. Which drug is contraindicated because of risk of spontaneous abortion?

Misoprostol
Prostaglandin analog (PGE2) that acts on the stomach to promote mucus protection of the stomach lining, but also acts in the uterus to encourage contraction, which makes it useful for abortion.

A 75 year old man – unconscious – collapsed 30 minutes ago. He has abdominal pain and history of abdominal mass. Smoker for 60 years. PE shows pulsatile abdominal mass and minimal bowel sounds. Which is cause?

Atherosclerotic
AAA – Atherosclerotic changes leads to aneurysmal dilatation of abdominal aorta

Image: A 75 year old man - unconscious - collapsed 30 minutes ago. He has abdominal pain and history of abdominal mass. Smoker for 60 years. PE shows pulsatile abdominal mass and minimal bowel sounds. Which is cause?

35-year old woman – fatigue and lethargy- 20lb weight gain – wears a coat because she is cold. PE shows coarse skin. Thyroid scan shows localization of radioiodine in the midline of the inferior aspect of the oral cavity. Radioiodine is most likely localized inferior to which structure?

Foramen Cecum
lingual thyroid, a persistence of thyroid tissue at the base of the tongue due to failure of migration

Image: 35-year old woman - fatigue and lethargy- 20lb weight gain - wears a coat because she is cold. PE shows coarse skin. Thyroid scan shows localization of radioiodine in the midline of the inferior aspect of the oral cavity. Radioiodine is most likely localized inferior to which structure?

A 58-year-old man – ER – 1 hour after being kicked in the side by a horse. 2 years ago – DX with chronic idiopathic myelofibrosis. BP 90/50 He drinks 12-ounce beers daily. Abdominal exam shows guarding and tenderness over the left upper quadrant. Hematocrit increased; Leukocyte count increased. Ultrasound shows intraperitoneal fluid. 4 hours later – a laparotomy is most likely to show which of the following?

Ruptured Spleen
myelofibrosis can cause an enlarged spleen.

A 10-year-old – intermittent dyspnea – Pt turns blue. Episodes resolves when she squats. A systolic murmur is heard at birth. Today, 4/6 systolic murmur is heard at the pulmonic area. Which is most likely in this patient?

Palpable Right Ventricular Lift
PROVE – Pulmonic Stenosis, Right Ventricular Hypertrophy, Overriding Aorta, VSD

The graph shows expiratory pressure-volume curves that compare a healthy adult with a patient who has a 9-month history of progressive shortness of breath. Graph TLC decreased. These findings are most consistent with which of the following?

Diffuse Pulmonary Fibrosis
TLC Decreased
restrictive lung disease

A 26-year-old man – comes to physician for a follow-up exam – Physician prescribes a HMG CoA reductase inhibitor. total serum cholesterol is decreased from 325mg/dl to 235 mg/dl. Cause of the decrease?

Increased number of LDL receptors on hepatocytes
HMG CoA reductase inhibitors block the ability of the body to produce its own cholesterol

A 24 year old man – fever and chills for 1 day -history of UTI – Ciprofloxacin is begun. Two days later urine culture grows E Coli resistant to ciprofloxacin. Cause of organisms resistance?

Alteration in DNA Gyrase
Cipro = MOA of fluoroquinolones is to inhibit prokaryotic topoisomers II (DNA Gyrase) occasionally Gram (-) organisms like e-coli develop resistance by mutating their DNA gyrase so drugs like cipro cannot inhibit

Results of a 5-year screen program for HIV in a population of 10,000 sex workers. Which is closet to average incidence:
Time 5 years Pop at Risk – 4800 New Patients 250

What is approximation of average annual incidence of infection in this population?

5%
(incidence = new cases / total)
The average of these %s for all the years = 5.58%. So that’s close enough to 5%.

A 43 year old woman with a 2 year history of labile HTN – very drastic changes in BP – One day after propranolol is started bp is 214/132. Serum cateccholamine concentrations are 4X upper limit. A Ct scan shows 2-cm nodule in the right adrenal gland. Which is most likely responsible for increased blood pressure after the propranolol therapy?

Unopposed a-adrenergic tone with beta blockers
making excess catecholamines – pheochromocytoma
Starting beta blockers before alpha blockade in pheochromocytoma is contraindicated.

Image: A 43 year old woman with a 2 year history of labile HTN - very drastic changes in BP - One day after propranolol is started bp is 214/132. Serum cateccholamine concentrations are 4X upper limit. A Ct scan shows 2-cm nodule in the right adrenal gland. Which is most likely responsible for increased blood pressure after the propranolol therapy?

An 80 y/o old -intermittent headache and aching pain in her jaw while chewing. Which is next step in establishing diagnosis?

Determination of erythrocyte sedimentation rate
Giant Cell Arteritis – jaw pain with eating and unilateral headache

An 80 y/o old -intermittent headache and aching pain in her jaw while chewing. Which is next step in establishing diagnosis?

Determination of erythrocyte sedimentation rate
Giant Cell Arteritis – jaw pain with eating and unilateral headache
Identified by determining the erythrocyte sedimentation rate.

Image: An 80 y/o old -intermittent headache and aching pain in her jaw while chewing. Which is next step in establishing diagnosis?

A 28 year-old-man – with AIDs – has a high fever, headache, lethargy. Exam shows papilledema and nuchal rigidity. A mucicarmine-stained smear of CSF is shown. Which is the primary site of infection with this organism?

Lungs
Cryptococcal meningitis that is transferred through respiratory droplets
Mucicarmine stain

Image: A 28 year-old-man - with AIDs - has a high fever, headache, lethargy. Exam shows papilledema and nuchal rigidity. A mucicarmine-stained smear of CSF is shown. Which is the primary site of infection with this organism?

A 7-year-old boy – metabolic acidosis and persistent phosphaturia. Patient most likely to have a primary defect of which labeled site in the drawing of the nephron?

Proximal tubal

Image: A 7-year-old boy - metabolic acidosis and persistent phosphaturia. Patient most likely to have a primary defect of which labeled site in the drawing of the nephron?

A 55 year old man – 1- hour history of severe chest pain, nausea, and vomiting. He is agitated, clammy, and sweating profusely. JVP is 12cm and crackles heard in lung bases. Which hemodynamic changes (Pulmonary Capillary Wedge Pressure, Cardiac Output, SVR)?

Pulmonary Capillary Wedge Pressure – Increased
Cardiac Output – decreased
Systemic Vascular Resistance – increased
cardiogenic shock. Characteristic features of cardiogenic shock: increased cardiac pressures and decreased cardiac output due to impeded forward flow, as well as increased systemic vascular resistance

35 year old male – 1 week of history of severe daily headaches. Pain is located around the left eye and associated with excessive tearing and redness, Headaches last for 30 minutes to 2 hours. – No focal finding. DX?

Cluster Headache.
excessive tearing

Image: 35 year old male - 1 week of history of severe daily headaches. Pain is located around the left eye and associated with excessive tearing and redness, Headaches last for 30 minutes to 2 hours. - No focal finding. DX?

A 52 year old – severe pain in midback for 2 weeks. History of left breast cancer treated with mastectomy and chemotherapy. Tenderness in palpation over the thoracic spine. MRI shows metastases to thoracic vertebral bodies. Which veins is the most likely path for tumor cells in the breast to metastasize to the vertebral bodies?

Intercostal Veins
Most direct pathfor breast cancer to metastasize to the vertebra are the intercostal veins.
Breast cancer will cause mixed, lytic and blastic lesions once in bone.

Image: A 52 year old - severe pain in midback for 2 weeks. History of left breast cancer treated with mastectomy and chemotherapy. Tenderness in palpation over the thoracic spine. MRI shows metastases to thoracic vertebral bodies. Which veins is the most likely path for tumor cells in the breast to metastasize to the vertebral bodies?

A 57-year-old woman – intubation in ICU after developing progressive dyspnea and hypoxemia. Pt was admitted for cirrhosis and was admitted to the hospital because of melena. Transfusion was initiated. Bilateral crackles are heard on inspiration. There are no wheezes. PE shows spider angiomata on chest and abdomen. Bilateral basilar crackles . CXR shows bilateral diffuse airspaces without pleural effusions or cardiomegaly. The patint develope which of the following transfusions reactions?

Acute Lung Injury
Transfusion-related acute lung injury (TRALI)
within six hours after blood product administration

A 3-year-old boy – epigastric pain and bile stained vomitus. Child had been well prior to episode. Cause of problem?

Annular Pancreas

Image: A 3-year-old boy - epigastric pain and bile stained vomitus. Child had been well prior to episode. Cause of problem?

The 35-year-old woman – indicated by arrow – family history of retinitis pigmentosa (Male-to-Male transmission). Which mode of inheritance can be eliminated based on pedigree?

X-Linked Recessive

Image: The 35-year-old woman - indicated by arrow - family history of retinitis pigmentosa (Male-to-Male transmission). Which mode of inheritance can be eliminated based on pedigree?

A 50-year-old man – surgical transection of the obturator nerve for unbalance muscle spasm affecting the hip. Which muscles is most likely paralyzed by this procedure?

Adductor Brevis
obturator nerve
“Innervates the muscles of the medial compartment of the thigh (obturator externus, adductor longus, adductor brevis, adductor magnus and gracilis).”

34 year old – 2 weeks history of severe back pain, decreased appetite, and malaise. Six weeks ago began infliximab. An MRI of spine shows osteomyelitis of L1-l2 with destruction of the interverterbral discs and 1.2 cm abscess. Gram stain is negative. Bone specimen shows a inflammation with granuloma formation. Cause of findings?

Tuberculous Osteomyelitis
TNF-a inhibitor always check for TB or autoimmune

Image: 34 year old - 2 weeks history of severe back pain, decreased appetite, and malaise. Six weeks ago began infliximab. An MRI of spine shows osteomyelitis of L1-l2 with destruction of the interverterbral discs and 1.2 cm abscess. Gram stain is negative. Bone specimen shows a inflammation with granuloma formation. Cause of findings?

An 80-year-old woman – T2DM, HTN, CKGD. BP 165/95. PE shows edema. Compared with a healthy individual – which lab finding (hematocrit, PTH, Calictrol) is most likely in this patient?

Hematocrit Decreased
Serum Parathyroid Increased
Serum Calcitriol Decreased
chronic kidney disease –> decreased EPO –> decreased hematocrit chronic kidney disease –> decreased PO4- excretion –> increased PTH chronic kidney disease –> decreased 1,25 dihydrovitD (calcitriol) –> increased PTH

A researcher wants to design an experiment to study whether there is an increase in cancer in patients with exposure to 50 ug/ml in drinking water compared with a persons who has less than 5ug/l. Which is the most efficient experimental design?

Case Control Study

A 71-year-old man – sudden onset of dysarthria and dysphagia, dizziness. Exam shows markedly decreased pain sensation on the right side of the face and complete loss of pain sensation of the left side of the body. Occlusion in which labeled vessels?

PICA

Image: A 71-year-old man - sudden onset of dysarthria and dysphagia, dizziness. Exam shows markedly decreased pain sensation on the right side of the face and complete loss of pain sensation of the left side of the body. Occlusion in which labeled vessels?

A 4-year-old boy – urine is red. 15 days ago – he had a sore throat, fever, and cough. Lab show: URINE color brown, RBC Casts, RBCs 30-50, Protein 2. Most likely diagnosis?

Proliferative Glomerulonephritis
Group A Strep – URI
RBC casts===> glomerulonephritis
PSGN can progressive to a proliferative glomerulonephritis mechanism

A 3-year-old boy – short stature and normal size trunk, large head with prominent forehead and nasal bridge and exaggerated lordosis. Cause of the findings is a genetic abnormality in which?

Endochondral Ossification
Achondroplasia- autosomal dominant mutation in Fibroblast Growth Factor Receptor 3.

Image: A 3-year-old boy - short stature and normal size trunk, large head with prominent forehead and nasal bridge and exaggerated lordosis. Cause of the findings is a genetic abnormality in which?

A 4-year-old boy – 1-cm – round midline neck mass just inferior to hyoid bone. Mass is most likely derived from which structures?

Tongue
The thyroglossal tract arises from the foramen cecum at the junction of the anterior two-thirds and posterior one-third of the tongue

A public health consultant – contacted for recommendations about primary prevention techniques for population of women 30 to 40 years old. Which is the most appropriate primary preventive recommendation for this group?

Regular Exercise
Primary prevention is an active step taken to head off or mitigate a disease (i.e vaccination and lifestyle modification

Seconday prevention is early detection of a disease, promoting early intervention. HIV testing every 6 months, or or annual mammography)

35-year-old African American Man – read pamphlet about importance of screening for skin cancer – likes to sail and does not burn – PE shows no abnormalities – increased risk for melanoma at which of the following locations?

Palms

68-year-old man concerned about change in sexual performance. He started dating s 40-year-old woman – My orgasms are shorter and less intense. Symmetric enlargement of prostate. Which is likely cause of patient dysfunction?

Normal Aging
Shorter, less intense orgasms, as well as increased time needed between sex could be related to a slight drop in testosterone with age

The breakdown of dipeptides and tripeptides to free amino acids takes place in which area of the gastrointestinal tract?

Intestinal Mucosa
absorption of nutrients – dipeptides and tripeptides are cotransported with hydrogen into the Intestinal mucosal cells while single AAs are cotransported with Na+.

A 68 has a loss of pain and temperature sensations on right side of the face and left side of the body, paralysis of the vocal cord on the right, and absence of the gag reflex on the right. Which region of brainstem involved?

Right Dorsolateral Medulla
Pain and temperature sensation of the body is part of the spinothalamic tract,
The loss of gag reflex and paralysis of the vocal cords imply impairment of cranial nerves IX and X, both of which localize to the medulla
Midbrain 1-4, Pons 5-8, Medulla 9-12

A 57 year old man – hoarsness and difficulty swallowing for 3 days – He is unable to elevate the right side of the palate. Which cranial nerve is most likely damaged? (IMAGE SHOWN

Area Labeled F – Vagus
CN 1,2,3,4 – Midbrain CN 5,6,7,8 – Pons CN 9,10,11,12 – Medulla
Inability to elevate the palate suggests damage of the vagus nerve

Image: A 57 year old man - hoarsness and difficulty swallowing for 3 days - He is unable to elevate the right side of the palate. Which cranial nerve is most likely damaged? (IMAGE SHOWN

52-year-old man massive MI. TX with oxygen, beta blockers, aspirin, and reperfusion therapy. 18 days later he develops ventricular fibrillation and dies. A photomicrograph of cardiac tissue from the site in site is shown. Which describes the appearance of the heart?

Granulation Tissue
highly vascular – characteristic of granulation tissue. Scar tissue formation will be closer to 1 month, plus you will see lots of fibrosis on histology.

Image: 52-year-old man massive MI. TX with oxygen, beta blockers, aspirin, and reperfusion therapy. 18 days later he develops ventricular fibrillation and dies. A photomicrograph of cardiac tissue from the site in site is shown. Which describes the appearance of the heart?

A 35-year-old woman – difficulty urinating 2 days post-partum. She has a long history of asthma. Which of the following is both and indication and a contraindication for bethanechol in this patient?

Parasympathomimetic Stimulation
Bethanechol agonist on M3 receptors which can treat her urinary problems but it might exacerbate her asthma symptoms since there are M3 receptors on the lungs.

17 year old brought in by ambulance after head injury – in coma – PE show multiple bleeding lacerations over the head. Emergent craniotomy is indicated, but physician unable to contact parents. A decision is made to proceed with operation without permission – the decision is most consistent with which ethical principle?

Beneficence
health care providers have a duty to be of a benefit to the patient and should take positive steps to prevent and to remove harm from the patient.

Consent for minors (FA2020 pg 265): Consent should be obtained from parents, except for Emergency Medicine.

A 35-year-old man – small cell carcinoma has systemic hypertension and hypokalemia. The most likely cause of these findings is ectopic secretion of the following hormones?

ACTH
ACTH increases cortisol → hypertension (alpha-1 upregulation & cortisol can bind to aldosterone receptors at high concentrations)
ACTH increases aldosterone → hypertension + hypokalemia (K+ dumped in collecting duct)

A 35-year-old woman – undergoes left oophorectomy because of a 5 x5 cm ovarian mass. Which structure at greatest risk when dividing the suspensory ligament?

Ureter
suspensory ligament has ovarian vessels –> ovarian artery + ureter can be damaged in oophorectomy

cardinal ligament has uterine vessels –> uterine artery + ureter can be damaged in hysterectomy

Image: A 35-year-old woman - undergoes left oophorectomy because of a 5 x5 cm ovarian mass. Which structure at greatest risk when dividing the suspensory ligament?

. 40-year-old man – horseback riding – develops swollen mass in left thigh. Mass becomes circumscribed and firm. DX?

Myositis Ossificans
b. bone formation in muscle
c. Condition where bone tissue forms inside muscle or other soft tissue after an injury. It tends to develop in young adults and athletes who are more likely to experience traumatic injuries

=In which of the following stages of the cell cycle are mitotic cyclins synthesized?

G2
b. G1 checkpoint: Cyclin D/CDK4 complex with p53 and rb G2 checkpoint: Cyclin B and CDK 1

G1 is associated with higher numbers D and 4
G2 is associated with B and 1

Image: =In which of the following stages of the cell cycle are mitotic cyclins synthesized?

59-year-old with gout – repeated episodes of gouty arthritis unresponsive to preventative therapy. Which increase uric acid excretion in this man?

Probenecid
Probenecid makes you Pee
Colchicine clenches your microtubules
Probenecid and high-dose salicylates inhibit reabsorption of uric acid in proximal convoluted tubule

40-year-old man – horseback riding – develops swollen mass in left thigh. Mass becomes circumscribed and firm. DX?

Myositis Ossificans
bone formation in muscle

Image: 40-year-old man - horseback riding - develops swollen mass in left thigh. Mass becomes circumscribed and firm. DX?

A cancer patient- TX with severe high dose chemotherapy – has severe bone marrow suppression. Which cytokine most beneficial to stimulating bone marrow?

Granulocyte Colony Stimulating factor
Chemo. patient with bone marrow suppression > give G-CSF.

An 80-year-old man – 2-week history of severe, persistent urinary tract infection. An 80 mg dose of Drug X is administered intravenously after the infusion is complete. 30 minutes after the infusion his serum concentration is 4ug/ml. Assuming half-life of 3-minutes and elimination half-life of 24, the volume of distribution of this drug in L the peripheral compartment is closest to which of the following?

20
Vd = [drug administered] ÷ [plasma drug concentration]
First convert it all to g/L because this is how the answer will be:
administered: 80 mg = 0.08 g plasma concentration: 4 ug/ml = 0.004 g/L,
Vd = 0.08 grams ÷ 0.004 g/L = 20 L

A 55-year-old man – diagnosed with coronary artery disease. Begins treatment with aspirin to prevent thrombus. Which effects of aspirin on platelet function is most likely to decrease this patient risk for thrombus?

Decreased Adherence
Aspirin inhibits platelet aggregation and produces a mild bleeding defect by inhibiting cyclooxygenase, a platelet enzyme that is required for TXA2 synthesis.”
closest thing to decreased aggregation that still made sense with aspirin’s mechanism of action

. A 3-month old boy – yellow eyes and skin and weakness since birth. PE shows jaundice, large fontanels, a flat midfacial area, hypotonia, and hepatomegaly.

Serum Studies:
Total Bilirubin Increased
AST Increased
ALT Increased
Very-long Chain Fatty Acids Increased
A liver biopsy shows foamy, lipid-filled hepatocytes, necrosis, and absence of specific organelle. The absent organelle is most likely?

Peroxisomes
Peroxisome = Membrane-enclosed organelle involved in β-oxidation of very-long-chain fatty acids (VLCFA) (strictly peroxisomal process).
Zellweger syndrome

Image: . A 3-month old boy - yellow eyes and skin and weakness since birth. PE shows jaundice, large fontanels, a flat midfacial area, hypotonia, and hepatomegaly.

Serum Studies:
Total Bilirubin Increased
AST Increased
ALT Increased
Very-long Chain Fatty Acids Increased
A liver biopsy shows foamy, lipid-filled hepatocytes, necrosis, and absence of specific organelle. The absent organelle is most likely?

In a culture of motile spore-forming bacteria, which inhibits bacterial growth by causing double stranded breaks in DNA?

Irradiation with X-Rays
more energy in a light wave the more likely it is to cause cellular (DNA) damage.
Ultraviolet has less energy than X-rays (goes through your flesh)
X-Ray > UVA (also able to cause double strand break) > UVB (most likely single strand break – repaired)

80-year-old – decreased frequency of bowl movements. (1 or 2 bowel movements weekly). No benefit from fiber intake. She manually supports the posterior vaginal wall in order to defecate. Which is cause of patient’s symptoms?

Damage to rectovaginal Septum
A rectocele is a herniation (bulge) of the front wall of the rectum into the back wall of the vagina. The tissue between the rectum and the vagina is known as the rectovaginal septum

Image: 80-year-old - decreased frequency of bowl movements. (1 or 2 bowel movements weekly). No benefit from fiber intake. She manually supports the posterior vaginal wall in order to defecate. Which is cause of patient's symptoms?

A 65-year-old woman – T2DM – follow-up exam – PE shows no abnormalities. BP 135/80. Abdominal ultrasound shows decreased size of both kidneys. MR aortography shows bilateral proximal renal artery stenosis. DX?

Atherosclerosis
MOST common cause of renal artery stenosis…with fibromuscular dysplasia being the SECOND most common cause
Atherosclerotic plaques—proximal 1/3 of renal artery, usually in older males, smokers.
Fibromuscular dysplasia—distal 2/3 of renal artery or segmental branches, usually young or middle-aged females

Image: A 65-year-old woman - T2DM - follow-up exam - PE shows no abnormalities. BP 135/80. Abdominal ultrasound shows decreased size of both kidneys. MR aortography shows bilateral proximal renal artery stenosis. DX?

A 65-year-old man – comes to physician because of difficulty sleeping for the past month. He has impaired concentration and decreased energy. He has recurrent thoughts of death since having MI 5 months ago. Which drug to add to regime?

Paroxetine
major depressive disorder: loss of interest/anhedonia (need to have this or depressed mood),sleep problems, weight changes, decreased energy, thoughts of death. Meets criteria because > 2 weeks timeframe. SSRIs are first-line; paroxetine i

75-year-old man with a 10-year history of progressive renal failure – Lab studies show a serum urea nitrogen concentration of 40 mg/dl and serum concentration of 3mg/dl. Ultrasound shows Hydronephrotic and a Dilated Ureter. Most likely cause of patient’s renal failure is an increase in which?

Hydrostatic pressure in Bowman Space
the Bowman’s capsule space exerts hydrostatic pressure of its own that pushes against the glomerulus. Increased Bowman’s capsule hydrostatic pressure will decrease GFR, while decreased Bowman’s capsule hydrostatic pressure will increase GFR.

3-year boy with AIDS develops giant cell pneumonia 3 months after exposure to an unimmunized cousin who had rash, conjunctivitis, rhinitis, and kolpik spots. The virus responsible for the pneumonia has which of the following types of genomes?

Negative Stranded RNA
Measles = Negative ssRNA virus

. In a study of drug action on neoplastic cells in culture, drug X markedly inhibits cell replication. A microscopic view of a typical cell incubated with drug X is shown. Drug X is most likely to be which

Vincristine
chemotherapeutic drug that inhibit B-tubulin and polymerization into microtubules. The cell in the picture is stuck in anaphase, with microtubules attached to its chromosomes,

Image: . In a study of drug action on neoplastic cells in culture, drug X markedly inhibits cell replication. A microscopic view of a typical cell incubated with drug X is shown. Drug X is most likely to be which

29-year-old jaw pain- 1 hour history of bilateral jaw pain that began after he tried to bite into a cheeseburger. PE shows extensive drooling and an inability to elevate the mandible. Xray shows bilateral anterior dislocation of the temporomandibular joint. Reduction of the TMJ is recommended. Relaxation of which is most likely to facilitate this procedure?

Lateral Pterygoid
3 muscles close the jaw: Masseter, teMporalis, Medial pterygoid 1 muscle opens the jaw:

Lateral pterygoid loosen jaw
ALL are innervated by trigeminal nerve, V3

Mnemonic: M’s munch (close the jaw), L’s lower (loosen/relax the jaw)

17-year girl comes to the physician because she has never had a menstrual period. She is not sexually active. She is 5’11 and 110 lb. BMI is 15. Breasts development is tanner stage 5 and pubic and axillary development is Tanner Stage 1. Pelvic exam shows blind vaginal pouch – Tanner Stage 1. Which is found on further testing?

Increased Serum Testosterone Concentration
Androgen insensitivity syndrome
Defect in androgen receptor resulting in normal-appearing female (46,XY DSD); female external genitalia with scant axillary and pubic hair, rudimentary vagina; uterus and fallopian tubes absent due to persistence of anti-Müllerian hormone from testes

A 70-year-old man – 1 year history of shortness of breath and nonproductive cough – unable to tolerate mild activity. PE shows clubbing of the fingers. Inspiratory crackles are heard at both lungs. A CT scan shows patchy subpleural reticular opacities. Which pulmonary functions will show a result greater than predicted?

Alveolar-Arterial PO2 difference
pulmonary fibrosis = restrictive (not obstructive)-type disease.
no occupational exposure = idiopathic pulmonary fibrosis

Image: A 70-year-old man - 1 year history of shortness of breath and nonproductive cough - unable to tolerate mild activity. PE shows clubbing of the fingers. Inspiratory crackles are heard at both lungs. A CT scan shows patchy subpleural reticular opacities. Which pulmonary functions will show a result greater than predicted?

A 50-year-old woman with HIV – follow-up exam. Receiving antiretroviral therapy including zidovudine and nelfinavir. She is also receiving pentamidine for pneumocystis and azithromycin for Myocbacterium Avium Complex. Drug induced bone marrow suppression is suspected. Cause?

Zidovudine
NRTIs are the main HIV therapy drug that can cause bone marrow suppression (not as common with NNRTIs). This class includes zidovudine, didanosine, emtricitabine, lamivudine, stavudine, abacavir. Zidovudine is most known for this side effect.

47-year-old – recurrent joint pain – the onset is abrupt involving the ankles and knee. PE shows yellow white nodules at the tips of several fingers. A photomicrograph of the joint fluid is shown. The crystals (YIPA) are most like composed of which?

Monosodium Urate
Gout [Monosodium urate] Crystals are needle shaped and ⊝ birefringent under polarized light (yellow under parallel light, blue under perpendicular light

Image: 47-year-old - recurrent joint pain - the onset is abrupt involving the ankles and knee. PE shows yellow white nodules at the tips of several fingers. A photomicrograph of the joint fluid is shown. The crystals (YIPA) are most like composed of which?

50-year-old – admitted to hospital for management of an acute exacerbation of heart failure. Multiple treatment for similar episodes (8x times). Bilateral crackles are heard on pulmonary exam. There is pitting edema of the lower extremities. She is placed on restricted 2L of liters. Her fluid balance is closely monitored. Nurse find pt drinking in bathroom drinking directly from faucet and holding a pitcher of water. Explanation for patient’s lack of clinical improvement?

Factitious Disorder
trying to fake a disease or symptoms -> they’re thirsty and selfish with no ability to regulate their own behavior

58-year-old – 3 day history of malaise, increased urinary frequency, dribbling of urine, and feeling of incomplete bladder emptying, deep, and dull pelvic pain. Digital rectal exam shows an enlarged, tender prostate. Which infectious agents cause these findings?

Escherichia Coli
Most common cause of acute prostatitis in older men is E. Coli

A 35 year old man- disoriented and hallucinating. He has a 20 year history of alcoholism. On admission, he has seizures. His blood pressure is 180/100. Serum potassium is 2.5 mEq/L, and urine potassium is 40 mEq/L. Alcohol withdrawal is suspected. Which of the following cause the hypokalemia?

Catecholamine -mediated intracellular shifts of K+
mediated intracellular shifts of K+
Catecholamine : 1. increases insulin release –> glu and K will enter cells. Causing hypokalemia in the plasma 2. Can trigger the beta 1 receptor causing the release of Renin—> Angiotensin 2 (can be contributing / causing an increase in BP)–> aldosterone –> you waste K in the urine.

A 73-year-old woman – has easy fatigability for 2 years. She had an ileal resection of Crohn disease. She has pale oral mucosa. A peripheral blood smear is shown. Most likely mechanism of this disorder?

Failure of conversion of N5- methyl tetrahydrofolate to tetrahydrofolate
“Structural abnormalities of the terminal ileum, such as Crohn disease and surgical resection, can cause decreased absorption of vitamin B12.” – First Aid General Principle

Image: A 73-year-old woman - has easy fatigability for 2 years. She had an ileal resection of Crohn disease. She has pale oral mucosa. A peripheral blood smear is shown. Most likely mechanism of this disorder?

A 27-year-old woman -underwent a cesarean delivery. PE shows a firm subcutaneous nodule adjacent to surgical incision. Microscopic exam shows fibrous connective tissue, macrophages, multinucleated giant cells, fibroblasts, a few lymphocytes. Which substances promotes fibroblast migration and proliferation most likely led the development of this lesion?

Transforming Growth Factor -B
TGF-Beta is an important fibroblast growth factor. Pathoma Ch2, page 20.
hypertropic scar or keloid. Both arise due to over-expression of TGF-beta

A 73-year-old woman – has easy fatigability for 2 years. She had an ileal resection of Crohn disease. She has pale oral mucosa. A peripheral blood smear is shown. Most likely mechanism of this disorder

Failure of conversion of N5- methyl tetrahydrofolate to tetrahydrofolate
b. “Structural abnormalities of the terminal ileum, such as Crohn disease and surgical resection, can cause decreased absorption of vitamin B12.” – First Aid General Principle

Image: A 73-year-old woman - has easy fatigability for 2 years. She had an ileal resection of Crohn disease. She has pale oral mucosa. A peripheral blood smear is shown. Most likely mechanism of this disorder

A 25 year old man – 3 day history of abdominal cramps and diarrhea. He appears anxious. PE shows piloerection and abdominal tenderness. Neuro exam normal. Most likely cause of patient’s condition is withdrawal from which substances?

Heroin
opioid withdrawal sxs (= sweating, dilated pupils, piloerection (“cold turkey”), rhinorrhea, lacrimation, yawning, nausea, stomach cramps, diarrhea (“flu-like” symptoms)”An obese 57-year-old man – smoked 2.5 packs of cigarettes daily for 40 years. PE shows mild dyspnea. A CXR shows a mass in one lobe of right lung. Exam shows squamous metaplasia of the bronchial mucosa. Which best describes changes in bronchial mucosa?

55-year-old woman – 3 month history of difficulty using her hands and a 3 week history of muscle cramps. She has 20 lb weight loss Neuro exam shows tongue fasciculations and lower extremity weakness and atrophy. Sensory exam is normal. A lesion at which of the site most likely cause these findings?

Lower Motoneurons
“LMN deficits… dysarthria, dysphagia, asymmetric limb weakness, fasciculations, atrophy
=UMN deficits… pseudobulbar palsy (i.e. dysarthria, dysphagia, emotional lability, spastic gait, clonus])”

Image: 55-year-old woman - 3 month history of difficulty using her hands and a 3 week history of muscle cramps. She has 20 lb weight loss Neuro exam shows tongue fasciculations and lower extremity weakness and atrophy. Sensory exam is normal. A lesion at which of the site most likely cause these findings?

An obese 57-year-old man – smoked 2.5 packs of cigarettes daily for 40 years. PE shows mild dyspnea. A CXR shows a mass in one lobe of right lung. Exam shows squamous metaplasia of the bronchial mucosa. Which best describes changes in bronchial mucosa?

Normal Ciliated columnar replace by normal Squamous epithelium
Metaplasia – A reversible adaptive response in which there is “Reprogramming of stem cellsreplacement of one cell type by another that can adapt to a new stress.”
Squamous metaplasia (SQM) is a pre-neoplastic change of the bronchial epithelium observed in the lungs in response to toxic injury induced by cigarette smoke

Image: An obese 57-year-old man - smoked 2.5 packs of cigarettes daily for 40 years. PE shows mild dyspnea. A CXR shows a mass in one lobe of right lung. Exam shows squamous metaplasia of the bronchial mucosa. Which best describes changes in bronchial mucosa?

A 56-year-old – pneumonia found to have advanced lung cancer. Pt refuses palliative radiation therapy and wants to be discharged. He is alone. He says he will return in 6 months after he makes money from selling his invention to cure arthritis. Most appropriate next step?

Determine whether the patient has decision making capacity
Does he understand risks and benefits
Autonomy is the most important ethics principle .

In this situation, a patient with advanced disease unlikely to be cured is refusing treatment, which is his right under the principle of autonomy. However, his comments about “returning in 6 months after curing arthritis” are questionable, and warrant determining if he has decision making capacity.

Image: A 56-year-old - pneumonia found to have advanced lung cancer. Pt refuses palliative radiation therapy and wants to be discharged. He is alone. He says he will return in 6 months after he makes money from selling his invention to cure arthritis. Most appropriate next step?

A 73-year-old man incurable malignant neoplasm of the lung, and his condition is slowly deteriorating. He request the respirator to be removed. After discussing with his children, physician decides to remove the respirator. Which describes his action?

Both legal and ethical
most important ethical principle that supersedes all others is autonomy. From an ethical standpoint, this patient has the right to refuse further treatment as he is mentally competent, in this case in the form of having the respiratory removed. From a legal standpoint, the physician is allowed to discontinue treatment for a patient if that is what the patient wants.

28-year-old with Chronic Hep. C – fever and progressive shortness of breath during the past 2 days. His temperature is 102F. PE shows wasting and intercostal retractions. Crackles heard on both lung fields. CXR shows diffuse hazy infiltrates. Leukocyte count is 2000. Lymphocyes are 5% To explain the cause of the illness, the patient should be tested for which chronic viral infections?

HIV
AIDS diagnosis: ≤ 200 CD4+ cells/mm3 (normal: 500-1500 cells/mm3). This patient has 5% lymphocytes out of 2000 overall = 100 cells = AIDS.

An investigator- studying new drug for patients undergoing adjuvant radiation therapy. The drug is administered prior to irradiation to minimize localized tissue damage at the irradiated site. The drug most likely inhibits which effects of external beam radiation?

Free Radical Formation
Radiation- damage by generating hydroxyl free radicals
two main of radiation = ionizing and nonionizing.
ionizing = radiotherapy – free radical formation
non-iodizing = is from the sun- DNA damage by formation of pyrimidine dimers

3-week-old female – increased concentration pf immunoreactive trypsin. Cystic fibrosis is suspected. At 4 months, her sweat chloride is greater than 60 mmol/L. Analysis of the 70 most common CF transmembrane gene mutations are done. Results show a mutation in one allele. Which explains the findings?

The patient has another mutation not included in the previous analysis
you need a mutation in 2 alleles to get CF [since it is autosomal recessive]
If you still have 1 functional copy of the CFTR gene, you can still make the CFTR protein (the chloride channel/transporter), hence your body won’t have any issu

A 13-year-old boy- chest swelling the began 6 months ago. Pt reluctant to take off his shirt and embarrassed. PE shows bilateral 1-cm, mildly tender, rubbery nodules under the areolar region. Sexual development is Tanner stage 3. Most appropriate for physician to state which of the following?

This will typically resolve within the next 12 to 18 months
Gynecomastia due to puberty (excess testosterone → estrogen) This goesaway naturally (apparently in 12 to 18 months)

A 35-year-old man – quadriplegia – develops UTI. A culture of urine grows an organism that is susceptible to aminoglycosides. Gentamicin is administered. If nephrotoxicity occurs in the patient, which of the labeled sites in the drawing of the nephron shown?

PCT
Aminoglycosides are nephrotoxic; nephrotoxic chemicals/drugs cause acute tubular necrosis (ATN), characterized by damage to the PCT
Aminoglycosides are nephrotoxic; nephrotoxic chemicals/drugs cause acute tubular necrosis (ATN), characterized by damage to the PCT

Image: A 35-year-old man - quadriplegia - develops UTI. A culture of urine grows an organism that is susceptible to aminoglycosides. Gentamicin is administered. If nephrotoxicity occurs in the patient, which of the labeled sites in the drawing of the nephron shown?

A 20-year-old woman (college student) with asthma – exacerbations of symptoms over last 3 months. Classes are stressful. I have a poodle, My roommate smokes. Advise the patient to do which of the following?

Ask the roommate not to smoke in the apartment
Symptoms started 3 months ago and she moved in with a roommate who smokes indoors 3 months ago.

A Cohort study is done to evaluate the association between use of video display terminals (VDTS) by women and the risk of congenital heart disease in their offspring. The relative risk ratio for congenital heart disease in newborns born to women who work 6 or more hours. VDT 1.1 (95% confidence interval: 0.8 -14). – which is of the following is the p vale calculated for a chi square test?

05 < p < 1.9
P value refers to probability of type 1 error. When the 95% confidence interval includes
the null value
Null (H 0 ) Hypothesis = Hypothesis of no difference or relationship (eg, there is no association between the disease and the risk factor in the population).
If the 95% CI for odds ratio or relative risk includes 1, H 0 is not rejected.
Since the CI here includes 1, it means that its not significant and therefore p value should not be < 0.5. Therefore p = 0.5-1.

Drug X is given to a 25-year-old normal patient. Drug cause increase in resting heart from 62 to 74 min. Prior to administration of the drug heart rate increased to 150/min with exercise. With the drug, heart rate increased – 98 min (less with drug). Mechanism of action of the drug X most likely involves?

Partial Agonist at B-adrenergic Receptors
In the presence of agonist
partial agonist act like an antagonist

A 34-year-old brought to ER – fall with outstretched hand- Examination of UE shows hard mass proximal to wrist joint anteriorly. X-ray shown of the following bones is most likely dislocated in this patient?

Lunate
FOOSH – Scaphoid most common fracture,
Lunate most common dislocation
scaphoid is most common one to be fractured, lunate is most common to be dislocated.
Straight line to Pinky, Here Comes The Thumb
(Scaphoid, Lunate, Triquetrum, Pisiform)
(Hamate, Capitate, Trapezoid, Trapezium)

A 22-year-old man – involved in a motorcycle collision. PE Shows dysmetria on the right. Muscle strength is normal. Which labeled structures in the photograph – most likely site of injury?

(Area E) cerebellum
Dysmetria ( also, dysdiadokinesia and intention tremor) is lateral cerebellum. (E).
D – Truncal Ataxia is a problem with the central cerebellum

Image: A 22-year-old man - involved in a motorcycle collision. PE Shows dysmetria on the right. Muscle strength is normal. Which labeled structures in the photograph - most likely site of injury?

37-year-old – farmworker – 12 hour severe pain in abdomen and legs and spasms in jaw. PE shows spasms of the masseter and abdominal musculature. Loud noise triggers spasms causing pt to be intubated. Administration of antitoxin prevents further symptoms that involve bidning of a toxin to which?

Synaptobrevin
tetatnus –
blocks release of GABA and Glycine
Renshaw cells SNARE proteins

A 55-year-old man with alcoholism is brought to emergency department 30 minutes after consuming a bottle of methanol. Physician recommends patient be treated with ethanol. To increase Km of hepatic alcohol dehydrogenase for methanol in this patient the ethanol must act as which?

Competitive Inhibitor (Increase KM for substrate).
b. Ethanol or Fomepizole prevents binding of methanol to active site
instead of using fomepizole they just gonna get him real drunk

Image: A 55-year-old man with alcoholism is brought to emergency department 30 minutes after consuming a bottle of methanol. Physician recommends patient be treated with ethanol. To increase Km of hepatic alcohol dehydrogenase for methanol in this patient the ethanol must act as which?

A 7-year-old girl – brought to the emergency department because of a 1-hour history of excessive sleepiness and slurring of speech. Mother states girl was well and had eaten 2 hours ago. Blood glucose is 42 mg/dl. IV dextrose is administered – 5 minutes later her symptoms resolve. PE shows no abnormalities. C peptide 0.5ng/ml (N=0.8-3.1), Insulin 32 – N= 5-20. DX?

Factitious Disorder Imposed on another
(No history of T1DM),
Exogenous Insulin C Peptide is low and Insulin High

A 54-year-old, MI – progressive hypotensive. Which physiologic events is most likely to cause swelling of parenchymal cells in his vital organs within minutes??

Decrease in Intracellular ATP Concentrations
Hypoxia impairs oxidative phosphorylation—less ATP—Less Na / K + (Na builds up causing swelling)

Theme of pathology: hypoxia impairs oxidative phosphorylation –> less ATP –> less Na-K pump activity so sodium builds up in the cell, causing swelling.

Calcium Builds up causing Lactic Acidosis, anaerobic glycolysis, lactic acid production

A 35 year left pelvic fracture in MVA accident. A 3-week period of bed rest for this patient- most likely to cause which physiologic change?

Decrease Blood Volume
Pt is supine, increase preload back to heart
ANP secreted due to RA Stretch, increase diuresis, reduction blood volume

33-year-old women comes to the physician because of a 2-month history of easy fatigability and dark urine in the mornings. No fever or weight loss. No meds. Vitals normal. Urine blood and protein.

Direct Antiglobulin (Coombs) test is negative. The result of the acidified serum test is positive. Which describes the underlying cellular cause of this patients condition?

Defect cell membrane anchor protein
Paroxysmal Nocturnal Hemaglobinuria – complement activity against RBC – impaired GPI anchor – associated with aplastic anemia
Paroxysmal Nocturnal Hemaglobinuria

38-year-old – blood tinged discharge from right breast for 3 months. Menses have occurred at regular 28-day intervals. She takes no medications. PE shows no breast masses and no palpable axillary adenopathy. Bloody discharge from the upper outer corner of the right nipple. Mammography shows no abnormalities. Cause of discharge?

Intraductal Papilloma
most common cause of blood tinged discharge, no changes on mammography or radiology (benign tumor)

Image: 38-year-old - blood tinged discharge from right breast for 3 months. Menses have occurred at regular 28-day intervals. She takes no medications. PE shows no breast masses and no palpable axillary adenopathy. Bloody discharge from the upper outer corner of the right nipple. Mammography shows no abnormalities. Cause of discharge?

A 32-year-old woman – diagnosed with AIDS is at greatest risk for developing which neoplasms?

Epstein-Barr Virus Induced Brain Lymphoma
CNS lymphoma AIDS defining illness

A 62-year-old man – 2-month history severe pain of his left leg began shortly after the leg was amputated below the knee. Throbbing, aching, and shooting pain Located in distal portion of absent extremity. Cause of patient’s symptoms

Phantom Limb Pain
(distal portion of absent extremity)
chronic pre-amputation pain, post-operative surgical pain and psychological distress.
d. Phantom pains – crushing, toes twisting, hot iron, burning, tingling, cramping, shocking, shooting, “pins & needles” Tends to localize to more distal phantom structures (e.g. fingers and toes)

Image: A 62-year-old man - 2-month history severe pain of his left leg began shortly after the leg was amputated below the knee. Throbbing, aching, and shooting pain Located in distal portion of absent extremity. Cause of patient's symptoms

A 59-year-old woman – 10-year history of progressive right sided hearing loss. An MRI shows a large Cerebellopontine Angle mass that has compressed Vestibulocochlear nerve. The mass arose from which of the following cell types?

Neural Crest Cells
Cerebellopontine angle mass = Vestibular schwannoma (AKA acoustic neuroma). Derived from Schwann cells, which are of neural crest origin.
c. Schwannoma – derived from neural crest
COPS- CNS Oligodendroctes, PNS Schwan Cells

Image: A 59-year-old woman - 10-year history of progressive right sided hearing loss. An MRI shows a large Cerebellopontine Angle mass that has compressed Vestibulocochlear nerve. The mass arose from which of the following cell types?

A 27-year-old man – he and wife not able to conceive a child. Poor libido and unable to maintain an erection. Receiving thyroid replacement hormone and corticosteroid therapy since removal of pituitary adenoma. Testosterone is 0.05 nmol/L (N=10-35). Azoospermia. What treatment to restore patients fertility

Injection of gonadotropins.
Removal pituitary removed- loss of FSH and LH
need FSH and LH to produce sperm
FSH stimulates the sertoli cells (line seminiferous tubules) spermatogonia produce spermatocytes.
Testosterone – Leydig cells stimulated by LH

Serum cholesterol concentrations are measure as part of a community study. The means and standard deviations are given. Which is the probability that a woman between the age of 50 and 54 has a serum cholesterol concentration greater than 296 mg/Dl?

Age Years 50-54 246 + or – 50

16%
Concentration greater than 296
Age Years 50-54 246 + or – 50 =
296 = 1 STD above the mean
1 SD = 68% of the population
2 SD= 95% of the population (95% -68%) = 27/2 = 13.5% )
3 SD= 99.7% of the population 5%/2 = 2.5

Image: Serum cholesterol concentrations are measure as part of a community study. The means and standard deviations are given. Which is the probability that a woman between the age of 50 and 54 has a serum cholesterol concentration greater than 296 mg/Dl?

Age Years 50-54 246 + or - 50

A 43-year-old woman – 1 week history of abdominal pain, nausea, vomiting, itching, fatigue. PE shows scleral icterus and RUQ tenderness. No rash. Ultrasound shows a large stone in common bile duct. Serum total bilirubin increased. Serum concentration of which of the following is most likely increased?

Alkaline Phosphatase
(concentrated in liver, bile duct, kidney, intestinal mucosa)
gall stone in common bile duct- Cholestasis

A group of physician submits a report to a medical journal – 3 patients with idiopathic pulmonary fibrosis who developed hepatoxicity following treatment with new drug. In report, Physicians state they are unaware of any previous adverse effects. What is the study design?

Case Series
history and treatment of small group without randomization or controls
in this study – researchers present the history and treatment of a small group of similar patients, without describing any sorting into groups or randomization.

A 3 year old girl – history of recurrent infections. In vitro, neutrophils form patient can’t kill Staphylococcus Aureus. Defect involving which enzymes?

NADPH Oxidase
Chronic Granulomatous Disease –
bad when dealing with catalase positive organisms (like Staph
STAPH is catalase + breaks down ROS

A 37-year-old woman with HIV infection – 6 hours of chest pain and abdominal pain and shortness of breath with exertion. Receiving antiretroviral therapy. Lab show: Lactate 90, AST and ALT elevated. A drug from which class caused findings?

Nucleoside Reverse Transcriptase Inhibitor’
NRTIs (specifically the nucleosides) cause lactic acidosis
causes lactic acidosis, bone marrow suppression
NRTI – Bone marrow supression, Lactic Acidosis, Anaemia
NNRTI – Hepatotoxicity, rash
Integrase inhibitor (tegras) – Myopathy (causing Increased Creatinine Kinase)
Protease Inhibitors (navir) – Lipodystrophy, Hyperlycemia, GI intolerance (think of hormonal effecs)

A man touches hot stove – 20 minutes later a blister develops at the site. Light and electron microscopy of the inflamed tissue is most likely to show?

Interendothelial Gaps in Venules
b. pathoma “vascular permeability occurs at post capillary venules.” Burns disintegrate post capillary venules
c. inflammatory response such as in burns, anaphylaxis or sepsis, intercellular contacts disintegrate in post-capillary venules leading to intercellular gap formation.

15-year-old cystic fibrosis – mutation in CFTR does not prevent synthesis of CFTR protein but does prevent CFTR from folding properly. The improperly folded CFTR protein will accumulate in which of the following cellular compartments?

Endoplasmic Reticulum
site of protein folding
protein folding happens at the RER and the stem says the protein doesn’t fold properly.
CF mutation is a misfolded protein and the protein is retained in the RER and not transported to the cell membrane

3-month-old brought to ED because of shortness of breath for 3 hours. He has upper respiratory infection for the past 3 days. Pe shows lethargy and hepatomegaly. Labs shows hypoglycemia, lactic acidemia, ketonemia, and metabolic acidosis. Admin of glycerol or fructose do not increase glucose concentrations. Glucose goes up after IV admin of galactose. A defect in which of the following liver metabolic pathwsys is most likely cause of findings?

Gluconeogenesis
b. fasting hypoglycemia corrected with sugar, enters gluconeogenesis pathway – fructose bisphosphate deficiency
c. fructose bisphosphatase deficiency =gluconeogenesis pathway below fructose bisphophatase.
d. Galactose on the other hand enters above it.

Image: 3-month-old brought to ED because of shortness of breath for 3 hours. He has upper respiratory infection for the past 3 days. Pe shows lethargy and hepatomegaly. Labs shows hypoglycemia, lactic acidemia, ketonemia, and metabolic acidosis. Admin of glycerol or fructose do not increase glucose concentrations. Glucose goes up after IV admin of galactose. A defect in which of the following liver metabolic pathwsys is most likely cause of findings?

4-year-old history of multiple bone fractures and poor wound healing- Photograph (Blue sclera). Which of the components of wound healing is most likely to be affected as a direct result of her underlying diseases?

Scar Formation
Osteogenesis Imperfecta –
Brittle Type 1 collagen – blue sclera
Need type 1 collagen to get scars

Image: 4-year-old history of multiple bone fractures and poor wound healing- Photograph (Blue sclera). Which of the components of wound healing is most likely to be affected as a direct result of her underlying diseases?

18-year-old – sharp chest pain and shortness of breath 1 hour after receiving a non-penetrating injury in rugby game. PE shows decreased breath sounds and increased tympany to percussion. CXR show. Pt at greatest risk of which complication?

Respiratory Acidosis
increased CO2, Low Po2 compensation with resp

Image: 18-year-old - sharp chest pain and shortness of breath 1 hour after receiving a non-penetrating injury in rugby game. PE shows decreased breath sounds and increased tympany to percussion. CXR show. Pt at greatest risk of which complication?

Which of the following best describes fbrous proteins that form two-dimensional network for inner surface of nuclear membrane?

Lamins
b. lamin is an intermediate filament that specifically provides support to the cell nucleus.
c. Fibronectin is an extracellular matrix glycoprotein, while
d. Don’t confuse lamin with laminin (science hates us clearly);
e. laminin is like fibronectin, an ECM glycoprotein and a major component of the basal lamina of basement membranes.

17-year-old girl – significant blood loss after being injured in an automobile collision. Blood loss stimulates bone marrow to synthesize which of the following?

Aminolevulinate
blood loss leads to hemapoiesis. 1st step in heme synthesis FA p 417 ALA)
condensation of glycine and succinyl CoA into delta-aminolevulinate. This is rate-limiting step of heme synthesis.

Image: 17-year-old girl - significant blood loss after being injured in an automobile collision. Blood loss stimulates bone marrow to synthesize which of the following?

2-year-old- 12-hour history of headache, loss of appetite, and vomiting. Temp 103.8 BP is 90/50 mm Hg, PE shows nuchal rigidity. Lumbar puncture is done. CSF shows increased protein, decreased glucose, abundant neutrophils, and gram positive diplococci. Which vaccine would have prevented the patients infection?

Polysaccharide protein conjugate vaccine
b. (Conjugate vaccine given to children under 5)
c. Meningitis caused by Strep pneumoniae, the most common cause of infectious meningitis in general. The vaccine for Strep pneumo is a polysaccharide protein conjugate vaccine.

30 year old man – develops tingling around lips and mouth after consuming – puffer fish at a Japanese restaurant. Fish contains a tetrodotoxin that causes cardiac arrest in high doses. This patient’s symptoms are most likely due to the blocking action of tetrodotoxin on which ion channels?

Sodium
tetrodotoxin inhibits sodium channels, prevents depolarization of cardiac muscles
Tetrodotoxin, found in puffer fish, inhibits sodium channels. This prevents depolarization of cardiac muscle and neurons, which leads to death if consumed in high enough quantity. The symptoms are vague (nausea, diarrhea, paresthesia)

35 year old with Asthma – 1 week history of abdominal pain, diarrhea, progressive cough, and wheezing. She recently returned from a trip in papua new guinea. PE shows cutaneous larvae over the abdomen – CXR shows bilateral central alveolar infiltrates. Stool sample shows (image of parasite). 23% eosinophils- Which is most appropriate pharmacotherapy?

Thiabendazole
intestinal parasites get treated with bendazole
Strongyloides infection, as this is the intestinal parasite that shows larva on stool sample

Image: 35 year old with Asthma - 1 week history of abdominal pain, diarrhea, progressive cough, and wheezing. She recently returned from a trip in papua new guinea. PE shows cutaneous larvae over the abdomen - CXR shows bilateral central alveolar infiltrates. Stool sample shows (image of parasite). 23% eosinophils- Which is most appropriate pharmacotherapy?

35-year-old with history of asthma – dies from Histoplasmosis – exam shows lungs 1.5 times the normal weight. Extensive focal areas of fibrosis and 2-5mm nodules throughout lungs. Biopsy of nodules show which findings?

Infiltration of Lymphocytes and Monocytes
Histoplasma – act like TB and cause cavitary lesions and calcified nodules with fibrotic scarring.
=fungi are combatted by lymphocytes and macrophages, not eosinophils or neutrophils.

32-year-old – painful swelling of right cheek for 24 hours. PE shows swollen and tender area overlying the parotid gland, Sialolithiasis is suspected. The calculus is most likely present in a duct that passes through which muscle?

Buccinator
stensens duct – pierces parotid gland
duct pierces through the bucc. (you can feel it with your tongue)
secretions of the parotid gland are transported to the oral cavity by the Stensen duct. It

Image: 32-year-old - painful swelling of right cheek for 24 hours. PE shows swollen and tender area overlying the parotid gland, Sialolithiasis is suspected. The calculus is most likely present in a duct that passes through which muscle?

83-year-old brought to physician with daughter to discuss complete work-up for dementia. The patient has mild memory problems. She takes no medications. Mini mental status is 23/30. Treponema pallidum is positive. Best next step?

Discussion of the Diagnosis with the Patient Privately
(talk to her first alone to determine capacity) –
cognitive impairment could be the manifestation of neurosyphilis. In addition, the doctor should talk directly to the patient to check for sexual abuse.
d. main goal for this question is avoid discussing issues with relatives without the patient’s permission.

. 15 year old girl – 3 day history of fever, sore throat, and malaise. PE shows pharyngeal erythema, enlarged tonsils, and tender cervical lymphadenopathy. Incubation with sheep erythrocytes results in agglutination. The atypical lymphocytes are most likely to be what type?

CD8+ lymphocytes
EBV infects B Cells -causes CD8 cells become abnormal’ with Mono – monospot test sheep agglutinin – which are atypical lymphocytes
Lymphocytosis associated with infectious mononucleosis is caused by an increase in the number of circulating activated T and B lymphocytes.
The atypical lymphocytes are CD8+ T- cells that are activated to eliminate EBV infected B- lymphocytes.

37 y/o – admitted to hospital because of gradually progressive weakness, anorexia, weight loss over 6 months. BP 74/40 and skin is Hyperpigmented. Morning serum cortisol concentration is 2 ug/dl (Low Cholesterol) Which is most likely diagnosis?

Autoimmune Adrenalitis
low cortisol, which causes body wasting.

hyperpigmented since low cortisol increases ACTH secretion from the pituitary, but this shares a common precursor protein with melanocyte-stimulating hormone (MSH)
often a sign of either adrenal failure or an ACTH-secreting tumor

65-year-old – sudden onset of shortness of breath and discomfort in his chest. PE shows diaphoresis. An ECG show ST elevation in the anterior leads. Compared with healthy – which cardiopulmonary change?

Increased Systemic Vascular Resistance
Decreased Pulmonary Vascular Resistance
Increased Pulmonary Capillary Wedge Pressure
Left Atrial Pressure
cardiogenic shock – an anterior wall MI isn’t pumping.

.27-year-old works from home as software engineer- Avoids meetings, “I wish I could have relationships, but I can’t I know they hate me once they get to know me”. Which personality disorder?

Avoidant
Avoidant personality disorder is characterized by a desire for social relationships, but a fear of being rejected or feeling inadequate for others.

diaphoresis. An ECG show ST elevation in the anterior leads. Compared with healthy – which cardiopulmonary change?

Increased Systemic Vascular Resistance
Pulmonary Vascular Resistance- Decreased
Pulmonary Capillary Wedge Pressure- Increased
Left Atrial Pressure
cardiogenic shock – an anterior wall MI isn’t pumping- SVR increased. CO decreased.

Image: diaphoresis. An ECG show ST elevation in the anterior leads. Compared with healthy - which cardiopulmonary change?

28-year-old man in blood pressure cuff placed around his left arm, the cuff is inflated to occlude arterial blood flow for 2 minutes and is then removed. Blood flow in the left arm increases by 50% during the next 3 minutes and then decreases to control values – which humoral substance is most likely involved?

Adenosine
CHALK causes vasodilation = (CO2, H+, Adenosine, Lactate, K+, K-)

A 22 year old woman – follow-up exam. 5 year history of migraines. She began taking oral contraceptive 6 months ago, but stopped the contraceptive 1 month ago. The headaches have since improved. She does not what hormmonal birth control, and she asks for the most effectibve alternative. It is most appropriate for the physician to recommend which of the following contraceptive methods for this patient?

Intrauterine Device
non hormonal most effective alternative COPPER IUD

most effective treatment. Copper IUD = long-acting reversible contraception, most effective emergency contraception.

21-year-old with asthma – comes to the physician because her current medication regime is not relieving her symptoms. The physician tells the patient about a clinical trial of a new drug for asthma. The trial is a large, randomized, prospective, double blinded study of volunteers with asthma. The trial is most likely occurring at which of the following phases of drug development?

Phase 3
large number random assignment, with placebo, any Improvement

The pedigrees of patients with schizophrenia most closely resemble those of patients with which of the following?

Diabetes Mellitus, Type 1
Schizophrenia doesn’t have a clear pattern of inheritance.
only Type 1 DM doesn’t have a clear inheritance pattern

Hox genes play a role in embryogenesis through which of the following processes?

Regulation of Transcription
Homebox genes are transcription factors

A 1 month old – male newborn is brought to the physician for a routine examination. Parents have dark skin and eyes. PE shows hypopigmentation of the skin, light blonde hair, and translucent irises. The inherited disorder causes this phenotypic expression is most likely due to a defect in the metabolism of which?

Tyrosine
albinism, which is due to decreased tyrosinase activity.

If he has a problem metabolizing Phenylalanine, he would be presenting with the PKU sx like intellectual disability, musty body odor, etc., in addition to his fair complexion

Image: A 1 month old - male newborn is brought to the physician for a routine examination. Parents have dark skin and eyes. PE shows hypopigmentation of the skin, light blonde hair, and translucent irises. The inherited disorder causes this phenotypic expression is most likely due to a defect in the metabolism of which?

A 55-year-old – 2 weeks history of palpitations and anxiety. He has primary hypothyroidism and treated with Levothyroxine. Pt is taking 2x dosage of Levothyroxine for 2 months. Which are most likely set of findings?

Increased Free T4
Increased Free T3
Decreased Thyroid Iodine Uptake

the patient has been taking excess of his levothyroxine medication. Levothyroxine is the exogenous form of T4. Therefore, free T4 must be elevated. T4 is converted to T3 at most peripheral tissues, so T3 will also be elevated.

12-year-old boy – brought to the physician by his mother for a well-child exam. His mother has outgrown the clothes that she bought him 6 months ago. 50th percentile for height and weight. Pt shows growth of 2.3 in/year. PE shows mild acne. Pubic hair and testes development are Tanner stage 2. Cause of onset of physical changes in this patient?

Nocturnal Luteinizing Hormone Pulses
Gonadotropin pulses occur only during sleep, but as puberty progresses they can be detected during the day. By the end of puberty, there is little day-night difference in the amplitude and frequency of gonadotropin pulses.

Image: 12-year-old boy - brought to the physician by his mother for a well-child exam. His mother has outgrown the clothes that she bought him 6 months ago. 50th percentile for height and weight. Pt shows growth of 2.3 in/year. PE shows mild acne. Pubic hair and testes development are Tanner stage 2. Cause of onset of physical changes in this patient?

A 23-year-old primgravid woman at 22 weeks gestation is brought to a small rural hospital by her husband becase of intermittent bleeding during past week. Pt woke up in copius amounts of blood and BP 90/60 mm/Hg. Her female obstetrician is a woman who practices 2 hours away. Pt is a conservative muslin and dressed in a burka, and she doesn’t want a male physician. Only physician available is a male. Which of the following is the most appropriate action by the physician?

Ask the patient if she would allow the examination if her husband is present at all times
Patient needs medical attention immediately, which eliminates obtaining a court order, or transferring her. A nurse does not have the same training and qualifications as a physician best option among those given is to ask the patient if she will allow with her husband present.

A 43 year old woman – nonproductive cough for 3 weeks. She has 6.8 kg (15 lb) weight loss. A CXR shows 3 nodules in right lung. FNA suggest a malignant neoplasm. A photograph in the patient is shown. Which of the following is most likely diagnosis

Metastatic Carcinoma
metastasis was the best option here because there are multiple malignant neoplasms

Image: A 43 year old woman - nonproductive cough for 3 weeks. She has 6.8 kg (15 lb) weight loss. A CXR shows 3 nodules in right lung. FNA suggest a malignant neoplasm. A photograph in the patient is shown. Which of the following is most likely diagnosis

A 62-year-old man – comes to physician for a follow-up exam. One month ago, he was prescribed a proton pump inhibitor (PPI) for epigastric pain that relieved when he easts. Pt states that the symptoms are still present and cause discomfort. Serum studies shows gastrin 4x reference range. Physician discontinues the PPI therapy in this patient. 3 weeks later, most appropriate to measure the concentration of which?

Serum Gastrin
The use of any agent that inhibits gastric secretion of acid
rule out Zollinger-Ellison syndrome you need to stop the PPIs first, then measure Gastrin to make that your intial measure of elevated Gastrin was not due to your drug rather than the patients disease.

Image: A 62-year-old man - comes to physician for a follow-up exam. One month ago, he was prescribed a proton pump inhibitor (PPI) for epigastric pain that relieved when he easts. Pt states that the symptoms are still present and cause discomfort. Serum studies shows gastrin 4x reference range. Physician discontinues the PPI therapy in this patient. 3 weeks later, most appropriate to measure the concentration of which?

Leave a Comment

Scroll to Top